You are on page 1of 247

QUANTITATIVE APTITUDE

Solve the following and check with the answers given at the end.

1. It was calculated that 75 men could complete a piece of work in 20 days. When work was
scheduled to commence, it was found necessary to send 25 men to another project. How
much longer will it take to complete the work?

2. A student divided a number by 2/3 when he required to multiply by 3/2. Calculate the
percentage of error in his result.

3. A dishonest shopkeeper professes to sell pulses at the cost price, but he uses a false weight
of 950gm. for a kg. His gain is …%.

4. A software engineer has the capability of thinking 100 lines of code in five minutes and can
type 100 lines of code in 10 minutes. He takes a break for five minutes after every ten
minutes. How many lines of codes will he complete typing after an hour?

5. A man was engaged on a job for 30 days on the condition that he would get a wage of Rs. 10
for the day he works, but he have to pay a fine of Rs. 2 for each day of his absence. If he
gets Rs. 216 at the end, he was absent for work for ... days.

6. A contractor agreeing to finish a work in 150 days, employed 75 men each working 8 hours
daily. After 90 days, only 2/7 of the work was completed. Increasing the number of men by
________ each working now for 10 hours daily, the work can be completed in time.

7. what is a percent of b divided by b percent of a?


(a) a (b) b (c) 1 (d) 10 (d) 100

8. A man bought a horse and a cart. If he sold the horse at 10 % loss and the cart at 20 % gain,
he would not lose anything; but if he sold the horse at 5% loss and the cart at 5% gain, he
would lose Rs. 10 in the bargain. The amount paid by him was Rs._______ for the horse and
Rs.________ for the cart.

9. A tennis marker is trying to put together a team of four players for a tennis tournament out
of seven available. males - a, b and c; females – m, n, o and p. All players are of equal
ability and there must be at least two males in the team. For a team of four, all players must
be able to play with each other under the following restrictions:
b should not play with m,
c should not play with p, and
a should not play with o.
Which of the following statements must be false?
1. b and p cannot be selected together
2. c and o cannot be selected together
3. c and n cannot be selected together.

Kumaraguru College, Coimbatore


10-12. The following figure depicts three views of a cube. Based on this, answer questions
10-12.

6 5 4

1 22 3 6
2 2 3

10. The number on the face opposite to the face carrying 1 is _______ .

11. The number on the faces adjacent to the face marked 5 are _______ .

12. Which of the following pairs does not correctly give the numbers on the opposite faces.
(1) 6,5 (2) 4,1 (3) 1,3 (4) 4,2

13. Five farmers have 7, 9, 11, 13 & 14 apple trees, respectively in their orchards. Last year,
each of them discovered that every tree in their own orchard bore exactly the same number
of apples. Further, if the third farmer gives one apple to the first, and the fifth gives three to
each of the second and the fourth, they would all have exactly the same number of apples.
What were the yields per tree in the orchards of the third and fourth farmers?

14. Five boys were climbing a hill. J was following H. R was just ahead of G. K was between G
& H. They were climbing up in a column. Who was the second?

15-18 John is undecided which of the four novels to buy. He is considering a spy
thriller, a Murder mystery, a Gothic romance and a science fiction novel. The books are
written by Rothko, Gorky, Burchfield and Hopper, not necessary in that order, and published
by Heron, Piegon, Blueja and sparrow, not necessary in that order.
(1) The book by Rothko is published by Sparrow.
(2) The Spy thriller is published by Heron.
(3) The science fiction novel is by Burchfield and is not published by Blueja.
(4)The Gothic romance is by Hopper.

15. Pigeon publishes ____________.

16. The novel by Gorky ________________.

17. John purchases books by the authors whose names come first and third in alphabetical order.
He does not buy the books ______.

18. On the basis of the first paragraph and statement (2), (3) and (4) only, it is possible to
deduce that
1. Rothko wrote the murder mystery or the spy thriller
2. Sparrow published the murder mystery or the spy thriller
3. The book by Burchfield is published by Sparrow.

19. If a light flashes every 6 seconds, how many times will it flash in ¾ of an hour?

Kumaraguru College, Coimbatore


20. If point P is on line segment AB, then which of the following is always true?
(1) AP = PB (2) AP > PB (3) PB > AP (4) AB > AP (5) AB > AP + PB

21. All men are vertebrates. Some mammals are vertebrates. Which of the following
conclusions drawn from the above statement is correct.
All men are mammals
All mammals are men
Some vertebrates are mammals.
None

22. Which of the following statements drawn from the given statements are correct?
Given:
All watches sold in that shop are of high standard. Some of the HMT watches are sold in
that shop.
a) All watches of high standard were manufactured by HMT.
b) Some of the HMT watches are of high standard.
c) None of the HMT watches is of high standard.
d) Some of the HMT watches of high standard are sold in that shop.

23-27.
1. Ashland is north of East Liverpool and west of Coshocton.
2. Bowling green is north of Ashland and west of Fredericktown.
3. Dover is south and east of Ashland.
4. East Liverpool is north of Fredericktown and east of Dover.
5. Fredericktown is north of Dover and west of Ashland.
6. Coshocton is south of Fredericktown and west of Dover.

23. Which of the towns mentioned is furthest of the north – west


(a) Ashland (b) Bowling green (c) Coshocton
(d) East Liverpool (e) Fredericktown

24. Which of the following must be both north and east of Fredericktown?
(a) Ashland (b) Coshocton (c) East Liverpool
I a only II b only III c only IV a & b Va&c

25. Which of the following towns must be situated both south and west of at least one other
town?
A. Ashland only
B. Ashland and Fredericktown
C. Dover and Fredericktown
D. Dover, Coshocton and Fredericktown
E. Coshocton, Dover and East Liverpool.

26. Which of the following statements, if true, would make the information in the numbered
statements more specific?
(a) Coshocton is north of Dover.
(b) East Liverpool is north of Dover
(c) Ashland is east of Bowling green.
(d) Coshocton is east of Fredericktown
Kumaraguru College, Coimbatore
(e) Bowling green is north of Fredericktown

27. Which of the numbered statements gives information that can be deduced from one or more
of the other statements?
(A) 1 (B) 2 (C) 3 (D) 4 (E) 6

28. Eight friends Harsha, Fakis, Balaji, Eswar, Dhinesh, Chandra, Geetha, and Ahmed are
sitting in a circle facing the center. Balaji is sitting between Geetha and Dhinesh. Harsha is
third to the left of Balaji and second to the right of Ahmed. Chandra is sitting between
Ahmed and Geetha and Balaji and Eshwar are not sitting opposite to each other. Who is
third to the left of Dhinesh?

29. If every alternative letter starting from B of the English alphabet is written in small letter,
rest all are written in capital letters, how the month “ September” be written.
(1) SeptEMbEr (2) SEpTeMBEr (3) SeptembeR
(4) SepteMber (5) None of the above.

30. The length of the side of a square is represented by x+2. The length of the side of an
equilateral triangle is 2x. If the square and the equilateral triangle have equal perimeter, then
the value of x is _______.

31. It takes Mr. Karthik y hours to complete typing a manuscript. After 2 hours, he was called
away. What fractional part of the assignment was left incomplete?

32. Which of the following is larger than 3/5?


(1) ½ (2) 39/50 (3) 7/25 (4) 3/10 (5) 59/100

33. The number that does not have a reciprocal is ____________.

34. There are 3 persons Sudhir, Arvind, and Gauri. Sudhir lent cars to Arvind and Gauri as
many as they had already. After some time Arvind gave as many cars to Sudhir and Gauri as
many as they have. After sometime Gauri did the same thing. At the end of this transaction
each one of them had 24. Find the cars each originally had.

35. A man bought a horse and a cart. If he sold the horse at 10 % loss and the cart at 20 % gain,
he would not lose anything; but if he sold the horse at 5% loss and the cart at 5% gain, he
would lose Rs. 10 in the bargain. The amount paid by him was Rs._______ for the horse and
Rs.________ for the cart.

Answers:

1. Answer:
30 days.
Explanation:
Before:
One day work = 1 / 20
One man’s one day work = 1 / ( 20 * 75)
Now:
No. Of workers = 50
Kumaraguru College, Coimbatore
One day work = 50 * 1 / ( 20 * 75)

The total no. of days required to complete the work = (75 * 20) / 50 = 30

2. Answer:
0%
Explanation:
Since 3x / 2 = x / (2 / 3)

3. Answer:
5.3 %
Explanation:
He sells 950 grams of pulses and gains 50 grams.
If he sells 100 grams of pulses then he will gain (50 / 950) *100 = 5.26

4. Answer:
250 lines of codes

5. Answer:
7 days
Explanation:
The equation portraying the given problem is:
10 * x – 2 * (30 – x) = 216 where x is the number of working days.
Solving this we get x = 23
Number of days he was absent was 7 (30-23) days.

6. Answer:
150 men.
Explanation:
One day’s work = 2 / (7 * 90)
One hour’s work = 2 / (7 * 90 * 8)
One man’s work = 2 / (7 * 90 * 8 * 75)

The remaining work (5/7) has to be completed within 60 days, because the total
number of days allotted for the project is 150 days.

So we get the equation

(2 * 10 * x * 60) / (7 * 90 * 8 * 75) = 5/7 where x is the number of men working


after the 90th day.

We get x = 225
Since we have 75 men already, it is enough to add only 150 men.

7. Answer:
(c) 1
Explanation:
a percent of b : (a/100) * b
b percent of a : (b/100) * a
Kumaraguru College, Coimbatore
a percent of b divided by b percent of a : ((a / 100 )*b) / (b/100) * a )) = 1

8. Answer:
Cost price of horse = Rs. 400 & the cost price of cart = 200.
Explanation:-
Let x be the cost price of the horse and y be the cost price of the cart.
In the first sale there is no loss or profit. (i.e.) The loss obtained is equal to the gain.

Therefore (10/100) * x = (20/100) * y

X = 2 * y -----------------(1)
In the second sale, he lost Rs. 10. (i.e.) The loss is greater than the profit by Rs. 10.

Therefore (5 / 100) * x = (5 / 100) * y + 10 -------(2)


Substituting (1) in (2) we get
(10 / 100) * y = (5 / 100) * y + 10
(5 / 100) * y = 10
y = 200
From (1) 2 * 200 = x = 400

9. Answer:
3.
Explanation:
Since inclusion of any male player will reject a female from the team. Since there
should be four member in the team and only three males are available, the girl, n should
included in the team always irrespective of others selection.

10. Answer:
5

11. Answer:
1,2,3 & 4

12. Answer:
B

13. Answer:
11 & 9 apples per tree.
Explanation:
Let a, b, c, d & e be the total number of apples bored per year in A, B, C, D & E ‘s
orchard. Given that a + 1 = b + 3 = c – 1 = d + 3 = e – 6
But the question is to find the number of apples bored per tree in C and D ‘s orchard. If is
enough to consider c – 1 = d + 3.
Since the number of trees in C’s orchard is 11 and that of D’s orchard is 13. Let x
and y be the number of apples bored per tree in C & d ‘s orchard respectively.
Therefore 11 x – 1 = 13 y + 3
By trial and error method, we get the value for x and y as 11 and 9

14. Answer:
Kumaraguru College, Coimbatore
G.
Explanation:
The order in which they are climbing is R – G – K – H – J

15 – 18
Answer:
Novel Name Author Publisher
Spy thriller Rathko Heron
Murder mystery Gorky Piegon
Gothic romance Burchfield Blueja
Science fiction Hopper Sparrow

Explanation:
Given
Novel Name Author Publisher
Spy thriller Rathko Heron
Murder mystery Gorky Piegon
Gothic romance Burchfield Blueja
Science fiction Hopper Sparrow

Since Blueja doesn’t publish the novel by Burchfield and Heron publishes the novel
spy thriller, Piegon publishes the novel by Burchfield.
Since Hopper writes Gothic romance and Heron publishes the novel spy thriller,
Blueja publishes the novel by Hopper.
Since Heron publishes the novel spy thriller and Heron publishes the novel by
Gorky, Gorky writes Spy thriller and Rathko writes Murder mystery.

19. Answer:
451 times.
Explanation:
There are 60 minutes in an hour.
In ¾ of an hour there are (60 * ¾) minutes = 45 minutes.
In ¾ of an hour there are (60 * 45) seconds = 2700 seconds.
Light flashed for every 6 seconds.
In 2700 seconds 2700/6 = 450 times.
The count start after the first flash, the light will flashes 451 times in ¾ of an hour.

20. Answer:
(4)
Explanation:
P
A B
Since p is a point on the line segment AB, AB > AP

21. Answer: (c)

22. Answer: (b) & (d).

Kumaraguru College, Coimbatore


Ahmed
23 - 27.Answer:
Fakis Chandra
28. Answer: Fakis
Explanation: Harsha Geetha

Eswar Balaji

Dhinesh

29. Answer:
(5).
Explanation:
Since every alternative letter starting from B of the English alphabet is written in
small letter, the letters written in small letter are b, d, f...
In the first two answers the letter E is written in both small & capital letters, so they
are not the correct answers. But in third and fourth answers the letter is written in small
letter instead capital letter, so they are not the answers.

30. Answer:
x=4
Explanation:
Since the side of the square is x + 2, its perimeter = 4 (x + 2) = 4x + 8
Since the side of the equilateral triangle is 2x, its perimeter = 3 * 2x = 6x
Also, the perimeters of both are equal.
(i.e.) 4x + 8 = 6x
(i.e.) 2x = 8  x = 4.

31. Answer:
(y – 2) / y.
Explanation:
To type a manuscript karthik took y hours.
Therefore his speed in typing = 1/y.
He was called away after 2 hours of typing.
Therefore the work completed = 1/y * 2.
Therefore the remaining work to be completed = 1 – 2/y.
(i.e.) work to be completed = (y-2)/y

32. Answer:
(2)

33. Answer:
1
Explanation:
Kumaraguru College, Coimbatore
One is the only number exists without reciprocal because the reciprocal of one is one
itself.

34. Answer:
Sudhir had 39 cars, Arvind had 21 cars and Gauri had 12 cars.
Explanation:
Sudhir Arvind Gauri

Finally 24 24 24
Before Gauri’s transaction 12 12 48
Before Arvind’s transaction 6 42 24
Before Sudhir’ s transaction 39 21 12

35. Answer:
Cost price of horse: Rs. 400 &
Cost price of cart: Rs. 200
Explanation:
Let x be the cost of horse & y be the cost of the cart.
10 % of loss in selling horse = 20 % of gain in selling the cart
Therefore (10 / 100) * x = (20 * 100) * y
 x = 2y -----------(1)
5 % of loss in selling the horse is 10 more than the 5 % gain in selling the cart.
Therefore (5 / 100) * x - 10 = (5 / 100) * y
 5x - 1000 = 5y
Substituting (1)
10y - 1000 = 5y
5y = 1000
y = 200
x = 400 from (1)

Exercise 2.1
For the following, find the next term in the series

1. 6, 24, 60,120, 210

a) 336 b) 366 c) 330 d) 660

Answer : a) 336
Explanation : The series is 1.2.3, 2.3.4, 3.4.5, 4.5.6, 5.6.7, ..... ( '.' means product)

2. 1, 5, 13, 25

Answer : 41
Explanation : The series is of the form 0^2+1^2, 1^2+2^2,...

3. 0, 5, 8, 17

Answer : 24
Explanation : 1^2-1, 2^2+1, 3^2-1, 4^2+1, 5^2-1
Kumaraguru College, Coimbatore
4. 1, 8, 9, 64, 25 (Hint : Every successive terms are related)

Answer : 216
Explanation : 1^2, 2^3, 3^2, 4^3, 5^2, 6^3

5. 8,24,12,36,18,54

Answer : 27

6. 71,76,69,74,67,72
Answer : 67

7. 5,9,16,29,54
Answer : 103
Explanation : 5*2-1=9; 9*2-2=16; 16*2-3=29; 29*2-4=54; 54*2-5=103

8. 1,2,4,10,16,40,64 (Successive terms are related)


Answer : 200
Explanation : The series is powers of 2 (2^0,2^1,..).
All digits are less than 8. Every second number is in octal number system.
128 should follow 64. 128 base 10 = 200 base 8.

Exercise 2.2

Find the odd man out.

1. 3,5,7,12,13,17,19
Answer : 12
Explanation : All but 12 are odd numbers

2. 2,5,10,17,26,37,50,64
Answer : 64
Explanation : 2+3=5; 5+5=10; 10+7=17; 17+9=26; 26+11=37; 37+13=50; 50+15=65;

3. 105,85,60,30,0,-45,-90
Answer : 0
Explanation : 105-20=85; 85-25=60; 60-30=30; 30-35=-5; -5-40=-45; -45-45=-90;

Exercise 3
Solve the following.

1. What is the number of zeros at the end of the product of the numbers from 1 to 100?
Answer : 127
2. A fast typist can type some matter in 2 hours and a slow typist can type the same in 3 hours. If
both type combinely, in how much time will they finish?
Answer : 1 hr 12 min
Explanation : The fast typist's work done in 1 hr = 1/2
Kumaraguru College, Coimbatore
The slow typist's work done in 1 hr = 1/3
If they work combinely, work done in 1 hr = 1/2+1/3 = 5/6
So, the work will be completed in 6/5 hours. i.e., 1+1/5 hours = 1hr 12 min

3. Gavaskar's average in his first 50 innings was 50. After the 51st innings, his average was 51.
How many runs did he score in his 51st innings. (supposing that he lost his wicket in his 51st
innings)
Answer : 101
Explanation : Total score after 50 innings = 50*50 = 2500
Total score after 51 innings = 51*51 = 2601
So, runs made in the 51st innings = 2601-2500 = 101
If he had not lost his wicket in his 51st innings, he would have scored an unbeaten 50
in his 51st innings.

4. Out of 80 coins, one is counterfeit. What is the minimum number of weighings needed to find out
the counterfeit coin?
Answer : 4

5. What can you conclude from the statement : All green are blue, all blue are red. ?
(i) some blue are green
(ii) some red are green
(iii) some green are not red
(iv) all red are blue
(a) i or ii but not both
(b) i & ii only
(c) iii or iv but not both
(d) iii & iv

Answer : (b)

6. A rectangular plate with length 8 inches, breadth 11 inches and thickness 2 inches is available.
What is the length of the circular rod with diameter 8 inches and equal to the volume of the
rectangular plate?
Answer : 3.5 inches
Explanation : Volume of the circular rod (cylinder) = Volume of the rectangular plate
(22/7)*4*4*h = 8*11*2
h = 7/2 = 3.5

7. What is the sum of all numbers between 100 and 1000 which are divisible by 14 ?
Answer : 35392
Explanation : The number closest to 100 which is greater than 100 and divisible by 14 is
112, which is the first term of the series which has to be summed.
The number closest to 1000 which is less than 1000 and divisible by 14 is 994, which
is the last term of the series.
112 + 126 + .... + 994 = 14(8+9+ ... + 71) = 35392

8. If s(a) denotes square root of a, find the value of s(12+s(12+s(12+ ...... upto infinity.
Answer : 4
Explanation : Let x = s(12+s(12+s(12+.....
Kumaraguru College, Coimbatore
We can write x = s(12+x). i.e., x^2 = 12 + x. Solving this quadratic equation, we get x = -3 or
x=4. Sum cannot be -ve and hence sum = 4.

9. A cylindrical container has a radius of eight inches with a height of three inches. Compute how
many inches should be added to either the radius or height to give the same increase in volume?
Answer : 16/3 inches
Explanation : Let x be the amount of increase. The volume will increase by the same
amount if the radius increased or the height is increased.
So, the effect on increasing height is equal to the effect on increasing the radius.
i.e., (22/7)*8*8*(3+x) = (22/7)*(8+x)*(8+x)*3
Solving the quadratic equation we get the x = 0 or 16/3. The possible increase would be by
16/3 inches.

10. With just six weights and a balance scale, you can weigh any unit number of kgs from 1 to 364.
What could be the six weights?
Answer : 1, 3, 9, 27, 81, 243 (All powers of 3)

11. Diophantus passed one sixth of his life in childhood, one twelfth in youth, and one seventh more
as a bachelor; five years after his marriage a son was born who died four years before his father at
half his final age. How old is Diophantus?
Answer : 84 years
Explanation : x/6 + x/12 + x/7 + 5 + x/2 + 4 = x

12 . If time at this moment is 9 P.M., what will be the time 23999999992 hours later?
Answer : 1 P.M.
Explanation : 24 billion hours later, it would be 9 P.M. and 8 hours before that it would be 1
P.M.

13. How big will an angle of one and a half degree look through a glass that magnifies things three
times?
Answer : 1 1/2 degrees
Explanation : The magnifying glass cannot increase the magnitude of an angle.

14. Divide 45 into four parts such that when 2 is added to the first part, 2 is subtracted from the
second part, 2 is multiplied by the third part and the fourth part is divided by two, all result in the
same number.
Answer: 8, 12, 5, 20
Explanation: a + b + c + d =45; a+2 = b-2 = 2c = d/2; a=b-4; c = (b-2)/2; d = 2(b-2);
b-4 + b + (b-2)/2 + 2(b-2) = 45;

15. I drove 60 km at 30 kmph and then an additional 60 km at 50 kmph. Compute my average speed
over my 120 km.
Answer : 37 1/2
Explanation : Time reqd for the first 60 km = 120 min.; Time reqd for the second 60 km =
72 min.; Total time reqd = 192 min
Avg speed = (60*120)/192 = 37 1/2

Questions 16 and 17 are based on the following :


Kumaraguru College, Coimbatore
Five executives of European Corporation hold a Conference in Rome
Mr. A converses in Spanish & Italian
Mr. B, a spaniard, knows English also
Mr. C knows English and belongs to Italy
Mr. D converses in French and Spanish
Mr. E , a native of Italy knows French

16. Which of the following can act as interpreter if Mr. C & Mr. D wish to converse
a) only Mr. A b) Only Mr. B c) Mr. A & Mr. B d) Any of the other three

Answer : d) Any of the other three.


Explanation : From the data given, we can infer the following.
A knows Spanish, Italian
B knows Spanish, English
C knows Italian, English
D knows Spanish, French
E knows Italian, French
To act as an interpreter between C and D, a person has to know one of the combinations
Italian&Spanish, Italian&French, English&Spanish, English&French
A, B, and E know atleast one of the combinations.

17. If a 6th executive is brought in, to be understood by maximum number of original five he should
be fluent in
a) English & French b) Italian & Spanish c) English & French d) French & Italian
Answer : b) Italian & Spanish
Explanation : No of executives who know
i) English is 2
ii) Spanish is 3
iii) Italian is 3
iv) French is 2
Italian & Spanish are spoken by the maximum no of executives. So, if the 6th executive is fluent
in Italian & Spanish, he can communicate with all the original five because everybody knows either
Spanish or Italian.

18. What is the sum of the first 25 natural odd numbers?


Answer : 625
Explanation : The sum of the first n natural odd nos is square(n).
1+3 = 4 = square(2) 1+3+5 = 9 = square(3)

19. The sum of any seven consecutive numbers is divisible by


a) 2 b) 7 c) 3 d) 11

Exercise 3
Try the following.

1. There are seventy clerks working in a company, of which 30 are females. Also, 30 clerks are
married; 24 clerks are above 25 years of age; 19 married clerks are above 25 years, of which

Kumaraguru College, Coimbatore


7 are males; 12 males are above 25 years of age; and 15 males are married. How many
bachelor girls are there and how many of these are above 25?

2. A man sailed off from the North Pole. After covering 2,000 miles in one direction he turned
West, sailed 2,000 miles, turned North and sailed ahead another 2,000 miles till he met his
friend. How far was he from the North Pole and in what direction?

3. Here is a series of comments on the ages of three persons J, R, S by themselves.


S : The difference between R's age and mine is three years.
J : R is the youngest.
R : Either I am 24 years old or J 25 or S 26.
J : All are above 24 years of age.
S : I am the eldest if and only if R is not the youngest.
R : S is elder to me.
J : I am the eldest.
R : S is not 27 years old.
S : The sum of my age and J's is two more than twice R's age.
One of the three had been telling a lie throughout whereas others had spoken the truth.
Determine the ages of S,J,R.

4. In a group of five people, what is the probability of finding two persons with the same
month of birth?

5. A father and his son go out for a 'walk-and-run' every morning around a track formed by an
equilateral triangle. The father's walking speed is 2 mph and his running speed is 5 mph. The
son's walking and running speeds are twice that of his father. Both start together from one
apex of the triangle, the son going clockwise and the father anti-clockwise. Initially the
father runs and the son walks for a certain period of time. Thereafter, as soon as the father
starts walking, the son starts running. Both complete the course in 45 minutes. For how long
does the father run? Where do the two cross each other?

6. The Director of Medical Services was on his annual visit to the ENT Hospital. While going
through the out patients' records he came across the following data for a particular day : "
Ear consultations 45; Nose 50; Throat 70; Ear and Nose 30; Nose and Throat 20; Ear and
Throat 30; Ear, Nose and Throat 10; Total patients 100." Then he came to the conclusion
that the records were bogus. Was he right?

7. Amongst Ram, Sham and Gobind are a doctor, a lawyer and a police officer. They are
married to Radha, Gita and Sita (not in order). Each of the wives have a profession. Gobind's
wife is an artist. Ram is not married to Gita. The lawyer's wife is a teacher. Radha is married
to the police officer. Sita is an expert cook. Who's who?

8. What should come next?


1, 2, 4, 10, 16, 40, 64,

Questions 9-12 are based on the following :


Three adults – Roberto, Sarah and Vicky – will be traveling in a van with five children –
Freddy, Hillary, Jonathan, Lupe, and Marta. The van has a driver’s seat and one passenger
seat in the front, and two benches behind the front seats, one beach behind the other. Each
Kumaraguru College, Coimbatore
bench has room for exactly three people. Everyone must sit in a seat or on a bench, and
seating is subject to the following restrictions: An adult must sit on each bench.
Either Roberto or Sarah must sit in the driver’s seat.
Jonathan must sit immediately beside Marta.

9. Of the following, who can sit in the front passenger seat ?


(a) Jonathan (b) Lupe (c) Roberto (d) Sarah (e) Vicky

10. Which of the following groups of three can sit together on a bench?
(a) Freddy, Jonathan and Marta (b) Freddy, Jonathan and Vicky
(c) Freddy, Sarah and Vicky (d) Hillary, Lupe and Sarah
(e) Lupe, Marta and Roberto

11. If Freddy sits immediately beside Vicky, which of the following cannot be true ?
a. Jonathan sits immediately beside Sarah
b. Lupe sits immediately beside Vicky
c. Hillary sits in the front passenger seat
d. Freddy sits on the same bench as Hillary
e. Hillary sits on the same bench as Roberto

12. If Sarah sits on a bench that is behind where Jonathan is sitting, which of the following must
be true ?
a. Hillary sits in a seat or on a bench that is in front of where Marta is sitting
b. Lupe sits in a seat or on a bench that is in front of where Freddy is sitting
c. Freddy sits on the same bench as Hillary
d. Lupe sits on the same bench as Sarah
e. Marta sits on the same bench as Vicky

13. Make six squares of the same size using twelve match-sticks. (Hint : You will need an
adhesive to arrange the required figure)

14. A farmer has two rectangular fields. The larger field has twice the length and 4 times the
width of the smaller field. If the smaller field has area K, then the are of the larger field is
greater than the area of the smaller field by what amount?
(a) 6K (b) 8K (c) 12K (d) 7K

15. Nine equal circles are enclosed in a square whose area is 36sq units. Find the area of each
circle.

16. There are 9 cards. Arrange them in a 3*3 matrix. Cards are of 4 colors. They are red, yellow,
blue, green. Conditions for arrangement: one red card must be in first row or second row. 2
green cards should be in 3rd column. Yellow cards must be in the 3 corners only. Two blue
cards must be in the 2nd row. At least one green card in each row.

17. Is z less than w? z and w are real numbers.


(I) z2 = 25
(II) w = 9
To answer the question,
a) Either I or II is sufficient
Kumaraguru College, Coimbatore
b) Both I and II are sufficient but neither of them is alone sufficient
c) I & II are sufficient
d) Both are not sufficient

18. A speaks truth 70% of the time; B speaks truth 80% of the time. What is the probability that
both are contradicting each other?

19. In a family 7 children don't eat spinach, 6 don't eat carrot, 5 don't eat beans, 4 don't eat
spinach & carrots, 3 don't eat carrot & beans, 2 don't eat beans & spinach. One doesn't eat all
3. Find the no. of children.

20. Anna, Bena, Catherina and Diana are at their monthly business meeting. Their occupations
are author, biologist, chemist and doctor, but not necessarily in that order. Diana just told the
neighbour, who is a biologist that Catherina was on her way with doughnuts. Anna is sitting
across from the doctor and next to the chemist. The doctor was thinking that Bena was a
good name for parent's to choose, but didn't say anything. What is each person's
occupation?

1 ) There are 3 temples and a pond in front of each temple. Anything you dip
in the pond gets doubled. A person dips X flowers in pond 1 and offers it
to the God in temple 1. He then dips the remaining flowers in pond 2 and
offers the same number of flowers to God in temple 2. He then dips the
remaining flowers in pond 3 and offers it to God in temple 3. He is now left with
no flowers. How many flowers did he have in the beginning.
a)7 ///// b)6 c)5 d)4 (consider that he offers 8 flowers in each temple)

5. In the investigation regarding one theft among the suspects A,B,C one who is the thief always
tells lies, the helper to the thief tells both truth and lies , the other fellow who is innocent always
tell the truth. They made the following statements
A: I'am a doctor, B is a barber and C is the taxi driver.
B: I'am an employee of water works dept, A is the taxi driver and C is the doctor.
C: I'am a lawyer A is a taxi driver and B is the doctor.
Who is the thief?
Ans : A is the thief
A tells lies, b is true, c tells false : true : false.

2) a cube of 3 units is painted on all sides.If this cube is divided


into cubes of 1 unit,how many cube have none of their faces painted?
(a)... (b)2 (c)1///// (d)0 (e)none of these

3)if a person sells a product for Rs 141/- he suffers a loss of 6%.if he


has to have a profit of 10%, at what price should he sell it?
(a) (b)rs.175 (c).. (d)rs.165
Kumaraguru College, Coimbatore
calculate from multiple choice….

4)a ball falls from a height of 8ft ,bounces back to half the distance
& continues till it comes to rest.what is the total distance travelled by the ball?
(a)24ft (b)... (c)infinite (d)cannot be determined

5)which of the following is the sum of 3 consecutive prime nos?


(a)49 (b)59 (c)both a &b ( Prime No’s : 13, 17, 19, 23 )

6)if the area of a square has increased by 69%,by what % has its side
increased?
Ans : 30% (consider for 100 and calculate)

7)in a class the average age is 16yrs.if the teacher who is 40 yrs of
age is also included ,the average becomes 17yrs,how many students were there?
Ans : 40 - 17 = 23 students

8)if 3 houses are to be painted,mr A can paint a house in 6 days (nos are not same)...mr B can do
the same in 8 days...& mr.C in 12 days.if mr A does the work for 8 days & leaves for vacation, &
mr B continues the work for the next 6 days, for how many days should mr.C work?
ans:11 days ((1 – (8/18)) – (6/24)) – (C/36).

10)if the 1rst day of 1999 is a sunday,what is the last day?


(a)sunday (b)monday (c).... (d)cannot be determined.
Ans : If leap year then Monday else Sunday so its SUNDAY

11) If A driver drives a car four times a lap 10,20 30,60 kmph what
is his average speed.
ans:20kmph
Apply 2xy/x+y for 10, 20 and 30,60 AND finally for both answers

16)six squares of same dimension are kept side by side to make a


rectangle with a perimeter of 182 cm. what is the perimeter of each square.
ans) 52. 14x = 182 => side x = 13

18)how many rational numbers are there between 0 & 5


ans:infinite

13) There is a room with 12 slots for painting. The details about the paintings are as follows:
18th century paintings – 3 19th century paintings - 2 20th century paintings - 4
Make up a sequence so that the paintings may be arranged with the following rules.
1) The 5th slot is always empty.
2) There must be an empty slot between paintings of different groups (century).
3) The paintings of the same group can be placed together.
A set of questions were asked with the pattern that was framed.
Answer : 19,19,null,18,null,20,20,20,20,null,18,18

Kumaraguru College, Coimbatore


12.There are 2 cars moving in opposite directions. the distance between them is 300cm. They move
forward for 100cm with a speed of 50 cm/s, and comes back 50 cms at 25cm/s. after how many
seconds will the cars collide?
Answer : 12 seconds --- first 50cms 4 sec then 50 cms 4 sec and so…on

10.there is a pack of cards.Find the sum of integers excluding all the picture cards. Ans.216
Excluding A’s, kings, jacks, queens

8.There are 31 kgs of rice .With a 1 kg weight,Find out the minimum number of weighings
required to weigh 31 kgs.
Ans : 5 weighings ie., 1-1, 2-2, 4-4, 8-8, 16-16.

7.There are 80 coins.One coin is counterfeit in weight.How many minimum weighings are required
to find out the counterfeit coin?
Ans 5 weighings - - - divide into 3 parts and then start weighing.

6.There is a cube of side 4 cm.The faces are coloured in red.the cube is cut into pieces of size 1
cm.How many cubes have no colour
Ans: 8 cubes
1.Train A starts from station A at 11.am.Train B starts from station B at 12 noon. which is 15kms
apart, the speed of train A is 4 kmph,train B is 1 kmph for the first hour,2 kmph for the 2nd hour,3
kmph for the 3rd hour etc.At what time do the two trains meet?
ans:2 PM Train A travels 12kms and Train B travels 3 kms

Find sum of digits of D. Let A= 19991999 , B = sum of digits of A, C = sum of digits of B


D = sum of digits of C
(HINT : A = B = C = D (mod 9))

Answer
The sum of the digits od D is 1.
Let E = sum of digits of D.
It follows from the hint that A = E (mod 9)

Kumaraguru College, Coimbatore


Consider,
A = 19991999
< 20002000
= 22000 * 10002000
= 1024200 * 106000
< 10800 * 106000
= 106800
i.e. A < 106800
i.e. B < 6800 * 9 = 61200
i.e. C < 5 * 9 = 45
i.e. D < 2 * 9 = 18
i.e. E <= 9
i.e. E is a single digit number.
Also,
1999 = 1 (mod 9)
so 19991999 = 1 (mod 9)
Therefore we conclude that E=1.

There is a 50m long army platoon marching ahead. The last person in the platoon wants to give a
letter to the first person leading the platoon. So while the platoon is marching he runs ahead, reaches
the first person and hands over the letter to him and without stopping he runs and comes back to his
original position.
In the mean time the whole platoon has moved ahead by 50m.
The question is how much distance did the last person cover in that time. Assuming that he ran the
whole distance with uniform speed.
Answer
The last person covered 120.71 meters.
It is given that the platoon and the last person moved with uniform speed. Also, they both
moved for the identical amount of time. Hence, the ratio of the distance they covered - while
person moving forward and backword - are equal.
Let's assume that when the last person reached the first person, the platoon moved X meters
forward.
Thus, while moving forward the last person moved (50+X) meters whereas the platoon moved
X meters.
Similarly, while moving back the last person moved [50-(50-X)] X meters whereas the platoon
moved (50-X) meters.
Now, as the ratios are equal,
(50+X)/X = X/(50-X)
(50+X)*(50-X) = X*X
Solving, X=35.355 meters
Thus, total distance covered by the last person
= (50+X) + X
= 2*X + 50
= 2*(35.355) + 50
= 120.71 meters
Note that at first glance, one might think that the total distance covered by the last person is
100 meters, as he ran the total lenght of the platoon (50 meters) twice. TRUE, but that's the
relative distance covered by the last person i.e. assuming that the platoon is stationary.

Kumaraguru College, Coimbatore


if you take a marker & start from a corner on a cube, what is the maximum number of edges you
can trace across if you never trace across the same edge twice, never remove the marker from the
cube, & never trace anywhere on the cube, except for the corners & edges?

Answer
9
To verify this, you can make a drawing of a cube, & number each of its 12 edges. Then, always
starting from 1 corner & 1 edge, you can determine all of the possible combinations for
tracing along the edges of a cube.
There is no need to start from other corners or edges of the cube, as you will only be repeating
the same combinations. The process is a little more involved than this, but is useful for solving
many types of spatial puzzles.

26. A car travels from B at a speed of 20 km/hr. The bus travel starts from A at a time of 6 A.M.
There is a bus for every half an hour interval. The car starts at 12 noon. Each bus travels at a speed
of 25 km/hr. Distance between A and B is 100 km. During its journey , The number of buses that
the car encounter is
Solution : 16 Buses buses between 8-30AM to 4-30 pm encounters with the car

A-B+c>A+B-C i) B is +ve, ii) B is –ve when it will hold true.


This holds true when B is Negative.

The ratio of white balls and black balls is 1:2. If 9 gray balls is added it becomes 2:4:3. Then what is
number of black balls. Ans:12

There are 80 coins, among them one coin weighs less compared to other. You are given a physical
balance to weigh. In how many wieghings the odd coin can be found. Ans: 5 weighings

If all the 6 are replaced by 9, then the algebraic sum of all the numbers from 1 to 100(both
inclusive) varies by
Ans: 330

The total no. of numbers that are divisible by 2 or 3 between 100 and 200(both inclusive) Ans:67

The average temperature of days from Monday to Wednesday is 37 degree Celsius and that of from
Tuesday to Thursday is 34 degrees. The temperature of Thursday is 4/5th of Monday. Then the
temperature of Thursday is
Ans: 36 degrees

B is 50% faster than A. If A starts at 9 A.M. and B starts at 10 A.M. A travels at a speed of 50
km/hr. If A and B are 300 kms apart, The time when they meet when they travel in opposite
direction is
Ans:12 noon

1. The following information is available about a group of young men in a colony. [ Marks: 8 ]

Kumaraguru College, Coimbatore


All handsome, fair skinned, muscular, lean, employed, and rich men are tall.
All handsome men are fair skinned.
Some muscular men are handsome.
Some muscular men are not fair skinned.
All lean men are muscular.
No lean man is handsome.
No fair skinned man who is not handsome is rich.
All tall men who are neither fair skinned nor muscular are employed.

1. Pramod is not fair skinned. Which of the following must be true?


a) Pramod is employed.
b) if Pramod is muscular, he is neither handsome nor lean.
c) if Pramod is tall, he is employed or muscular.
d) if Pramod is not employed, he is muscular.
e) if Pramod is tall, he may be muscular or handsome, but not both.

2. Which must be false if the information given is true ?


a) no lean men are fair skinned.
b) some fair skinned men are lean.
c) some rich men are both fair skinned and muscular.
d) some tall men are neither fair skinned nor employed.
e) some rich men are lean.

3. Which of the following can be deduced from the information given?


a) all rich men are handsome.
b) some rich men are handsome.
c) some rich men are employed.
d) some rich men are muscular.
e) all rich men are handsome, muscular, or employed.

4. Which cannot be shown to be true or false on the basis of the information given?
I. No fair skinned or muscular man is employed
II. Some muscular men are fair skinned but not handsome
III. No fair skinned man is both handsome and lean

a) I only b) II only c) III only d) I and II e) II and III

Soln.

Handsome Fair Muscular Lean Employed Rich Tall


skinned
Handsome ******* ALL ALL
Fair NO ****** NO ALL
skinned
Muscular SOME SOME ******* ALL
Lean ALL ******* ALL
Employed ****** ALL
Rich ***** ALL

Kumaraguru College, Coimbatore


Tall NO NO ALL ******

1. (c)
2. (a)
3. (a)
4. ( )
---------------------------------------------------------------------------------------------------------------------

2. [ Marks : 4 ]

After gathering 770 chestnuts, three girls divided them up so that amounts were in the same
proportion as their ages. As often as Mary took four chestnuts, Nelli took three, and for every six
that Mary received, Susie took seven. How many chestnuts did each girl get?
Soln.

Mary Nelli Susie

4 3 0+
6 3+ 7
8 6 7+
12 9 14

Therefore,
Mary’s share : Nelli’s share : Susie’s share :: 12 : 9 : 14

Thus,
Mary’s share = 12/ ( 12 + 9 + 14 ) * 770 = (12 / 35) * 770 = 264 chestnuts.
Nelli’s share = 9/ ( 12 + 9 + 14 ) * 770 = ( 9 / 35) * 770 = 198 chestnuts.
Susie’s share = 14/ ( 12 + 9 + 14 ) * 770 = (12 / 35) * 770 = 298 chestnuts.

---------------------------------------------------------------------------------------------------------------------

3. [ Marks : 5 ]

A supportive young hare and tortoise raced in opposite directions around a circular track that was
100 yards in diameter. They started at the same spot, but the hare did not move until the tortoise had
a start of one eighth of the distance ( that is, the circumference of the circle). The hare held such a
poor opinion of the other’s racing ability that he sauntered along, nibbling the grass until he met the
tortoise. At this point the hare had gone one sixth of the distance. How many times faster than he
went before must the hare now run in order to win the race ?

Soln:

It says the hare moves only after the tortoise covers 1/8 of distance, i.e., 100Pi.

Time traveled is same. (T)

Kumaraguru College, Coimbatore


Tortoise:

Distance traveled = 100Pi – 100Pi/8 – 100Pi/6 = 100Pi(17/24).


Speed T = 100Pi(17/24)/T

Hare:

Distance traveled = 100Pi/6.


SpeedH =100Pi/6T.

To win the race hare should cover 100Pi * (5/6) distance by the time the tortoise covers
100Pi/6 distance in the same time T2

As the speed of tortoise is constant,

T2 = (100Pi/6) / SpeedT

= (100Pi/6) / (1700Pi/24T)

= 4T / 17

SpeedH 2 = (500Pi/6) / T2

= (500Pi/6) / (4T / 17)

= 2125 Pi/ 6T

SpeedH 2/Speed H = (2125 Pi / 6T) / (100Pi/6T)

= 21.25

Thus, to win the race the hare must now run 21.25 times faster than before in order to win the race.
---------------------------------------------------------------------------------------------------------------------

4. [ Marks : 4 ]

Ajit was driving down the country side when he saw a farmer tending his pigs and ducks in his
yard. Ajit asked the farmer how many of each he had.
The farmer replied that there were 60 eyes and 86 feet between them.
How many ducks and how many pigs were there?

Answer:

Let ducks be D and pigs be P.

No. of eyes = 60 => 2D+2P=60 =>D + P=30…………………………………….(1)


No. of feet = 86 => 2D+4P=86 =>D +2P=43…………………………………….(2)
Kumaraguru College, Coimbatore
On solving (1) & (2), we get :
P=13, D=17

---------------------------------------------------------------------------------------------------------------------5.
[ Marks : 4 ]

In Mulund, the shoe store is closed every Monday, the boutique is closed every Tuesday, the
grocery store is closed every Thursday and the bank is open only on Monday, Wednesday and
Friday. Everything is closed on Sunday.

One day A, B, C and D went shopping together, each with a different place to go. They made the
following statements:

A D and I wanted to go earlier in the week but there wasn’t day when we could both take care of
our errands.
B I did not want to come today but tomorrow I will not be able to do what I want to do.
C I could have gone yesterday or the day before just as well as today.
D Either yesterday or tomorrow would have suited me.

Which place did each person visit?


Soln.

Shoe store Boutique Grocery Bank


Monday × √ √ √
Tuesday √ × √ ×
Wednesday √ √ √ √
Thursday √ √ × ×
Friday √ √ √ √
Saturday √ √ √ ×
Sunday × × × ×

From Statements 1, 2 , 3 & 4:


All are open.
The day can be either Wednesday or Friday.

From statement 1:
Friday is eliminated.

Therefore, it was Wednesday.

From statement 2:
B visited either the Grocery or the Bank.

From statement 3:

C visited the Grocery.

Kumaraguru College, Coimbatore


Therefore, B visited the Bank.

From statement 4:

D visited the Shoe store.

Therefore, A visited the Boutique.

---------------------------------------------------------------------------------------------------------------------

6. [ Marks : 6 ]

The Novice hockey tournaments are on for beginners. Just three teams are in the league, and each
plays the other two teams just once. Only part of the information appears in the result chart, which
is given below.
Team Games Won Lost Tied Goals For Goals against
A 2 1 0
B 2 1 1 2
C 2

The scoring pattern in the tournament is as follows:

Two points are awarded to the winning team. In case of a tie, both teams are awarded one point, so
the total points in the standings should always equal the total number of games played ( since each
game played is counted as one for each of the two participating teams). Of course, total goals scored
for and goals scored against must be the same, since every goal scored for one team is scored
against another.

The games are played in the following order: Game 1: A Vs B; Game 2: A Vs C; Game B Vs C

Can you determine the score of each of the above games ?

Soln.

For B:
Since, Goals for are 1 & Goals against are 2 & since it had won a Game, it means that it won
one match 1-0 & lost the other 0-2.

For A:
It is given that A played a tied match. Thus, the tied match was played between A & C. Also,
since Goals for A are 0, it means it played a 0-0 tied match with C.
Goals against A are 1. Thus, it lost its match to B by 0-1.

For C:
It played a 0-0 tied match with A.
Goals for are 2 & Goals against are 0.

Kumaraguru College, Coimbatore


Therefore, it beat B 2-0.

Thus, the results of the three games are as follows:


Game 1 : B bt. A 1-0 ; Game 2 : A tied with C 0-0; Game 3: C bt. B 2-0

Team Games Won Lost Tied Goals For Goals against


A 2 0 1 1 0 1
B 2 1 1 0 1 2
C 2 1 0 1 2 0

7. [ Marks : 8 ]
A recent murder case centered around the six men: Clam, Flip, Gront, Herm, Mast, and Walt. In one
order or another these men were the victim, the murderer, the witness, the police, the judge, and the
hangman. The facts of the case were simple. The victim had died instantly from the effect of
gunshot wound inflicted by a shot. After a lengthy trial the murderer was convicted, sentenced to
death, and hanged.
1. Mast knew both the victim and the murderer.
2. In court the judge asked Clam his account of the shooting.
3. Walt was the last of the six to see Flip alive.
4. The police testified that he picked up Gront near the place where the body was found.
5. Herm and Walt never met.
What role did each of the following play in this melodrama ?
a) Murderer
b) Victim
c) Judge
d) Witness
Soln.
From Stat 2:
Clam can’t be Judge, Hangman, Victim.

From Stat 3:
Walt can be the Murderer and Flip the Victim.
Or
Walt can be the Hangman and Flip the Murderer.

From Stat 5:
Walt cannot be the murderer, because, irrespective of who Herm was, Walt would have met
Herm in that case.
Thus, Walt is the Hangman and Flip the Murderer.
Also, as the Victim and the Hangman never met, Herm is the Victim.

Thus, now from Stat 2:


Clam is either the Witness or the Police.

From Stat 5:
Gront is the Witness, since he can’t be the Judge or the Police.

Kumaraguru College, Coimbatore


Thus, Clam is the Police.

This leaves only one option for Mast : The Judge.


These role assignments do not contradict any of the given 5 clues.

Clam Flip Gront Herm Mast Walt


Murderer ** YES ** ** ** **
Victim ** ** ** YES ** **
Judge ** ** ** ** YES **
Hangman ** ** ** ** ** YES
Police YES ** ** ** ** **
Witness ** ** YES ** ** **

Q3)when the asctual time pass 1hr wall clock ic 10 min behind it
when 1 hr is shown by wall clock, table clock shows 10 min ahead
of ih when table clock shows 1 hr the alarm closck goes 5minbehind
it,when alarm clock goes 1 hr wrist watch is 5 min ahead of it assuming
that all clocks are correcrt with actual tinme at 12 noon what will
be time shown by wrist watch after 6 hr
ans---5:47:32.5
(n X 60 )50/60 X 70/60 X 55/60 X 65/60

8. [ Marks : 5 ]

Fodder, Pepsi and Cereale often eat dinner out.

a) each orders either coffee or tea after dinner.


b) if Fodder orders coffee, then Pepsi orders the drink that Cereale orders.
c) if Pepsi orders coffee, then Fodder orders the drink that Cereale does not order.
d) if Cereale orders tea, then Fodder orders the drink that Pepsi orders

Which person/persons always order(s) the same drink after dinner ?

Soln.

The following cases arise:

Fodder Pepsi Cereale


Case 1 Coffee Coffee Coffee
Case 2 Coffee Tea Tea
Case 3 Coffee Coffee Tea
Case 4 Tea Coffee Coffee

Kumaraguru College, Coimbatore


Case 5 Tea Tea Tea
Case 6 Coffee Coffee Tea

From (b), Cases 3 & 6 are ruled out.


From (c), Case 1 is ruled out.
From (d), Case 2 is ruled out.

Thus, only Cases 4 & 5 are possible.

In either case, Fodder orders Tea.


Thus, Fodder always orders Tea after dinner.

-----------------------------------------------------------------------------------------------------------------

9. [ Marks : 6 ]

At a recent birthday party there were four mothers and their children aged 1,2,3 and 4. From the
clues below can you work out whose child is who and their relevant ages?

 It was Jane’s child’s birthday party.


 Brian is not the oldest child.
 Sarah had Anne just over a year ago.
 Laura’s child’s will be the next birthday.
 Daniel is older than Charlie is.
 Teresa’s child is the oldest.
 Charlie is older than Laura’s child.

Soln.

Brian(2) Anne(1) Daniel(4) Charlie(3)


Jane(3) ------------- ------------ --------------- Yes
Sarah(1) --------- Yes ------------ -----------
Laura(2) Yes ------------- ------------ ------------
Teresa(4) ---------- ------------ Yes ----------

2> Four perssons are three to cross a bridge they have one torch light
one person A can cross in 1 min
person B can cross in 2 min
person C can cross in 5 min
person D can cross in 10 min
They have to cross bridge with in 17 min.
At a time only two persons can cross
I A&B ----> 2
II A <----1
III C&D ---->10
IV B <----2

Kumaraguru College, Coimbatore


V A&B ---->2
__________
Total 17

5> Three are three boxes ,


In one box Two white balls
In two box 2 black balls
In three box 1 white &1 black
The lables on the boxes are not correct.Then you have to open
one box and to find the colour of the balls in all boxes.
Solution: Open the box labled black& white
If white balls are there then the box labled with white balls
contain black balls and labled with black balls contain one
black and one white ball and viceversa if two black ballsare there.

4> What is the maximum number of slices can you obtain by cutting
a cake with only 4 cuts?
ANS: 16

Question # 1
-------------
A Couple decided to travel a north country side .so they decide to travel
a minimum amount on car the first day and the second and sbsequent day
a distance of 20 miles .If they travel a total amount of 1080 miles.Find
he distance travlled on the 4th day and the 9 day.
Solution :
1st day - 20
2nd day - 40......4rth day - 80....9th day - 180miles

Question # 2
-------------
A card boarb of 34 * 14 has to be attached to a wooden box and a total
of 35 pins are to be used on the each side of the cardbox.Find the total
number of pins used .
Solution : 136 pins

Question # 3
-------------
During a Pizza buffet where A eats more times 2.4 than B, and B eats 6 times
less than C.find the leat number of times all the three has to eat.
Solution : A eats 2.4 times, B eats 1 time, C eats 6 times.

Question # 4
-------------
Last Year mu cousin came to my place and we played a game where the
Kumaraguru College, Coimbatore
loosing one has to give one choclate to the person who won the game .At the
end of the vacation,i.e the day my cousin was leaving she counted number of
games that i won an she won.At last she gave me a total of 8 choclates even
though she won about 12 games.Find the mumber of games that we played.
Solution : 32 games

1.
there are two clocks one runs 1min/hrs faster and other 1min/hr slower
when will the two clocks have time time difference of 1 hr :
ans : 30hrs

2.
i take a taxi whose no is 3 digit no. it is not divisible by 2,3,5,7
but divisible by 11 it is the smallest no possible:
ans : 121

1.i participated in a race.1/5th of those who are before me are


equal to 5/6th of those behind me.what were the total number of
contestants in the race?(3 Marks)
Solution : behind me are 6, before me are 25, so total is 32

2.Find the 3 digit number.Third digit is square root of first


digit.Second digit is sum of first and third digits.Find the
number(3 Marks)
Solution : 462

6. H&J play marbles beginning with same no. of


marbles. H wins 20 & then looses 2/3 of total. Now, J
has 4 times that of H. How many he had initially.
ANS.: 100

9. Five thieves. Each steals half the total + two


more, except the fifth who finds nothing to steal.
Initially , how many diamonds?
ANS.: 60

1a) BE * BE = ACB
A,B,C,E ARE NON ZERO NUMBERS FIND B,E.
ANS) B=1 E=9

3) THERE ARE SIX CARDS IN WHICH IT HAS TWO KING CARDS. ALL CARDS ARE
TURNED DOWN AND TWO CARDS ARE OPENED
a) WHAT IS THE POSSOBILITY TO GET AT LEAST ONE KING.1/6
b) WHAT IS THE POSSIBILITY TO GET TWO KINGS.1/3

4) A PERSON WENT TO A SHOP AND ASKED FOR CHANGE FOR 1.15PAISE.


BUT HE SAID THAT HE COULD NOT ONLY GIVE CHANGE FOR ONE RUPEE.
Kumaraguru College, Coimbatore
BUT ALSO FOR 50P,25P,10P AND 5P. WHAT WERE THE COINS HE HAD
ans) 1-->50 4--->10P 1--->25P

5) THERE ARE 3 NURSES AND THEY WORK ALTOGETHER ONLY ONCE IN A


WEEK.
NO NURSE IS CALLED TO WORK FOR 3 CONSECUTIVE DAYS.
NURSE 1 IS OFF ON TUESEDAY,THURSDAY AND SUNDAY.
NURSE 2 IS OFF ON SATURDAY.
NURSE 3 IS OFF ON THURSDAY,SUNDAY.
NO TWO NURSES ARE OFF MORE THAN ONCE A WEEK.
FIND THE DAY ON WHICH ALL THE 3 NURSES WERE ON WORK.
Solution : Friday

6) THERE ARE 5 PERSONS A,B,C,D,E AND EACH IS WEARING A BLOCK OR WHITE


CAP ON HIS HEAD. A PERSON CAN SEE THE CAPS OF THE REMAINING 4 BUT
CAN'T SEE HIS OWN CAP. A PERSON WEARING WHITE SAYS TRUE AND WHO
WEARS BLOCK SAYS FALSE.
i) A SAYS I SEE 3 WHITES AND 1 BLOCK
ii) B SAYS I SEE 4 BLOCKS
iii) E SAYS I SEE 4 WHITES
iiii) C SAYS I SEE 3 BLOCKS AND 1 WHITE.
NOW FIND THE CAPS WEARED BY A,B,C,D AND E are black, black, white, white,
black.

8) 1/3 ED OF THE CONTENTS OF A CONTAINER EVAPORATED ON THE 1 ST DAY.


3/4 TH OF THE REMAINING CONTENTS OF THE CONTAINER EVAPORATED THE
SECOND DAY. WHAT PART OF THE CONTENTS OF THE CONTAINER ARE LEFT
AT
THE END OF THE SECOND DAY.
Solution : 5/12 of x

q1) a frog jumps 3 ft comres bacfk 2ft in a day in how many day
it will come out of 30ft deep well (2 marks )ns
ans 28 day

q2) A-B=C, D/E=F, G+H=I, C.F=I


ANS : A=9, B=5, C=4, F=2, d=6, e=3, I=8, g=1, h=7.

q4)a soft. engr just returned from US has eaten too mucgh fat &put
a lot of weight everfy sunday he starts walking 4 km/hr on level
ground then up at 3 km\hr then back down hill at 6km\hr then again
on level grounggd at 4km\hr till he reaches his destination if he
returned home at 9 p.m. e what distance did he covered
Solution : 24km
it is aslo asked in some parer fingd half of the dist covered by hoim
in this case ans 12km
Kumaraguru College, Coimbatore
1. Five people A ,B ,C ,D ,E are related to each other.
Four of them make one true statement each as
follows. ( 8 marks)
(i) B is my father's brother.
(ii) E is my mother-in-law.
(iii)C is my son-in-law's brother
(iv)A is my brother's wife.
who made these statemens and what are the
relationships among them?
Ans: (i) D
(ii) B
(iii) E
(iv) C

2. fathers age is reverse of son`s age . one year


back faters age was twice of son`s age . what`s the
fathers current age
ans : 73

4. a man asks a weatherman what`s the past five days temp? he says i didn`t remember but i
can say their product is 12 and all are diff temperatures. what are the five temperatures?
ans: -2,-1,1,2,3

6: there ia truck which should reach some place at


11`o clock , if it travels with 30 mph it reaches i
hour before , if it travles with 20 mph it reaches 1
hour late. what is the distance it must be travlled
and what is the speed it must maintain to rech at
exact time?
ans: 120 miles and 24 mph

7: there are some stones (may be 6231) with eqaul


weight, but one of them having more weight than
others,how many times we need to weigh to find that
overweigheted stone?
ans: 12 ( as per the number given there, that no of
stones i didn`t remember exactly).

2) A car is traveling at a uniform speed. The driver sees a milestone showing a 2-digit number.
After traveling for an hour the driver sees another milestone with the same digits in reverse
order. After another hour the driver sees another milestone containing the same two digits
separated by a zero. What is the average speed of the driver?
ans : 45 kmph ie., at first 16 then 61 then 106

3) There are 3 societies A, B, C.


A lent cars to B and C as many as they had already.
Kumaraguru College, Coimbatore
After some time B gave as many tractors to A and C
as many as they have. After sometime c did the same thing. At the end of this transaction
each one of them had 24.
Find the cars each orginally had.
Ans: A had 39 cars, B had 21 cars & C had 12 cars

8) Find length and colour of serpent


if not green or 35m long, then it is 55m long
if not black or 45m long, then it is brown
if not black or 35m long, then it is 55m long
Solution : Brown 55m long

9) Three man A,B,C went in three direction and had stolen a mule , a horse and a camel they
ware caught by C.B.I and arrested . During their interrogation they gave the following
statements.
A: B had stolen a horse
B: A and C are both lying and I had stolen nothing.
C: A is lying and B had stolen a mule
One who had stolen a camel is telling lie and one who had stolen a horse is telling truth.
Among A,B,C who had stolen which animal ?
(ans : A-Camel, B-Mule, C-Horse)

2.In the 4 digits 1,2,3,4 how many 4 digited numbers are


> possible which are divisable by 4? Repetetions are allowed
 Ans 64

> 4. Theno. of children > adults . Theno .of adults > the no .of boys .
The no.of boys > no. of girls .The no.of girls > no.of familyi conditions
1.No family is without a child
> 2 Every girl has atleast one brotherand sister .
> Ans c>a>b>g>f; 9 6 5 4 3 .

7) 3 persons say these statements.


> > A says either Democratic or liberal wins the elections.
> > B says Democratic wins.C says neither democratic nor liberal wins
> > the election.of these only one is wrong.who wins the election?5 marks).
> > Solution : Democratic and C is wrong

3)complete the table.


> > Played won lost draw goals goals
>> for against
>>A 2 2 0 0 7 1
>>B 2 0 1 1 2 4
>>C 2 0 1 1 3 7
> > A,B,C are 3 hockey teams.(2 marks).

Kumaraguru College, Coimbatore


4) A says Party was held on :Thursday ,May 8th.correct
> > B says Party was held on :Tuesday,May 10th.
> > C says party was held on :Friday ,June 8th.
> > Given April 1 st was Tuesday.one of A,B,C says 1 correct.one says 1
wrong.and one was completely wrong of date,Month and day. Find the Day the
party held. (5marks).

6) Every station in N railroad issue everyother station's ticket.some stations are added.Now
they have to issue (if 36)46 more tickets.say the No.of stations after and before added.(5
marks).
Solution : 11 stations before and 13 stations after

2.one boy tells three ppl to guess mule color..


number one says its not blk
number two says its eiterh brown or grey
number 3 says its brown.
..the boy then says one is atleast lying ans atleast telling truth...finf mule color
ans grey?

9)Two men ride alternatively on a horse.The 1st man travel for a particular time and ties the
horse for the other person,then the other person will ride for the same time.The horse rides 12
miles an hr,and the person walks 4 miles an hr...how long will the horse be idle??
Solution ; 2 hrs half of the time

2.THERE WERE TWO CLOCK ONE IS GETTING SLOW BY TWO MINUTES/HR AND
ANOTHER ONE IS GAINING 1 MINUTE EVERY HOUR AND EXACTLY AFTER HOW
MANY HOURS THERE TWO CLOCK HAS A 1HR DIFFERENCE.
ANS(20HR)4MARKS

4.4 PERSONS ED,FRANK,GEORGE,HARRY BRING THEIR WIFE TO A PARTY AND


THEY
EXCHANGE EACH OTHER WIFE WHILE DANCING THEY GIVE CONDITIONS LIKE
THIS BETTY IS DANCING WITH ED,DOROTHY DANCING WITH ALICE
HUSBAND,ALICE DANCING WITH CAROLS HUSBAND,FRANK WAS DANCING
WITH GEORGE'S WIFE THE ANSWER IS(8 MARKS)
WIFES MARRIEDTO DANCING WITH
ALICE GEORGE FRANK
BETTY HARRY ED
CAROL FRANK HARRY
DOROTHY ED GEORGE

Kumaraguru College, Coimbatore


8.THERE IS A NUMBER WHICH WHEN ADDED WITH 33/4 GIVES THE SAME ANS
AS MULTIPLIED WITH 33/4.(3 MARKS) Ans : 33/29

9.A MAN HAS 50 MANGOES AND PLACE THE 50 MANGOES LIKE THIS THE
DISTANCE BETN 1AND 2 IS 1 YARD AND THE DISTANCE BETN.2&3 IS 3YARD AND
THE DISTANCE BETN. 3&4 IS 5YARD LIKE THIS HE INCREASES THE DISTANCE
BETN . MANGOES IN STEPS OF TWO.AFTER ARRANGING THEM LIKE THAT AND
IF HE START PICKING THEM UP HOW MUCH YARD HE HAS TO WALK.(5 MARKS)
ANS(2401)

10.A MAN SAYS THAT HE GAINED 10 PERCENT AS PROFIT IN SELLING A


SUITABLE
CLOTH MATERIAL.HE SAYS IF HE HAD PURCHASED THE SAME ONE 10 PERCENT
CHEAPER THAN IT WAS ACTUAL AND IF HE HAD SOLD IT FOR 20% PROFIT HE
GETS 25PAISE LESS.
FIND AT WHAT PRICE HE SELLED THE SUIT.
Solution : Consider for 100/- and then calculate for 25 paise less
ANS(Rs12.50)

1. A man was travelling to a place 30 miles away from starting point. he was speeding at 60
miles/hr. but when he came back, his car got breakdown and half an hour was wasted in
reparing that. altogether he took 1 hr for return journey. Find the avg. speed of the whole
journey.
Solution : consider for two conditions ie along the repair OR eliminate that.
Ans : 40miles/hr

1.Each alphabet A,B.. Z is a constant.A=1,B=2,C=3^2,D=4^9 n so


on.Each letter is assigned a value -the position of that letter
raised to the value of preceding alphabet.(C = 3 ^B,D= 4 ^ C n so on)
Compute the numerical value of (X-A)(X-B)(X-C)....(X-Y)(X-Z).
Solution : ZERO

5. 5 letters A,B,C,D,E represent different digit


given;
AB * CD = EEE and CC = sumthing (i don remember).
AB* D = ?(get 2nd relation from other frnds)
Solution :A is 1, b is 2, c is 3 and d is 7

8.A cyclist got his tyre punctured when he had covered two third of
the distance to be covered.Finishing on foot,He takes twice the time
taken before to reach the destination.How fast does he ride than
walking?
Solution : 6 times faster then walking

1. Find the next number in the series:


0,1,5,14,30,55,91,140,__204 (4 Marks).

Kumaraguru College, Coimbatore


3. There are two barbers who work at the same speed i.e. each one takes a quarter of an hour for a
haircut and 5 minutes for a shave. Now, there are three customers waiting to be attended to and the
barbers are in a hurry to finish their job as soon as possible. What is the least amount of time in
which they can accomplish their task??
Solution : in 30 min….one person two hair cuts and other 3 shave’s and one hair cut

7. A certain factory for manufacturing parts has 9 workers where each worker produces 15 sets per
day. The manager of the company works harder and produces 9 more sets than the average of all the
ten workers together. What is the total no. of sets produced by the workers?
Ans : 159 sets per day

8. There are four people: John, Brown, Clark, Sam with the posts of president, accountant, cashier
and manager, not necessarily in the same order. The president is always beaten by the cashier ina
game of chess. Despite this the president does not play with anybody else except the cashier.The
cashier and the manager are both better players than the accountant. John and Sam are next-door
neighbours and play chess every evening. Clark is a better player thanJohn. The accountant lives
close to the president's place but not close to the others.Find the designation of each person??? (5
Marks).
Solution : John – President, Brown – Accountant, Clark – Manager, Sam – Cashier

10. A company owns a safe. There are three persons entrusted with the task of looking after the
valuable documents inside the safe. But, each person does not trust the other. So, they decide to put
locks on the door in such a way that:
-- No one person alone can open the safe.
-- Any two persons together should be able to open the safe.
How many total number of locks and keys are required???
Solution : 2 Locks and for each lock 2 Keys

2. 3 members expect which will be the elected party among 4 parties namely Flying party, Eatwell,
Good sleep, Loosing parties
X expects that either f.p or e.w.p will win.
Y expects that either f.p or g.s will win
Z expects that neither e.w.p nor g.s.p will win
One part of each members statement is correct. Which party will win?
Ans: F.P

3. Three members are hunting in a forest. While crossing a stream cartridges of two of them gets
wetted up. Then the third person divides his cartidges equally. After 4 shots of each person, total
cartridges remaining are equal to what each had initially after sharing. How many total cartridges
they initially they shared?
Ans: 18
Y
Y/3 Y/3 Y/3
Y/3-4 Y/3-4 Y/3-4
3(Y/3-4) =Y/3
Y=18

Kumaraguru College, Coimbatore


5. Boys are allowed to watch football at c.v.Raman auditorium subjected to conditions.
1. the boy over age 16 can wear overcoat
2. no boy over age 15 can wear cap
3. to watch the football either he has to wear overcoat or cap or both
4. a boy with an umbrella or above 16 or both cannot wear sweater.
5. boys must either not watch football or wear sweater.
What is the appearance of the boy who is watching football.
Ans: Who are above 15 and below 16 with cap. :

1.My neighbor has seven children. Every brother has equal no. of brothers and sisters, but each
sister has twice as many brothers as sisters?
Ans: 4,3

2.There are 11more animals than birds in a pet shop. If there are as many birds as animals and if
there are as many animals as birds the no. of legs is 4/5 of the original. How many birds and
animals are there?
Ans: 22,11

3. One soap can be made out of the scraps obtained while preparing 11 soaps. How many soaps can
be prepared out of the scrap obtained out of 251 soaps. (Take all possibilities)
Ans: 25 ie 22 + 2 + 1

4. There is a 5digit no. 3 pairs of sum is eleven each.


Last digit is 3 times the first one.
3 rd digit is 3 less than the second.
4 th digit is 4 more than the second one.
Find the digit.
Ans : 25296.

5. 2XX
3XX
---
5XX
X4X
XX3
-------
XXXXX
-------
Find out the X's and seven is not there in the calculation.
Ans: 281x332

8) 1/3 ed of the contents of a container evaporated on the 1 st day.


3/4 th of the remaining contents of the container evaporated the
second day. what part of the contents of the container are left at
the end of the second day.
Kumaraguru College, Coimbatore
Solution : 1/6th of content is left

1) be * be = acb a,b,c,e are non zero numbers find b,e.


ans) b=1 e=9

6. There are 5 burglars and once went to a bakery to rob it obviously The first guy ate 1/2 of the
total bread and 1/2 of the bread The second guy ate 1/2 of the remaining and 1/2 of the bread. The
third guy ,fourth guy and fifth guy did the same. After fifth
guy there is no bread left out. How many bread are there?
Ans:31 ie., 31, 15, 7, 3, 1 and last 0

7. The main line train starts at 5.00AM and the harbor line train starts at 5.02AM.Each train has the
frequency of 10 minutes. If a guy goes in the morning at a random time what is the probability of he
getting main line train?
Ans: 0.8

A Couple decided to travel a north country side .so they decide to travel
a minimum amount on car the first day and the second and sbsequent day
a distance of 20 miles .If they travel a total amount of 1080 miles.Find
he distance travlled on the 4th day and the 9 day.
Solution : 80 miles and 160 miles

9.Two people used to ride on a horse and walk


alternatively.Riding speed is 12m/hr and walking speed
is 4 m/hr.First person rides the horse through some
distance and get down and tied the horse there and
walked the remaining distance.The second one walked
this distance and reaches near the horse and then
complete the rem distance on horse.Then what portion
of the total time did horse rests?
Solution : Horse rests, Half of the total time

7.There is a square cabbage patch.He told his sister


that i have a larger patch than last year and hence
211 more cabbages thios year.Then how many cabbages i
have this year.?
Ans:last year 105 * 105 = 11025 and this year 106*106=11236ie 211 difference

Question # 1
-------------
There r 100 nations competing for a world-cup.
The board decided to make Knock-out series.
How many matches to be played for deciding the world champion?
Solutions : 99 matches

Analytical.
Kumaraguru College, Coimbatore
1.AN old man spend 1/6th life as boy 1/12th as graduation 1/7th and 5 years as a political
career &
marriage then his son john is born. 4 yrs before his son is selected as governer as age whichis
equal to half old man's age.
Ans:-84yrs.
(eq:- x-(1/6x+1/12x+1/7x+5)-4=1/2x)

2:-Puzzle from shakuntala devi


Genrous woman spends1/2 the money on shopping & gives 1$ to begger.
then 1/2 remaining money & 2$ to restaurant.then1/2 remaining & 3$ to charity.
She left with only 1$.
How much $ she started with.
Ans:- 42$ ie., (((1+3)2 + 2)*2+1)*2

4.sons age is 5 times daughters.mother is five times son.father is 5 times wife.Total of all age is
Grandpas who is celebrating 81st B`day.
Ans:-5 yrs

6:-One woman buys plates worth 1.30$ at 2cent discount each plate.
Then she exchanged the plates for sausers & bowl where one bowl & one sausers costs equal
to one plate.no of sausers which costs 3 cent is10 more than bowl.no of saucers & bowl is 16
more than no of plates.
ns:-10Plates, 13 saucers, 3 bowls

10.Hart Is friend of AMY AXY.


1.Either Amy or Axy is older of three.
2.Either Hart or Amy is younger of three.
Find who is younger & older.
Answer:-Axy Older,Amy younger.

3. Bill, Ed, & Derek, along with their wives - Grace,


Helen, & Mary - in one order or the other were playing
18-holes golf. There score were as follows -
i) Grace, Helen, Mary, & Ed scored 106, 102, 100, &
94 respectively.
ii) Bill & Derek scored 98 & 96, but forgot who
scored what.
iii) Bill's wife was beaten by Ed's wife.
iv) When Bill & Derek remembered their scores, they
came to know that exactly two couples had same
total scores.
What was scored by Bill, & by Derek.
ANS.: Bill:96, Derek: 98

8. Following are the statements of three of the


participants in a race. One among them is the winner
Kumaraguru College, Coimbatore
of the race. One of them is a Sororrean, who always
tells the truth, one of them is a Nororrean, who
always lies, & the third one is Midrorrean, who lies
alternately. You have to find who is Sororrean, who is
Nororrean, & who is Miderorrean.
A: i) C obstructed my way, & hence I lost.
ii) C always tells the truth.
iii) C won the race.
B: i) C is not a Nororrean.
ii) A won the race.
C: i) B won the race.
ii) I didn't obstruct the way of C, because of
which he lost.
ANS.: A:Nororrean, B:Sororrean, C:Midrorrean
*******************************************************
If A is Sx, C is also Sx.
If A is Mx (with 2nd stat. true) then none is Nx.
If A is Mx (with 2nd stat. false) then none is Sx.
Hence, A is Nx, then C is Mx, & therefore, B is Sx.

1) There is a five digit number.


The fifth digit is one fourth of the third digit and
one half of the fourth digit. Third digit is one half of the first digit.
second digit is 5 more than the fifth digit.
What is that 5 digit no.?
Solution : 89464

4) A boy goes to school from his house.on one fourth oh his way to school, he crosses a
machinery station. And on one third of his way to school, he crosses a Railway station. He
crossed the machinery station at 7:30 and he crosses the Railway station at 7:35. when does
he leave the house & when does he reach the school ? (5M)
Solution : 7-15 to 8-15

7) Series...

a) 3,6,13,26,33,66, 73, 146?


b) 364, 361, 19, 16, 4, 1, -1, -4?

8) A large cube is painted on all sides with Red color. It is then cut into 27 small cubes.
How many sides of the cube will have...
a) 3 red faces. 8 cubes
b) 2 red faces. 12 cubes
c) 1 red face. 6 cubes
e) No red faces. (8M) 1 cube.

Kumaraguru College, Coimbatore


9) Four persons A,B,C,D were there. All were of different weights. All Four gave a
statement.Among the four statements only the person who is lightest in weight of all say lie
others gave a true statement.
A Says : B is heavier than D.
B Says : A is heavier than C.
C Says : I am heavier than D.
D Says : C is heavier than B.
Find the lightest & List the persons in ascending order according to their weights. (5M)
Order of weights is : A, B, C, D from heavy to ligthtest.

A, B, C, D&E are having their birthdays on consecutive days of the week


not necessarily in the same order.
A 's birthday comes before G's as many days as B's birthday comes after E's.
D is older than E by 2 days.
This time G's birthday came on Wednesday.
Then find the day of each of their birthdays?
Ans:
Birthday of D on SUNDAY
Birthday of A on MONDAY
Birthday of E on TUESDAY
Birthday of G on WEDNESDAY
Birthday of B on THURSDAY

Question # 6
-------------
A girl 'A' told to her friend about the size and color of a snake she has seen in the beach. It is
one of the colors brown/black/green and one of the sizes 35/45/55.
If it were not green or if it were not of length 35 it is 55.
If it were not black or if it were not of length 45 it is 55.
If it were not black or if it were not of length 35 it is 55.
a) What is the color of the snake?
b) What is the length of the snake?
Ans:
a) brown
b) 55

3)If a person sells two cows at Rs 210 making on one 10 % profit and on other 10% loss
.Overall he gets 5 %profit .Whats original cost of cows .143 and 67

4)Their are stamps of 2,20,10,15,7 .3 sets to be bought in no of 5 and two type of atamps in
number of 6 .total cost 3 rs .Find which stamps of 5 and which are in nos 6.
Solution : 2 * 5, 15 * 5, 7 * 5, 20 * 6, 10 * 6.

Kumaraguru College, Coimbatore


8)A is half full of wine another vessel B which is twice of A is 1/4rth fill ed of Wine .Both vessel
filled with water and put in C.What share in mixture is wine in C.
water : wine is 2 : 1

Q) Two dice are rolled. If the score is calculated as a product of the number
appeared. The score for the second role is six more than that of the 1st roal, the score for 3rd
roll is 6 less than the 2nd roll, the score for 4 th role is 11 more than the 3rd roll, the score on
5th role is five more than the 4th roll. Find the scores of 1,2,3,4 rolls
Solution : 9, 16, 9, 20, 25

A,B,C,D,E wore shirts of colours red,blue,green, yellow and orange


respectively. They stood on a ladder. Yellow is not the colour at the tail end
of the ladder. Yellow is between those who wore green and red. Green is
between those who wore yellow and orange and who is behind blue. Which
color is at the end of the ladder?
Solution : Red, yellow, green, orange, blue.

There is log weighing 30kgs. The log having twice thickness and twice short as first one will
weigh howmuch ??
Solution : 30Kgs

In a business, A invested 31/2 times more than B.They wanted to include C by taking an
amount of Rs2500 from C. How should tha amount be divided among A and B such that three
of them get the same interest?
Solution : A = 4250, B =750, c = 2500

5) Adding 1/4 of the time from midnight to the present time, to 1/2 of the time from present
until midnight, gives the present time. what is the present time. (ans 9.36 plz check)
Solution : x/4 + (24-x)/2 = x.

Q) One of the arms of a balance is longer than the other. If 1kg is kept on the left pan,it is
equal to 8 water melons. If the 1kg is kept on a a right pan, it is equal to 2 water melons on
right. If the weight of all the watermelons are sam, what is the weight of the water melon?
Solution : consider the wt. of melon as 200 grams....

8. A, B, C are the husbands and D, E, F are their wives not in that order. They are playing the
Golf following these conditions. D, E, F and B scores are as follows 106,102,100 and 94.A and
C scores are 98 and 96 not in that order as their names are not displayed. Two couples get the
same score. B wife beat the A wife list out the wives names and the scores they got.
Hus Wife Score Total
A F 98 102 200 100 198
B D 94 106 200 106 200
C E 96 100 96 102 198

(1) 9 cards are there. u have to arrange them in a 3*3 matrix.


Kumaraguru College, Coimbatore
cards are of 4 colors.they are red,yellow,blue,green.
conditions for arrangement: one red card must be in first row
or second row.2 green cards should be in 3rd column.Yellow
cards must be in the 3 corners only. Two blue cards must be in
the 2nd row. Atleast one green card in each row.
Solution:
Yello Red Gren
Blu Blu Gren
Yello Gren Yello

3. In a soap company a soap is manufactured with 11 parts.


For making one soap you will get 1 part as scrap. At the
end of the day u have 251 such scraps. From that how many
soaps can be manufactured? ans: 22 + 2+ 1 = 25.

3)A ship went on a voyage after 180 miles a plane statrted with 10 times
speed that of the ship. Find the distance when they meet from
starting point.

Ans. 180 + (x/10) = x


x = 200 miles 2 marks

6) There N stations on a railroad. After adding x stations 46 additional


tickets have to be printed. Find N and X.

Ans. let N(N-1) = t;


(N+x)(N+x-1) = t+46;
trail and error method x=2 and N=11 4 marks

2. No. of animals is 11 more than the no. of birds. If the no. of birds were the no. of animals
and no. of animals were the no. of birds( ie., interchanging no.s of animals and birds.), the
total no. of legs get reduced by one fifth (1/5). How many no. of birds and animals were there?
ans: birds:11,animals:22

One of Mr. Bajaj, his wife, their son and Mr. Bajaj's mother is an Engineer and another is a Doctor.

• If the Doctor is a male, then the Engineer is a male.

Kumaraguru College, Coimbatore


• If the Engineer is younger than the Doctor, then the Engineer and the Doctor are not blood relatives.
• If the Engineer is a female, then she and the Doctor are blood relatives.

Can you tell who is the Doctor and the Engineer?


Answer

Mr. Bajaj is the Engineer and either his wife or his son is the Doctor.

Mr. Bajaj's wife and mother are not blood relatives. So from 3, if the Engineer is a female, the Doctor is a male. But
from 1, if the Doctor is a male, then the Engineer is a male. Thus, there is a contradiction, if the Engineer is a female.
Hence, either Mr. Bajaj or his son is the Engineer.

Mr. Bajaj's son is the youngest of all four and is blood relative of each of them. So from 2, Mr. Bajaj's son is not the
Engineer. Hence, Mr. Bajaj is the Engineer.

Now from 2, Mr. Bajaj's mother can not be the Doctor. So the Doctor is either his wife or his son . It is not possible to
determine anything further.

Three men - Sam, Cam and Laurie - are married to Carrie, Billy and Tina, but not necessarily in the same order.

Sam's wife and Billy's Husband play Carrie and Tina's husband at bridge. No wife partners her husband and Cam does
not play bridge.

Who is married to Cam?

Answer

Carrie is married to Cam.

"Sam's wife and Billy's Husband play Carrie and Tina's husband at bridge."

It means that Sam is not married to either Billy or Carrie. Thus, Sam is married to Tina.

As Cam does not play bridge, Billy's husband must be Laurie.

Hence, Carrie is married to Cam.

There are 3 persons X, Y and Z. On some day, X lent tractors to Y and Z as many as they had. After a month Y gave as
many tractors to X and Z as many as they have. After a month Z did the same thing. At the end of this transaction each
one of them had 24.

Find the tractors each originally had?

Answer

One way to solve it is by making 3 equations and solve them simultaneously. But there is rather easier way to solve it
using Backtracing.

It's given that at the end, each had 24 tractors (24, 24, 24) i.e. after Z gave tractors to X & Y as many as they had. It
means that after getting tractors from Z their tractors got doubled. So before Z gave them tractors, they had 12 tractors
each and Z had 48 tractors. (12, 12, 48)

Similarly, before Y gave tractors to X & Z, they had 6 & 24 tractors respectively and Y had 42 tractors i.e. (6, 42, 24)

Kumaraguru College, Coimbatore


Again, before X gave tractors to Y & Z, they had 21 & 12 tractors respectively and X had 39 tractors i.e. (39, 21, 12)

Hence, initially X had 39 tractors, Y had 21 tractors and Z had 12 tractors.

A certain street has 1000 buildings. A sign-maker is contracted to number the houses from 1 to 1000. How many zeroes
will he need?

Answer

The sign-maker will need 192 zeroes.

Divide 1000 building numbers into groups of 100 each as follow:


(1..100), (101..200), (201..300), ....... (901..1000)

For the first group, sign-maker will need 11 zeroes.


For group numbers 2 to 9, he will require 20 zeroes each.
And for group number 10, he will require 21 zeroes.

The total numbers of zeroes required are


= 11 + 8*20 + 21
= 11 + 160 + 21
= 192

There are 9 coins. Out of which one is odd one i.e weight is less or more. How many iterations of weighing are required
to find odd coin?

Answer

It is always possible to find odd coin in 3 weighings and to tell whether the odd coin is heavier or lighter.

1. Take 8 coins and weigh 4 against 4.


o If both are not equal, goto step 2
o If both are equal, goto step 3

2. One of these 8 coins is the odd one. Name the coins on heavier side of the scale as H1, H2, H3 and H4.
Similarly, name the coins on the lighter side of the scale as L1, L2, L3 and L4. Either one of H's is heavier or
one of L's is lighter. Weigh (H1, H2, L1) against (H3, H4, X) where X is one coin remaining in intial
weighing.

o If both are equal, one of L2, L3, L4 is lighter. Weigh L2 against L3.
 If both are equal, L4 is the odd coin and is lighter.
 If L2 is light, L2 is the odd coin and is lighter.
 If L3 is light, L3 is the odd coin and is lighter.

o If (H1, H2, L1) is heavier side on the scale, either H1 or H2 is heavier. Weight H1 against H2
 If both are equal, there is some error.
 If H1 is heavy, H1 is the odd coin and is heavier.
 If H2 is heavy, H2 is the odd coin and is heavier.

Kumaraguru College, Coimbatore


o If (H3, H4, X) is heavier side on the scale, either H3 or H4 is heavier or L1 is lighter. Weight H3
against H4
 If both are equal, L1 is the odd coin and is lighter.
 If H3 is heavy, H3 is the odd coin and is heavier.
 If H4 is heavy, H4 is the odd coin and is heavier.

3. The remaining coin X is the odd one. Weigh X against the anyone coin used in initial weighing.
o If both are equal, there is some error.
o If X is heavy, X is the odd coin and is heavier.
o If X is light, X is the odd coin and is lighter.

In a sports contest there were m medals awarded on n successive days (n > 1).

1. On the first day 1 medal and 1/7 of the remaining m - 1 medals were awarded.
2. On the second day 2 medals and 1/7 of the now remaining medals was awarded; and so on.
3. On the nth and last day, the remaining n medals were awarded.

How many days did the contest last, and how many medals were awarded altogether?
Answer

Total 36 medals were awarded and the contest was for 6 days.

On day 1: Medals awarded = (1 + 35/7) = 6 : Remaining 30 medals


On day 2: Medals awarded = (2 + 28/7) = 6 : Remaining 24 medals
On day 3: Medals awarded = (3 + 21/7) = 6 : Remaining 18 medals
On day 4: Medals awarded = (4 + 14/7) = 6 : Remaining 12 medals
On day 5: Medals awarded = (5 +7/7) = 6 : Remaining 6 medals
On day 6: Medals awarded 6

I got this answer by writing small program. If anyone know any other simpler method, do submit it.

A number of 9 digits has the following properties:

• The number comprising the leftmost two digits is divisible by 2, that comprising the leftmost three digits is
divisible by 3, the leftmost four by 4, the leftmost five by 5, and so on for the nine digits of the number i.e. the
number formed from the first n digits is divisible by n, 2<=n<=9.
• Each digit in the number is different i.e. no digits are repeated.
• The digit 0 does not occur in the number i.e. it is comprised only of the digits 1-9 in some order.

Find the number.

Answer

The answer is 381654729

One way to solve it is Trial-&-Error. You can make it bit easier as odd positions will always occupy ODD numbers and
even positions will always occupy EVEN numbers. Further 5th position will contain 5 as 0 does not occur.

The other way to solve this problem is by writing a computer program that systematically tries all possibilities

Answer

Assume that contents of the container is X

Kumaraguru College, Coimbatore


On the first day 1/3rd is evaporated.
(1 - 1/3) of X is remaining i.e. (2/3)X

On the Second day 3/4th is evaporated. Hence,


(1- 3/4) of (2/3)X is remaining
i.e. (1/4)(2/3)X = (1/6) X

Hence 1/6th of the contents of the container is remaining

Vipul was studying for his examinations and the lights went off. It was around 1:00 AM. He lighted two uniform
candles of equal length but one thicker than the other. The thick candle is supposed to last six hours and the thin one
two hours less. When he finally went to sleep, the thick candle was twice as long as the thin one.

For how long did Vipul study in candle light?

Answer

Vipul studied for 3 hours in candle light.

Assume that the initial lenght of both the candle was L and Vipul studied for X hours.

In X hours, total thick candle burnt = XL/6


In X hours, total thin candle burnt = XL/4

After X hours, total thick candle remaining = L - XL/6


After X hours, total thin candle remaining = L - XL/4

Also, it is given that the thick candle was twice as long as the thin one when he finally went to sleep.
(L - XL/6) = 2(L - XL/4)
(6 - X)/6 = (4 - X)/2
(6 - X) = 3*(4 - X)
6 - X = 12 - 3X
2X = 6
X=3

Hence, Vipul studied for 3 hours i.e. 180 minutes in candle light.

If you started a business in which you earned Rs.1 on the first day, Rs.3 on the second day, Rs.5 on the third day, Rs.7
on the fourth day, & so on.

How much would you have earned with this business after 50 years (assuming there are exactly 365 days in every
year)?

Answer

Rs.333,062,500

To begin with, you want to know the total number of days: 365 x 50 = 18250.

By experimentation, the following formula can be discovered, & used to determine the amount earned for any particular
day: 1 + 2(x-1), with x being the number of the day. Take half of the 18250 days, & pair them up with the other half in
the following way: day 1 with day 18250, day 2 with day 18249, & so on, & you will see that if you add these pairs
together, they always equal Rs.36500.

Multiply this number by the total number of pairs (9125), & you have the amount you would have earned in 50 years.

Kumaraguru College, Coimbatore


Math gurus may use series formula to solve it.(series: 1,3,5,7,9,11.....upto 18250 terms)
A worker earns a 5% raise. A year later, the worker receives a 2.5% cut in pay, & now his salary is Rs. 22702.68

What was his salary to begin with?


Answer

Rs.22176

Assume his salary was Rs. X

He earns 5% raise. So his salary is (105*X)/100

A year later he receives 2.5% cut. So his salary is ((105*X)/100)*(97.5/100) which is Rs. 22702.68

Hence, solving equation ((105*X)/100)*(97.5/100) = 22702.68


X = 22176

Answer

66 seconds

It is given that the time between first and last ticks at 6'o is 30 seconds.
Total time gaps between first and last ticks at 6'o = 5
(i.e. between 1 & 2, 2 & 3, 3 & 4, 4 & 5 and 5 & 6)

So time gap between two ticks = 30/5 = 6 seconds.

Now, total time gaps between first and last ticks at 12'o = 11
Therefore time taken for 12 ticks = 11 * 6 = 66 seconds (and not 60 seconds)

500 men are arranged in an array of 10 rows and 50 columns according to their heights.

Tallest among each row of all are asked to come out. And the shortest among them is A.

Similarly after resuming them to their original positions, the shortest among each column are asked to come out. And
the tallest among them is B.

Now who is taller A or B ?


Answer

No one is taller, both are same as A and B are the same person.

As it is mentioned that 500 men are arranged in an array of 10 rows and 50 columns according to their heights. Let's
assume that position numbers represent their heights. Hence, the shortest among the 50, 100, 150, ... 450, 500 is person
with height 50 i.e. A. Similarly the tallest among 1, 2, 3, 4, 5, ..... 48, 48, 50 is person with height 50 i.e. B

Now, both A and B are the person with height 50. Hence both are same.

In Mr. Mehta's family, there are one grandfather, one grandmother, two fathers, two mothers, one father-in-law, one
mother-in-law, four children, three grandchildren, one brother, two sisters, two sons, two daughters and one daughter-
in-law.

How many members are there in Mr. Mehta's family? Give minimal possible answer.
Answer

Kumaraguru College, Coimbatore


There are 7 members in Mr. Mehta's family. Mother & Father of Mr. Mehta, Mr. & Mrs. Mehta, his son and two
daughters.

Mother & Father of Mr. Mehta


|
|
Mr. & Mrs. Mehta
|
|
One Son & Two Daughters
When Alexander the Great attacked the forces of Porus, an Indian soldier was captured by the
Greeks. He had displayed such bravery in battle, however, that the enemy offered to let him choose
how he wanted to be killed. They told him, "If you tell a lie, you will put to the sword, and if you tell
the truth you will be hanged."

The soldier could make only one statement. He made that statement and went free. What did he say?

Answer

The soldier said, "You will put me to the sword."

The soldier has to say a Paradox to save himself. If his statement is true, he will be hanged, which is not the sword and
hence false. If his statement is false, he will be put to the sword, which will make it true. A Paradox !!!

A person wanted to withdraw X rupees and Y paise from the bank. But cashier made a mistake and gave him Y rupees
and X paise. Neither the person nor the cashier noticed that.

After spending 20 paise, the person counts the money. And to his surprise, he has double the amount he wanted to
withdraw.

Find X and Y. (1 Rupee = 100 Paise)

Submit Users BrainVista


Answer Answer (2) Answer

The game of Tic-Tac-Toe is being played between two players. Only the last mark to be placed in the game as shown.

Who will win the game, O or X? Can you tell which was the sixth mark and at which position? Do explain your answer.
At the Party:

1. There were 9 men and children.


2. There were 2 more women than children.
3. The number of different man-woman couples possible was 24. Note that if there were 7 men and 5 women,
then there would have been 35 man-woman couples possible.

Also, of the three groups - men, women and children - at the party:
4. There were 4 of one group.
5. There were 6 of one group.
6. There were 8 of one group.

Exactly one of the above 6 statements is false.

Can you tell which one is false? Also, how many men, women and children are there at the party

Kumaraguru College, Coimbatore


Assume that both the players are intelligent enough.

Answer

O will win the game. The sixth mark was X in square 9.

The 7th mark must be placed in square 5 which is the win situation for both X and O. Hence, the 6th mark must be
placed in a line already containing two of the opponents marks. There are two such possibilities - the 6th mark would
have been either O in square 7 or X in square 9.

As we know both the players are intelligent enough, the 6th mark could not be O in square 7. Instead, he would have
placed O in square 5 and would have won.

Hence, the sixth mark must be X placed in square 9. And the seventh mark will be O. Thus O will win the game.

Answer

Statement (4) is false. There are 3 men, 8 women and 6 children.

Assume that Statements (4), (5) and (6) are all true. Then, Statement (1) is false. But then Statement (2) and (3) both can
not be true. Thus, contradictory to the fact that exactly one statement is false.

So Statement (4) or Statement (5) or Statement (6) is false. Also, Statements (1), (2) and (3) all are true.

From (1) and (2), there are 11 men and women. Then from (3), there are 2 possible cases - either there are 8 men and 3
women or there are 3 men and 8 women.

If there are 8 men and 3 women, then there is 1 child. Then Statements (4) and (5) both are false, which is not possible.

Hence, there are 3 men, 8 women and 6 children. Statement (4) is false.

There is a shortage of tubelights, bulbs and fans in a village - Kharghar. It is found that

• All houses do not have either tubelight or bulb or fan.


• exactly 19% of houses do not have just one of these.
• atleast 67% of houses do not have tubelights.
• atleast 83% of houses do not have bulbs.
• atleast 73% of houses do not have fans.

What percentage of houses do not have tubelight, bulb and fan?


Answer

42% houses do not have tubelight, bulb and fan.

Let's assume that there are 100 houses. Hence, there should be total 300 items i.e. 100 tubelights, 100 bulbs and 100
fans.

From the given data, we know that there is shortage of atleast (67+83+73) 223 items in every 100 houses.

Also, exactly 19 houses do not have just one item. It means that remaining 81 houses should account for the shortage of
remaining (223-19) 204 items. If those remaining 81 houses do not have 2 items each, there would be a shortage of 162
items. But total of 204 items are short. Hence, atleast (204-162) 42 houses do not have all 3 items - tubelight, bulb and
fan.

Kumaraguru College, Coimbatore


Thus, 42% houses do not have tubelight, bulb and fan.
Mr. Subramaniam rents a private car for Andheri-Colaba-Andheri trip. It costs him Rs. 300 everyday.

One day the car driver informed Mr. Subramaniam that there were two students from Bandra who wished to go from
Bandra to Colaba and back to Bandra. Bandra is halfway between Andheri and Colaba. Mr. Subramaniam asked the
driver to let the students travel with him.

On the first day when they came, Mr. Subramaniam said, "If you tell me the mathematically correct price you should
pay individually for your portion of the trip, I will let you travel for free."

How much should the individual student pay for their journey?
Answer

The individual student should pay Rs. 50 for their journey.

Note that 3 persons are travelling between Bandra and Colaba.

The entire trip costs Rs. 300 to Mr. Subramanian. Hence, half of the trip costs Rs. 150.

For Andheri-Bandra-Andheri, only one person i.e. Mr. Subramaniam is travelling. Hence, he would pay Rs. 150.

For Bandra-Colaba-Bandra, three persons i.e Mr. Subramaniam and two students, are travelling. Hence, each student
would pay Rs. 50.

Substitute digits for the letters to make the following Division true
OUT

-------------

STEM|DEMISE

|DMOC

-------------

TUIS

STEM

----------

ZZZE

ZUMM

--------

IST
Note that the leftmost letter can't be zero in any word. Also, there must be a one-to-one mapping between digits and
letters. e.g. if you substitute 3 for the letter M, no other letter can be 3 and all other M in the puzzle must be
Answer

C=0, U=1, S=2, T=3, O=4, M=5, I=6, Z=7, E=8, D=9

It is obvious that U=1 (as U*STEM=STEM) and C=0 (as I-C=I).

S*O is a single digit and also S*T is a single digit. Hence, their values (O, S, T) must be 2, 3 or 4 (as they can not be 0

Kumaraguru College, Coimbatore


or 1 or greater than 4).

Consider, STEM*O=DMOC, where C=0. It means that M must be 5. Now, its simple. O=4, S=2, T=3, E=8, Z=7, I=6
and D=9.
OUT 413

------------- -------------

STEM|DEMISE 2385|985628

|DMOC |9540

------------- -------------

TUIS 3162

STEM 2385

---------- ----------

ZZZE 7778

ZUMM 7155

-------- --------

IST 623
Also, when arranged from 0 to 9, it spells CUSTOMIZED.
At what time after 4.00 p.m. is the minutes hand of a clock exactly aligned with the hour hand?

Answer

4:21:49.5

Assume that X minutes after 4.00 PM minute hand exactly aligns with and hour hand.

For every minute, minute hand travels 6 degrees.


Hence, for X minutes it will travel 6 * X degrees.

For every minute, hour hand travels 1/2 degrees.


Hence, for X minutes it will travel X/2 degrees.

At 4.00 PM, the angle between minute hand and hour hand is 120 degrees. Also, after X minutes, minute hand and hour
hand are exactly aligned. So the angle with respect to 12 i.e. Vertical Plane will be same. Therefore,

6 * X = 120 + X/2
12 * X = 240 + X
11 * X = 240
X = 21.8182
X = 21 minutes 49.5 seconds

Hence, at 4:21:49.5 minute hand is exactly aligned with the hour hand.

A soldier looses his way in a thick jungle. At random he walks from his camp but mathematically in an interesting
fashion.

First he walks one mile East then half mile to North. Then 1/4 mile to West, then 1/8 mile to South and so on making a
loop.

Kumaraguru College, Coimbatore


Finally how far he is from his camp and in which direction?

Answer

The soldier is 0.8944 miles away from his camp towards East-North.

It is obvious that he is in East-North direction.

Distance travelled in North and South directions


= 1/2 - 1/8 + 1/32 - 1/128 + 1/512 - 1/2048 + and so on... (a geometric series with r = (-1/4) )

(1/2) * ( 1 - (-1/4)n )
= ---------------------------
( 1 - (-1/4) )

= 1 / ( 2 * ( 1 - (-1/4) ) )
= 2/5

Similarly in East and West directions


= 1 - 1/4 + 1/16 - 1/64 + 1/256 - and so on... (a geometric series with r = (-1/4) )

(1) * ( 1 - (-1/4)n )
= ---------------------------
( 1 - (-1/4) )

= 1 / ( ( 1- (-1/4) )
= 4/5

So the soldier is 4/5 miles away towards East and 2/5 miles away towards North. So using right angled triangle, soldier
is 0.8944 miles away from his camp.

Raj has a jewel chest containing Rings, Pins and Ear-rings. The chest contains 26 pieces. Raj has 2 1/2 times as many
rings as pins, and the number of pairs of earrings is 4 less than the number of rings.

How many earrings does Raj have?

Answer

12 earrings

Assume that there are R rings, P pins and E pair of ear-rings.

It is given that, he has 2 1/2 times as many rings as pins.


R = (5/2) * P or P = (2*R)/5

And, the number of pairs of earrings is 4 less than the number of rings.
E = R - 4 or R = E + 4

Also, there are total 26 pieces.


R + P + 2*E = 26
R + (2*R)/5 + 2*E = 26
5*R + 2*R + 10*E = 130
7*R + 10*E = 130
7*(E + 4) + 10*E = 130
7*E + 28 + 10*E = 130
17*E = 102
E=6

Kumaraguru College, Coimbatore


Hence, there are 6 pairs of Ear-rings i.e. total 12 Ear-rings
How many ways are there of arranging the sixteen black or white pieces of a standard international chess set on the first
two rows of the board?

Given that each pawn is identical and each rook, knight and bishop is identical to its pair.
Submitted

Answer

6,48,64,800 ways

There are total 16 pieces which can be arranged on 16 places in 16P16 = 16! ways.
(16! = 16 * 15 * 14 * 13 * 12 * ..... * 3 * 2 * 1)

But, there are some duplicate combinations because of identical pieces.

• There are 8 identical pawn, which can be arranged in 8P8 = 8! ways.


• Similarly there are 2 identical rooks, 2 identical knights and 2 identical bishops. Each can be arranged in 2P2 =
2! ways.

Hence, the require answer is


= (16!) / (8! * 2! * 2! * 2!)
= 6,48,64,800

A person with some money spends 1/3 for cloths, 1/5 of the remaining for food and 1/4 of the remaining for travel. He
is left with Rs 100/-

How much did he have with him in the begining?

Answer

Rs. 250/-

Assume that initially he had Rs. X


He spent 1/3 for cloths =. (1/3) * X
Remaining money = (2/3) * X

He spent 1/5 of remaining money for food = (1/5) * (2/3) * X = (2/15) * X


Remaining money = (2/3) * X - (2/15) * X = (8/15) * X

Again, he spent 1/4 of remaining maoney for travel = (1/4) * (8/15) * X = (2/15) * X
Remaining money = (8/15) * X - (2/15) * X = (6/15) * X

But after spending for travel he is left with Rs. 100/- So


(6/15) * X = 100
X = 250

Grass in lawn grows equally thick and in a uniform rate. It takes 24 days for 70 cows and 60 days for 30 cows to eat the
whole of the grass.

How many cows are needed to eat the grass in 96 days?


Answer

20 cows

g - grass at the beginning

Kumaraguru College, Coimbatore


r - rate at which grass grows, per day
y - rate at which one cow eats grass, per day
n - no of cows to eat the grass in 96 days

From given data,


g + 24*r = 70 * 24 * y ---------- A
g + 60*r = 30 * 60 * y ---------- B
g + 96*r = n * 96 * y ---------- C

Solving for (B-A),


(60 * r) - (24 * r) = (30 * 60 * y) - (70 * 24 * y)
36 * r = 120 * y ---------- D

Solving for (C-B),


(96 * r) - (60 * r) = (n * 96 * y) - (30 * 60 * y)
36 * r = (n * 96 - 30 * 60) * y
120 * y = (n * 96 - 30 * 60) * y [From D]
120 = (n * 96 - 1800)
n = 20

Hence, 20 cows are needed to eat the grass in 96 days.


There is a safe with a 5 digit number as the key. The 4th digit is 4 greater than the second digit, while the 3rd digit is 3
less than the 2nd digit. The 1st digit is thrice the last digit. There are 3 pairs whose sum is 11.

Find the number.


Answer

65292

As per given conditions, there are three possible combinations for 2nd, 3rd and 4th digits. They are (3, 0, 7) or (4, 1, 8)
or (5, 2, 9)

It is given that there are 3 pairs whose sum is 11. All possible pairs are (2, 9), (3, 8), (4, 7), (5, 6). Now required number
is 5 digit number and it contains 3 pairs of 11. So it must not be having 0 and 1 in it. Hence, the only possible
combination for 2nd, 3rd and 4th digits is (5, 2, 9)

Also, 1st digit is thrice the last digit. The possible combinations are (3, 1), (6, 2) and (9, 3), out of which only (6, 2) with
(5, 2, 9) gives 3 pairs of 11. Hence, the answer is 65292.
Four friends - Arjan, Bhuvan, Guran and Lakha were comparing the number of sheep that they owned.

It was found that Guran had ten more sheep than Lakha.

If Arjan gave one-third to Bhuvan, and Bhuvan gave a quarter of what he then held to Guran, who then passed on a fifth
of his holding to Lakha, they would all have an equal number of sheep.

How many sheep did each of them possess? Give the minimal possible answer

Answer

Arjan, Bhuvan, Guran and Lakha had 90, 50, 55 and 45 sheep respectively.

Assume that Arjan, Bhuvan, Guran and Lakha had A, B, G and L sheep respectively. As it is given that at the end each
would have an equal number of sheep, comparing the final numbers from the above table.

Arjan's sheep = Bhuvan's sheep


2A/3 = A/4 + 3B/4

Kumaraguru College, Coimbatore


8A = 3A + 9B
5A = 9B

Arjan's sheep = Guran's sheep


2A/3 = A/15 + B/5 + 4G/5
2A/3 = A/15 + A/9 + 4G/5 (as B=5A/9)
30A = 3A + 5A + 36G
22A = 36G
11A = 18G

Arjan's sheep = Lakha's sheep


2A/3 = A/60 + B/20 + G/5 + L
2A/3 = A/60 + A/36 + 11A/90 + L (as B=5A/9 and G=11A/18)
2A/3 = A/6 + L
A/2 = L
A = 2L

Also, it is given that Guran had ten more sheep than Lakha.
G = L + 10
11A/18 = A/2 + 10
A/9 = 10
A = 90 sheep

Thus, Arjan had 90 sheep, Bhuvan had 5A/9 i.e. 50 sheep, Guran had 11A/18 i.e. 55 sheep and Lakha had A/2 i.e. 45
sheep.
Consider a number 235, where last digit is the sum of first two digits i.e. 2 + 3 = 5.

How many such 3-digit numbers are there?

Answer

There are 45 different 3-digit numbers.

The last digit can not be 0.

If the last digit is 1, the only possible number is 101. (Note that 011 is not a 3-digit number)

If the last digit is 2, the possible numbers are 202 and 112.

If the last digit is 3, the possible numbers are 303, 213 and 123.

If the last digit is 4, the possible numbers are 404, 314, 224 and 134.

If the last digit is 5, the possible numbers are 505, 415, 325, 235 and 145.

Note the pattern here - If the last digit is 1, there is only one number. If the last digit is 2, there are two numbers. If the
last digit is 3, there are three numbers. If the last digit is 4, there are four numbers. If the last digit is 5, there are five
numbers. And so on.....

Thus, total numbers are


1 + 2 + 3 + 4 + 5 + 6 + 7 + 8 + 9 = 45

Altogether then, there are 45 different 3-digit numbers, where last digit is the sum of first two digits.

Find the smallest number such that if its rightmost digit is placed at its left end, the new number so formed is precisely
50% larger than the original number.
Answer

Kumaraguru College, Coimbatore


The answer is 285714.

If its rightmost digit is placed at its left end, then new number is 428571 which is 50% larger than the original number
285714.

The simplest way is to write a small program. And the other way is trial and error !!!
Two identical pack of cards A and B are shuffled throughly. One card is picked from A and shuffled with B. The top
card from pack A is turned up. If this is the Queen of Hearts, what are the chances that the top card in B will be the King
of Hearts?

Answer

52 / 2703

There are two cases to be considered.

CASE 1 : King of Hearts is drawn from Pack A and shuffled with Pack B

Probability of drawing King of Hearts from Pack A = 1/51 (as Queen of Hearts is not to be drawn)
Probability of having King of Hearts on the top of the Pack B = 2/53

So total probability of case 1 = (1/51) * (2/53) = 2 / (51 * 53)

CASE 2 : King of Hearts is not drawn from Pack A

Probability of not drawing King of Hearts from Pack A = 50/51 (as Queen of Hearts is not to be drawn)
Probability of having King of Hearts on the top of the Pack B = 1/53

So total probability of case 2 = (50/51) * (1/53) = 50 / (51 * 53)

Now adding both the probability, the required probability is


= 2 / (51 * 53) + 50 / (51 * 53)
= 52 / (51 * 53)
= 52 / 2703
= 0.0192378
There are 3 ants at 3 corners of a triangle, they randomly start moving towards another corner.

What is the probability that they don't collide?

Answer

Let's mark the corners of the triangle as A,B,C. There are total 8 ways in which ants can move.

1. A->B, B->C, C->A


2. A->B, B->C, C->B
3. A->B, B->A, C->A
4. A->B, B->A, C->B
5. A->C, C->B, B->A
6. A->C, C->B, B->C
7. A->C, C->A, B->A
8. A->C, C->A, B->C

Out of which, there are only two cases under which the ants won't collide :

• A->B, B->C, C->A


• A->C, C->B, B->A

Kumaraguru College, Coimbatore


There is a 4-character code, with 2 of them being letters and the other 2 being numbers.

How many maximum attempts would be necessary to find the correct code? Note that the code is case-sensitive.
Answer

The maximum number of attempts required are 16,22,400

There are 52 possible letters - a to z and A to Z, and 10 possible numbers - 0 to 9. Now, 4 characters - 2 letters and 2
numbers, can be selected in 52*52*10*10 ways. These 4 characters can be arranged in 4C2 i.e. 6 different ways - the
number of unique patterns that can be formed by lining up 4 objects of which 2 are distinguished one way (i.e. they
must be letters) and the other 2 are distinguished another way (i.e. they must be numbers).

Consider an example : Let's assume that @ represents letter and # represents number. the 6 possible ways of arranging
them are : @@##, @#@#, @##@, #@@#, #@#@, ##@@

Hence, the required answer is


= 52*52*10*10*6
= 16,22,400 attempts
= 1.6 million approx.

Thanks to Tim Sanders for opening BrainVista's brain !!!


How many possible combinations are there in a 3x3x3 rubics cube?

In other words, if you wanted to solve the rubics cube by trying different combinations, how many might it take you
(worst case senerio)?

How many for a 4x4x4 cube?


Submitted

Answer

There are 4.3252 * 10^19 possible combinations for 3x3x3 Rubics and 7.4012 * 10^45 possible combinations for 4x4x4
Rubics.

Let's consider 3x3x3 Rubics first.

There are 8 corner cubes, which can be arranged in 8! ways.


Each of these 8 cubes can be turned in 3 different directions, so there are 3^8 orientations altogether. But if you get all
but one of the corner cube into chosen positions and orientations, only one of 3 orientations of the final corner cube is
possible. Thus, total ways corner cubes can be placed = (8!) * (3^8)/8 = (8!) * (3^7)

Similarly, 12 edge cubes can be arranged in 12! ways.


Each of these 12 cubes can be turned in 2 different directions, so there are 2^12 orientations altogether. But if you get
all but one of the edge cube into chosen positions and orientations, only one of 2 orientations of the final edge cube is
possible. Thus, total ways edge cubes can be placed = (12!) * (2^12)/2 = (12!) * (2^11)

Here, we have essentially pulled the cubes apart and stuck cubes back in place wherever we please. In reality, we can
only move cubes around by turning the faces of the cubes. It turns out that you can't turn the faces in such a way as to
switch the positions of two cubes while returning all the others to their original positions. Thus if you get all but two
cubes in place, there is only one attainable choice for them (not 2!). Hence, we must divide by 2.

Total different possible combinations are


= [(8!) * (3^7)] * [(12!) * (2^11)] / 2
= (8!) * (3^7) * (12!) * (2^10)

Kumaraguru College, Coimbatore


= 4.3252 * 10^19

Similarly, for 4x4x4 Rubics total different possible combinations are


= [(8!) * (3^7)] * [(24!)] * [(24!) / (4!^6)] / 24
= 7.4011968 * 10^45

Note that there are 24 edge cubes, which you can not turn in 2 orientations (hence no 2^24 / 2). Also, there are 4 center
cubes per face i.e. (24!) / (4!^6). You can switch 2 cubes without affecting the rest of the combination as 4*4*4 has
even dimensions (hence no division by 2). But pattern on one side is rotated in 4 directions over 6 faces, hence divide
by 24.
Substitute digits for the letters to make the following relation true.
N E V E R

L E A V E

+ M E

-----------------

A L O N E
Note that the leftmost letter can't be zero in any word. Also, there must be a one-to-one mapping between digits and
letters. e.g. if you substitute 3 for the letter M, no other letter can be 3 and all other M in the puzzle must be 3.

Answer

A tough one!!!

Since R + E + E = 10 + E, it is clear that R + E = 10 and neither R nor E is equal to 0 or 5. This is the only entry point to

solve it. Now use trial-n-error method.

N E V E R 2 1 4 1 9

L E A V E 3 1 5 4 1

+ M E + 6 1

----------------- -----------------

A L O N E 5 3 0 2 1

One of the four people - Mr. Clinton, his wife Monika, their son Mandy and their daughter Cindy - is a singer and
another is a dancer. Mr. Clinton is older than his wife and Mady is older than his sister.

1. If the singer and the dancer are the same sex, then the dancer is older than the singer.
2. If neither the singer nor the dancer is the parent of the other, then the singer is older than the dancer.
3. If the singer is a man, then the singer and the dancer are the same age.
4. If the singer and the dancer are of opposite sex then the man is older than the woman.
5. If the dancer is a woman, then the dancer is older than the singer.

Whose occupation do you know? And what is his/her occupation?


Answer

Cindy is the Singer. Mr. Clinton or Monika is the Dancer.

From (1) and (3), the singer and the dancer, both can not be a man. From (3) and (4), if the singer is a man, then the

Kumaraguru College, Coimbatore


dancer must be a man. Hence, the singer must be a woman.

CASE I : Singer is a woman and Dancer is also a woman


Then, the dancer is Monika and the singer is Cindy.

CASE II : Singer is a woman and Dancer is also a man


Then, the dancer is Mr. Clinton and the singer is Cindy.

In both the cases, we know that Cindy is the Singer. And either Mr. Clinton or Monika is the Dancer.
There are 20 people in your applicant pool, including 5 pairs of identical twins.

If you hire 5 people randomly, what are the chances you will hire at least 1 pair of identical twins? (Needless to say, this
could cause trouble ;))
Submitted

Answer

The probability to hire 5 people with at least 1 pair of identical twins is 25.28%

5 people from the 20 people can be hired in 20C5 = 15504 ways.

Now, divide 20 people into two groups of 10 people each :


G1 - with all twins
G2 - with all people other than twins

Let's find out all possible ways to hire 5 people without a single pair of indentical twins.
People from People from No of ways to hire G1 without a single pair of No of ways to hire Total
G1 G2 indentical twins G2 ways
0 5 10C0 10C5 252
1 4 10C1 10C4 2100
2 3 10C2 * 8/9 10C3 4800
3 2 10C3 * 8/9 * 6/8 10C2 3600
4 1 10C4 * 8/9 * 6/8 * 4/7 10C1 800
5 0 10C5 * 8/9 * 6/8 * 4/7 * 2/6 10C0 32
Total 11584

Thus, total possible ways to hire 5 people without a single pair of indentical twins = 11584 ways

So, total possible ways to hire 5 people with at least a single pair of indentical twins = 15504 - 11584 = 3920 ways

Hence, the probability to hire 5 people with at least a single pair of indentical twins
= 3920/15504
= 245/969
= 0.2528
= 25.28%
In a hotel, rooms are numbered from 101 to 550. A room is chosen at random. What is the probability that room number
starts with 1, 2 or 3 and ends with 4, 5 or 6?

Answer

There are total 450 rooms.

Out of which 299 room number starts with either 1, 2 or 3. (as room number 100 is not there) Now out of those 299
rooms only 90 room numbers end with 4, 5 or 6

Kumaraguru College, Coimbatore


So the probability is 90/450 i.e. 1/5 or 0.20
Draw 9 dots on a page, in the shape of three rows of three dots to form a square. Now place your pen on the page, draw
4 straight lines and try and cover all the dots.

You're not allowed to lift your pen.

Note: Don't be confined by the dimensions of the square.


Submitted

There are 3 persons X, Y and Z. On some day, X lent tractors to Y and Z as many as they had. After a month Y gave as
many tractors to X and Z as many as they have. After a month Z did the same thing. At the end of this transaction each
one of them had 24.

Find the tractors each originally had?

Answer

One way to solve it is by making 3 equations and solve them simultaneously. But there is rather easier way to solve it
using Backtracing.

It's given that at the end, each had 24 tractors (24, 24, 24) i.e. after Z gave tractors to X & Y as many as they had. It
means that after getting tractors from Z their tractors got doubled. So before Z gave them tractors, they had 12 tractors
each and Z had 48 tractors. (12, 12, 48)

Similarly, before Y gave tractors to X & Z, they had 6 & 24 tractors respectively and Y had 42 tractors i.e. (6, 42, 24)

Again, before X gave tractors to Y & Z, they had 21 & 12 tractors respectively and X had 39 tractors i.e. (39, 21, 12)

Hence, initially X had 39 tractors, Y had 21 tractors and Z had 12 tractors.

There is a 50m long army platoon marching ahead. The last person in the platoon wants to give a letter to the first
person leading the platoon. So while the platoon is marching he runs ahead, reaches the first person and hands over the
letter to him and without stopping he runs and comes back to his original position.

In the mean time the whole platoon has moved ahead by 50m.

The question is how much distance did the last person cover in that time. Assuming that he ran the whole distance with
uniform speed.
Submitted

Answer

The last person covered 120.71 meters.

It is given that the platoon and the last person moved with uniform speed. Also, they both moved for the identical
amount of time. Hence, the ratio of the distance they covered - while person moving forward and backword - are equal.

Let's assume that when the last person reached the first person, the platoon moved X meters forward.

Thus, while moving forward the last person moved (50+X) meters whereas the platoon moved X meters.

Similarly, while moving back the last person moved [50-(50-X)] X meters whereas the platoon moved (50-X) meters.

Kumaraguru College, Coimbatore


Now, as the ratios are equal,
(50+X)/X = X/(50-X)
(50+X)*(50-X) = X*X

Solving, X=35.355 meters

Thus, total distance covered by the last person


= (50+X) + X
= 2*X + 50
= 2*(35.355) + 50
= 120.71 meters

Note that at first glance, one might think that the total distance covered by the last person is 100 meters, as he ran the
total lenght of the platoon (50 meters) twice. TRUE, but that's the relative distance covered by the last person i.e.
assuming that the platoon is stationary.
Assume that you have enough coins of 1, 5, 10, 25 and 50 cents.

How many ways are there to make change for a dollar? Do explain your answer.

There are 292 ways to make change for a dollar using coins of 1, 5, 10, 25 and 50 cents.

Let's generalised the teaser and make a table as shown above.

If you wish to make change for 75 cents using only 1, 5, 10 and 25 cent coins, go to the .25 row and the 75 column to
obtain 121 ways to do this.

The table can be created from left-to-right and top-to-bottom. Start with the top left i.e. 1 cent row. There is exactly one
way to make change for every amount. Then calculate the 5 cents row by adding the number of ways to make change
for the amount using 1 cent coins plus the number of ways to make change for 5 cents less using 1 and 5 cent coins.

Let's take an example:


To get change for 50 cents using 1, 5 and 10 cent coins.
* 50 cents change using 1 and 5 cent coins = 11 ways
* (50-10) 40 cents change using 1, 5 and 10 cent coins = 25 ways
* 50 cents change using 1, 5 and 10 cent coins = 11+25 = 36 ways

Let's take another example:


To get change for 75 cents using all coins up to 50 cent i.e. 1, 5, 10, 25 and 50 cents coins.
* 75 cents change using coins upto 25 cent = 121 ways
* (75-50) 25 cents change using coins upto 50 cent = 13 ways
* 75 cents change using coins upto 50 cent = 121+13 = 134 ways

For people who don't want to tease their brain and love to do computer programming, there is a simple way. Write a
small multi-loop program to solve the equation: A + 5B + 10C + 25D + 50E = 100
where,
A = 0 to 100
B = 0 to 20
C = 0 to 10
D = 0 to 4
E = 0 to 2

The program should output all the possible values of A, B, C, D and E for which the equation is satisfied.

In a Road Race, one of the three bikers was doing 15km less than the first and 3km more than the third. He also finished
the race 12 minutes after the first and 3 minutes before the third.

Kumaraguru College, Coimbatore


Can you find out the speed of each biker, the time taken by each biker to finish the race and the length of the course?

Assume that there were no stops in the race and also they were driving with constant speeds through out the

Answer

Let us assume that


Speed of First biker = V1 km/min
Speed of Second biker = V2 km/min
Speed of Third biker = V3 km/min
Total time take by first biker = T1 min
Total distance = S km

Now as per the data given in the teaser, at a time T min


X1 = V1 * T ----> 1

X1 - 15 = V2 * T ----> 2

X1 - 18 = V3 * T ----> 3

At a Distance S Km.
S = V1 * T1 ----> 4

S = V2 * (T1 + 12) ----> 5

S = V3 * (T1 + 15) ----> 6

Thus there are 6 equations and 7 unknown data that means it has infinite number of solutions.

By solving above 6 equations we get,


Time taken by first biker, T1 = 60 Min.
Time taken by Second biker, T2 = 72 Min.
Time taken by first biker, T3 = 75 Min.

Also, we get
Speed of first biker, V1 = 90/T km/min
Speed of second biker, V2 = (5/6)V1 = 75/T km/min
Speed of third biker, V3 = (4/5)V1 = 72/T km/min

Also, the length of the course, S = 5400/T km

Thus, for the data given, only the time taken by each biker can be found i.e. 60, 72 and 75 minutes. For other quantities,
one more independent datum is required i.e. either T or V1 or V2 or V3

Thanks to Theertham Srinivas for the answer !!!


What is the four-digit number in which the first digit is 1/3 of the second, the third is the sum of the first and second,
and the last is three times the second?

Answer

The 4 digit number is 1349.

It is given that the first digit is 1/3 of the second. There are 3 such possibilities.

1. 1 and 3
2. 2 and 6
3. 3 and 9

Kumaraguru College, Coimbatore


Now, the third digit is the sum of the first and second digits.

1. 1+3=4
2. 2+6=8
3. 3 + 9 = 12

It is clear that option 3 is not possible. So we are left with only two options. Also, the last digit is three times the second,
which rules out the second option. Hence, the answer is 1349.

Difference between Bholu's and Molu's age is 2 years and the difference between Molu's and Kolu's age is 5 years.

What is the maximum possible value of the sum of the difference in their ages, taken two at a time?
Answer

The maximum possible value of the sum of the difference in their ages - taken two at a time - is 14 years.

It is given that -
"Difference between Bholu's and Molu's age is 2 years"
"Difference between Molu's and Kolu's age is 5 years"

Now, to get the maximum possible value, the difference between Bholu's and Kolu's age should be maximum i.e.
Molu's age should be in between Bholu's and Kolu's age. Then, the difference between Bholu's and Kolu's age is 7
years.

Hence, the maximum possible value of the sum of the difference in their ages - taken two at a time - is (2 + 5 + 7) 14
years.
If it is given that:
25 - 2 = 3
100 x 2 = 20
36 / 3 = 2

What is 144 - 3 = ?
Submitted
Answer

There are 3 possible answers to it.

Answer 1 : 9
Simply replace the first number by its square root.
(25) 5 - 2 = 3
(100) 10 x 2 = 20
(36) 6 / 3 = 2
(144) 12 - 3 = 9

Answer 2 : 11
Drop the digit in the tens position from the first number.
(2) 5 - 2 = 3
1 (0) 0 x 2 = 20
(3) 6 / 3 = 2
1 (4) 4 - 3 = 11

You will get the same answer on removing left and right digit alternatively from the first number i.e remove left digit
from first (2), right digit from second (0), left digit from third (3) and right digit from forth (4).
(2) 5 - 2 = 3
10 (0) x 2 = 20
(3) 6 / 3 = 2
14 (4) - 3 = 11

Kumaraguru College, Coimbatore


Answer 3 : 14
Drop left and right digit alternatively from the actual answer.
25 - 2 = (2) 3 (drop left digit i.e. 2)
100 * 2 = 20 (0) (drop right digit i.e. 0)
36 / 3 = (1) 2 (drop left digit i.e. 1)
144 - 3 = 14 (1) (drop right digit i.e. 1)

A 3 digit number is such that it's unit digit is equal to the product of the other two digits which are prime. Also, the
difference between it's reverse and itself is 396.

What is the sum of the three digits?

Answer

The required number is 236 and the sum is 11.

It is given that the first two digits of the required number are prime numbers i.e. 2, 3, 5 or 7. Note that 1 is neither prime
nor composite. Also, the third digit is the multiplication of the first two digits. Thus, first two digits must be either 2 or
3 i.e. 22, 23, 32 or 33 which means that there are four possible numbers - 224, 236, 326 and 339.

Now, it is also given that - the difference between it's reverse and itself is 396. Only 236 satisfies this condition. Hence,
the sum of the three digits is 11.

There are 4 mugs placed upturned on the table. Each mug have the same number of marbles and a statement about the
number of marbles in it. The statements are: Two or Three, One or Four, Three or One, One or Two.

Only one of the statement is correct. How many marbles are there under each mug?

Answer

A simple one.

As it is given that only one of the four statement is correct, the correct number can not appear in more than one
statement. If it appears in more than one statement, then more than one statement will be correct.

Hence, there are 4 marbles under each mug.


At University of Probability, there are 375 freshmen, 293 sophomores, 187 juniors, & 126 seniors. One student will
randomly be chosen to receive an award.

What percent chance is there that it will be a junior? Round to the nearest whole percent
Answer

19%

This puzzle is easy. Divide the number of juniors (187) by the total number of students (981), & then multiply the
number by 100 to convert to a percentage.

Hence the answer is (187/981)*100 = 19%

If you were to dial any 7 digits on a telephone in random order, what is the probability that you will dial your own
phone number?

Assume that your telephone number is 7-digits.


Answer

1 in 10,000,000

Kumaraguru College, Coimbatore


There are 10 digits i.e. 0-9. First digit can be dialed in 10 ways. Second digit can be dialed in 10 ways. Third digit can
be dialed in 10 ways. And so on.....

Thus, 7-digit can be dialed in 10*10*10*10*10*10*10 (=10,000,000) ways. And, you have just one telephone number.
Hence, the possibility that you will dial your own number is 1 in 10,000,000.

Note that 0123456 may not be a valid 7-digit telephone number. But while dialing in random order, that is one of the
possible 7-digit number which you may dial.
An anthropologist discovers an isolated tribe whose written alphabet contains only six letters (call the letters A, B, C, D,
E and F). The tribe has a taboo against using the same letter twice in the same word. It's never done.

If each different sequence of letters constitues a different word in the language, what is the maximum number of six-
letter words that the language can employ?
Submitted

Answer

The language can employ maximum of 720 six-letter words.

It is a simple permutation problem of arranging 6 letters to get different six-letter words. And it can be done in in 6!
ways i.e. 720 ways.

In otherwords, the first letter can be any of the given 6 letters (A through F). Then, whatever the first letter is, the
second letter will always be from the remaining 5 letters (as same letter can not be used twice), and the third letter
always be from the remaining 4 letters, and so on. Thus, the different possible six-letter words are 6*5*4*3*2*1 = 720

Kate, Demi, Madona, Sharon, Britney and Nicole decided to lunch together in a restaurant. The waiter led them to a
round table with six chairs.

How many different ways can they seat?

Answer

There are 120 different possible seating arrangments.

Note that on a round table ABCDEF and BCDEFA is the same.

The first person can sit on any one of the seats. Now, for the second person there are 5 options, for the third person there
are 4 options, for the forth person there are 3 options, for the fifth person there are 2 options and for the last person there
is just one option.

Thus, total different possible seating arrangements are


=5*4*3*2*1
= 120

Silu and Meenu were walking on the road.

Silu said, "I weigh 51 Kgs. How much do you weigh?"

Meenu replied that she wouldn't reveal her weight directly as she is overweight. But she said, "I weigh 29 Kgs plus half
of my weight."

How much does Meenu weigh?

Answer

Meenu weighs 58 Kgs.

Kumaraguru College, Coimbatore


It is given that Meenu weighs 29 Kgs plus half of her own weight. It means that 29 Kgs is the other half. So she weighs
58 Kgs.

Solving mathematically, let's assume that her weight is X Kgs.


X = 29 + X/2
2*X = 58 + X
X = 58 Kgs

Consider the sum: ABC + DEF + GHI = JJJ

If different letters represent different digits, and there are no leading zeros, what does J represent?

Answer

The value of J must be 9.

Since there are no leading zeros, J must be 7, 8, or 9. (JJJ = ABC + DEF + GHI = 14? + 25? + 36? = 7??)

Now, the remainder left after dividing any number by 9 is the same as the remainder left after dividing the sum of the
digits of that number by 9. Also, note that 0 + 1 + ... + 9 has a remainder of 0 after dividing by 9 and JJJ has a remainder
of 0, 3, or 6.

The number 9 is the only number from 7, 8 and 9 that leaves a remainder of 0, 3, or 6 if you remove it from the sum 0 +
1 + ... + 9. Hence, it follows that J must be 9.
A man has Ten Horses and nine stables as shown here.
[] [] [] [] [] [] [] [] []
The man wants to fit Ten Horses into nine stables. How can he fit Ten horses into nine stables?
Submitted
Answer

The answer is simple. It says the man wants to fit "Ten Horses" into nine stables. There are nine letters in the phrase
"Ten Horses". So you can put one letter each in all nine stables.
[T] [E] [N] [H] [O] [R] [S] [E] [S]

A man is at a river with a 9 gallon bucket and a 4 gallon bucket. He needs exactly 6 gallons of water.

How can he use both buckets to get exactly 6 gallons of water?

Note that he cannot estimate by dumping some of the water out of the 9 gallon bucket or the 4 gallon bucket
Answer

For the sack of explanation, let's identify 4 gallon bucket as Bucket P and 9 gallon bucket as Bucket Q.
4 gallon bucket 9 gallon bucket
Operation (Bucket P) (Bucket Q)

Initially 0 0

Fill the bucket Q with 9 gallon water 0 9

Pour 4 gallon water from bucket Q to bucket P 4 5

Empty bucket P 0 5

Pour 4 gallon water from bucket Q to bucket P 4 1

Empty bucket P 0 1

Kumaraguru College, Coimbatore


Pour 1 gallon water from bucket Q to bucket P 1 0

Fill the bucket Q with 9 gallon water 1 9

Pour 3 gallon water from bucket Q to bucket P 4 6

9 gallon bucket contains 6 gallon of water, as required.

Each of the five characters in the word BRAIN has a different value between 0 and 9. Using the given grid, can you find
out the value of each character?
B R A I N 31

B B R B A 31

N I A B B 32

N I B A I 30

I R A A A 23

37 29 25 27 29
The numbers on the extreme right represent the sum of the values represented by the characters in that row. Also, the
numbers on the last raw represent the sum of the values represented by the characters in that column. e.g. B + R + A + I
+ N = 31 (from first row)

There are 9 coins. Out of which one is odd one i.e weight is less or more. How many iterations of weighing are required
to find odd coin?
Answer

It is always possible to find odd coin in 3 weighings and to tell whether the odd coin is heavier or lighter.

1. Take 8 coins and weigh 4 against 4.


o If both are not equal, goto step 2
o If both are equal, goto step 3

2. One of these 8 coins is the odd one. Name the coins on heavier side of the scale as H1, H2, H3 and H4.
Similarly, name the coins on the lighter side of the scale as L1, L2, L3 and L4. Either one of H's is heavier or
one of L's is lighter. Weigh (H1, H2, L1) against (H3, H4, X) where X is one coin remaining in intial
weighing.

o If both are equal, one of L2, L3, L4 is lighter. Weigh L2 against L3.
 If both are equal, L4 is the odd coin and is lighter.
 If L2 is light, L2 is the odd coin and is lighter.
 If L3 is light, L3 is the odd coin and is lighter.

o If (H1, H2, L1) is heavier side on the scale, either H1 or H2 is heavier. Weight H1 against H2
 If both are equal, there is some error.
 If H1 is heavy, H1 is the odd coin and is heavier.
 If H2 is heavy, H2 is the odd coin and is heavier.

Kumaraguru College, Coimbatore


o If (H3, H4, X) is heavier side on the scale, either H3 or H4 is heavier or L1 is lighter. Weight H3
against H4
 If both are equal, L1 is the odd coin and is lighter.
 If H3 is heavy, H3 is the odd coin and is heavier.
 If H4 is heavy, H4 is the odd coin and is heavier.

3. The remaining coin X is the odd one. Weigh X against the anyone coin used in initial weighing.
o If both are equal, there is some error.
o If X is heavy, X is the odd coin and is heavier.
o If X is light, X is the odd coin and is lighter.

In a sports contest there were m medals awarded on n successive days (n > 1).

1. On the first day 1 medal and 1/7 of the remaining m - 1 medals were awarded.
2. On the second day 2 medals and 1/7 of the now remaining medals was awarded; and so on.
3. On the nth and last day, the remaining n medals were awarded.

How many days did the contest last, and how many medals were awarded altogether?
Answer

Total 36 medals were awarded and the contest was for 6 days.

On day 1: Medals awarded = (1 + 35/7) = 6 : Remaining 30 medals


On day 2: Medals awarded = (2 + 28/7) = 6 : Remaining 24 medals
On day 3: Medals awarded = (3 + 21/7) = 6 : Remaining 18 medals
On day 4: Medals awarded = (4 + 14/7) = 6 : Remaining 12 medals
On day 5: Medals awarded = (5 +7/7) = 6 : Remaining 6 medals
On day 6: Medals awarded 6

I got this answer by writing small program. If anyone know any other simpler method, do submit it.

A number of 9 digits has the following properties:

• The number comprising the leftmost two digits is divisible by 2, that comprising the leftmost three digits is
divisible by 3, the leftmost four by 4, the leftmost five by 5, and so on for the nine digits of the number i.e. the
number formed from the first n digits is divisible by n, 2<=n<=9.
• Each digit in the number is different i.e. no digits are repeated.
• The digit 0 does not occur in the number i.e. it is comprised only of the digits 1-9 in some order.

Find the number.

Answer

The answer is 381654729

One way to solve it is Trial-&-Error. You can make it bit easier as odd positions will always occupy ODD numbers and
even positions will always occupy EVEN numbers. Further 5th position will contain 5 as 0 does not occur.

The other way to solve this problem is by writing a computer program that systematically tries all possibilities.

1/3 rd of the contents of a container evaporated on the 1st day. 3/4th of the remaining contents of the container
evaporated on the second day.

What part of the contents of the container is left at the end of the second day?

Kumaraguru College, Coimbatore


Answer

Assume that contents of the container is X

On the first day 1/3rd is evaporated.


(1 - 1/3) of X is remaining i.e. (2/3)X

On the Second day 3/4th is evaporated. Hence,


(1- 3/4) of (2/3)X is remaining
i.e. (1/4)(2/3)X = (1/6) X

Hence 1/6th of the contents of the container is remaining

There are four people in a room (not including you). Exactly two of these four always tell the truth. The other two
always lie.

You have to figure out who is who IN ONLY 2 QUESTIONS. Your questions have to be YES or NO questions and can
only be answered by one person. (If you ask the same question to two different people then that counts as two
questions). Keep in mind that all four know each other's characteristics whether they lie or not.

What questions would you ask to figure out who is who? Remember that you can ask only 2 questions.
Submitted
You have 3 baskets, & each one contains exactly 4 balls, each of which is of the same size. Each ball is either red,
black, white, or purple, & there is one of each color in each basket.

If you were blindfolded, & lightly shook each basket so that the balls would be randomly distributed, & then took 1 ball
from each basket, what chance is there that you would have exactly 2 red balls?

Answer

There are 64 different possible outcomes, & in 9 of these, exactly 2 of the balls will be red. There is thus a slightly
better than 14% chance [(9/64)*100] that exactly 2 balls will be red.

A much faster way to solve the problem is to look at it this way. There are 3 scenarios where exactly 3 balls are red:

1 2 3

-----------

R R X

R X R

X R R

X is any ball that is not red.


There is a 4.6875% chance that each of these situations will occur.

Take the first one, for example: 25% chance the first ball is red, multiplied by a 25% chance the second ball is red,
multiplied by a 75% chance the third ball is not red.

Because there are 3 scenarios where this outcome occurs, you multiply the 4.6875% chance of any one occurring by 3,
& you get 14.0625%

Consider a state lottery where you get to choose 8 numbers from 1 to 80, no repetiton allowed. The Lottery Commission
chooses 11 from those 80 numbers, again no repetition. You win the lottery if atleast 7 of your numbers are there in the
11 chosen by the Lottery Commission.

Kumaraguru College, Coimbatore


What is the probablity of winning the lottery?
To move a Safe, two cylindrical steel bars 7 inches in diameter are used as rollers.

How far will the safe have moved forward when the rollers have made one revolution?
Answer

The safe must have moved 22 inches forward.

If the rollers make one revolution, the safe will move the distance equal to the circumference of the roller. Hence, the
distance covered by the safe is
= PI * Diameter (or 2 * PI * Radius)
= PI * 7
= 3.14159265 * 7
= 21.99115
= 22 inches approx.
SubmittIf a rook and a bishop of a standard chess set are randomly placed on a chessboard, what is the probability that
one is attacking the other?

Note that both are different colored pieces.


SubmAnswer

The probability of either the Rook or the Bishop attacking the other is 0.3611

A Rook and a Bishop on a standard chess-board can be arranged in 64P2 = 64*63 = 4032 ways

Now, there are 2 cases - Rook attacking Bishop and Bishop attacking Rook. Note that the Rook and the Bishop never
attack each other simultaneously. Let's consider both the cases one by one.

Case I - Rook attacking Bishop


The Rook can be placed in any of the given 64 positions and it always attacks 14 positions. Hence, total possible ways
of the Rook attacking the Bishop = 64*14 = 896 ways

Case II - Bishop attacking Rook


View the chess-board as a 4 co-centric hollow squares with the outermost square with side 8 units and the innermost
square with side 2 units.

If the bishop is in one of the outer 28 squares, then it can attack 7 positions. If the bishop is in one of the 20 squares at
next inner-level, then it can attack 9 positions. Similarly if the bishop is in one of the 12 squares at next inner-level, then
it can attack 11 positions. And if the bishop is in one of the 4 squares at next inner-level (the innermost level), then it
can attack 13 positions.

Hence, total possible ways of the Bishop attacking the Rook


= 28*7 + 20*9 + 12*11 + 4*13
= 560 ways

Thus, the required probability is


= (896 + 560) / 4032
= 13/36
= 0.3611
itted ed
Here in England McDonald's has just launched a new advertising campaign. The poster shows 8 McDonald's products
and underneath claims there are 40312 combinations of the above items.

Given that the maximum number of items allowed is 8, and you are allowed to have less than 8 items, and that the order
of purchase does not matter (i.e. buying a burger and fries is the same as buying fries and a burger)

How many possible combinations are there? Are McDonald's correct in claiming there are 40312 combinations?

Kumaraguru College, Coimbatore


Answer

Total possible combinations are 12869.

It is given that you can order maximum of 8 items and you are allowed to have less than 8 items. Also, the order of
purchase does not matter. Let's create a table for ordering total N items using X products.

Items Products Used (X)


Ordered
(N) 1 2 3 4 5 6 7 8

1 1 - - - - - - -

2 1 1 - - - - - -

3 1 2 1 - - - - -

4 1 3 3 1 - - - -

5 1 4 6 4 1 - - -

6 1 5 10 10 5 1 - -

7 1 6 15 20 15 6 1 -

8 1 7 21 35 35 21 7 1

Total (T) 8 28 56 70 56 28 8 1

Ways to choose 8C1 8C2 8C3 8C4 8C5 8C6 8C7 8C8
X products from
8 products (W)

Total combinations 64 784 3136 4900 3136 784 64 1


(T*W)

Thus, total possible combinations are


= 64 + 784 + 3136 + 4900 + 3136 + 784 + 64 + 1
= 12869
What are the chances that at least two out of a group of fifty people share the same birthday?
SubmittedAnswer

The probability of atleast two out of a group of 50 people share the same birthday is 97%

Probability of atleast two share the same birthday = 1 - probability of all 50 have different birthdays

Probability of all 50 have different birthday


= 365/365 * 364/365 * 363/365 * ... * 317/365 * 316/365
= (365 * 364 * 363 * 362 * ... * 317 * 316)/36550
= 0.0296264

Probability of atleast two share the same birthday


= 1 - 0.0296264
= 0.9703735
= 97% approx.

Thus, the probability of atleast two out of a group of 50 people share the same birthday is 97%

This explains why in a school/college with classrooms of 50 students, there are at least two students with a birthday on

Kumaraguru College, Coimbatore


the same day of the year. Also, if there are 23 people in the room, then there are 50% chances that atleast two of them
have a birthday on the same day of the year!!!

A tank can be filled by pipe A in 30 minutes and by pipe B in 24 minutes. Outlet pipe C can empty the full tank in X
minutes.

If the tank is empty initially and if all the three pipes A, B and C are opened simultaneously, the tank will NEVER be
full. Give the maximal possible value of X.
Answer

The maximum possible value of X is 13 minutes 20 seconds.

In one minute,
pipe A can fill 1/30 part of the tank.
pipe B can fill 1/24 part of the tank.

Thus, the net water level increase in one minute is


= 1/30 + 1/24
= 3/40 part of the tank

In order to keep the tank always empty, outlet pipe C should empty at least 3/40 part of the tank in one minute. Thus,
pipe C can empty the full tank in 40/3 i.e. 13 minutes 20 seconds.
A worker earns a 5% raise. A year later, the worker receives a 2.5% cut in pay, & now his salary is Rs. 22702.68

What was his salary to begin with?


Answer

Rs.22176

Assume his salary was Rs. X

He earns 5% raise. So his salary is (105*X)/100

A year later he receives 2.5% cut. So his salary is ((105*X)/100)*(97.5/100) which is Rs. 22702.68

Hence, solving equation ((105*X)/100)*(97.5/100) = 22702.68


X = 22176

500 men are arranged in an array of 10 rows and 50 columns according to their heights.

Tallest among each row of all are asked to come out. And the shortest among them is A.

Similarly after resuming them to their original positions, the shortest among each column are asked to come out. And
the tallest among them is B.

Now who is taller A or B ?


A person wanted to withdraw X rupees and Y paise from the bank. But cashier made a mistake and gave him Y rupees
and X paise. Neither the person nor the cashier noticed that.

After spending 20 paise, the person counts the money. And to his surprise, he has double the amount he wanted to
withdraw.

Find X and Y. (1 Rupee = 100 Paise)

Submit Users BrainVista Puzzle

Kumaraguru College, Coimbatore


Answer Answer (2) Answer

At the Party:

1. There were 9 men and children.


2. There were 2 more women than children.
3. The number of different man-woman couples possible was 24. Note that if there were 7 men and 5 women,
then there would have been 35 man-woman couples possible.

Also, of the three groups - men, women and children - at the party:
4. There were 4 of one group.
5. There were 6 of one group.
6. There were 8 of one group.

Exactly one of the above 6 statements is false.

Can you tell which one is false? Also, how many men, women and children are there at the party?
Answer

Statement (4) is false. There are 3 men, 8 women and 6 children.

Assume that Statements (4), (5) and (6) are all true. Then, Statement (1) is false. But then Statement (2) and (3) both can
not be true. Thus, contradictory to the fact that exactly one statement is false.

So Statement (4) or Statement (5) or Statement (6) is false. Also, Statements (1), (2) and (3) all are true.

From (1) and (2), there are 11 men and women. Then from (3), there are 2 possible cases - either there are 8 men and 3
women or there are 3 men and 8 women.

If there are 8 men and 3 women, then there is 1 child. Then Statements (4) and (5) both are false, which is not possible.

Hence, there are 3 men, 8 women and 6 children. Statement (4) is false.

Answer

42% houses do not have tubelight, bulb and fan.

Let's assume that there are 100 houses. Hence, there should be total 300 items i.e. 100 tubelights, 100 bulbs and 100
fans.

From the given data, we know that there is shortage of atleast (67+83+73) 223 items in every 100 houses.

Also, exactly 19 houses do not have just one item. It means that remaining 81 houses should account for the shortage of
remaining (223-19) 204 items. If those remaining 81 houses do not have 2 items each, there would be a shortage of 162
items. But total of 204 items are short. Hence, atleast (204-162) 42 houses do not have all 3 items - tubelight, bulb and
fan.

Thus, 42% houses do not have tubelight, bulb and fan.


What is the remainder left after dividing 1! + 2! + 3! + … + 100! By 7?

Think carefully !!!


Answer

A tricky one.

7! onwards all terms are divisible by 7 as 7 is one of the factor. So there is no remainder left for those terms i.e.
remainder left after dividing 7! + 8! + 9! + ... + 100! is 0.

Kumaraguru College, Coimbatore


The only part to be consider is
= 1! + 2! + 3! + 4! + 5! + 6!
= 1 + 2 + 6 + 24 + 120 + 720
= 873

The remainder left after dividing 873 by 7 is 5

Hence, the remainder is 5.

Imagine that you have 26 constants, labelled A through Z. Each constant is assigned a value in the following way: A =
1; the rest of the values equal their position in the alphabet (B corresponds to the second position so it equals 2, C = 3,
etc.) raised to the power of the preceeding constant value. So, B = 2 ^ (A's value), or B = 2^1 = 2. C = 3^2 = 9. D = 4^9,
etc.

Find the exact numerical value to the following equation: (X - A) * (X - B) * (X - C) * ... * (X - Y) * (X - Z)

Answer

(X - A) * (X - B) * (X - C) * ... * (X - Y) * (X - Z) equals 0 since (X - X) is zero


If three babies are born every second of the day, then how many babies will be born in the year 2001?
SuAnswer

9,46,08,000 babies

The total seconds in year 2001


= 365 days/year * 24 hours/day * 60 minutes/hours * 60 seconds/minute
= 365 * 24 * 60 * 60 seconds
= 3,15,36,000 seconds

Thus, there are 3,15,36,000 seconds in the year 2001. Also, three babies born are every second. Hence, total babies born
= 3 * 3,15,36,000 seconds
= 9,46,08,000bmitted

Replace the letters with the correct numbers.


TWO

X TWO

---------

THREE

Submitted by : Timmy Chan


Answer

T=1, W=3, O=8, H=9, R=2, E=4


138

x 138

------------

19044
You can reduce the number of trials. T must be 1 as there is multiplication of T with T in hundred's position. Also, O
can not be 0 or 1. Now, you have to find three digit number whose square satisfies above conditions and square of that
has same last two digits. Hence, it must be between 102 and 139.

Kumaraguru College, Coimbatore


Answer

The easiest way to solve this problem is by writing a computer program that systematically tries all
possible mappings from the numbers onto the letters. This will give you only one solution which
meets the condition that numbers 1 and 6 are most frequently used. mars m=4
venus a=5

uranus r=9

saturn s=3

-------- + v=2 4593

neptune e=0 20163

n=1 695163

u=6 358691

t=8 -------- +

p=7 1078610

There are 4 army men. They have been captured by a rebel group and have been held at ransom. An army intelligent
officer orders them to be burried deep in dirt up to their necks. The format of their burrial are as shown in the figure.

Conditions

• They each have hats on their heads. either black(b) or white (w) look at diagram above. There are total 2 white
hats and 2 black hats.
• They only look in front of them not behind. They are not allowed to communicate by talking.
• Between army man 1 and 2, there is a wall.
• Captive man 4 can see the colour of hats on 2 and 3
• 3 can only see 2's hat
• 2 can only see a wall and 1 can see a wall too, but is on the other side

The officer speaks up, "If one of you can correctly tell me the colour of your hat, you will all go scott free back to your
contries. If you are wrong, you will all be killed.

How can one of them be certain about the hat they are wearing and not risk the lives of their fellow souldiers by taking a
50/50 guess!
Submitted
Answer

Either soldier 3 or soldier 4 can save the life as soldier 1 and soldier 2 can not see colour of any hat, even not their own..
In our case soldier 3 will tell the colour of his hat.

Soldier 4 can see the hat on soldier 2 and soldier 3. If both are white, then he can be sure about colour of his hat which
will be black and vice-versa. But if one of them is white and one is black, then soldier 4 can not say anything as he can
have either of them. So he will keep mum.

If soldier 4 won't say anyhing for a while, then soldier 3 will know that soldier 4 is not in position to tell the colour of
hat on his hat. It means that colour of soldier 3's hat is opposite of colour of soldier 2's hat. So soldier 3 can tell correctly
the colour of hat on his head which is Black.

Kumaraguru College, Coimbatore


Here, we are assuming that all the soldiers are intelligent enough. Also, this solution will work for any combination of 2
Black hats and 2 White hats.
One side of the bottom layer of a triangular pyramid has 12 balls. How many are there in the whole pyramid?

Note that the pyramid is equilateral and solid.


Answer

There are total 364 balls.

As there are 12 balls along one side, it means that there are 12 layers of balls. The top most layer has 1 ball. The second
layer has 3 (1+2) balls. The third layer has 6 (1+2+3) balls. The fourth layer has 10 (1+2+3+4) balls. The fifth layer has
15 (1+2+3+4+5) balls. Similarly, there are 21, 28, 36, 45, 55, 66 and 78 balls in the remaining layers.

Hence, the total number of balls are


= 1 + 3 + 6 + 10 + 15 + 21 + 28 + 36 + 45 + 55 + 66 + 78
= 364 balls

A blindfolded man is asked to sit in the front of a carrom board. The holes of the board are shut with lids in random
order, i.e. any number of all the four holes can be shut or open.

Now the man is supposed to touch any two holes at a time and can do the following.

• Open the closed hole.


• Close the open hole.
• Let the hole be as it is.

After he has done it, the carrom board is rotated and again brought to some position. The man is again not aware of
what are the holes which are open or closed.

How many minimum number of turns does the blindfolded man require to either open all the holes or close all the
holes?

Note that whenever all the holes are either open or close, there will be an alarm so that the blindfolded man will know
that he has won.
Submitted
Answer

The blindfolded man requires 5 turns.

1. Open two adjacent holes.

2. Open two diagonal holes. Now atleast 3 holes are open. If 4th hole is also open, then you are done. If not, the
4th hole is close.

3. Check two diagonal holes.


o If one is close, open it and all the holes are open.
o If both are close, open any one hole. Now, two holes are open and two are close. The diagonal holes
are in the opposite status i.e. in both the diagonals, one hole is open and one is close.

4. Check any two adjacent holes.


o If both are open, close both of them. Now, all holes are close.
o If both are close, open both of them. Now, all holes are open.
o If one is open and one is close, invert them i.e. close the open hole and open the close hole. Now, the
diagonal holes are in the same status i.e. two holes in one diagonal are open and in other are close.

Kumaraguru College, Coimbatore


5. Check any two diagonal holes.
o If both are open, close both of them. Now, all holes are close.
o If both are close, open both of them. Now, all holes are open.

In the middle of the confounded desert, there is the lost city of "Ash". To reach it, I will have to travel overland by foot
from the coast. On a trek like this, each person can only carry enough rations for five days and the farthest we can travel
in one day is 30 miles. Also, the city is 120 miles from the starting point.

What I am trying to figure out is the fewest number of persons, including myself, that I will need in our Group so that I
can reach the city, stay overnight, and then return to the coast without running out of supplies.

How many persons (including myself) will I need to accomplish this mission?
Answer

Total 4 persons (including you) required.

It is given that each person can only carry enough rations for five days. And there are 4 persons. Hence, total of 20 days
rations is available.

1. First Day : 4 days of rations are used up. One person goes back using one day of rations for the return trip. The
rations remaining for the further trek is for 15 days.
2. Second Day : The remaining three people use up 3 days of rations. One person goes back using 2 days of
rations for the return trip. The rations remaining for the further trek is for 10 days.
3. Third Day : The remaining two people use up 2 days of rations. One person goes back using 3 days of rations
for the return trip. The rations remaining for the further trek is for 5 days.
4. Fourth Day : The remaining person uses up one day of rations. He stays overnight. The next day he returns to
the coast using 4 days of rations.

Thus, total 4 persons, including you are required.


At what time after 4.00 p.m. is the minutes hand of a clock exactly aligned with the hour hand?
Answer

4:21:49.5

Assume that X minutes after 4.00 PM minute hand exactly aligns with and hour hand.

For every minute, minute hand travels 6 degrees.


Hence, for X minutes it will travel 6 * X degrees.

For every minute, hour hand travels 1/2 degrees.


Hence, for X minutes it will travel X/2 degrees.

At 4.00 PM, the angle between minute hand and hour hand is 120 degrees. Also, after X minutes, minute hand and hour
hand are exactly aligned. So the angle with respect to 12 i.e. Vertical Plane will be same. Therefore,

6 * X = 120 + X/2
12 * X = 240 + X
11 * X = 240
X = 21.8182
X = 21 minutes 49.5 seconds

Hence, at 4:21:49.5 minute hand is exactly aligned with the hour hand.

Kumaraguru College, Coimbatore


Substitute digits for the letters to make the following Division true
OUT

-------------

STEM|DEMISE

|DMOC

-------------

TUIS

STEM

----------

ZZZE

ZUMM

--------

IST
Note that the leftmost letter can't be zero in any word. Also, there must be a one-to-one mapping between digits and
letters. e.g. if you substitute 3 for the letter M, no other letter can be 3 and all other M in the puzzle must be 3.
Submitted by : Calon
Answer

C=0, U=1, S=2, T=3, O=4, M=5, I=6, Z=7, E=8, D=9

It is obvious that U=1 (as U*STEM=STEM) and C=0 (as I-C=I).

S*O is a single digit and also S*T is a single digit. Hence, their values (O, S, T) must be 2, 3 or 4 (as they can not be 0
or 1 or greater than 4).

Consider, STEM*O=DMOC, where C=0. It means that M must be 5. Now, its simple. O=4, S=2, T=3, E=8, Z=7, I=6
and D=9.

Kumaraguru College, Coimbatore


OUT 413

------------- -------------

STEM|DEMISE 2385|985628

|DMOC |9540

------------- -------------

TUIS 3162

STEM 2385

---------- ----------

ZZZE 7778

ZUMM 7155

-------- --------

IST 623
Also, when arranged from 0 to 9, it spells CUSTOMIZED.
Answer

2025

It is given that no inhabitants have exactly 2025 hairs. Hence there are 2025 inhabitants with 0 to 2024 hairs in the head.

Suppose there are more than 2025 inhabitants. But these will violate the condition that "There are more inhabitants than
there are hairs on the head of any one inhabitants." As for any number more than 2025, there will be same number of
inhabitants as the maximum number of hairs on the head of any inhabitant.
There are four groups of Mangoes, Apples and Bananas as follows:
Group I : 1 Mango, 1 Apples and 1 Banana
Group II : 1 Mango, 5 Apples and 7 Bananas
Group III : 1 Mango, 7 Apples and 10 Bananas
Group IV : 9 Mango, 23 Apples and 30 Bananas

Group II costs Rs 300 and Group III costs Rs 390.

Can you tell how much does Group I and Group IV cost?
Answer

Group I costs Rs 120 and Group IV costs Rs 1710

Assume that the values of one mango, one apple and one banana are M, A and B respectively.

From Group II : M + 5A + 7B = 300


From Group III : M + 7A + 10B = 390

Subtracting above to equations : 2A + 3B = 90

For Group I :
=M+A+B
= (M + 5A + 7B) - (4A + 6B)
= (M + 5A + 7B) - 2(2A + 3B)
= 300 - 2(90)

Kumaraguru College, Coimbatore


= 300 - 180
= 120

Similarly, for Group IV :


= 9M + 23A + 30B
= 9(M + 5A + 7B) - (22A + 33B)
= 9(M + 5A + 7B) - 11(2A + 3B)
= 9(300) - 11(90)
= 2700 - 990
= 1710

Thus, Group I costs Rs 120 and Group IV costs Rs 1710.


Tic-Tac-Toe is being played. One 'X' has been placed in one of the corners. No 'O' has been placed yet.

Where does the player that is playing 'O' has to put his first 'O' so that 'X' doesn't win?

Assume that both players are very intelligent. Explain your answer
Answer

"O" should be placed in the center.

Let's number the positions as:


1|2|3

---------

4|5|6

---------

7|8|9
It is given that "X" is placed in one of the corner position. Let's assume that its at position 1.

Now, let's take each position one by one.

• If "O" is placed in position 2, "X" can always win by choosing position 4, 5 or 7.


• If "O" is placed in position 3, "X" can always win by choosing position 4, 7 or 9.
• If "O" is placed in position 4, "X" can always win by choosing position 2, 3 or 5.
• If "O" is placed in position 6, "X" can always win by choosing position 3, 5 or 7.
• If "O" is placed in position 7, "X" can always win by choosing position 2, 3 or 9.
• If "O" is placed in position 8, "X" can always win by choosing position 3, 5 or 7.
• If "O" is placed in position 9, "X" can always win by choosing position 3, or 7.

If "O" is placed in position 5 i.e. center position, "X" can't win unless "O" does something foolish ;))

Hence, "O" should be placed in the center.

Amit, Bhavin, Himanshu and Rakesh are sitting around a table.

• The Electonics Engineer is sitting to the left of the Mechanical Engineer.


• Amit is sitting opposite to Computer Engineer.
• Himanshu likes to play Computer Games.
• Bhavin is sitting to the right of the Chemical Engineer.

Can you figure out everyone's profession?


Answer

Kumaraguru College, Coimbatore


Amit is the Mechanical Engineer. Bhavin is the Computer Engineer. Himanshu and Rakesh are either Chemical
Engineer or Elecronics Engineer.

Amit and Bhavin are sitting opposite to each other. Whereas Chemical Engineer and Elecronics Engineer are sitting
opposite to each other.

We cannot find out who is Chemical Engineer and Elecronics Engineer as data provided is not sufficient

Five friends with surname Batliwala, Pocketwala, Talawala, Chunawala and Natakwala have their first name and
middle name as follow.

1. Four of them have a first and middle name of Paresh.


2. Three of them have a first and middle name of Kamlesh.
3. Two of them have a first and middle name of Naresh.
4. One of them have a first and middle name of Elesh.
5. Pocketwala and Talawala, either both are named Kamlesh or neither is named Kamlesh.
6. Either Batliwala and Pocketwala both are named Naresh or Talawala and Chunawala both are named Naresh.
7. Chunawala and Natakwala are not both named Paresh.

Who is named Elesh?


Answer

Pocketwala is named Elesh.

From (1) and (7), it is clear that Batliwala, Pocketwala and Talawala are named Paresh.

From (6) and (5), if Pocketwala or Talawala both are named Kamlesh, then either of them will have three names i.e.
Paresh, Kamlesh and Naresh. Hence, Pocketwala and Talawala both are not named Kamlesh. It means that Batliwala,
Chunawala and Natakwala are named Kamlesh.

Now it is clear that Talawala and Chunawala are named Naresh. Also, Pocketwala is named Elesh.

Mr. Wagle goes to work by a bus. One day he falls asleep when the bus still has twice as far to go as it has already gone.

Halfway through the trip he wakes up as the bus bounces over some bad potholes. When he finally falls asleep again,
the bus still has half the distance to go that it has already travelled. Fortunately, Mr. Wagle wakes up at the end of his
trip.

What portion of the total trip did Mr. Wagle sleep?

Answer

Mr. wagle slept through half his trip.

Let's draw a timeline. Picture the bus route on a line showen below:

---------------- ________ -------- ________________

Start 1/3 1/2 2/3 End

----- shows time for which Mr. Wagle was not sleeping

_____ shows time for which Mr. Wagle was sleeping

When Mr. Wagle fell asleep the first time, the bus sill had twice as far to go as it had already gone, that marks the first

Kumaraguru College, Coimbatore


third of his trip.

He wake up halfway through the trip i.e slept from 1/3 mark to the 1/2 mark. He fell sleep again when the bus still had
half the distance to go that it had already traveled i.e 2/3 mark.

Adding up, all sleeping times,


= (1/2 - 1/3) + (1 - 2/3)
= 1/6 + 1/3
= 1/2

Hence, Mr. wagle slept through half his trip.


4 If you don't agree, try it yourself!
You have a bucket of jelly beans. Some are red, some are blue, and some green. With your eyes closed, pick out 2 of a
like color.

How many do you have to grab to be sure you have 2 of the same?

You have a bucket of jelly beans. Some are red, some are blue, and some green. With your eyes closed, pick out 2 of a
like color.

How many do you have to grab to be sure you have 2 of the same?
If you select 4 Jelly beans you are guarenteed that you will have 2 that are the same color.

There are 70 employees working with BrainVista of which 30 are females. Also,

• 30 employees are married


• 24 employees are above 25 years of age
• 19 married employees are above 25 years, of which 7 are males
• 12 males are above 25 years of age
• 15 males are married.

How many unmarried females are there and how many of them are above 25?
Answer

15 unmarried females & none are above 25 years of age.

Simply put all given information into the table structure and you will get the answer.
Married Unmarried

Below 25 Above 25 Below 25 Above 25

Female 3 12 15 0

Male 8 7 20 5

There is a safe with a 5 digit number as the key. The 4th digit is 4 greater than the second digit, while the 3rd digit is 3
less than the 2nd digit. The 1st digit is thrice the last digit. There are 3 pairs whose sum is 11.

Find the number.


Answer

65292

As per given conditions, there are three possible combinations for 2nd, 3rd and 4th digits. They are (3, 0, 7) or (4, 1, 8)
or (5, 2, 9)

Kumaraguru College, Coimbatore


It is given that there are 3 pairs whose sum is 11. All possible pairs are (2, 9), (3, 8), (4, 7), (5, 6). Now required number
is 5 digit number and it contains 3 pairs of 11. So it must not be having 0 and 1 in it. Hence, the only possible
combination for 2nd, 3rd and 4th digits is (5, 2, 9)

Also, 1st digit is thrice the last digit. The possible combinations are (3, 1), (6, 2) and (9, 3), out of which only (6, 2) with
(5, 2, 9) gives 3 pairs of 11. Hence, the answer is 65292.
My friend collects antique stamps. She purchased two, but found that she needed to raise money urgently. So she sold
them for Rs. 8000 each. On one she made 20% and on the other she lost 20%.

How much did she gain or lose in the entire transaction?


Answer

She lost Rs 666.67

Consider the first stamp. She mades 20% on it after selling it for Rs 8000.

So the original price of first stamp is


= (8000 * 100) / 80
= Rs 6666.67

Similarly, consider second stamp. She lost 20% on it after selling it for Rs 8000

So the original price of second stamp is


= (8000 * 100) / 80
= Rs 10000

Total buying price of two stamps


= Rs 6666.67 + Rs 10000
= Rs 16666.67

Total selling price of two stamps


= Rs 8000 + Rs 8000
= Rs 16000

Hence, she lost Rs 666.67

Assume for a moment that the earth is a perfectly uniform sphere of radius 6400 km. Suppose a thread equal to the
length of the circumference of the earth was placed along the equator, and drawn to a tight fit.

Now suppose that the length of the thread is increased by 12 cm, and that it is pulled away uniformly in all directions.

By how many cm. will the thread be separated from the earth's surface?
Answer

The cicumference of the earth is


= 2 * PI * r
= 2 * PI * 6400 km
= 2 * PI * 6400 * 1000 m
= 2 * PI * 6400 * 1000 * 100 cm
= 1280000000 * PI cm

where r = radius of the earth, PI = 3.141592654

Hence, the length of the thread is = 1280000000 * PI cm

Now length of the thread is increasd by 12 cm. So the new length is = (1280000000 * PI) + 12 cm

Kumaraguru College, Coimbatore


This thread will make one concentric circle with the earth which is slightly away from the earth. The circumfernce of
that circle is nothing but (1280000000 * PI) + 12 cm

Assume that radius of the outer circle is R cm


Therefore,
2 * PI * R = (1280000000 * PI) + 12 cm

Solving above equation, R = 640000001.908 cm


Radius of the earth is r = 640000000 cm

Hence, the thread will be separatedfrom the earth by


= R - r cm
= 640000001.908 - 640000000
= 1.908 cm
Scientist decided to do a study on the population growth of rabbits. Inside a controlled environment, 1000 rabbits were
placed.

Six months later, there were 1000Z rabbits. At the beginning of the 3rd year, there were roughly 2828Z rabbits, which
was 4 times what the scientists placed in there at the beginning of the 1st year.

If Z is a positive variable, how many rabbits would be there at the beginning of the 11th year?
SubmiAnswer

At the beginning of the 11th year, there would be 1,024,000 rabbits.

At the beginning, there were 1000 rabbits. Also, there were 4000 rabbits at the beginning of third year which is equal to
2828Z. Thus, Z = 4000/2828 i.e. 1.414 (the square root of 2)

Note that 2828Z can be represented as 2000*Z*Z (Z=1.414), which can be further simplified as 1000*Z*Z*Z*Z

Also, it is given that at the end of 6 months, there were 1000Z rabbits.

It is clear that the population growth is 1.414 times every six months i.e. 2 times every year. After N years, the
population would be 1000*(Z^(2N)) i.e. 1000*(2^N)

Thus, at the beginning of the 11th year (i.e. after 10 years), there would be 1000*(2^10) i.e. 1,024,000 rabbits.
tted
A man is stranded on a desert island. All he has to drink is a 20oz bottle of sprite.

To conserve his drink he decides that on the first day he will drink one oz and the refill the bottle back up with water.
On the 2nd day he will drink 2oz and refill the bottle. On the 3rd day he will drink 3oz and so on...

By the time all the sprite is gone, how much water has he drunk?
SubmittAnswer

The man drunk 190oz of water.

It is given that the man has 20oz bottle of sprite. Also, he will drink 1oz on the first day and refill the bottle with water,
will drink 2oz on the second day and refill the bottle, will drink 3oz on the third day and refill the bottle, and so on till
20th day. Thus at the end of 20 days, he must have drunk (1 + 2 + 3 + 4 + ..... +18 + 19 + 20) = 210oz of liquid.

Out of that 210oz, 20oz is the sprite which he had initially. Hence, he must have drunk 190oz of water.ed
You have four 9's and you may use any of the (+, -, /, *) as many times as you like. I want to see a mathematical
expression which uses the four 9's to = 100

How many such expressions can you make?


Submitted

Kumaraguru College, Coimbatore


Answer

There are 5 such expressions.

99 + (9/9) = 100

(99/.99) = 100

(9/.9) * (9/.9) = 100

((9*9) + 9)/.9 = 100

(99-9)/.9 = 100
Two planes take off at the same exact moment. They are flying across the Atlantic. One leaves New York and is flying
to Paris at 500 miles per hour. The other leaves Paris and is flying to New York at only 450 miles per hour ( because of
a strong head wind ).

Which one will be closer to Paris when they meet?


They will both be the same distance from Paris when they meet!!!

12 members were present at a board meeting. Each member shook hands with all of the other members before & after
the meeting.

How many hand shakes were there?


Answer

132

Think of it this way: the first person shakes hands with 11 people, the second person also shakes hands with 11 people,
but you only count 10, because the hand shake with the first person was already counted. Then add 9 for the third
person, 8 for the fourth, & so on.

66 hand shakes took place before & 66 after the meeting, for a total of 132.
Arrange five planets such that 4 of them add up to 5th planet numerically. Each of the letters of the planet should
represent a unique number from the range 0 - 9. You have to use all ten digits.

There is an amazing mathematical relationship exists among the names of the planet.
Answer

The tought process is initially to find planets such that the total number of alphabets in them is 10.

The only possible combination of planets is Saturn, Uranus, Venus, Mars and Neptune because for other combinations
there will be more than 10 alphabets. Among these five, Neptune is the lenghtiest, so it must be the sum of the other
four.

Kumaraguru College, Coimbatore


SATURN

URANUS

VENUS

+ MARS

--------------

NEPTUNE

Now the only possible value for N is 1. By finding the value for S, we can reach the result:

358691

695163

20163

+ 4593

--------------

1078610

You have 14 apples. Your Friend Marge takes away 3 and gives you 2. You drop 7 but pick up 4. Bret takes 4 and gives
5. You take one from Marge and give it to Bret in exchange for 3 more. You give those 3 to Marge and she gives you an
apple and an orange. Frank comes and takes the apple Marge gave you and gives you a pear. You give the pear to Bret
in exchange for an apple. Frank then takes an apple from Marge, gives it to Bret for an orange, gives you the orange for
an apple.

How many pears do you have?


Submitted
Answer

None

Frank gave you a pear in exchange of the apple which Marge gave you. And you gave that pear to Bret in exchange for
an apple. All the others exchanges involved apples and/or organges.
Four couples are going to the movie. Each row holds eight seats. Betty and Jim don't want to sit next to Alice and Tom.
Alice and Tom don't want to sit next to Gertrude and Bill. On the otherhand, Sally and Bob don't want to sit next to
Betty and Jim.

How can the couples arrange themselves in a row so that they all sit where they would like?
Submitted by : Tara Smith

Answer

From the given data, it can be inferred that:


(Sally & Bob) NOT (Betty & Jim) NOT (Alice & Tom) NOT (Gertrude & Bill)

(A) NOT (B) means A and B can not seat next to each other.

Now, it is obvious that (Betty & Jim) and (Alice & Tom) will occupy the corner seats as both of them can have only one
neighbour. Therefore,

Kumaraguru College, Coimbatore


(Gertrude & Bill) will seat next to (Betty & Jim)
(Sally & Bob) will seat next to (Gertrude & Bill)
(Alice & Tom) will seat next to (Sally & Bob)

Thus, there are two possible arrangements - a mirror images of each other.

1. (Betty & Jim) - (Gertrude & Bill) - (Sally & Bob) - (Alice & Tom)
2. (Alice & Tom) - (Sally & Bob) - (Gertrude & Bill) - (Betty & Jim)
Substitute digits for the letters to make the following addition problem true.
W H O S E

T E E T H

A R E

+ A S

-------------------

S W O R D S
Note that the leftmost letter can't be zero in any word. Also, there must be a one-to-one mapping between digits and
letters. e.g. if you substitute 3 for the letter H, no other letter can be 3 and all other H in the puzzle must be 3.
Answer

It is obvious that S=1 and T=9.

Also, (H + E) should be greater than 10 and hence, (E + H + E) must 20. Thus, there are 3 possible values for (E, H)
pair: (6, 8) or (7, 6) or (8, 4). Use trial-n-error and everything will fit-in.

W H O S E 2 8 5 1 6

T E E T H 9 6 6 9 8

A R E 4 7 6

+ A S + 4 1

------------------- -------------------

S W O R D S 1 2 5 7 3 1

When Socrates was imprisoned for being a disturbing influence, he was held in high esteem by his guards. All four of
them hoped that something would occur that would facilitate his escape. One evening, the guard who was on duty
intentionally left the cell door open so that Socrates could leave for distant parts.

Socrates did not attempt to escape, as it was his philosophy that if you accept society's rules, you must also accept it's
punishments. However, the open door was considered by the authorities to be a serious matter. It was not clear which
guard was on that evening. The four guards make the following statements in their defense:

Aaron:
A) I did not leave the door open.
B) Clement was the one who did it.

Bob:
A) I was not the one who was on duty that evening.
B) Aaron was on duty.

Clement:

Kumaraguru College, Coimbatore


A) Bob was the one on duty that evening.
B) I hoped Socrates would escape.

David:
A) I did not leave the door open.
B) I was not surprised that Socrates did not attempt to escape.

Considering that, in total, three statements are true, and five statements are false, which guard is guiltyAnswer

David is the guilty.

Note that "All four of them hoped that something would occur that would facilitate his escape". It makes Clement's
statement B True and David's statement B False.

Now consider each of them as a guilty, one at a time.


Aaron Bob Clement David True
Stmts
A B A B A B A B

If Aaron is guilty False False True True False True True False 4

If Bob is guilty True False False False True True True False 4

If Clement is guilty True True True False False True True False 5

If David is guilty True False True False False True False False 3

Since in total, three statements are true and five statements are false. It is clear from the above table that David is?

There were N stations on a railroad. After adding X stations 46 additional tickets have to be printed.

Find N and X.
Answer

Let before adding X stations, total number of tickets


t = N(N-1)

After adding X stations total number of tickets are


t + 46 = (N+X)(N+X-1)

Subtracting 1st from 2nd


46 = (N+X)(N+X-1) - N(N-1)
46 = N2 + NX - N + NX + X2 - X - N2 + N
46 = 2NX + X2 - X
46 = (2N - 1)X + X2
X2 + (2N - 1)X - 46 = 0

Now there are only two possible factors of 46. They are (46,1) and (23,2)

Case I: (46,1)
2N - 1 = 45
2N = 46
N = 23
And X = 1

Case II: (23,2)


2N - 1 = 21
2N = 22
N = 11

Kumaraguru College, Coimbatore


And X = 2

Hence, there are 2 possible answers.

An emergency vehicle travels 10 miles at a speed of 50 miles per hour.

How fast must the vehicle travel on the return trip if the round-trip travel time is to be 20 minutes?
Answer

75 miles per hour

While going to the destination, the vehicle travels 10 mils at the speed of 50 miles per hour. So the time taken to travel
10 miles is
= (60 * 10) / 50
= 12 minutes

Now it's given that round-trip travel time is 20 minutes. So the vehicle should complete its return trip of 10 miles in 8
minutes. So the speed of the vehicle must
= (60 * 10) / 8
= 75 miles per hour
All of the students at a college are majoring in psychology, business, or both. 73% of the students are psychology
majors, & 62% are business majors.

If there are 200 students, how many of them are majoring in both psychology & business?
Answer

70 students are majoring in both, psychology & business

If 73% of the students are psychology majors, we know that 27% are not psychology majors. By the same reasoning,
38% are not business majors, because 62% of the students do major in business. So: 27 + 38 = 65

65% of the students are not majoring in both psychology & business, so 35% are double majors, a total of 70 students.
Two trains starting at same time, one from Bangalore to Mysore and other in opposite direction arrive at their
destination 1hr and 4hrs respectively after passing each other.

Answer

The speed of Bangalore-Mysore train is TWICE the speed of Mysore-Bangalore train.

Let the distance between Bangalore and Mysore is D kms.


Also, let speed of the train from Bangalore to Mysore is X km/hr and speed of the tain from Mysore to Bangalore is Y
km/hr.

Now, assume that both the trains met each other at T kms from the Bangalore (point P in figure)
Time taken by Bangalore-Mysore train to reach P = Time taken by Mysore-Bangalore train to reach P
( T / X ) = ( D - T ) / Y -----equ(I)

Also, Bangalore-Mysore train and Mysore-Bangalore train arrive destination 1 hr and 4 hrs respectively after passing
each other. It means that Bangalore-Mysore train travels (D - T) kms in 1 hr at X km/hr and Mysore-Bangalore train
travels T kms in 4 hrs at Y km/hr. Hence,
( D - T ) = X and
T=4*Y

Substituting these values in equation I, we get


(4*Y)/X=X/Y
4*Y*Y=X*X
2*Y=X

Kumaraguru College, Coimbatore


Hence, the speed of Bangalore-Mysore train is TWICE the speed of Mysore-Bangalore train.How much faster is one
train from other?

Answer

49 times

Let's assume that everyone clinked their mug with friend to his left only. It means that there are total 49 clinks. Now the
right clink of each person is left clink of the person on right which is already happened. Hence, there are only 49 clinks.

Mrs. Watsherface had a garage sale. A custmer named Gina bought an old lamp and a rug. She paid a total of $5.25 for
everything. The rug cost 25 cents more than the lamp.

How much did each cost?


Submitted by : Kimi
Answer

The lamp cost $ 2.50 and the rug cost $ 2.75

A simple one.

Assume that the lamp cost $ L.


Hence the rug must have cost $ (L + 0.25)
Also, total cost is $ 5.25, Hence the equation :
L + L + 0.25 = 5.25
2*L=5
L = 2.50

Hence, the lamp cost $ 2.50 and the rug cost $ 2.75

Brain Teaser No : 00518

Write 1111......(243 times) i.e. a 243 digit number with all 1s.

Prove that it is divisible by 243.


SubmittAnswer

Prove it using the mathematical induction.

First here are a couple of things to note:

[1] A number whose digits add up to a multiple of three is divisable by 3.


e.g. 369: 3+6+9=18: 1+8=9 which is a multiple of 3 hence 369 is divisable by 3.

[2] Whenever a number (X) is multiplied with another number (Y) then the product (X*Y) will have all the factors of X
as well as all the factors of Y in its set of factors.
e.g. if X has factors of (1,P,Q,X) and Y has factors of (1,Q,R,Y) then X*Y has factors of (1,P,Q,Q,R,X,Y).

Let
N = any series of digits (e.g. N=369)
D = the number of digits in N (e.g. if N=369 then D=3)
P = is a number constructed in the following way : a 1, followed by (D-1) 0s, followed by another 1, followed by (D-1)
0s, followed by another 1. (e.g. if N=369 then D=3 and P would be 1001001) Note that P will always be divisible by 3.

Also, if we multiply N with P we are essentially repeating N for (D-1) times.


e.g. if N=369 then D=3, P=1001001 and N*P=369369369

Kumaraguru College, Coimbatore


Let's start with N=111. It is clear that N is divisible by 3. (From [1])
Also, D=3 and P=1001001
N*P=111111111 (9 times)
The resulting number 111111111 must be divisible by 9 as N and P both are divisible by 3.

Now, let's start with N=111111111. It is clear that N is divisible by 9.


Also, D=9 and P=1000000001000000001
N*P=111111111... (27 times)
The resulting number 1111111... (27 times) must be divisible by 27 as N is divisible by 9 and P is divisible by 3.

Repeat the same procedure for N=1111111... (27 times) The resulting number 1111111... (81 times) must be divisible
by 81 as N is divisible by 27 and P is divisible by 3.

Similarly, for N=1111111... (81 times) The resulting number 1111111... (243 times) must be divisible by 243 as N is
divisible by 81 and P is divisible by 3.

Thus, 1111111... (243 times) is divisible by 243.

Thanks to Ryan Hutcherson for solution !!!


edKaran bought a little box of midget matches, each one inch in length. He found that he could arrange them all in the
form of a triangle whose area was just as many square inches as there were matches.

He then used up six of the matches, and found that with the remainder he could again construct another triangle whose
area was just as many square inches as there were matches.

And using another six matches he could again do precisely the same.

How many matches were there in the box originally?

Note that the match-box can hold maximum of 50 matches.


Answer

Initially, there were 42 or 36 matches in the match-box.

There are 42 matches in the box with which he could form a triangle 20, 15, 7, with an area of 42 square inches. After 6
matches had been used, the remaining 36 matches would form a triangle 17, 10, 9, with an area of 36 square inches.
After using another 6 matches, the remaining 30 matches would form a triangle 13, 12, 5, with an area of 30 square
inches. After using another 6, the 24 remaining would form a triangle 10, 8, 6, with an area of 24 square inches.

Thus, there are two possible answers. There were either 42 or 36 matches in the match-box.

Also it is interesting to know that there are just 5 such triangles for which the perimeter and the area is the same
(assuming all sides are integers) and they are :

1. 24 (10, 8, 6)
2. 30 (13, 12, 5)
3. 36 (17, 10, 9)
4. 42 (20, 15, 7)
5. 60 (29, 25, 6)

Find the values of each of the alphabets.

Kumaraguru College, Coimbatore


NOON

SOON

+ MOON

----------

JUNE
Answer

Using trial and error. There are 2 solutions to it and may be more.

2442

1442

+ 5442

----------

9326

4114

5114

+ 0114

----------

9342

We have to fill number from 1 to 12 at the intersection point of two or more lines. We have to construct a star using two
triangle. The sum of all number lying in straight lines should be same. This can be easilty understood by the fig. and
hence solved.
Submitted by : Vaibhav Gupta

Submit Users BrainVista Puzzle A Add to


Answer Answer (3) Answer Friend Favourite
We have one answer where sum of all the numbers lying in straight lines is 26.

If you have others, do submit them.

Answer

Bantu is telling the truth.

For a IF-THEN statement to be false, IF part has to be true and THEN part has to be false.

Since only one statement is true and remaining three are false, IF part of three statements are true & THEN part of one
statement is true. Let's put the given information in table. The pet-name in the normal text represents the IF part and the
pet-name in round brackets represents the THEN part.

Kumaraguru College, Coimbatore


Montu Bantu Chantu Pintu

Montu says Dog (Dog) (Dog) Dog

Bantu says (Dog) Cat Cat (Dog)

Chantu says Dog (Cat) Dog (Cat)

Pintu says Cat Cat


(Dog) (Dog)

It is clear that the IF part of the statements made by Montu, Chantu and Pintu are true as they do not contradict each
other. And the IF part of the statement made by Bantu is false.

Thus, Bantu is telling the truth.

Montu have a Dog and may or may not have a Cat.


Bantu have a Cat.
Chantu have a Dog.
Pintu have a Dog and a Cat.
Answer

Number 3 is face down on the table.

If Abu can see two even numbers i.e. number 2 twice, and if Dabu can see number 2 twice, then number 2 must be
facing up.

Now everything else is simple. (see the following diagram)

Dabu Abu

3 2 2

Calu Babu
Thus, the number hidden from the view is number 3 and hence the answer.
Two identical pack of cards A and B are shuffled throughly. One card is picked from A and shuffled with B. The top
card from pack A is turned up. If this is the Queen of Hearts, what are the chances that the top card in B will be the King
of Hearts?
Answer

52 / 2703

There are two cases to be considered.

CASE 1 : King of Hearts is drawn from Pack A and shuffled with Pack B

Kumaraguru College, Coimbatore


Probability of drawing King of Hearts from Pack A = 1/51 (as Queen of Hearts is not to be drawn)
Probability of having King of Hearts on the top of the Pack B = 2/53

So total probability of case 1 = (1/51) * (2/53) = 2 / (51 * 53)

CASE 2 : King of Hearts is not drawn from Pack A

Probability of not drawing King of Hearts from Pack A = 50/51 (as Queen of Hearts is not to be drawn)
Probability of having King of Hearts on the top of the Pack B = 1/53

So total probability of case 2 = (50/51) * (1/53) = 50 / (51 * 53)

Now adding both the probability, the required probability is


= 2 / (51 * 53) + 50 / (51 * 53)
= 52 / (51 * 53)
= 52 / 2703
= 0.0192378
How many possible combinations are there in a 3x3x3 rubics cube?

In other words, if you wanted to solve the rubics cube by trying different combinations, how many might it take you
(worst case senerio)?

How many for a 4x4x4 cube?


Submitted
Answer

There are 4.3252 * 10^19 possible combinations for 3x3x3 Rubics and 7.4012 * 10^45 possible combinations for 4x4x4
Rubics.

Let's consider 3x3x3 Rubics first.

There are 8 corner cubes, which can be arranged in 8! ways.


Each of these 8 cubes can be turned in 3 different directions, so there are 3^8 orientations altogether. But if you get all
but one of the corner cube into chosen positions and orientations, only one of 3 orientations of the final corner cube is
possible. Thus, total ways corner cubes can be placed = (8!) * (3^8)/8 = (8!) * (3^7)

Similarly, 12 edge cubes can be arranged in 12! ways.


Each of these 12 cubes can be turned in 2 different directions, so there are 2^12 orientations altogether. But if you get
all but one of the edge cube into chosen positions and orientations, only one of 2 orientations of the final edge cube is
possible. Thus, total ways edge cubes can be placed = (12!) * (2^12)/2 = (12!) * (2^11)

Here, we have essentially pulled the cubes apart and stuck cubes back in place wherever we please. In reality, we can
only move cubes around by turning the faces of the cubes. It turns out that you can't turn the faces in such a way as to
switch the positions of two cubes while returning all the others to their original positions. Thus if you get all but two
cubes in place, there is only one attainable choice for them (not 2!). Hence, we must divide by 2.

Total different possible combinations are


= [(8!) * (3^7)] * [(12!) * (2^11)] / 2
= (8!) * (3^7) * (12!) * (2^10)
= 4.3252 * 10^19

Similarly, for 4x4x4 Rubics total different possible combinations are


= [(8!) * (3^7)] * [(24!)] * [(24!) / (4!^6)] / 24
= 7.4011968 * 10^45

Note that there are 24 edge cubes, which you can not turn in 2 orientations (hence no 2^24 / 2). Also, there are 4 center

Kumaraguru College, Coimbatore


cubes per face i.e. (24!) / (4!^6). You can switch 2 cubes without affecting the rest of the combination as 4*4*4 has
even dimensions (hence no division by 2). But pattern on one side is rotated in 4 directions over 6 faces, hence divide
by 24.

Answer

A tough one!!!

Since R + E + E = 10 + E, it is clear that R + E = 10 and neither R nor E is equal to 0 or 5. This is the only entry point to

solve it. Now use trial-n-error method.

N E V E R 2 1 4 1 9

L E A V E 3 1 5 4 1

+ M E + 6 1

----------------- -----------------

A L O N E 5 3 0 2 1
There are 20 people in your applicant pool, including 5 pairs of identical twins.

If you hire 5 people randomly, what are the chances you will hire at least 1 pair of identical twins? (Needless to say, this
could cause trouble ;))
SubmAnswer

The probability to hire 5 people with at least 1 pair of identical twins is 25.28%

5 people from the 20 people can be hired in 20C5 = 15504 ways.

Now, divide 20 people into two groups of 10 people each :


G1 - with all twins
G2 - with all people other than twins

Let's find out all possible ways to hire 5 people without a single pair of indentical twins.
People from People from No of ways to hire G1 without a single pair of No of ways to hire Total
G1 G2 indentical twins G2 ways
0 5 10C0 10C5 252
1 4 10C1 10C4 2100
2 3 10C2 * 8/9 10C3 4800
3 2 10C3 * 8/9 * 6/8 10C2 3600
4 1 10C4 * 8/9 * 6/8 * 4/7 10C1 800
5 0 10C5 * 8/9 * 6/8 * 4/7 * 2/6 10C0 32
Total 11584

Thus, total possible ways to hire 5 people without a single pair of indentical twins = 11584 ways

So, total possible ways to hire 5 people with at least a single pair of indentical twins = 15504 - 11584 = 3920 ways

Hence, the probability to hire 5 people with at least a single pair of indentical twins
= 3920/15504
= 245/969

Kumaraguru College, Coimbatore


= 0.2528
= 25.28%itted
Veeru says to Jay, "Can you figure out how many Eggs I have in my bucket?" He gives 3 clues to Jay: If the number of
Eggs I have

1. is a multiple of 5, it is a number between 1 and 19


2. is not a multiple of 8, it is a number between 20 and 29
3. is not a multiple of 10, it is a number between 30 and 39

How many Eggs does Veeru have in his bucket?


Submit Users
BrainV
Answer Answer (14)

Mr. Black, Mr. White and Mr. Grey were chatting in the Yahoo conference. They were wearing a black suit, a white suit
and a grey suit, not necessarily in the same order.

Mr. Grey sent message, "We all are wearing suit that are of the same color as our names but none of us is wearing a suit
that is the same color as his name."

On that a person wearing the white suit replied, "What difference does that make?"

Can you tell what color suit each of the three persons had on?
Answer

Mr. Grey is wearing Black suit.


Mr. White is wearing Grey suit.
Mr. Black is wearing White suit.

Mr. Grey must not be wearing grey suit as that is the same colour as his name. Also, he was not wearing white suit as
the person wearing white suit responded to his comment. So Mr Grey must be wearing a black suit.

Similarly, Mr. White must be wearing either black suit or grey suit. But Mr. Grey is wearing a black suit. Hence, Mr.
White must be wearing a grey suit.

And, Mr. Black must be wearing white suit.


Substitute numbers for the letters so that the following mathematical expressions are correct.
ABC DEF GHI

--- = IE --- = IE --- = IE

3 6 9
Note that the same number must be used for the same letter whenever it appears.
Answer

A=2, B=1, C=9, D=4, E=3, F=8, G=6, H=5, I=7

Let's start with GHI = 9 * IE. Note that I appears on both the side. Also, after multiplying IE by 9 the answer should
have I at the unit's place. The possible values of IE are 19, 28, 37, 46, 55, 64, 73, 82 and 91; out of which only 64, 73
and 82 satisfies the condition. (as all alphabet should represent different digits)

Now, consider DEF = 6 * IE. Out of three short-listed values, only 73 satisfies the equation. Also, ABC = 3 * IE is
satisfied by 73.

Hence, A=2, B=1, C=9, D=4, E=3, F=8, G=6, H=5, I=7

Kumaraguru College, Coimbatore


219 438 657

--- = 73 --- = 73 --- = 73

3 6 9

Brain Teaser No : 00374

A, B, C and D are related to each other.

• One of the four is the opposite sex from each of the other three.
• D is A's brother or only daughter.
• A or B is C's only son.
• B or C is D's sister.

Answer

A, B & D are males; C is female. B is C's only son. A & D are C's brothers.
A(male) --- C(female) --- D(male)

B(male)
Work out which relation can hold and discard the contradictory options.

From (2) and (4), D can not be a only daughter and have a sister (B or C). Hence, D is A's brother i.e. D is a Male.

From (4), let's say that B is D's sister i.e. B is Female.


From (3), A is C's only son i.e. A is Male.
But D is A's brother which means that A is not C's only son. Hence, our assumption was wrong.

Thus, C is D's sister i.e. C is Female. And B must be C's only son.

Now it is clear that D & B are Males and C is Female. A must be a Male as only one of them is of opposite sex from
each of the other three. And he is C & D's brother.How are they related to each other?

Dr. DoLittle always goes walking to the clinic and takes the same time while going and while coming back. One day he
noticed something.

When he left the home, the hour hand and the minute hand were exactly opposite to each other and when he reached the
clinic, they were together.

Similarly, when he left the clinic, the hour hand and the minute hand were together and when he reached the home, they
were exactly opposite to each other.

How much time does Dr. DoLittle take to reach home from the clinic? Give the minimal possible answer.
Answer

32 minutes 43.6 seconds

In twelve hours, the minute hand and the hour hand are together for 11 times. It means that after every 12/11 hours, both
the hands are together.

Similarly in twelve hours, the minute hand and the hour hand are exactly opposite to each other for 11 times. It means
that after every 12/11 hours, both the hands are opposite.

Kumaraguru College, Coimbatore


Now, let's take an example. We know that at 12 both the hands are together and at 6 both the hands are exactly opposite
to each other.

After 6, both the hands are in opposition at [6+(12/11)] hours, [6+2*(12/11)] hours, [6+3*(12/11)] hours and so on. The
sixth such time is [6+6*(12/11)] hours which is the first time after 12. Thus after 12, both the hands are opposite to each
other at 12:32:43.6

Hence, Dr. DoLittle takes 32 minutes and 43.6 seconds to reach home from the clinic.

SlowRun Express runs between Bangalore and Mumbai, For the up as well as the down journey, the train leaves the
starting station at 10:00 PM everyday and reaches the destination at 11:30 PM after three days.

Mr. Haani once travelled by SlowRun Express from Mumbai to Bangalore. How many SlowRun Express did he cross
during his journey?
Answer

Mr. Haani crossed 7 SlowRun Expresses during his journey.

Let's say that Mr. Haani travelled by SlowRun Express on Wednesday 10:00PM from Mumbai. The first train he would
have crossed is the one scheduled to arrive at Mumbai at 11:30 PM the same day i.e. the one that left Bangalore at 10:00
PM on last Sunday.

Also, he would have crossed the last train just before reaching Bangalore on Saturday.

Thus, Mr. Haani must have crossed 7 SlowRun Expresses during his journey.
Six cabins numbered 1-6 consecutively, are arranged in a row and are separated by thin dividers. These cabins must be
assigned to six staff members based on following facts.

1. Miss Shalaka's work requires her to speak on the phone frequently throughout the day.
2. Miss Shudha prefers cabin number 5 as 5 is her lucky number.
3. Mr. Shaan and Mr. Sharma often talk to each other during their work and prefers to have adjacent cabins.
4. Mr. Sinha, Mr. Shaan and Mr. Solanki all smoke. Miss Shudha is allergic to smoke and must have non-
smokers adjacent to her.
5. Mr. Solanki needs silence during work.

Can you tell the cabin numbers of each of them?


Answer

The cabins from left to right (1-6) are of Mr. Solanki, Mr. Sinha, Mr. Shaan, Mr. Sharma, Miss Shudha and Miss
Shalaka.

From (2), cabin number 5 is assigned to Miss Shudha.

As Miss Shudha is allergic to smoke and Mr. Sinha, Mr. Shaan & Mr. Solanki all smoke, they must be in cabin numbers
1, 2 and 3 not necessarily in the same order. Also, Miss Shalaka and Mr. Sharma must be in cabin 4 and 6.

From (3), Mr. Shaan must be in cabin 3 and Mr. Sharma must be in cabin 4. Thus, Miss Shalaka is in cabin 6.

As Mr. Solanki needs silence during work and Mr. Shaan is in cabin 3 who often talks to Mr. Sharma during work, Mr.
Solanki must be in cabin 1. Hence, Mr. Sinha is in cabin 2.

Thus, the cabins numbers are


1# Mr. Solanki,
2# Mr. Sinha,
3# Mr. Shaan,
4# Mr. Sharma,

Kumaraguru College, Coimbatore


5# Miss Shudha,
6# Miss Shalaka
SkyFi city is served by 6 subway lines - A, E, I, O, U and Z.

• When it snows, morning service on line E is delayed.


• When it rains or snows, service on the lines A, U and Z is delayed both morning and afternoon.
• When the temperature drops below 20 C, afternoon service is cancelled on either line A or line O, but not both.
• When the temperature rises above 40 C, afternoon service is cancelled on either line I or line Z, but not both.
• When service on line A is delayed or cancelled, service on line I is also delayed.
• When service on line Z is delayed or cancelled, service on line E is also delayed.

On February 10, it snows all day with the temperature at 18C. On how many lines service will be delayed or cancelled,
including both morning and afternoon?
SkyFi city is served by 6 subway lines - A, E, I, O, U and Z.

• When it snows, morning service on line E is delayed.


• When it rains or snows, service on the lines A, U and Z is delayed both morning and afternoon.
• When the temperature drops below 20 C, afternoon service is cancelled on either line A or line O, but not both.
• When the temperature rises above 40 C, afternoon service is cancelled on either line I or line Z, but not both.
• When service on line A is delayed or cancelled, service on line I is also delayed.
• When service on line Z is delayed or cancelled, service on line E is also delayed.

On February 10, it snows all day with the temperature at 18C. On how many lines service will be delayed or cancelled,
including both morning and afternoon?
In a certain game, if 2 wixsomes are worth 3 changs, and 4 changs are worth 1 plut, then 6 plutes are worth how many
wixsomes?

Answer

It is given that
2 wixsomes = 3 changs
8 wixsomes = 12 changs ----- (I)

Also, given that


4 changs = 1 plut
12 changs = 3 plutes
8 wixsomes = 3 plutes ----- From (I)

Therefore,
6 plutes = 16 wixsomes

In a certain year, the number of girls who graduated from City High School was twice the number of boys. If 3/4 of the
girls and 5/6 of the boys went to college immediately after graduation, what fraction of the graduates that year went to
college immediately after graduation?

Answer

Assume that number of boys graduated from City High School = B


Therefore, number of girls graduated from City High School = 2*B

It is given that 3/4 of the girls and 5/6 of the boys went to college immediately after graduation.
Hence, total students went to college
= (3/4)(2*B) + (5/6)(B)
= B * (3/2 + 5/6)
= (7/3)B

Fraction of the graduates that year went to college immediately after graduation

Kumaraguru College, Coimbatore


= [(7/3)B] / [3*B]
= 7/9

Therefore, the answer is 7/9

A mule and a donkey were carrying full sacks on their backs.

The mule started complaining that his load was too heavy. The donkey said to him "Why are you complaining? If you
gave me one of your sacks I'd have double what you have and if I give you one of my sacks we'd have an even amount."

How many sacks were each of them carrying? Give the minimal possible answer.
SubmittAnswer

The mule was carrying 5 sacks and the donkey was carrying 7 sacks.

Let's assume that the mule was carrying M sacks and the donkey was carrying D sacks.

As the donkey told the mule, "If you gave me one of your sacks I'd have double what you have."
D + 1 = 2 * (M-1)
D + 1 = 2M - 2
D = 2M - 3

The donkey also said, "If I give you one of my sacks we'd have an even amount."
D-1=M+1
D=M+2

Comparing both the equations,


2M - 3 = M + 2
M=5

Substituting M=5 in any of above equation, we get D=7

Hence, the mule was carrying 5 sacks and the donkey was carrying 7 sacks.
edTwo people enter a race in whick you run to a point and back. Person A runs 20 mph to and from the point. Person B
runs to the point going 10 mph and 30 mph going back.

Who came in first?


Submitted
Answer

Person A came in first.

Let's assume that the distance between start and the point is D miles.

Total time taken by Person A to finish


= (D/20) + (D/20)
= D/10
= 0.1D

Total time taken by Person B to finish


= (D/10) + (D/30)
= 2D/15
= 0.1333D

Thus, Person A is the Winner.

Kumaraguru College, Coimbatore


Alternatively (if you don't like mathematics ;)), analyse the situation as follow:

Note that initially speed of Person A (20 mph) was twice the speed of Person B (10 mph). Hence, when Person A (20
mph forward) reached the point, Person B (10 mph forward) was halfway. When Person A (20 mph back) finished,
Person B (still 10 mph forward) reached the point.

Thus, Person A wins the race and by that time Person B covers only half the distance, no matter how far the point is!!!
Mark ate half of a pizza on Monday. He ate half of what was left on Tuesday and so on. He followed this pattern for one
week.

How much of the pizza would he have eaten during the week?
Submitted
Answer

Mark would have ate 127/128 (99.22%) of the pizza during the week.

Mark ate half the pizza on Monday. On Tuesday, he would have ate half of the remaining pizza i.e. 1/4 of the original
pizza. Similarly, he would have ate 1/8 of the original pizza on Wednesday and so on for the seven days.

Total pizza Mark ate during the week is


= 1/2 + 1/4 + 1/8 + 1/16 + 1/32 + 1/64 + 1/128
= 127/128
= 99.22% of the original pizza
In the General meeting of "Friends Club", Sameer said, "The repairs to the Club will come to a total of Rs 3120 and I
propose that this amount should be met by the members, each paying an equal amount."

The proposal was immediately agreed. However, four members of the Club chose to resign, leaving the remaining
members to pay an extra Rs 26 each.

How many members did the Club originally have?


Answer

The Club originally had 24 members.

Assume that there were initially N members.

As 4 members resigned and remaining members paid Rs 26 each, it means that total amount of 4 members is equal to Rs
26 each from remaining (N-4) members. Thus,

4 * (3120 / N) = 26 * (N - 4)
12480 = 26N2 - 104N
26N2 - 104N - 12480 = 0

Solving the quadratic equation we get N=24.

Hence, the Club originally had 24 members.


Answer

The tank will be full in 16 minutes.

In one minute,
pipe A can fill 1/30 part of the tank.
pipe B can fill 1/24 part of the tank.
pipe C can empty 1/80 part of the tank.

Thus, the net water level in one minute is


= 1/30 + 1/24 - 1/80
= 15/240 part of the tank

Kumaraguru College, Coimbatore


Hence, the tank will be full in 240/15 i.e. 16 minutes.

A rich old Arab has three sons. When he died, he willed his 17 camels to the sons, to be divided as follows:

First Son to get 1/2 of the camels Second Son to get 1/3rd of the camels Third Son to get 1/9th of the camels.

The sons are sitting there trying to figure out how this can possibly be done, when a very old wise man goes riding by.
They stop him and ask him to help them solve their problem. Without hesitation he divides the camels properly and
continues riding on his way.

How did he do it?


Answer

The old man temporarily added his camel to the 17, making a total of 18 camels.

First son got 1/2 of it = 9

Second son got 1/3 of it = 6

Third son got 1/9 of it = 2

For a total of 17. He then takes his camel back and rides away......

There were two men standing on a street. The one says to the other, "I have 3 daughters, the product of their ages is 36.
What is the age of the OLDEST daughter?"

The second guy says, "I need more information." So, the first guy says, "The sum of their ages is equal to the address of
the house across the street."

The second guy looks at the address and says, "I still need more information." So, the first guy says, "My oldest
daughter wears a red dress."

Submit
User
Answer
There are 3 colored boxes - Red, Green and Blue. Each box contains 2 envelopes. Each envelope contains money - two
of them contain Rs. 25000 each, two of them contain Rs. 15000 each and remaining two contain Rs. 10000 each.

There is one statement written on the cover of each box.


* Red Box: Both, a red box and a blue box contain Rs. 10000 each.
* Green Box: Both, a green box and a red box contain Rs. 25000 each.
* Blue Box: Both, a blue box and a green box contain Rs. 15000 each.

Only one of the above 3 statements is true and the corresponding box contains the maximum amount.

Can you tell which box contains the maximum amount and how much?
Answer

Blue box contains the maximum amount Rs. 40000

As it is given that only one of the given 3 statements is true; assume in turn, each statement to be true & the other 2 false
and check whether the corresponding box contains the maximum amount.

Let's assume that the statement on the Blue box is true. Thus, the given 3 statements can be interpreted as
* Atmost one, a red box or a blue box contains Rs. 10000.

Kumaraguru College, Coimbatore


* Atmost one, a green box or a red box contains Rs. 25000.
* Both, a blue box and a green box contain Rs. 15000 each.

Going through all possible combinations, we can conclude that


Red Box : Rs. 10000 + Rs. 25000 = Rs. 35000
Green Box : Rs. 10000 + Rs. 15000 = Rs. 25000
Blue Box : Rs. 15000 + Rs. 25000 = Rs. 40000

You can test out for other two statements i.e. assuming Red box statement true and then Green box statement true. In
both the cases, other statements will contradict the true statement.

Sachin, Dravid and Ganguly played in a Cricket match between India and England.

• None of them scored more than 99 runs.


• If you add the digits of the runs scored by Sachin to his own score, you will get the runs scored by Dravid.
• If you reverse the digits of the runs scored by Dravid, you will get the runs scored by Ganguly.
• The total runs scored by them is 240.

Can you figure out their individual scores?


Answer

Sachin, Dravid and Ganguly scored 75, 87 and 78 respectively.

Sachin's score must be less than 86, otherwise Dravid's score would be more than 99. Also, he must have scored atleast
42 - incase Dravid and Ganguly scored 99 each.

Also, as none of them scored more than 99 and the total runs scored by them is 240; their individual scores must be
around 80.

Now, use trial-n-error method to solve the teaser.


Three men, including Gianni and three woman, including Sachi are in line at the BrentWood post office. Each has two
different pieces of business to conduct.

1. The first person is a woman.


2. Carlos wants to send an overnight package.
3. Lau is just ahead of Pimentelli who is the same sex as Lau.
4. Gianni is two places ahead of the person who wants to buy stamps.
5. Knutson - who is the opposite sex than Rendler - isn't the person who wanted to complain about a mail carrier.
6. The six people, not necessarily in the same order are - Anthony, Donna, the person who wants to fill out a
change-of-address form, the one who wants to buy a money order, the one who wants to send Airmail to Tibet
and the second person in the line.
7. The four tasks of the last two people in line, not necessarily in the same order are - sending books fourth class,
buying a money order, picking up a package and complaining about a mail carrier.
8. The person who wants to send books fourth class is just behind a person of the same sex.
9. Mary is just behind a person who wants to send an insured package.
10. The person who wants to send Airmail to Tibet is either two places ahead of or two places behind the one who
wants to add postage to his or her meter.
11. Anthony isn't two places behind the who wants to pickup a registered letter.
12. Toriseza is two places ahead of the person who wants to pick up a package.
13. Knutson isn't just ahead of the person who wants to send an item parcel post.

Can you figure out where each customer is in the line, his or her full name (one surname is Loti) and the two things he
or she wants to accomplish? Provide your answer is POSITION - FIRST NAME -
LAST NAME - BUSINESS format.
Submit Users
Answer Answer Answer (8)

Kumaraguru College, Coimbatore


A very TOUGH puzzle !!!

POS FIRST NAME LAST NAME BUSINESS

1 Sachi Loti • Fill Out a Change-of-Address Form


• Add Postage to Meter

2 Gianni Lau • Pick Up a Registered Letter


• Send an Item Parcel Post

3 Carlos Pimentelli • Overnight Package


• Send Airmail to Tibet

4 Donna Toriseza • Buy Stamps


• Send an Insured Package

5 Mary Knutson • Buy a Money Order


• Send Books fourth Class

6 Anthony Rendler • Complain About a Mail Carrier


• Pick Up a Package

Answer

A tough one.

It is obvious that C=1. Also, the maximum possible value of E is 7. Now, start putting possible values of D, E and R as
they occure frequently and use trial-n-error.

WORLD 53684

+ TRADE + 76042

------------ ------------

CENTER 129726
Answer

7.5 degrees

At 3:15 minute hand will be perfactly horizontal pointing towards 3. Whereas hour hand will be towards 4. Also, hour
hand must have covered 1/4 of angle between 3 and 4.

The angle between two adjacent digits is 360/12 = 30 degrees.

Hence 1/4 of it is 7.5 degrees.


An apple vendor has 1000 apples and 10 empty boxes. He asks his son to place all the 1000 apples in all the 10 boxes in
such a manner that if he asks for any number of apples from 1 to 1000, his son should be able to pick them in terms of
boxes.

How did the son place all the apples among the 10 boxes, given that any number of apples can be put in one box.
Answer

1, 2, 4, 8, 16, 32, 64, 128, 256, 489

Let's start from scratch.

Kumaraguru College, Coimbatore


• The apple vandor can ask for only 1 apple, so one box must contain 1 apple.
• He can ask for 2 apples, so one box must contain 2 apples.
He can ask for 3 apples, in that case box one and box two will add up to 3.
• He can ask for 4 apples, so one box i.e. third box must contain 4 apples.
• Now using box number one, two and three containing 1, 2 and 4 apples respectively, his son can give upto 7
apples. Hence, forth box must contain 8 apples.
• Similarly, using first four boxes containing 1, 2, 4 and 8 apples, his son can give upto 15 apples. Hence fifth
box must contain 16 apples.
• You must have noticed one thing till now that each box till now contains power of 2 apples. Hence the answer
is 1, 2, 4, 8, 16, 32, 64, 128, 256, 489. This is true for any number of apples, here in our case only upto 1000.

Answer

The sequence of letters from the lowest value to the highest value is TUSQRPV.

From (3), Q is the middle term.


___ ___ ___ _Q_ ___ ___ ___

From (4), there must be exactly 2 numbers between P and S which gives two possible positions.

[1] ___ _S_ ___ _Q_ _P_ ___ ___

[2] ___ ___ _S_ _Q_ ___ _P_ ___

From (1), the number of letters between U and Q must be same as the number of letters between S and R. Also, the
number of letters between them can be 1, 2 or 3.

Using trial and error, it can be found that there must be 2 letters between them. Also, it is possible only in option [2]
above.

[2] ___ _U_ _S_ _Q_ _R_ _P_ ___

From (2) V must be the highest and the remaining T must be the lowest number.

_T_ _U_ _S_ _Q_ _R_ _P_ _V_

Thus, the sequence of letters from the lowest value to the highest value is TUSQRPV.
A contractor had employed 100 labourers for a flyover construction task. He did not allow any woman to work without
her husband. Also, atleast half the men working came with their wives.

He paid five rupees per day to each man, four ruppes to each woman and one rupee to each child. He gave out 200
rupees every evening.

How many men, women and children were working with the constructor?
Answer

16 men, 12 women and 72 children were working with the constructor.

Let's assume that there were X men, Y women and Z children working with the constructor. Hence,

X + Y + Z = 100
5X + 4Y + Z = 200

Eliminating X and Y in turn from these equations, we get


X = 3Z - 200

Kumaraguru College, Coimbatore


Y = 300 - 4Z

As if woman works, her husband also works and atleast half the men working came with their wives; the value of Y lies
between X and X/2. Substituting these limiting values in equations, we get

if Y = X,
300 - 4Z = 3Z - 200
7Z = 500
Z = 500/7 i.e. 71.428

if Y = X/2,
300 - 4Z = (3Z - 200)/2
600 - 8Z = 3Z - 200
11Z = 800
Z = 800/11 i.e. 72.727

But Z must be an integer, hence Z=72. Also, X=16 and Y=12

There were 16 men, 12 women and 72 children working with the constructor.

Because cigars cannot be entirely smoked, a Bobo who collects cigar butts can make a cigar to smoke out of every 3
butts that he finds.

Today, he has collected 27 cigar butts. How many cigars will he be able to smoke?
Answer

13 not 12

He makes 9 originals from the 27 butts he found, and after he smokes them he has 9 butts left for another 3 cigars. And
then he has 3 butts for another cigar.

So 9+3+1=13
In a small town, there are three temples in a row and a well in front of each temple. A pilgrim came to the town with
certain number of flowers.

Before entering the first temple, he washed all the flowers he had with the water of well. To his surprise, flowers
doubled. He offered few flowers to the God in the first temple and moved to the second temple. Here also, before
entering the temple he washed the remaining flowers with the water of well. And again his flowers doubled. He offered
few flowers to the God in second temple and moved to the third temple. Here also, his flowers doubled after washing
them with water. He offered few flowers to the God in third temple.

There were no flowers left when pilgrim came out of third temple and he offered same number of flowers to the God in
all three temples.

What is the minimum number of flowers the pilgrim had initially? How many flower did he offer to each God?
Answer

The pilgrim had 7 flowers, initially and he offered 8 flowers to each God.

Assume that the pilgrim had X flowers initially and he offered Y flowers to each God.

From the above figure, there are (8X - 7Y) flowers when the pilgrim came out of the third temple. But it is given that
there were no flowers left when he came out of third temple. It means that
(8X - 7Y) = 0
8X = 7Y

The minimum values of X and Y are 7 and 8 respectively to satisfy above equation. Hence, the pilgrim had 7 flowers

Kumaraguru College, Coimbatore


and he offered 8 flowers to each God.

In general, the pilgrim had 7N flowers initially and he offered 8N flowers to each God, where N = 1, 2, 3, 4, .....

Answer

Cruz is Tanya's date.

As no two of them have the same number of preferred traits - from (1), exactly one of them has none of the preferred
traits and exactly one of them has all the preferred traits.

From (4) and (5), there are only two possibilities:


* Adam & Cruz both are tall and Bond & Dumbo both are fair.
* Adam & Cruz both are short and Bond & Dumbo both are dark.

But from (2), second possibility is impossible. So the first one is the correct possibility i.e. Adam & Cruz both are tall
and Bond & Dumbo both are fair.

Then from (3), Bond is short and handsome.

Also, from (1) and (2), Adam is tall and fair. Also, Dumbo is the person without any preferred traits. Cruz is Dark.
Adam and Cruz are handsome. Thus, following are the individual preferred traits:

Cruz - Tall, Dark and Handsome


Adam - Tall and Handsome
Bond - Handsome
Dumbo - None :-(

Hence, Cruz is Tanya's date.


Consider a game of Tower of Hanoi (like the one that you can play on BrainVista).

If the tower has 2 discs, the least possible moves with which you can move the entire tower to another peg is 3.

If the tower has 3 discs, the least possible moves with which you can move the entire tower to another peg is 7.

What is the least possible moves with which you can move the entire tower to another peg if the tower has N discs?
Submitted
Answer

There are number of ways to find the answer.

To move the largest disc (at level N) from one tower to the other, it requires 2(N-1) moves. Thus, to move N discs from
one tower to the other, the number of moves required is
= 2(N-1) + 2(N-2) + 2(N-3) + ..... + 22 + 21 + 20
= 2N - 1

For N discs, the number of moves is one more than two times the number of moves for N-1 discs. Thus, the recursive
function is
F(1) = 1
F(N) = 2*[F(N-1)] + 1
where N is the total number of discs

Also, one can arrive at the answer by finding the number of moves for smaller number of discs and then derive the
pattern.
For 1 disc, number of moves = 1

Kumaraguru College, Coimbatore


For 2 discs, number of moves = 3
For 3 discs, number of moves = 7
For 4 discs, number of moves = 15
For 5 discs, number of moves = 31

Thus, the pattern is 2N – 1


A boy found that he had a 48 inch strip of paper. He could cut an inch off every second.

How long would it take for him to cut 48 pieces? He can not fold the strip and also, can not stack two or more strips and
cut them together.
SubmiAnswer

47 seconds.

To get 48 pieces, the boy have to put only 47 cuts. i.e. he can cut 46 pieces in 46 seconds. After getting 46 pieces, he
will have a 2 inches long piece. He can cut it into two with just a one cut in 1 second. Hence, total of 47 seconds.tted by
: Kimi

The cricket match between India and Pakistan was over.

• Harbhajan scored more runs than Ganguly.


• Sachin scored more runs than Laxman but less than Dravid
• Badani scored as much runs as Agarkar but less than Dravid and more than Sachin.
• Ganguly scored more runs than either Agarkar or Dravid.

Each batsman scored 10 runs more than his immediate batsman. The lowest score was 10 runs. How much did each one
of them score
Answer

A simple one. Use the given facts and put down all the players in order. The order is as follow with Harbhajan, the
highest scorer and Laxman, the lowest scorer.

1. Harbhajan
2. Ganguly
3. Dravid
4. Badani, Agarkar
5. Sachin
6. Laxman

Also, as the lowest score was 10 runs. Laxman must have scored 10, Sachin 20, Badani & Agarkar 30 and so on.

1. Harbhajan - 60 runs
2. Ganguly - 50 runs
3. Dravid - 40 runs
4. Badani, Agarkar - 30 runs each
5. Sachin - 20 runs
6. Laxman - 10 runs

There are 10 statements written on a piece of paper:

1. At least one of statements 9 and 10 is true.


2. This either is the first true or the first false statement.
3. There are three consecutive statements, which are false.
4. The difference between the numbers of the last true and the first true statement divides the number, that is to be
found.
5. The sum of the numbers of the true statements is the number, that is to be found.
6. This is not the last true statement.

Kumaraguru College, Coimbatore


7. The number of each true statement divides the number, that is to be found.
8. The number that is to be found is the percentage of true statements.
9. The number of divisors of the number, that is to be found, (apart from 1 and itself) is greater than the sum of
the numbers of the true statements.
10. There are no three consecutive true statements.

Find the minimal possible number?


Submitted
Answer

The numebr is 420.

If statement 6 is false, it creates a paradox. Hence, Statement 6 must be true.

Consider Statement 2:

• If it is true, it must be the first true statement. Otherwise, it creates a paradox.


• If it is false, it must be the second false statement. Otherwise, it creates a paradox.

In both the cases, Statement 1 is false.

As Statement 1 is false, Statement 9 and Statement 10 both are false i.e. there are three consecutive true statements.

1 2 3 4 5 6 7 8 9 10
False - - - - True - - False False

Let\'s assume that Statement 3 is false i.e. there are no three consecutive false statements. It means that Statement 2 and
Statement 8 must be true, else there will be three consecutive false statements.

1 2 3 4 5 6 7 8 9 10
False True False - - True - True False False

Also, atleast two of Statements 4, 5 and 7 must be true as there are three consecutive true statements.

According to Statement 8, the number that is to be found is the percentage of true statements. Hence, number is either
50 or 60. Now if Statement 7 is true, then the number of each true statement divides the number, that is to be found. But
7 and 8 do not divide either 50 or 60. Hence, Statement 7 is false which means that Statement 4 and 5 are true. But
Statement 5 contradicts the Statement 8. Hence, our assumption that Statement 3 is false is wrong and Statement 3 is
true i.e. there are 3 consecutive false statements which means that Statement 8 is false as there is no other possibilities
of 3 consecutive false statements.

Also, Statement 7 is true as Statement 6 is not the last true statement.

1 2 3 4 5 6 7 8 9 10
False - True - - True True False False False

According to Statement 7, the number of each true statement divides the number, that is to be found. And according to
Statement 5, the sum of the numbers of the true statements is the number, that is to be found. For all possible
combinations Statement 5 is false.

There 3 consecutive true statements. Hence, Statement 2 and Statement 4 are true.

1 2 3 4 5 6 7 8 9 10

Kumaraguru College, Coimbatore


False True True True False True True False False False

Now, the conditions for the number to be found are:

1. The numebr is divisible by 5 (Statement 4)


2. The numebr is divisible by 2, 3, 4, 6, 7 (Statement 7)
3. The number of divisors of the number, that is to be found, (apart from 1 and itself) is not greater than the sum
of the numbers of the true statements. (Statement 9)

The minimum possible number is 420.

The divisors of 420, apart from 1 and itself are 2, 3, 4, 5, 6, 7, 10, 12, 14, 15, 20, 21, 28, 30, 35, 42, 60, 70, 84, 105, 140,
210. There are total of 22 divisors. Also, the sum of the numbers of the true statements is 22 (2+3+4+6+7=22), which
satisfies the third condition.
Ankit and Tejas divided a bag of Apples between them.

Tejas said, "It's not fair! You have 3 times as many Apples I have." Ankit said, "OK, I will give you one Apple for each
year of your age." Tejas replied, "Still not fair. Now, you have twice as many Apples as I have." "Dear, that's fair
enough as I am twice older than you.", said Ankit.

Ankit went to Kitchen to drink water. While Ankit was in Kitchen, Tejas took apples from Ankit's pile equal to Ankit's
age.

Who have more apples now?


Answer

At the end, Ankit and Tejas, both have the same number of apples.

Let's assume that initially Tejas got N apples and his age is T years. Hence, initially Ankit got 3N apples and his age is
2T years.

Operation Ankit's Apples Tejas's Apples

Initially 3N N

Ankit gave T apples to Tejas


3N - T N+T
(equals age of Tejas)

Tejas took 2T apples from Ankit's pile


3N - 3T N + 3T
(equals age of Ankit)

It is given that after Ankit gave T apples to Tejas, Ankit had twice as many apples as Tejas had.
3N - T = 2*(N + T)
3N - T = 2N + 2T
N = 3T

From the table, at the end Ankit have (3N - 3T) apples and Tejas have (N + 3T) apples. Substituting N = 3T, we get
Ankit's apples = 3N - 3T = 9T - 3T = 6T
Tejas's apples = N + 3T = 3T + 3T = 6T

Thus, at the end Ankit and Tejas, both have the same number of apples.

On evey Sunday Amar, Akbar and Anthony lunch together at Preetam-Da-Dhaba where they order lassi based on
following facts.

Kumaraguru College, Coimbatore


1. Unless neither Amar nor Akbar have lassi, Anthony must have it.
2. If Amar does not have lassi, either Akbar or Anthony or both have it.
3. Anthony has lassi only if either Amar or Akbar or both have it.
4. Akbar and Anthony never have lassi together.

Who order(s) lassi?


Answer

Amar and Anthony both have lassi whereas Akbar never does.

Fact (2) can be alternatively stated that "either Amar or Akbar or Anthony must have lassi".

From Fact (3), it can be infered that either Amar or Akbar must have lassi.

Now, from Fact (1), it is apparent that Anthony too must have lassi. But according to Fact (4), Akbar cannot have lassi
when Anthony does.
Answer

Start with ZBG and ZBGJ. It should be either "the/then" or "you/your" combination as they appear more.

BRWQHLFKW HJK QIBWK

obstacles are those

QICEDW ZBG WKK MIKE

things you see when

ZBG QHSK ZBGJ KZKW

you take your eyes

B U U Z B G J D B H F W.

o f f y o u r g o a l s.
Answer

It is obvious that between 5 O'clock and 6 O'clock the hands will not be exactly opposite to each other. It is also obvious
that the hands will be opposite to each other just before 5 O'clock. Now to find exact time:

The hour hand moves 1 degree for every 12 degrees that the minute hand moves. Let the hour hand be X degree away
from 5 O'clock. Therefore the minute hand is 12X degree away from 12 O'clock.

Therefore solving for X

Angle between minute hand and 12 O'clock + Angle between 12 O'clock and 4 O'clock + Angle between 4 O'clock and
hour hand = 180
12X + 120 + (30-X) = 180
11X = 30
Hence X = 30/11 degrees
(hour hand is X degree away from 5 O'clock)

Kumaraguru College, Coimbatore


Now each degree the hour hand moves is 2 minutes.

Therefore minutes are


= 2 * 30/11
= 60/11
= 5.45 (means 5 minutes 27.16 seconds)

Therefore the exact time at which the hands are opposite to each other is
= 4 hrs. 54 min. 32.74 seconds

Ali Baba had four sons, to whom he bequeathed his 39 camels, with the proviso that the legacy be divided in the
following way :

The oldest son was to receive one half the property, the next a quarter, the third an eighth and the youngest one tenth.
The four brothers were at a loss as how to divide the inheritance among themselves without cutting up a camel, until a
stranger appeared upon the scene.
Dismounting from his camel, he asked if he might help, for he knew just what to do. The brothers gratefully accepted
his offer.

Adding his own camel to Ali Baba's 39, he divided the 40 as per the will. The oldest son received 20, the next 10, the
third 5 and the youngest 4. One camel remained : this was his, which he mounted and rode away.

Scratching their heads in amazement, they started calculating. The oldest thought : is not 20 greater than the half of 39?
Someone must have received less than his proper share ! But each brother discovered that he had received more than his
due. How is it possible?
Answer

They took their percentages from 40 and not from 39, so they got more than their share.

The oldest son got 1/2 of 40 = 20 which is 0.5 more


The second son got 1/4 of 40 = 10 which is 0.25 more
The third son got 1/8 of 40 = 5 which is 0.125 more
The youngest son got 1/10 of 40 = 4 which is 0.1 more

And the stranger got 1/40 of 40 = 1 which is 0.025 more (As he is not supposed to get anything)

All these fractions add to = 0.5 + 0.25 + 0.125 + 0.1 + 0.025 = 1 which stranger took away.
There is a family party consisting of two fathers, two mothers, two sons, one father-in-law, one mother-in-law, one
daughter-in-law, one grandfather, one grandmother and one grandson.

What is the minimum number of persons required so that this is possible?


Answer

There are total 2 couples and a son. Grandfather and Grand mother, their son and his wife and again their son. So total 5
people.

Grandfather, Grandmother
|
|
Son, wife
|
|
Son

A man went into a fast food restaurant and ate a meal costing Rs. 105, giving the accountant a Rs. 500 note. He kept the
change, came back a few minutes later and had some food packed for his girl friend. He gave the accountant a Rs. 100

Kumaraguru College, Coimbatore


note and received Rs. 20 in change. Later the bank told the accountant that both the Rs. 500 and the Rs. 100 notes were
counterfeit.

How much money did the restaurant lose? Ignore the profit of the food restaurant.
Answer

He lost Rs.600

First time restaurant has given food worth Rs.105 and Rs. 395 change. Similarly second time, food worth Rs.80 and
Rs.20 change. Here, we are not considering food restaurant profits.

SLIDE

- DEAN

---------

3651
Each of seven digits from 0-9 are represented by a different letter above such that the subtraction is true.

What word represents 3651?


Answer

3651 represents LENS.

Let's assign possible values to each letter and then use trial-n-error.

S must be 1.

Then D (under L) must be greater than 5. If D is 6, then L is 0. But then A must be 0 or 1 which is impossible. Hence,
the possible values of D are 7, 8 or 9.

N must be E + 1. Also, D must be A + 5 as the possible values of D are 7, 8 or 9, D can not be (10+A) + 5.

Now using trial-n-error, we get S=1, I=2, L=3, A=4, N=5, E=6 and D=9

S L I D E 1 3 2 9 6

- D E A N - 9 6 4 5

-------------- --------------

3 6 5 1 L E N S
Hence, 3651 represents LENS.
Adam, Burzin, Clark and Edmund each live in an apartment. Their apartments are arranged in a row numbered 1 to 4
from left to right. Also, one of them is the landlord.

1. If Clark's apartment is not next to Burzin's apartment, then the landlord is Adam and lives in apartment 1.
2. If Adam's apartment is right of Clark's apartment, then the landlord is Edmund and lives in apartment 4.
3. If Burzin's apartment is not next to Edmund's apartment, then the landlord is Clark and lives in apartment 3.
4. If Edmund's apartment is right of Adam's apartment, then the landlord is Burzin and lives in apartment 2.

Who is the landlord?

Answer

Clark is the landlord.

Kumaraguru College, Coimbatore


Assume each statement true, one at a time and see that no other statement is contradicted.

Let's assume that Statement (1) is true. Then, Adam is the landlord and lives in apartment 1. Also, other three's
apartments will be on the right of his apartment - which contradicts Statement (4) i.e. If Edmund's apartment is right of
Adam's apartment, then the landlord is Burzin. Thus, Adam is not the landlord.

Let's assume that Statement (2) is true. Then, Edmund is the landlord and lives in apartment 4. Also, other three's
apartments will be on the left of his apartment - which again contradicts Statement (4) i.e. If Edmund's apartment is
right of Adam's apartment, then the landlord is Burzin. Thus, Edmund is not the landlord either.

Let's assume that Statement (3) is true. Then, Clark is the landlord and lives in apartment 3. It satisfies all the statements
for
(1) Adam - (2) Edmund - (3) Clark - (4) Burzin

Hence, Clark is the landlord.

Similarly, you can assume Statement (4) true and find out that it also contradicts.
Answer

J is the married man.

Note that a person's sister-in-law may be the wife of that person's brother or the sister of that person's spouse.

There are 2 cases:

1. If B's legal spouse is J, then J's sibling must be P and P's sister-in-law must be B.
2. If B's legal spouse is P, then P's sister-in-law must be J and J's sibling must be B.

It is given that B's legal spouse and J's sibling are of the same sex. Also, it is obvious that P's sister-in-law is female.
Then, B's legal spouse and J's sibling both must be males.

B's spouse J's sibling P's sister-in-law

(male) (male) (female)

------------------------------------------------------

Case I J P B

Case II P B J

Case II is not possible as B & P are married to each other and both are male. Hence, J is the married man.
Answer

The formula to find number of diagonals (D) given total number of vertices or sides (N) is

N * (N - 3)

D = -----------

Using the formula, we get


1325 * 2 = N * (N - 3)
N2 - 3N - 2650 = 0

Solving the quadratic equation, we get N = 53 or -50

Kumaraguru College, Coimbatore


It is obvious that answer is 53 as number of vertices can not be negative.

Alternatively, you can derive the formula as triange has 0 diagonals, quadrangel has 2, pentagon has 5, hexagon has 9
and so on......

Hence the series is 0, 0, 0, 2, 5, 9, 14, ........ (as diagram with 1,2 or 3 vertices will have 0 diagonals).

Using the series one can arrive to the formula given above.
Answer

44

36 of the cubes have EXACTLY 2 of their sides painted black, but because a cube with 3 of its sides painted black has 2
of its sides painted black, you must also include the corner cubes. This was a trick question, but hopefully the title of the
puzzle tipped you off to this.
Answer

It takes 5 moves to make the triangle with 5 rows point the other way.

0 = a coin that has not been moved.


X = the old position of the moved coin
8 = the new position of the moved coin.

________X
_______X X
____8 0 0 0 8
_____0 0 0 0
____X 0 0 0 X
_______8 8
________8

For traingle of any number of rows, the optimal number of moves can be achieved by moving the vertically symmetrical
coins i.e. by moving same number of coins from bottom left and right, and remaining coins from the top.

For a triangle with an odd number of rows, the total moves require are :
(N2/4) - (N-4) Where N = 4, 6, 8, 10, ...

For a triangle with even number of rows, the total moves require are :
((N2-1)/4) - (N-4) Where N = 5, 7, 9, 11, ...

Thanks to Alex Crosse for submitting above formulas.


Answer

Each envelope contains the money equal to the 2 raised to the envelope number minus 1. The sentence "Each envelope
contains the least number of bills possible of any available US currency" is only to misguide you. This is always
possible for any amount !!!

One more thing to notice here is that the man must have placed money in envelopes in such a way that if he bids for any
amount less than $25000, he should be able to pick them in terms of envelopes.

First envelope contains, 20 = $1


Second envelope contains, 21 = $2
Third envelope contains, 22 = $4
Fourth envelope contains, 23 = $8 and so on...

Hence the amount in envelopes are $1, $2, $4, $8, $16, $32, $64, $128, $256, $512, $1024, $2048, $4096, $8192,

Kumaraguru College, Coimbatore


$8617

Last envelope (No. 15) contains only $8617 as total amount is only $25000.

Now as he bids for $8322 and gives envelope number 2, 8 and 14 which contains $2, $128 and $8192 respectively.

Envelope No 2 conrains one $2 bill


Envelope No 8 conrains one $100 bill, one $20 bill, one $5 bill, one $2 bill and one $1 bill
Envelope No 14 conrains eighty-one $100 bill, one $50 bill, four $10 bill and one $2 bill

Hence the auctioneer will find one $1 bill in the envelopes.

Answer

The minute and the hour hand meet 11 times in 12 hours in normal watch i.e. they meet after every
= (12 * 60) / 11 minutes
= 65.45 minutes
= 65 minutes 27.16 seconds

But in our case they meet after every 65 minutes means the watch is gaining 27.16 seconds.
Answer

The number is 45, simply because


45 = 5 * (4 + 5)
How does one find this number?

Let T be the digit in the tens place and U be the digit in the units place. Then, the number is 10*T + U, and the sum of
its digits is T + U.

The following equation can be readily written:


10*T + U = 5*(T + U) or
10*T + U = 5*T + 5*U or
5*T = 4*U

Thus, T / U = 4 / 5

Since T and U are digits, T must be 4 and U must be 5.


There are six boxes containing 5, 7, 14, 16, 18, 29 balls of either red or blue in colour. Some boxes contain only red
balls and others contain only blue.

One sales man sold one box out of them and then he says, "I have the same number of red balls left out as that of blue."

Which box is the one he solds out?


Answer

Total no of balls = 5 + 7 + 14 + 16 + 18 + 29 = 89

Total number of balls are odd. Also, same number of red balls and blue balls are left out after selling one box. So it is
obvious that the box with odd number of balls in it is sold out i.e. 5, 7 or 29.

Now using trial and error method,


(89-29) /2 = 60/2 = 30 and
14 + 16 = 5 + 7 + 18 = 30

So box with 29 balls is sold out.

Answer

Kumaraguru College, Coimbatore


47 Chocolates and 16 Friends

Let's assume that there are total C chocolates and F friends.

According to first case, if she gives 3 chocolates to each friend, one friend will get only 2 chocolates.
3*(F - 1) + 2 = C

Similarly, if she gives 2 chocolates to each friends, she will left with 15 chocolates.
2*F + 15 = C

Solving above 2 equations, F = 16 and C = 47. Hence, Ekta got 47 chocolates and 16 friends
Pooja and Esha met each other after long time. In the course of their conversation, Pooja asked Esha her age. Esha
replied, "If you reverse my age, you will get my husbund's age. He is of course older than me. Also, the difference
between our age is 1/11th of the sum of our age."

Can you help out Pooja in finding Esha's age?


Answer

Esha's age is 45 years.

Assume that Esha's age is 10X+Y years. Hence, her hunsbands age is (10Y + X) years.

It is given that difference between their age is 1/11th of the sum of their age. Hence,
[(10Y + X) - (10X + Y)] = (1/11)[(10Y + X) + (10X + Y)]
(9Y - 9X) = (1/11)(11X + 11Y)
9Y - 9X = X + Y
8Y = 10X
4Y = 5X

Hence, the possible values are X=4, Y=5 and Esha's age is 45 years.
A fish had a tail as long as its head plus a quarter the lenght of its body. Its body was three-quarters of its total length.
Its head was 4 inches long.

What was the length of the fish?


Submitted

The fish is 128 inches long.

It is obvious that the lenght of the fish is the summation of lenghts of the head, the body and the tail. Hence,
Fish (F) = Head (H) + Body (B) + Tail (T)

But it is given that the lenght of the head is 4 inches i.e. H = 4. The body is three-quarters of its total length i.e. B =
(3/4)*F. And the tail is its head plus a quarter the lenght of its body i.e. T = H + B/4. Thus, the equation is
F=H+B+T
F = 4 + (3/4)*F + H + B/4
F = 4 + (3/4)*F + 4 + (1/4)*(3/4)*F
F = 8 + (15/16)*F
(1/16)*F = 8
F = 128 inches

Thus, the fish is 128 inches long.

Assume that you have just heard of a scandal and you are the first one to know. You pass it on to four person in a matter
of 30 minutes. Each of these four in turn passes it to four other persons in the next 30 minutes and so on.

How long it will take for everybody in the World to get to know the scandal?

Assume that nobody hears it more than once and the population of the World is approximately 5.6 billions.

Kumaraguru College, Coimbatore


Answer

Everybody in the World will get to know the scandal in 8 hours.

You came to know of a scandal and you passed it on to 4 persons in 30 minutes. So total (1+4) 5 persons would know
about it in 30 minutes.

By the end of one hour, 16 more persons would know about it. So total of (1+4+16) 21 persons would know about it in
one hour.

Similarly, the other (1+4+16+64) persons would have know about it in one and a half hours. (1+4+16+64+256) persons
would have know about it in two hours and so on...

It can be deduced that the terms of the above series are the power of 4 i.e. 4^0, 4^1, 4^2, 4^3 and so on upto (2N+1)
terms. Also, the last term would be 4^2N where N is the number of hours.

Sum of the above mentioned series = [4^(2N+1)-1]/3

The sum of the series must be 5.6 billions. Hence, equating the sum of the series with 5.6 billions, we get N=8 hours.

Scandals travel FAST !!!


A B C

E F G

I
Each of the digits from 1 to 9 is represented by a different letter above. Also, A + B + C = C + D + E = E + F + G = G +
H + I = 13

Which digit does E represent?


Answer

E represents 4.

Find out all possible groups of three different numbers that add up to 13 and arrange them according to given condition.

If one number is 9, it must go with 1 and 3.


If one number is 8, it must go with either 1 and 4 or 2 and 3.
If one number is 7, it must go with either 1 and 5 or 2 and 4.
If one number is 6, it must go with either 2 and 5 or 3 and 4.

It is clear that 9 must go with 1 and 3. Also, no digit may be used in more than two sums. Hence, there are 2 cases:
Case I: If 8 goes with 1 and 4, then 7 goes with 2 and 4, then 6 goes with 2 and 5.
Case II: If 8 goes with 2 and 3, then 7 goes with 2 and 4, then 6 goes with 3 and 4.

But in case II, 3 is used in three sums. Hence, Case I is correct. And the possible arrangements are:

Kumaraguru College, Coimbatore


9 3 1 5 6 2

8 7

4 7 2 4 8 1

6 3

5 9
Thus, E must be 4.
A, B and C are three points on a straight line, not necessarily equidistant with B being between A and C. Three
semicircles are drawn on the same side of the line with AB, BC and AC as the diameters. BD is perpendicular to the line
ABC, and D lies on the semicircle AC.

If the funny shaped diagram between the three semicircles has an area of 1000 square cms, find the length of BD.

Submit Users BrainVista


P
Answer Answer (33) Answer

Brain Teaser No : 00660

Gomzi has 3 timepieces in his house - a wall clock, an alarm clock and a wristwatch. The wristwatch is always
accurate, whereas the wall clock gains 2 minutes everyday and the alarm clock loses 2 minutes everyday.

At exactly midnight last night, all three watches were showing the same time.

If today is 25 July 2003, then on which date all three clocks will show the same time again?

Answer

All three clocks will show the same time again on midnight between 19 July 2004 and 20 July 2004.

A clock finishes on round in 12*60 i.e. 720 minutes.

If a clock gains 2 minutes everyday, then it would be 720 minutes ahead after 360 days. Thus, after 360 days, it will
show the same time again.

Similary, if a clock loses 2 minutes everyday, then it would be 720 minutes behind after 360 days. Thus, after 360 days,
it will show the same time again.

Thus, after 360 days all three clocks will show the same time again i.e. midnight between 19 July 2004 and 20 July
2004.
You have 9 marbles. 8 marbles weigh 1 ounce each, & one marble weighs 1.5 ounces. You are unable to determine
which is the heavier marble by looking at them. You have a weighing scale that consists of 2 pans, but the scale is only
good for 2 total weighings.

How can you determine which marble is the heaviest one using the scale & in 2 weighings?
Answer

Divide 9 marbles into 3 groups of 3 marbles each.

Take any 2 groups and place them on each pan. If they balance, remove the marbles from the pans, & place any 2 of the
marbles from the remaining unweighed group on the pans, 1 on each pan.

Kumaraguru College, Coimbatore


If one is heavier, it is the heavier marble, but if they balance, the remaining unweighed marble is the heavier one.

If your first weighing does not balance, remove the marbles from the lighter pan, & place 1 marble on each pan from the
heavier pan. The heavier 1 is the 1.5 ounce marble, but if they balance, then the marble from the heavy pan from the
first weighing that was not weighed in the second weighing is the heavy 1.
Once a week a wagon driver leaves his hut and drives his wagon to the river dock to pick up supplies for his town. At
4:05 PM, one-fifth of the way to the dock, he passes the Temple. At 4:15 PM, one-third of the way, he passes the
Preetam-Da-Dhabaa.

At what time does he reached the dock?

Answer

5:05 PM

At 4:05 PM, the wagon driver passes the temple, one-fifth of the way to the dock. Also, at 4:15 PM, he passes the
Preetam-Da-Dhabaa, one-third of the way. Thus, he travels 2/15 (1/3 - 1/5) of the distance in 10 minutes.

At 4:15 PM, he has already travelled 1/3 of the distance. Thus 2/3 of the way is remaining, which can be travelled in
= ( (2/3) * 10 ) / (2/15)
= 50 minutes

At 4:15, he was at Preetam-Da-Dhabaa.and remaining way will take 50 more minutes. Hence, the driver will reach at
5:05 PM to the dock.
Answer

Put all the given information into the table structure as follow:

North Road South Road East Road West Road

Mr. North No

Mr. South No

Mr. East No No No

Mr. West No No

Now from table, two things are obvious and they are:

• Mr.North took the South Road


• Mr.East took the North Road

Put this information into the table, Also keep in mind that the prisoners head towards different directions after escaping.

North Road South Road East Road West Road

Mr. North No YES No No

Mr. South No No

Mr. East YES No No No

Mr. West No No No

Now from the table:

Kumaraguru College, Coimbatore


• Mr.West took the East Road
• Mr.South took the West Road

So the answer is:

• Mr.North took the South Road


• Mr.South took the West Road
• Mr.East took the North Road
• Mr.West took the East Road

Shahrukh speaks truth only in the morning and lies in the afternoon, whereas Salman speaks truth only in the afternoon
and lies in the morning.

A says that B is Shahrukh.

Is it morning or afternoon and who is A - Shahrukh or Salman?


Answer

It is Afternoon and A can be Salman or Shahrukh. If A is Salman, he is speaking truth. If A is Shahrukh, he is lying.

Want to confirm it? Consider following 4 possible answers and check for its truthness individually.

1. It is Morning and A is Shahrukh


2. It is Morning and A is Salman
3. It is Afternoon and A is Shahrukh
4. It is Afternoon and A is Salman

A rich man died. In his will, he has divided his gold coins among his 5 sons, 5 daughters and a manager.

According to his will: First give one coin to manager. 1/5th of the remaining to the elder son. Now give one coin to the
manager and 1/5th of the remaining to second son and so on..... After giving coins to 5th son, divided the remaining
coins among five daughters equally.

All should get full coins. Find the minimum number of coins he has?
Answer

We tried to find out some simple mathematical method and finally we wrote small C program to find out the answer.
The answer is 3121 coins.

Here is the breakup:


First son = 624 coins
Second son = 499 coins
Third son = 399 coins
Forth son = 319 coins
Fifth son = 255 coins
Daughters = 204 each
Manager = 5 coins
There is a grid of 20 squares by 10 squares. How many different rectangles are possible?

Note that square is a rectangle.


Answer

11550

The Generic solution to this is:

Kumaraguru College, Coimbatore


Total number of rectangles = (Summation of row numbers) * (Summation of column numbers)

Here there are 20 rows and 10 columns or vice versa. Hence, total possible rectangles
= ( 20 + 19 + 18 + 17 + 16 + .... + 3 + 2 + 1 ) * ( 10 + 9 +8 + 7 + .... + 3 + 2 + 1)
= ( 210 ) * (55)
= 11550

Hence, total 11,550 different rectangles are possible.

If you don't believe it, try formula on some smaller grids like 4x2, 3x2, 3x3 etc...
If A+B=C, D-C=A and E-B=C, then what does D+F stands for? Provide your answer in letter terms as well as in
number terms.
Submitted by : David
Answer

J or 10

A simple one.

Assume that each character represents the number equivalent to the position in the alphabet i.e. A = 1, B = 2, C = 3, D =
4 and so on. Now let's check our assumption.

A + B = C i.e. 1 + 2 = 3
D - C = A i.e. 4 - 3 = 1
E - B = C i.e. 5 - 2 = 3

Thus, our assumption was Correct. Hence, D + F = J i.e. 4 + 6 = 10


A woman took a certain number of eggs to the market and sold some of them.

The next day, through the industry of her hens, the number left over had been doubled, and she sold the same number as
the previous day.

On the third day the new remainder was tripled, and she sold the same number as before.

On the fourth day the remainder was quadrupled, and her sales the same as before.

On the fifth day what had been left over were quintupled, yet she sold exactly the same as on all the previous occasions
and so disposed of her entire stock.

What is the smallest number of eggs she could have taken to market the first day, and how many did she sell daily?
Note that the answer is not zero.
Submitted
Answer

She took 103 eggs to market on the first day and sold 60 eggs everyday.

Let's assume that she had N eggs on the first day and she sold X eggs everyday. Putting down the given information in
the table as follow.

Days Eggs at the start of the day Eggs Sold Eggs Remaining

Day 1 N X N-X

Day 2 2N-2X X 2N-3X

Day 3 6N-9X X 6N-10X

Day 4 24N-40X X 24N-41X

Kumaraguru College, Coimbatore


Day 5 120N-205X X 120N-206X

It is given that she disposed of her entire stock on the fifth day. But from the table above, the number of eggs remaining
are (120N-206X). Hence,
120N - 206X = 0
120N = 206X
60N = 103X

The smallest value of N and X must be 103 and 60 respectively. Hence, she took 103 eggs to market on the first day and
sold 60 eggs everyday.
John lives in "Friends Society" where all the houses are in a row and are numbered sequentially starting from 1. His
house number is 109.

Jessy lives in the same society. All the house numbers on the left side of Jessy's house add up exactly the same as all the
house numbers on the right side of her house.

What is the number of Jessy's house? Find the minimal possible answer.
Answer

There are 288 houses and Jessy's house number is 204.

Let's assume that in the "Friends Society" there are total N houses numbered from 1 to N and Jessy's house number is X.

Now it is given that all the house numbers on the left side of Jessy's house add up exactly the same as all the house
numbers on the right side of her house. Hence,
1 + 2 + 3 + ..... + (X-1) = (X+1) + (X+2) + (X+3) + ..... + N

Both the sides of the above equations are in A.P. Hence, using A.P. summation formaula,

[(X-1)/2][2*(1) + (X-1-1)] = [(N-X)/2][2*(X+1) + (N-X-1)]


[X-1][(2) + (X-2)] = [N-X][(2X+2) + (N-X-1)]
(X-1)(X) = (N-X)(N+X+1)
X2 - X = N2 + NX + N - NX - X2 - X
X 2 = N2 + N - X 2
2X2 = N2 + N
X2 = (N2 + N)/2
X2 = N(N+1)/2

Now, using Trial and Error method to find values of N and X such that above equation is satisfied, we get

1. N = 8, X = 6
2. N = 49, X = 35
3. N = 288, X = 204
4. N = 1681, X = 1189
5. N = 9800, X = 6930

But we require minimal possible answer and it is given that John's house number is 109. It means that there are atleast
109 houses. Hence, first two are not possible. And the answer is : there are 288 houses and Jessy's house number is 204.
Makayla had $1.19 in change. None of the coins was a dollar.

Nicole ask her for change for a dollar, but Makayla could not make change.

What coins did she have?


Submitted
Answer

As it is given that Makayla had $1.19, it means she would have four pennies. Now, the remaining $1.15 in coins must

Kumaraguru College, Coimbatore


not add up for exactly a dollar. Therefore she would not have 4 quarters or 2 quarters and 5 dimes. But she would have
either 1 quarter or 3 quarters. Hence, there are 2 solutions.

Solution I
1 Quarter, 9 Dimes, 4 Pennies (0.25 + 0.90 + 0.04 = $1.19)

Solution II
3 Quarters, 4 Dimes, 4 Pennies (0.75 + 0.40 + 0.04 = $1.19)
A group of friends went on a holiday to a hill station. It rained for 13 days. But when it rained in the morning, the
afternoon was lovely. And when it rained in the afternoon, the day was preceded by clear morning.

Altogether there were 11 very nice mornings and 12 very nice afternoons. How many days did their holiday last?
Answer

The holiday last for 18 days.

Let's assume the number of days as follows:


Rain in the morning and lovely afternoon = X days
Clear morning and rain in the afternoon = Y days
No rain in the morning and in the afternoon = Z days

Number of days with rain = X + Y = 13 days


Number of days with clear mornings = Y + Z = 11 days
Number of days with clear afternoons = X + Z = 12 days

Solving above 3 equations, we get X = 7, Y = 6 and Z = 5

Hence, total number of days on holiday = 18 days

Answer

See the pattern of the Y. AY * Y = NLY i.e. Y is multiplied by Y and the last digit of the answer is also Y. Thus, the
value of Y would be 5 or 6.

Also,
H=0 as L - H = L
P = 2N as P - N = N
L - Y = P = 2N
E-L=p

Let's find out the minimum possible values. If N=1, then P=2, Y=5, L=7 and E=9. Note that the value of Y can not be 6
as it makes L=8 and E=10 which is not possible. Hence, Y=5, N=1, P=2, L=7, E=9, H=0

Now, using trial-n-error or rather solving F*AY=PNH, we get F=6 and A=3.

Kumaraguru College, Coimbatore


565 YFY

----------- -----------

35|19775 AY|NELLY

|175 |NLY

----------- -----------

227 PPL

210 PNH

----------- -----------

175 NLY

175 NLY

----------- -----------

000 000

Answer

TRILOK have 5 children.

Name of the person and number of his children are related by some pattern.

Assign each vowel following values.


A=0 E=1 I=2 O=3 U=4

The number of children to any person is the sum of the values represented by vowels in his name.
RATISH = 0(A) + 2(I) = 2
OM = 3(O) = 3
AMIT = 0(A) + 2(I) = 2
ASHOK = 0(A) + 3(O) = 3
TRILOK = 2(I) + 3(O) = 5

Hence, TRILOK have 5 children.

Major Jasbir is forming five-person Special Task Group. The group must contain one leader, two bomb-experts and two
soldiers.

P, Q and R are possible bomb-experts. R, S and T are possible leaders. U, V and W are possible soldiers. Also, P and R
prefers to work with each other in the same team. T prefers to work only if V works.

How many different possible Groups, Major Jasbir can make?


Answer

Major Jasbir can make 8 different possible groups.

As 2 bomb-experts to be selected from the given 3 and also P & R prefers to work together, PR must be there in all the
possible Groups. Also, T prefers to work only if V works. It doesn't mean that V won't work without T.

Kumaraguru College, Coimbatore


Hence, possible groups are:
PR - S - UV
PR - S - VW
PR - S - WU

PR - T - UV
PR - T - VW

PQ - R - UV
PQ - R - VW
PQ - R - WU

Hence, there 8 different groups are possible.


The secret agent X emailed some code to his head office. They are "RADAR, LEVEL, ROTOR, REDIVIDER,
MOTOR". But four of these five words have something in common and one is fake.

Can you tell which one is fake? Ignore the fact that four of the code-words are of the same length.
Answer

The fake code-word is MOTOR.

All the code-words except MOTOR are Palindromes.

Answer

300 viewers watch all three channels.

Let's assume that total X viewers watch all three channels.

total viewers who watch only Moon Plus and Mony = 500-X
total viewers who watch only Moon Plus and Mee TV = 800-X
total viewers who watch only Mony and Mee TV = 1000-X

total viewers who watch only Moon Plus


= 1500 - (500-X) - (800-X) - X
= 200 + X

total viewers who watch only Mony


= 2000 - (500-X) - (1000-X) - X
= 500 + X

total viewers who watch only Mee TV


= 2500 - (1000-X) - (800-X) - X
= 700 + X

We know that total viewers are 4000. Summing up all 7 values,


X + (500 - X) + (800 - X) + (1000 - X) + (200 + X) + (500 + X) + (700 + X) = 4000
X + 3700 = 4000
X = 300

Hence, total 300 viewers watch all three channels.


A man was looking at a portrait. Someone asked him, "Whose picture are you looking at?"

He replied, pointing at the portrait: "Brothers and sisters have I none, but this man's son is my father's son."

Now whose picture is the man looking at?


Answer

Kumaraguru College, Coimbatore


The man is looking at his FATHER's portrait.

"my father's son" is the man himself as he do not have any brothers and sisters. So the statement reduces to "this man's
son is myself." Now it is clear that the portrait is of his father.
Given the following facts:

1. Dinesh is younger than Farukh and older than Gurmit.


2. Jatin is younger than Chandu and older than Eshrat.
3. Amit is younger than Irfan and older than Chandu.
4. Farukh is younger than Bhavin and older than Hemant.
5. Irfan is younger than Gurmit and older than Jatin.
6. Hemant is older than Gurmit.

Who is the Youngest?


Answer

Eshrat is the youngest.

Discard whoever are older than someone.

From (1) Gurmit is younger than Dinesh and Farukh.


From (5) Jatin is younger than Irfan and Gurmit.
From (2) Eshrat is younger than Jatin and Chandu.

From above 3 deductions, Eshrat is younger than Dinesh, Farukh, Irfan, Gurmit, Jatin and Chandu.

Also,
From (3) Chandu is younger than Amit and Irfan.
From (4) Hemant is younger than Farukh and Bhavin.
From (6) Gurmit is younger than Hemant.

From above 3 deductions, Gurmit is younger than Farukh, Bhavin and Hemant. Also, Chandu is younger than Amit and
Irfan. But as seen earlier, Eshrat is younger than Gurmit and Chandu.

Hence, Eshrat is the youngest.


Last Saturday Milan went for the late night show and came late. In the morning family members asked him which
movie did he see. He gave different answers to everyone.

• He told to his father that he had gone to see MONEY.


• According to his mom, he saw either JOHNY or BABLU.
• His elder brother came to know that he saw BHABI.
• To his sister, he told ROBOT.
• And his grandpa heard that he saw BUNNY.

Thus, Milan gave six movie names, all five letter words. But he saw some other movie with five letter word. Moreover,
each of the six movie names mentioned above has exactly two letters common with the movie he saw. (with the same
positions)

Can you tell which movie did Milan see?


Answer

Milan saw BOBBY.

The six movie names are - MONEY, JOHNY, BABLU, BHABI, ROBOT and BUNNY.

Compare MONEY and JOHNY. They have O common at the second place and Y common at the fifth place. Also, they
can't have two different letters each, common with the required movie as the letters in remaining three places are all

Kumaraguru College, Coimbatore


different. Thus, the required movie must have either O at the second place or Y at the fifth place or both.

Similarly, comparing JOHNY and BUNNY - the required movie must have either N at the fourth place or Y at the fifth
place or both. Also, comparing MONEY and BUNNY - the required movie must have either N at the third place or Y at
the fifth place or both.

From the above 3 deduction, either Y is at fifth place or O is at the second place and N is at the third & fourth place.
The later combination is not possible as BABLU, BHABI & ROBOT will need at least 3 other letters which makes the
required movie 6 letter long. Hence, the required movie must have Y at the fifth place.

Now Y is not there in BABLU and BHABI at the fifth place and they have only B common at the first place. Hence, B
must be the first letter.

As B is at the first place and Y is at the fifth place and every movie has exactly 2 letters common with the required
movie. From BUNNY, the required movie do not have U at the second place and N at the third and fourth place. Now
looking at JOHNY and MONEY, they must have O common at the second place.

Using the same kind of arguments for BABLU, BHABI and ROBOT, we can conclude that Milan saw BOBBY.

Jim lies a lot. He tells the truth on only one day in a week.

One day he said: "I lie on Mondays and Tuesdays."


The next day he said: "Today is either Sunday, Saturday or Thursday."
The next day he said: "I lie on Fridays and Wednesdays."

On which day of the week does Jim tell the truth?


Answer

Jim tells the truth on Tuesday.

As Jim tells truth only on one day in a week, his statement on day 1 and day 3 both can not be false. Otherwise he tells
truth on more than one days in a week. Also, all three statements are mad on three consecutive days, statement made on
day 1 and day 3 both can not be true. Thus, either the statement made on day 1 or day 3 is true and other is false. Also,
the statement made on day 2 must be false i.e. day 1 is not Saturday, Friday or Wednesday.

Let's assume that the statement 1 is true. Then from the statement 3, day 1 must be either Friday or Wednesday. But it is
already deduced that day 1 is not Saturday, Friday or Wednesday.

Hence, the statement made on day 1 is false and the last statement is true. then from the statement 1, day 3 must be
either Monday or Tuesday. But it is already deduced that day 1 can not be Saturday i.e. day 3 can't be Monday. Hence,
Jim tells the truth on Tuesday.

4 men can dig 4 holes in 4 days.

How many hours does it take for 1 man to dig half a hole?
Submitted
Answer

There is nothing like "HALF HOLE".


Consider a chessboard with a single Rook. A Rook can move any number of square sideways/forward, but not
diagonally.

What is the minimum number of moves the Rook needs to make, in order to pass over all the squares on the chessboard
and return to the original position?
Answer

16 moves

Kumaraguru College, Coimbatore


As a Rook can move any number of square sideways/forward, but not diagonally and there are 8 rows and 8 columns on
the chessboard; the Rook needs minimum 16 moves to pass over all the squares and return to the original position.
A farmer needs 8 gallons of water. He has only three unmared buckets, two 6 gallon and one 11 gallon bucket.

How can he collect 8 gallons of water using three unmarked buckets? Provide solution with minimal water wastage.
Answer

Here is the solution with 10 gallon water wastage.


OPERATIONS 6 6 11
Fill 6 gallon bucket with water 6 0 0
Empty 6 gallon bucket into 11 gallon bucket 0 0 6
Fill 6 gallon bucket with water 6 0 6
Fill 11 gallon bucket to full using filled 6 gallon bucket. This will leave 1 gallon water in 6 gallon bucket 1 0 11
Empty 11 gallon bucket into second 6 gallon bucket. 1 6 5
Empty 11 gallon bucket - wastage of 5 gallon water 1 6 0
Empty second 6 gallon bucket into 11 gallon bucket 1 0 6
Fill seccond 6 gallon bucket with water 1 6 6
Fill 11 gallon bucket to full using filled second 6 gallon bucket. This will leave 1 gallon water in second 6
1 1 11
gallon bucket
Fill first 6 gallon bucket with 1 gallon water which is in second 6 gallon bucket 2 0 11
Empty 11 gallon bucket into second 6 gallon bucket. 2 6 5
Empty 11 gallon bucket - wastage of 5 gallon water 2 6 0
Fill 11 gallon bucket with water in both the 6 gallon buckets 0 0 11
I bought a car with a peculiar 5 digit numbered licence plate which on reversing could still be read. On reversing value
is increased by 78633.

Whats the original number if all digits are different?


Answer

Only 0 1 6 8 and 9 can be read upside down. So on rearranging these digits we get the answer as 10968.
Jack and Jill are playing cards for a stake of $1 a game. At the end of the evening, Jack has won 3 games and Jill has
won $3. How many games did they play?
Submitted by : Nathalie Drouin
Answer

They played total of 9 games. Jack won 3 games and Jill won 6 games.

If Jack has won three games and Jill has won $3, she lost a dollar for each loss, therefore she has won 6 and lost 3 to
make $3 and he won the other 3 that she lost!

Sam and Mala have a conversation.

• Sam says I am certainly not over 40


• Mala says I am 38 and you are atleast 5 years older than me
• Now Sam says you are atleast 39

All the statements by the two are false. How old are they really?
Answer

Sam is 41 and Mala is 37.

Kumaraguru College, Coimbatore


Let's invert the teaser and read it like this :

• Sam says I am certainly over 40


• Mala says I am not 38 and you are atmost 4 years older than me
• Now Sam says you are atmost 38

From first statement it is clear that Sam is over 40. Also, from next 2 statements it is clear that Mala is less then 38.
Hence the possibilities are :
Sam = 41, 42, 43, 44, 45, ......
Mala = 37, 36, 35, 34, 33, ......

It also says that the difference between their age is maximum 4 years. Hence, there is only one possible pair i.e. 41 and
37, all other combination have differences more then 4.

Hence the answer - Sam is 41 and Mala is 37.


A person travels on a cycle from home to church on a straight road with wind against him. He took 4 hours to reach
there.

On the way back to the home, he took 3 hours to reach as wind was in the same direction.

If there is no wind, how much time does he take to travel from home to church?
Answer

Let distance between home and church is D.

A person took 4 hours to reach church. So speed while travelling towards church is D/4.

Similarly, he took 3 hours to reach home. So speed while coming back is D/3.

There is a speed difference of 7*D/12, which is the wind helping person in 1 direction, & slowing him in the other
direction. Average the 2 speeds, & you have the speed that person can travel in no wind, which is 7*D/24.

Hence, person will take D / (7*D/24) hours to travel distance D which is 24/7 hours.

Answer is 3 hours 25 minutes 42 seconds


There are N secret agents each know a different piece of secret information. They can telephone each other and
exchange all the information they know. After the telephone call, they both know anything that either of them knew
before the call.

What are the minimum number of telephone calls needed so that all of the them know everything?
Answer

(2N - 3) telephone calls, for N = 2,3


(2N - 4) telephone calls, for N > 3

Divide the N secret agents into two groups. If N is odd, one group will contain one extra agent.

Consider first group: agent 1 will call up agent 2, agent 2 will call up agent 3 and so on. Similarly in second group,
agent 1 will call up agent 2, agent 2 will call up agent 3 and so on. After (N - 2) calls, two agents in each the group will
know anything that anyone knew in his group, say they are Y1 & Y2 from group 1 and Z1 & Z2 from group 2.

Now, Y1 will call up Z1 and Y2 will call up Z2. Hence, in next two calls total of 4 agents will know everything.

Now (N - 4) telephone calls are reqiured for remaining (N - 4) secret agents.

Total telephone calls require are


= (N - 2) + 2 + (N - 4)

Kumaraguru College, Coimbatore


= 2N - 4

Let\'s take an example. Say there are 4 secret agents W, X, Y & Z. Divide them into two groups of 2 each i.e. (W, X)
and (Y, Z). Here, 4 telephone calls are required.

1. W will call up X.
2. Y will call up Z.
3. W, who knows WX will call up Y, who knows YZ.
4. X, who knows WX will call up Z, who knows YZ.

Take an another example. Say there are 5 secret agents J, K, L, M & N. Divide them into two groups i.e. (J, K) and (L,
M, N). Here, 6 telephone calls are required.

1. J will call up K.
2. L will call up M.
3. M will call up N. Now M and N know LMN.
4. J, who knows JK will call up M, who knows LMN.
5. K, who knows JK will call up N, who knows LMN.
6. L will call up to anyone of four.

Mrs. F has invited several wives of delegates to the United Nations for an informal luncheon. She plans to seat her 9
guests ina row such that each lady will be able to converse with the person directly to her left and right. She has
prepared the following list.

Mrs. F speaks English only.


Mrs. G speaks English and French.
Mrs. H speaks English and Russian.
Mrs. J speaks Russian only.
Mrs. K speaks English only.
Mrs. L speaks French only.
Mrs. M speaks French and German.
Mrs. N speaks English and German.
Mrs. O speaks English only.

How many distinct seating arrangements are possible? Give all possible seating arrangements.

Note that ABCD and DCBA are the same.


Answer

126 distinct seating arrangements are possible.

Mrs. J and Mrs. H must be together and Mrs. J must be at the end as Mrs. J speaks only Russian and Mrs. H is the only
other Russian speaker.

Mrs. L speaks only French and there are two others - Mrs. G and Mrs. M - who speak French. Here there are 2 cases.

• CASE A : Mrs. L is at the other end


If Mrs. L is at the other end, either Mrs. G or Mrs. M must seat next to her.

o CASE AA : Mrs. G seats next to Mrs. L


Then, Mrs. M must seat next to Mrs. G and Mrs. N must seat next to Mrs. M. This is because Mrs. M
speaks French and German, and Mrs. N is the only other German speaker. Thus, the possible seating
arrangement is JHxxxNMGL, where x is the English speakers. Mrs. F, Mrs. K and Mrs. O can be
arranged in remaining 3 positions in 3! different ways i.e. 6 ways.

o CASE AB : Mrs. M seats next to Mrs. L


If so, then either Mrs. N or Mrs. G must seat next to Mrs. M

Kumaraguru College, Coimbatore


 CASE ABA : Mrs. N seats next to Mrs. M
Thus, the possible seating arrangement is JHxxxxNML, where x is the English speakers.
Mrs. F, Mrs. G, Mrs. K and Mrs. O can be arranged in remaining 4 positions in 4! different
ways i.e. 24 ways.

 CASE ABB : Mrs. G seats next to Mrs. M


Thus, the possible seating arrangement is JHxxxxGML, where x is the English speakers.
Mrs. F, Mrs. K, Mrs. N and Mrs. O can be arranged in remaining 4 positions in 4! different
ways i.e. 24 ways.

• CASE B : Mrs. L does not seat at the end


It means that Mrs. G, Mrs. L and Mrs. M must seat together. Also, Mrs. L must seat between Mrs. G and Mrs.
M.

o CASE BA : Mrs. G seats left and Mrs. M seats right to Mrs. L i.e. GLM

 CASE BAA : GLM is at the other end


Thus, the possible seating arrangement is JHxxxxGLM, where x is the English speakers.
Mrs. F, Mrs. K, Mrs. N and Mrs. O can be arranged in remaining 4 positions in 4! different
ways i.e. 24 ways.

 CASE BAB : GLM is not at the other end


Then Mrs. N must seat next to Mrs. M. Now, we have a group of four GLMN where Mrs. G
and Mrs. N speak English. Thus, the possible seating arrangement is JHxxxX, where x is the
individual English speakers and X is the group of four females with English speakers at the
both ends. Thus, there are 4! different ways i.e. 24 ways.

o CASE BB : Mrs. M seats left and Mrs. G seats right to Mrs. L i.e. MLG
Then, Mrs. N must seat next to Mrs. M. Now, we have a group of four NMLG where Mrs. G and Mrs.
N speak English. Thus, the possible seating arrangement is JHxxxX, where x is the individual English
speakers and X is the group of four females with English speakers at the both ends. Thus, there are 4!
different ways i.e. 24 ways.

Thus, total different possible seating arrangements are :


= 6 (case AA) + 24 (case ABA) + 24 (case ABB) + 24 (case BAA) + 24 (case BAB) + 24 (case BB)
= 126 seating arrangements

Thus, 126 distinct seating arrangements are poosible.


What is the smallest number which when divided by 10 leaves a remainder of 9, when divided by 9 leaves a remainder
of 8, when divided by 8 leaves a remainder of 7, when divided by 7 leaves a remainder of 6 and so on until when
divided by 2 leaves a remainder of 1?
Answer

The smallest such number is 2519.

The easiest way is to find the Least Common Multiple (LCM) of 2, 3, 4, 5, 6, 7, 8 and 9. And subtract 1 from it.

The LCM of 2, 3, 4, 5, 6, 7, 8 and 9 is given by 2520. Hence, the required number is 2519

Kumaraguru College, Coimbatore


Three friends divided some bullets equally. After all of them shot 4 bullets the total number of bullets remaining is
equal to the bullets each had after division. Find the original number divided.
Answer

18

Assume that initial there were 3*X bullets.

So they got X bullets each after division.

All of them shot 4 bullets. So now they have (X - 4) bullets each.

But it is given that,after they shot 4 bullets each, total number of bullets remaining is equal to the bullets each had after
division i.e. X

Therefore, the equation is


3 * (X - 4) = X
3 * X - 12 = X
2 * X = 12
X=6

Therefore the total bullets before division is = 3 * X = 18


Brain Teaser No : 00114

Everyday in his business a merchant had to weigh amounts from 1 kg to 121 kgs, to the nearest kg. What are the
minimum number of different weights required and how heavy should they be?

The minimum number is 5 and they should weigh 1, 3, 9, 27 and 81 kgs


Replace each letter by a digit. Each letter must be represented by the same digit and no beginning letter of a word can be
0.

ONE

ONE

ONE

+ONE

-------

TEN
Answer

Use trial and error. 0 =1, N = 8 ,E = 2, T = 7

Kumaraguru College, Coimbatore


182

182

182

+182

------

728
A man is on a search for Atlantis and comes upon an island where all the inhabitants know whether Atlantis is still
around or not.

However, all of the inhabitants are either Fairies or Trolls and they all use a spell to appear humanoid so you cannot tell
which is which. And the Faries always tell the truth and the Trolls always lie, but there is a slight complication, some of
the Fairies have gone insane and always lie and some of the Trolls have also gone insane and always tell the truth.

So here is your task: you must ask the first inhabitant that you come to ONE question and from that ONE question you
must determine wether Atlantis is still around or not.

What is the question that you must ask?


Answer

There are 2 answers to it:

Answer I"Is the statement that you are reliable equivalent to the statement that Atlantis is still around?"

Answer II"Do you believe that the Statement that you are a Fairy is equivalent to the statement that Atlantis is still
around?"
Answer

19 hours

A frog climbs 1 foot per 1 1/2 hours as during 30 minutes rest he slips back 2 feet. This way he will climb 12 feet in 18
hours. In next hour he will climb 3 more feet i.e. he will complete 15 feet in 19 hours and will reach the top of the wall.
If a bear eats 65 pounds in fish every day EXCEPT every 6th day which it only eats 45 pounds of fish.

If the bear continues this, how many pounds of fish will it eat in 200 days?
Submitted by : David
Answer

The bear will eat 12,340 pounds of fish in 200 days.

It is given that on every 6th day beareats 45 pounds of fish i.e. on day number 6, 12, 18, 24, .... 192, 198 the bear eats 45
pounds of fish.

Total number of 6th days = 200/6 = 33 (the bear eats 45 pounds)


Hence, the normal days are = 200 - 33 = 167 (the bear eats 65 pounds)

Thus, in 200 days, the bear will eat


= (167) * (65) + (33) * (45)
= 10855 + 1485
= 12,340 pounds
You have 3 points labelled A, B and C. You then have another 3 points labelled 1, 2 and 3. The aim of the puzzle is to
connect point A with point 1, 2 and 3. Point B with point 1, 2 and 3 and point C with point 1, 2 and 3.

Kumaraguru College, Coimbatore


Now while connecting the points you have to follow one rule - the lines cannot cross over each other.
A B C

1 2 3
PS : You can arrange the points in order as long as the lines DO NOT cross over each other.
Answer

There is no solution to it, if you consider 2 dimensions. It is impossible to join each of points A, B and C with points 1,
2 and 3 without lines crossing each other.

There is solution, if you consider 3 dimensions. Consider a circular base and a line perpendicular to it passing from the
center. Now take any 3 points along the perimeter of the circular base as points 1, 2 and 3. Similarly take any 3 points
along the perpendicular line as points A, B and C. Now it is quite simple to join each of points A, B and C with points 1,
2 and 3 without any of the lines crossing each other.

The other possible 3D structure is Pyramid. Take points 1, 2 and 3 as a vertices of the triangular base and points A, B
and C along the height of the Pyramid which is perpendicular to the triangular base and passing through the apex.

Answer

They weigh 54, 56, 58, 59, 62 pounds.

Let's assume that the weight of five bales are B1, B2, B3, B4 and B5 pounds respectively. Also, B1 <= B2 <= B3 <= B4
<= B5

It is given that five bales of hay are weighed two at a time in all possible ways. It means that each of the bale is
weighted four times.
Thus,
4*(B1 + B2 + B3 + B4 + B5) = (110 + 112 + 113 + 114 + 115 + 116 + 117 + 118 + 120 + 121)
4*(B1 + B2 + B3 + B4 + B5) = 1156
(B1 + B2 + B3 + B4 + B5) = 289 pounds

Now, B1 and B2 must add to 110 as they are the lightest one.
B1 + B2 = 110

Similarly, B4 and B5 must add to 121 as they are the heaviest one.
B4 + B5 = 121

From above three equation, we get B3 = 58 pounds

Also, it is obvious that B1 and B3 will add to 112 - the next possible higher value. Similarly, B3 and B5 will add to 120
- the next possible lower value.
B1 + B3 = 112
B3 + B5 = 120

Substituting B3 = 58, we get B1 = 54 and B5 = 62


From 2 & 3 equations, we get B2 = 56 and B4 = 59

Hence, the weight of five bales are 54, 56, 58, 59 and 62 pounds.

Pinto says, "The horse is not Black."


Sandy says, "The horse is either Brown or Grey."
Andy says, "The horse is Brown."

At least one is telling truth and at least one is lying.

Can you tell the color of the horse?

Kumaraguru College, Coimbatore


Answer

The color of the horse can be any color other than Black and Brown.

If the color of the horse is Black - all are lying.

If the color of the horse is Brown - all are telling truth.

Thus, the horse is neither Black nor Brown.

If the color of the horse is Grey - Pinto and Sandy are telling truth whereas Andy is lying.

If the color of the horse is other than Black, Brown and Grey - Pinto is telling truth whereas Sandy and Andy are lying.

You must have noticed that for the given conditions, Pinto is always telling truth whereas Andy is always lying

Three Gold (G) coins, three Silver (S) coins and three Copper (C) coins are arranged in a single row as follow:
G S C G S C G S C

• Only 2 adjacent unlike coins can be moved at any one time.


• The moved coins must be in contact with at least one other coin in line. i.e. no pair of coins is to be moved and
placed away from the remaining ones.
• No coin pairs can be reversed i.e. a S-C combination must remain in that order in its new positionwhen it is
moved.

What is the minimum number of moves required to get all the coins in following order?
C C C S S S G G G
Show all moves.
Answer

Minimum number of moves are 8.

Move Order of Coins

0 G S C G S C G S C

1 G S G S C G S C C

2 G S C G S C C S G

3 G S C G S C C S G

4 G S C C S G S C G

5 G S C C S C S G G

6 G S C C C S S G G

7 C C C S S G G S G

8 C C C S S S G G G
A fly is flying between two trains, each travelling towards each other on the same track at 60 km/h. The fly reaches one
engine, reverses itself immediately, and flies back to the other engine, repeating the process each time.

The fly is flying at 90 km/h. If the fly flies 180 km before the trains meet, how far apart were the trains initially?

Kumaraguru College, Coimbatore


Answer

Initially, the trains were 240 km apart.

The fly is flying at the speed of 90 km/h and covers 180 km. Hence, the fly flies for 2 hours after trains started.

It's obvious that trains met 2 hours after they started travelling towards each other. Also, trains were travelling at the
speed of 60 km/h. So, each train traveled 120 km before they met.

Hence, the trains were 240 km apart initially.

What is the minimum number of numbers needed to form every number from 1 to 7,000?

Example: To form 4884, you would need 2 4s & 2 8s. 4822 requires a 4, a 8, & 2 2s, but you would not count the
numbers again that you had already counted from making 4884.
Answer

36

You will need 3 of numbers 0, 7, 8 & 9, & 4 of numbers 1-6.


A drinks machine offers three selections - Tea, Coffee or Random (Either tea or Coffee) but the machine has been wired
up wrongly so that each button does not give what it claims.

If each drink costs 50p, how much minimum money do you have to put into the machine to work out which button
gives which selection?
Submitted
Answer

You have to put just 50p.

Put 50p and push the button for Random. There are only 2 possibilities. It will give either Tea or Coffee.

• If it gives Tea, then the button named Random is for Tea. The button named Coffee is for Random selection.
And the button named Tea is for Coffee.
• If it gives Coffee, then the button named Random is for Coffee. The button named Tea is for Random
selection. And the button named Coffee is for Tea.

Thus, you can make out which button is for what by putting just 50p and pressing Random selection first.
You have 13 balls which all look identical. All the balls are the same weight except for one. Using only a balance scale,
can find the odd one out with only 3 weighings?

Is it possible to always tell if the odd one out is heavier or lighter than the other balls?
Submitted by : Brett Hurrell
Answer

It is always possible to find odd ball in 3 weighings and in most of the cases it is possible to tell whether the odd ball is
heavier or lighter. Only in one case, it is not possible to tell the odd ball is whether heavier or lighter.

1. Take 8 balls and weigh 4 against 4.


o If both are not equal, goto step 2
o If both are equal, goto step 3

2. One of these 8 balls is the odd one. Name the balls on heavier side of the scale as H1, H2, H3 and H4.
Similarly, name the balls on the lighter side of the scale as L1, L2, L3 and L4. Either one of H's is heavier or

Kumaraguru College, Coimbatore


one of L's is lighter. Weigh (H1, H2, L1) against (H3, H4, X) where X is one ball from the remaining 5 balls in
intial weighing.

o If both are equal, one of L2, L3, L4 is lighter. Weigh L2 against L3.
 If both are equal, L4 is the odd ball and is lighter.
 If L2 is light, L2 is the odd ball and is lighter.
 If L3 is light, L3 is the odd ball and is lighter.

o If (H1, H2, L1) is heavier side on the scale, either H1 or H2 is heavier. Weight H1 against H2
 If both are equal, there is some error.
 If H1 is heavy, H1 is the odd ball and is heavier.
 If H2 is heavy, H2 is the odd ball and is heavier.

o If (H3, H4, X) is heavier side on the scale, either H3 or H4 is heavier or L1 is lighter. Weight H3
against H4
 If both are equal, L1 is the odd ball and is lighter.
 If H3 is heavy, H3 is the odd ball and is heavier.
 If H4 is heavy, H4 is the odd ball and is heavier.

3. One of the remaining 5 balls is the odd one. Name the balls as C1, C2, C3, C4, C5. Weight (C1, C2, C3)
against (X1, X2, X3) where X1, X2, X3 are any three balls from the first weighing of 8 balls.

o If both are equal, one of remaining 2 balls is the odd i.e. either C4 or C5. Weigh C4 with X1
 If both are equal, C5 is the odd ball. But you can not tell whether it is heavier or lighter.
 If C4 is heavy, C4 is the odd ball and is heavier.
 If C4 is light, C4 is the odd ball and is lighter.

o If (C1, C2, C3) is heavier side, one of C1, C2, C3 is the odd ball and is heavier. Weigh C1 and C2.
 If both are equal, C3 is the odd ball and is heavier.
 If C1 is heavy, C1 is the odd ball and is heavier.
 If C2 is heavy, C2 is the odd ball and is heavier.

o If (C1, C2, C3) is lighter side, one of C1, C2, C3 is the odd ball and is lighter. Weigh C1 and C2.
 If both are equal, C3 is the odd ball and is heavier.
 If C1 is light, C1 is the odd ball and is lighter.
 If C2 is light, C2 is the odd ball and is lighter.

How many squares are there in a 5 inch by 5 inch square grid? Note that the grid is made up of one inch by one inch
squares.
Submitted by : Kristin Monroe
Answer

There are 55 squares in a 5 by 5 grid.

There are 25 squares of one grid.


There are 16 squares of four grids i.e. 2 by 2

Kumaraguru College, Coimbatore


There are 9 squares of nine grids i.e. 3 by 3
There are 4 squares of sixteen grids i.e. 4 by 4
There is 1 square of twenty-five girds i.e. 5 by 5

Hence, there are total 25 + 16 + 9 + 4 + 1 = 55 squares.

You must have noticed one thing that total number squares possible of each size is always a perfact square i.e. 25, 16, 9,
4, 1

For a grid of N by N, the possible number of squares are


= N2 + (N - 1)2 + (N - 2)2 + (N - 3)2 + ......... + 32 + 22 + 12

For 1 by 1 grid, total squares = 12 = 1


For 2 by 2 grid, total squares = 22 + 12 = 5
For 3 by 3 grid, total squares = 32 + 22 + 12 = 14
For 4 by 4 grid, total squares = 42 + 32 + 22 + 12 = 30
For 5 by 5 grid, total squares = 52 + 42 + 32 + 22 + 12 = 55
Five horses ran in the race.

• There were no ties.


• Sikandar did not come first.
• Star was neither first nor last.
• Mughal Glory came in one place after Sikandar.
• Zozo was not second.
• Rangila was two place below Zozo.

In what order did the horses finish?


Answer

It's simple.

Let's find the possible places horses can finish. Possibilities are:
Sikandar - 2,3,4 (not 5th as Mughal Glory came one place after him)
Star - 2,3,4
Mughal Glory - 3,4,5
Zozo - 1,3 (not 4th & 5th as Rangila is two place after him)
Rangila - 3,5

So the result is:


1 Zozo
2 Star
3 Rangila
4 Sikandar
5 Mughal Glory

If you added together the number of 2's in each of the following sets of numbers, which set would contain the most 2's:
1-333, 334-666, or 667-999?
Answer

1-333

The reason why is because 200-299 each begins with a 2!

If one person sends the e-mail to two friends, asking each of them to copy the mail and send it to two of their friends,
those in turn send it to two of their friends and so on.

How many e-mails would have been sent by the time it did 30 sets?

Kumaraguru College, Coimbatore


Answer

2147483646

First person sent the mail to 2 persons. Those 2 sent the mail to 2 persons each, total 4 persons. Now, those 4 person
sent mail to total 8 persons, then 8 to 16 persons, 16 to 32 persons and so on.... Hence, it a series of 2, 4, 8, 16, 32 upto
30 numbers

It is a Geometric series with common ratio 2 and first number is also 2. Summation of such series is given by A * (Rn -
1) / (R - 1) where
A = First term
R = Common Ratio
n = total numbers

So total number of times mail sent by the time it did 30 sets


= 2 * (230 - 1) / (2 - 1)
= 2 * (1073741824 - 1)
= 2 * 1073741823
= 2147483646
Brain Teaser No : 00347

At the entrance to a members club stands a stranger seeking admission. A friend told him that it's easy to get in. You
just have to answer a question corrcetly! Answering wrong, however, will result in being shot!

To live a little longer, the man waits in a back alley near the entrance for people to go in. After a while a man comes
to the entrance. The door warden asks him: "Twelve?" to which he replies "Six!" and goes in.

"That's easy." our friend thinks, but he waits a little longer.

Another man comes to the door. "Six?" the door warden asks, to which he replies "Three!" and goes in.

"That's too good to be true" our friend thinks, and he was right. Because, when asked "Four?", he answered "Two!"
and was found dead in the alley.

What was the correct answer?


Submitted by : Milind Gadagkar

Answer

The correct answer was "Four".

The answer is the number of letters in the word spoken by the door warden.

"Twelve" contains "Six" letters i.e. T, W, E, L, V, E


"Six" contains "Three" letters i.e. S, I, X
Similarly, "Four" contains "Four" letters i.e. F, O, U, R
There is a perfect sphere of diameter 40 cms. resting up against a perfectly straight wall and a perfectly straight floor i.e.
the wall and the floor make a perfect right angle.

Can a perfect sphere of diameter 7 cms. pass through the space between the big sphere, the wall and the floor? Support
your answer with valid arguments.
Don't submit just "Yes" or "No".
Submit Users BrainVista Puzzle A Add to
Answer Answer (23) Answer Friend Favourite

Kumaraguru College, Coimbatore


Submit Users BrainVista Puzzle A
Add
Answer Answer (23) Answer Friend
Sarika multiplied 414 by certain number and obtained 69958 as the answer. But she found that there is some error in the
answer - both the 9s in the answer are wrong and all the other digits are correct.

Can you find the correct answer?


Answer

The correct answer is 60858.

If you divide 69958 by 414, you will get 168.98. Hence, assume some three digit number and multiply it by 414 and use
6**58 as the answer.

Assume three digit number such that


***

414

-------------

***

***0

***00

-------------

6**58
It is obvious that the last digit of the assumed number must be 7.
**7

414

-------------

**8

**70

**800

-------------

6**58
Now, the second last digit of the assumed number must be 4 or 9. Also, the first digit of the assumed number must be 1
as the first digit of the answer is 6. Using trial and error for above two conditions, the answer is

Kumaraguru College, Coimbatore


147

414

-------------

588

1470

58800

-------------

60858
Find the least number which when divided by 35, leaves remainder 25; when divided by 45, leaves remainder 35 and
when divided by 55, leaves remainder 45.

Answer

3455

The answer is LCM of (35, 45, 55) minus 10.


LCM of (35, 45, 55) is 3465.
Hence, the answer is 3455.
The ratio of Boys to Girls is 6:4. 60% of the boys and 40% of the girls take lunch in the canteen. What % of class takes
lunch in canteen?
Answer

Assume there are 6X boys and 4X Girls

Total Students taking lunch in canteen


= (6X)(60/100) + (4X)(40/100)
= 36(X/10) + 16(X/10)
= 52(X/10)

Total students are = 6X + 4X = 10X

% of class taking lunch in canteen


= ((52X/10) * 100 ) / 10X
= 52 %
In the following multiplication, certain digits have been replaced with asterisks (*). Replace all the asterisks such that
the problem holds the result.

Kumaraguru College, Coimbatore


* * 7

X 3 * *

----------

* 0 * 3

* 1 *

* 5 *

-------------

* 7 * * 3
Answer

A simple one.
1 1 7

X 3 1 9

----------

1 0 5 3

1 1 7

3 5 1

-------------

3 7 3 2 3
How long would it take you to count 1 billion orally if you could count 200 every minute and were given a day off
every four years?

Assume that you start counting on 1 January 2001.


SubmitteAnswer

9 Years, 187 Days, 5 Hours, 20 minutes

As you can count 200 per minute, to count 1 billion you require
= 1,000,000,000/200 minutes
= 5,000,000 minutes
= 83,333.3333 hours
= 3,472.2222 days
= 9.512937 years
= 9 Years, 187 Days, 5 Hours, 20 minutes

Note that a day off every four year will be a Leap day. Hence, no need to consider leap year.
dFive students - Akash, Chintan, Jignesh, Mukund and Venky - appeared for an exam. There were total five questions -
two multiple choice (a, b or c) and three true/false questions. They answered five questions each and answered as
follow.

Kumaraguru College, Coimbatore


I II III IV V

--------------------------------------------------

Chintan c b True True False

Akash c c True True True

Jignesh a c False True True

Mukund b a True True False

Venky b b True False True

--------------------------------------------------
Also, no two students got the same number of correct answers.

Can you tell which are the correct answers? What are their individual score?
Answer

The correct answers are b, a, True, False and False. Also, the scores are Jignesh (0), Akash (1), Chintan (2), Venky (3)
and Mukund (4).

As no two students got the same number of correct answers, the total number of correct answers must be either 15
(1+2+3+4+5) or 10 (0+1+2+3+4).

Let's find out the maximum number of correct answers possible from the answers given by them.
For Question I = 2 (b or c)
For Question II = 2 (b or c)
For Question III = 4 (True)
For Question IV = 4 (True)
For Question V = 3 (True)

Thus, the maximum number of correct answers possible are 15 (2+2+4+4+3) which means that Akash would have
given all correct answers as only he answered True for questions III, IV and V. But then Chintan and Jignesh would
have exactly 3 correct answers. And also, Mukund and Venky would have 2 correct answers. So no one got all five
correct. One can also arrive at this conclusion by trial-and-error, but that would be bit lengthy.

Now, it is clear that total number of correct answers are 10 (0+1+2+3+4). Questions III and IV both can not be False. If
so, total number of correct answers would not be 10. So the student who got all wrong can not be Chintan, Akash and
Mukund.

If Venky got all wrong, then Chintan, Jignesh and Mukund each would have atleast 2 correct answers. It means that
Akash would have to be the student with only one correct answer and the correct answers for questions I and II would
be a and a respectively. But then the total number of correct answers would be 1 (a) + 1 (a) + 1 (False) + 4 (True) + 2
(Flase) = 9.

Thus, Jignesh is the student with all wrong answers. The correct answers are b, a, True, False and False. Also, the scores
are Jignesh (0), Akash (1), Chintan (2), Venky (3) and Mukund (4).

Eleven boys and girls wait to take their seats in the same row in a movie theater. There are exactly 11 seats in the row.

They decided that after the first person sits down, the next person has to sit next to the first. The third sits next to one of
the first two and so on until all eleven are seated. In other words, no person can take a seat that separates him/her from
at least one other person.

How many different ways can this be accomplished? Note that the first person can choose any of the 11 seats.

Kumaraguru College, Coimbatore


Answer

There are 1024 different ways.

This is the type of Brain Teaser that can be solved using the method of induction.

If there is just a one person and one seat, that person has only one option.

If there are two persons and two seats, it can be accomplished in 2 different ways.

If there are three persons and three seats, it can be accomplished in 4 different ways. Remember that no person can take
a seat that separates him/her from at least one other person.

Similarly, four persons and four seats produce 8 different ways. And five persons with five seats produce 16 different
ways.

It can be seen that with each additional person and seat, the different ways increase by the power of two. For six persons
with six seats, there are 32 different ways.

For any number N, the different possible ways are 2(N-1)

Thus, for 11 persons and 11 seats, total different ways are 210 i.e. 1024
The secret agent X emailed a code word to his head office. They are "AIM DUE OAT TIE MOD". But four of these
five words are fake and only one contains the information.

The agent X also mailed a sentence as a clue - if I tell you any one character of the code word, you would be able to tell
the number of vowels in the code word.

Can you tell which is the code word?

Answer

The code word is TIE.

If you were told any one character of MOD, then you would not be able to determine whether the number of vowels are
one or two. e.g. if you were told M, there are two words with M - AIM with 2 vowels and MOD with 1 vowel. So you
would not be able to say the number of vowels. Same arguments can be given for characters O and D.

Hence, the word with any one of M, O or D is not a code word i.e. AIM, DUE, OAT and MOD are not the code word.
Thus, TIE is the code word.
T : two words - TIE and OAT, both with 2 vowels
I : two words - TIE and AIM, both with 2 vowels
E : two words - TIE and DUE, both with 2 vowels.
Brain Teaser No : 00361

Four men - Abraham, Bobby, Clinton and Denial - are standing in a straight line.

1. One man is fair, handsome and unscarred.


2. Two men who are not fair, are each standing next to Abraham.
3. Bobby is the only man standing next to exactly one handsome man.
4. Clinton is the only man not standing next to exactly one scarred man.

Who is fair, handsome and unscarred?

Answer

Kumaraguru College, Coimbatore


Clinton is fair, handsome and unscarred.

From (2), both the men standing next to Abraham are not fair. Also, exactly one man is fair, handsom and unscarred.
Hence, there are two cases:

Case 1 :: ? (N, ?, ?) : Abraham (Y, Y, N) : ? (N, ?, ?) : ? (?, ?, ?)


Case 2 :: ? (N, ?, ?) : Abraham (?, ?, ?) : ? (N, ?, ?) : ? (Y, Y, N)

Note the representation - Name (Fair, Handsome, Scarred). "Y" stands for Yes and "N" stabds for No. Abraham (Y, Y,
N) means Abraham is Fair, Handsome and Unscarred.

It is clear that either Abraham or the man at the extreme right is fair, handsome and unscarred.

From (4), it is deduced that Clinton is standing next to unscarred man and each of the other men standing next to exactly
one scarred man.

Case 1 :: Clinton (N, ?, N) : Abraham (Y, Y, N) : ? (N, ?, Y) : ? (?, ?, Y)


Case 2 :: ? (N, ?, Y) : Abraham (?, ?, Y) : ? (N, ?, N) : Clinton (Y, Y, N)

From (3), Bobby is the only man standing next to exactly one handsome man. But in Case 1, Clinton is standing next to
exactly one handsome man. Hence, Case 1 is not possible and Case 2 is the correct one.

Case 2 :: ? (N, ?, Y) : Abraham (?, ?, Y) : ? (N, ?, N) : Clinton (Y, Y, N)

Again from (3) and (4), there are 2 possibilities as shown below.

Case 2a :: Denial (N, N, Y) : Abraham (?, N, Y) : Bobby (N, N, N) : Clinton (Y, Y, N)


Case 2b :: Bobby (N, N, Y) : Abraham (?, Y, Y) : Denial (N, N, N) : Clinton (Y, Y, N)

Thus, Clinton is fair, handsome and unscarred. Also, Abraham may be either fair or not fair.
An orange colored glass has Orange juice and white colored glass has Apple juice both of equal volumes. 50ml of the
orange juice is taken and poured into the white glass. After that similarly, 50ml from the white glass is poured into the
orange glass.

Of the two quantities, the amount of apple juice in the orange glass and the amount of orange juice in the white glass,
which one is greater and by how much?
Answer

The two quantities are equal.

Solve it by taking example. Let's assume that both glasses contain 450 ml of juice each.

Now, 50ml of the orange juice is taken and poured into the White glass. Hence, orange colored glass contains 400 ml of
Orange juice and white glass contains 450 ml of Apple juice and 50 ml of Orange juice i.e. total of 500 ml from white
glass contains 450 ml of Apple juice and 50 ml of Orange juice. It means that every 50 ml from white glass contains 45
ml of Apple juice and 5 ml of Orange juice.

Similary, 50 ml of juice from white glass is poured into orange glass. Now this 50 ml is not a pure apple juice. It
contains 45 ml of Apple juice and 5 ml of Orange juice.

Hence, Orange glass contains 405 ml of Orange juice and 45 ml of Apple juice. Similary, white glass contains 405 ml of
Apple juice and 45 ml of Orange juice.

Orange Glass White Glass

Orange Juice Apple Juice Orange Juice Apple Juice

Initially 450 ml 0 ml 0 ml 450 ml

Kumaraguru College, Coimbatore


50 ml from Orange Glass is poured into White Glass 400 ml 0 ml 50 ml 450 ml

50 ml from White Glass is poured into Orange Glass 405 ml 45 ml 45 ml 405 ml

Now it is clear that the amount of apple juice in the orange glass and the amount of orange juice in the white glass are
the same.

P.S. Here we assumed 450 ml as initial quantity in both the glasses just for simplicity. You can try the same by
assuming any other number. But the answer is the same.
Brain Teaser No : 00433

Annie, Bunnie, Candy and Dina visited Edy on 14th February.

1. The time of each visit was as follows:


- Annie at 8:00
- Bunnie at 9:00
- Candy at 10:00
- Dina at 11:00
Each time mentioned above may be either AM or PM.
2. Candy did not visit Edy between Bunnie and Dina.
3. At least one female visited Edy between Annie and Bunnie.
4. Annie did not visit Edy before both Candy and Dina.

Can you tell at what time did they individually visit Edy?

Answer

Bunnie (9:00AM) - Dina (11:00AM) - Annie (8:00PM) - Candy (10:00PM)

From the given data, it is clear that at least one female visited Edy in the morning and at least one female visited Edy in
the evening. Also, from (4), Annie did not visit Edy first. It means that Annie visited Edy at 8:00 PM

From (3), Bunnie must have visited Edy at 9:00 AM. Also, either Candy or Dina or both visited Edy in the morning.

But from (2), only Dina must have visited Edy in the morning at 11:00 AM and hence, Candy visited Edy at 10:00 PM.

The order of visits must be:


Bunnie (9:00AM) - Dina (11:00AM) - Annie (8:00PM) - Candy (10:00PM)

In training for a competition, you find that swimming downstream (with the current) in a river, you can swim 2 miles in
40 minutes, & upstream (against the current), you can swim 2 miles in 60 minutes.

How long would it take you to swim a mile in still water?


Answer

You are able to swim downstream at 3 miles an hour, & upstream at 2 miles an hour. There is a difference of 1 mile an
hour, which is the river helping you in 1 direction, & slowing you in the other direction.

Average the 2 rates, & you have the rate that you can swim in still water, which is 2.5 miles an hour.

You can thus swim a mile in still water in 24 minutes.


Father's age is three years more than three times the son's age. After three years, father's age will be ten years more than
twice the son's age.

What is the father's present age?

Kumaraguru College, Coimbatore


Answer

Let son's present age is X years.


Hence, father's present age is (3X + 3) years.

After 3 years, son's age will be (X + 3) years.


and father's age will be (3X + 6) years.

But given that after 3 years father's age will be ten years more than twice the son's age.
(3X + 6) = 2 * (X + 3) + 10
3X + 6 = 2X + 16
X = 10

Therefore, father's present age is 33 years.

Submit Users
Brai
Answer Answer (17)

Brain Teaser No : 00570

• A is the father of two children - B and D who are of different sexes.


• C is B's spouse.
• E is the same sex as D.
• B and C have the two children - F who is the same sex as B and G who is the same sex as
C.
• E's mother, H who is married to L, is the sister of D's mother, M.
• E and E's spouse, I have two children - J and K who are the same sex as I.

Note that no persons have married more than once. Also, there are more number of females than
males. Can you tell how many females are there?

Answer

There are 7 females and 6 males.

Assume that there are four sexes - male, female, X and Y. Prepare the following tree based on the data given :

sister

L(m) - H(f) -------------------- M(f) - A(m)

| |

| |

E(x) - I(y) D(x) B(y) - C(x)

| |

| |

J(y) K(y) F(y) G(x)

It is clear that there are altogether 13 persons - 2 males, 2 females, 4 Xs and 5 Ys.

Kumaraguru College, Coimbatore


It is given that there are more number of females than male. Hence, all Y must represent female. Thus, there are 7
females and 6 males.
A positive integer that, when added to 1000 gives a sum which is greater than when multiplied by 1000.

Find the positive integer.


Answer

The positive integer is 1.

Sum of 1 and 1000 = 1 + 1000 = 1001


Multiplication of 1 and 1000 = 1 * 1000 = 1000

Thus, sum of 1 and 1000 is greater than the multiplication of 1 and 1000.
Mr. D'souza has bought four cars - Merc, Honda, Ford, Zen - as presents for his sons' birthdays, all of which are next
week. Given the following information, what will each son get?

Alan will not get the Honda unless Barry gets the Merc and Denzil gets the Ford. Barry will not get the Ford unless Carl
gets the Zen and Alan gets the Merc. Denzil will not get the Zen unless Alan gets the Honda and Barry gets the Merc.
Alan will not get the Merc unless Carl gets the Zen and Denzil gets the Ford. Barry will not get the Merc unless Alan
gets the Zen and Denzil gets the Ford. Alan will not get the Zen unless Barry gets the Honda and Carl gets the Merc.
Carl will not get the Zen unless Barry gets the Honda and Alan gets the Ford. Alan will not get the Ford unless Barry
gets the Zen and Denzil gets the Honda. Carl will not get the Merc unless Denzil gets the Honda.
Answer

Let's put given 9 information in a table. The person in Bold Font will not get the corresponding car unless the persons in
Normal Font get the corresponding cars. Also, the person will Italics will get the remaining car.

Merc Honda Ford Zen

1 Barry Alan Denzil Carl

2 Alan Denzil Barry Carl

3 Barry Alan Carl Denzil

4 Alan Barry Denzil Carl

5 Barry Carl Denzil Alan

6 Carl Barry Denzil Alan

7 Denzil Barry Alan Carl

8 Carl Denzil Alan Barry

9 Carl Denzil ? ?

Now, let's assume that Alan gets the Merc. Then from (4), Barry gets the Honda, Denzil gets the Ford and Carl gets the
Zen. But from (7), Carl will not get the Zen unless Barry gets the Honda and Alan gets the Ford. Thus, it contradicts the
original assumption. Hence, Alan will not get the Merc.

Let's assume that Alan gets the Honda. Then from (1), Barry gets the Merc, Denzil gets the Ford and Carl gets the Zen.
But from (5) or from (7), it contradicts the original assumption. Hence, Alan will not get the Honda.

Let's assume that Alan gets the Ford. Then from (8), Carl gets the Merc, Denzil gets the Ford and Barry gets the Zen -
which does not contradict any of the statement.

Similaly, you can assume that Alan gets the Zen. (which is contradictory to (9))

Kumaraguru College, Coimbatore


Hence, Alan gets the Ford, Barry gets the Zen, Carl gets the Merc and Denzil gets the Honda.
Yesterday in a party, I asked Mr. Shah his birthday. With a mischievous glint in his eyes he replied. "The day before
yesterday I was 83 years old and next year I will be 86."

Can you figure out what is the Date of Birth of Mr. Shah? Assume that the current year is 2000.
Answer

Mr. Shah's date of birth is 31 December, 1915

Today is 1 January, 2000. The day before yesterday was 30 December, 1999 and Mr. Shah was 83 on that day. Today
i.e. 1 January, 2000 - he is 84. On 31 December 2000, he will be 85 and next year i.e. 31 December, 2001 - he will be
86. Hence, the date of birth is 31 December, 1915.

Many people do think of Leap year and date of birth as 29th February as 2000 is the Leap year and there is difference of
3 years in Mr. Shah's age. But that is not the answer.

Brain Teaser No : 00800

There are 4 mathematicians - Brahma, Sachin, Prashant and Nakul - having lunch in a hotel. Suddenly, Brahma
thinks of 2 integer numbers greater than 1 and says, "The sum of the numbers is..." and he whispers the sum to
Sachin. Then he says, "The product of the numbers is..." and he whispers the product to Prashant. After that
following conversation takes place :

Sachin : Prashant, I don't think that we know the numbers.


Prashant : Aha!, now I know the numbers.
Sachin : Oh, now I also know the numbers.
Nakul : Now, I also know the numbers.

What are the numbers? Explain your answer.


Submitted
Answer

The numbers are 4 and 13.

As Sachin is initially confident that they (i.e. he and Prashant) don't know the numbers, we can conclude that -
1) The sum must not be expressible as sum of two primes, otherwise Sachin could not have been sure in advance that
Prashant did not know the numbers.
2) The product cannot be less than 12, otherwise there would only be one choice and Prashant would have figured that
out also.

Such possible sum are - 11, 17, 23, 27, 29, 35, 37, 41, 47, 51, 53, 57, 59, 65, 67, 71, 77, 79, 83, 87, 89, 93, 95, 97, 101,
107, 113, 117, 119, 121, 123, 125, 127, 131, 135, 137, 143, 145, 147, 149, 155, 157, 161, 163, 167, 171, 173, 177, 179,
185, 187, 189, 191, 197, ....

Let's examine them one by one.

If the sum of two numbers is 11, Sachin will think that the numbers would be (2,9), (3,8), (4,7) or (5,6).

Sachin : "As 11 is not expressible as sum of two primes, Prashant can't know the numbers."

Here, the product would be 18(2*9), 24(3*8), 28(4*7) or 30(5*6). In all the cases except for product 30, Prashant would
know the numbers.

- if product of two numbers is 18:


Prashant : "Since the product is 18, the sum could be either 11(2,9) or 9(3,6). But if the sum was 9, Sachin would have
deduced that I might know the numbers as (2,7) is the possible prime numbers pair. Hence, the numbers must be 2 and

Kumaraguru College, Coimbatore


9." (OR in otherwords, 9 is not in the Possible Sum List)

- if product of two numbers is 24:


Prashant : "Since the product is 24, the sum could be either 14(2,12), 11(3,8) or 10(4,6). But 14 and 10 are not in the
Possible Sum List. Hence, the numbers must be 3 and 8."

- if product of two numbers is 28:


Prashant : "Since the product is 28, the sum could be either 16(2,14) or 11(4,7). But 16 is not in the Possible Sum List.
Hence, the numbers must be 4 and 7."

- if product of two numbers is 30:


Prashant : "Since the product is 30, the sum could be either 17(2,15), 13(3,10) or 11(5,6). But 13 is not in the Possible
Sum List. Hence, the numbers must be either (2,15) or (5,6)." Here, Prashant won't be sure of the numbers.

Hence, Prashant will be sure of the numbers if product is either 18, 24 or 28.

Sachin : "Since Prashant knows the numbers, they must be either (3,8), (4,7) or (5,6)." But he won't be sure. Hence, the
sum is not 11.

Summerising data for sum 11:


Possible Sum PRODUCT Possible Sum
2+9 18 2+9=11 (possible)
3+6=9
3+8 24 2+12=14
3+8=11 (possible)
4+6=10
4+7 28 2+12=14
3+8=11 (possible)
4+6=10
5+6 30 2+15=17 (possible)
3+10=13
5+6=11 (possible)

Following the same procedure for 17:


Possible Sum PRODUCT Possible Sum
2+15 30 2+15=17 (possible)
3+10= 13
5+6=11 (possible)
3+14 42 2+21=23 (possible)
3+14=17 (possible)
6+7=13
4+13 52 2+26=28
4+13=17 (possible)
5+12 60 2+30=32
3+20=23 (possible)
4+15=19
5+12=17 (possible)
6+10=16
6+11 66 2+33=35 (possible)
3+22=25
6+11=17 (possible)

Kumaraguru College, Coimbatore


7+10 70 2+35=37 (possible)
5+14=19
7+10=17 (possible)
8+9 72 2+36=38
3+24=27 (possible)
4+18=22
6+12=18
8+9=17 (possible)

Here, Prashant will be sure of the numbers if the product is 52.

Sachin : "Since Prashant knows the numbers, they must be (4,13)."

For all other numbers in the Possible Sum List, Prashant might be sure of the numbers but Sachin won't.

Here is the step by step explaination:

Sachin : "As the sum is 17, two numbers can be either (2,15), (3,14), (4,13), (5,12), (6,11), (7,10) or (8,9). Also, as none
of them is a prime numbers pair, Prashant won't be knowing numbers either."

Prashant : "Since Sachin is sure that both of us don't know the numbers, the sum must be one of the Possible Sum List.
Further, as the product is 52, two numbers can be either (2,26) or (4,13). But if they were (2,26), Sachin would not have
been sure in advance that I don't know the numbers as 28 (2+26) is not in the Possible Sum List. Hence, two numbers
are 4 and 13."

Sachin : "As Prashant now knows both the numbers, out of all possible products - 30(2,15), 42(3,14), 52(4,13),
60(5,12), 66(6,11), 70(7,10), 72(8,9) - there is one product for which list of all possible sum contains ONLY ONE sum
from the Possible Sum List. And also, no such two lists exist. [see table above for 17] Hence, two numbers are 4 and
13."

Nakul figured out both the numbers just as we did by observing the conversation between Sachin and Prashant.

It is interesting to note that there are no other such two numbers. We checked all the possible sums till 500 !!!
Substitute digits for the letters to make the following subtraction problem true.
S A N T A

- C L A U S

-----------------

X M A S
Note that the leftmost letter can't be zero in any word. Also, there must be a one-to-one mapping between digits and
letters. e.g. if you substitute 3 for the letter M, no other letter can be 3 and all other M in the puzzle must be 3.
Answer

One of the simplest brain teaser as there are total 26 possible answers.

It is obvious that S=C+1. Since A-S=S, it is clear that A=2*S or 2*s-10. Also, L and X are interchangeable.
SANTA - CLAUS = XMAS

24034 - 16492 = 7542

24034 - 17492 = 6542

24074 - 15432 = 8642

Kumaraguru College, Coimbatore


24074 - 18432 = 5642

24534 - 16492 = 8042

24534 - 18492 = 6042

24794 - 16452 = 8342

24794 - 18452 = 6342

24804 - 15462 = 9342

24804 - 19462 = 5342

24974 - 16432 = 8542

24974 - 18432 = 6542

36806 - 27643 = 9163

36806 - 29643 = 7163

36156 - 27693 = 8463

36156 - 28693 = 7463

62132 - 54206 = 7926

62132 - 57206 = 4926

62172 - 53246 = 8926

62172 - 58246 = 3926

62402 - 53276 = 9126

62402 - 59276 = 3126

62712 - 53286 = 9426

62712 - 59286 = 3426

62932 - 58206 = 4726

62932 - 54206 = 8726

1) 101^100 -1 is divisible by.....


2) Question on boat ( stream velocity given...)
3) Train Question( Goods and Passenger train..their speeds given..)
4) Pipe question (with leak at the bottom..)
5) Salary & Proportion problem
6) Another problem on Salary & Proportion

Kumaraguru College, Coimbatore


7) Age question-father and son
8) Another age question
9) Question on ratios( Sachin:Saurav=Saurav:Rahul=3:2....together they scored some runs,you had to find the runs
scored by Sachin)
10) Angle between hands when time is 2:20
11) x^2 + 4 y^2 =4xy.Find x:y
12) A question on Arithmetic Progression(something like 5 times the 5th term is 8 times the 8th term..find 12th term...)
13) A and B's work units given.They were together gievn Rs.720.When C joined,they together completed the work in 5
days.Find C's wages
14) There was a circle.A square of max size was cut from it.From this square,a circle of max size was cut.What was the
ratio of this final size w.r.t initial size?
15) A runs 3/4th faster than B.One of them was placed some metres ahead.How far should the finishing post be placed
so that both of them finish at the same time?
16) Longest time one has to wait for next birthday?(366/365/4 years/8years)
17) Next no: in the seq: 7,11,__,19,23
18) Some question on steps...it was 10 ft high...an ant travelled upwards..and total time taken
19) Cricket-some data on runrate of the opposition being 15%....
20) Time & Distance..somebody was travelling along the circumference....

1. If a date oct,02,2001 is written as 10022001(mmddyy), Then which date before 01,oct,2000 gives
the maximum valued palindrome(the digits must be same when reversed) ?
Ans:12311321

2. There are eight differen coloured eggs in a basket, among them 23 are in red, 17 blue, 21
green, 18 yellow, 16 black,12 white, 25 voilet.What is the probability of getting atleast one pair of the same colour,
when you draw a least number?
Ans:...................

3. What is the number whose positive integer digits when multiplied gives the maximum value and
added gives the exact 100.
ans:(2+2+........+50 times)=100, i.e 2 power 50.

4. In my garden the gross grows equally in every year. It takes 24 days for the 70 cows to eat
and it takes 60 days for the 30 cows to eat. Then how many number of cows can eat in 96 days?
Ans:17.5 so considered as 18

Maximum numbers that can be formed using all the 4 digits 6 4 8 1


without repetition and which is divisible by 9.( ans none)

2.) Find the number of sides of a regular convex polygon whose angle
is 40degrees.

3.) a+b+c=0, then roots of ax^2+bx+c=0 is


1.imag 2.real 3.coincidental 4.zero

4.) Difference b/w the compound interest and simple interest for
Rs.2500 for 2 years is given-----. find the rate of interest.

5.) there was one more question on S.I and C.I

6.) the minimum number by which 60 is to be multiplied to generate a


square.
ans 15

7.) A monkey climbs 6 mts and falls 3mts in alternate minutes.Then


time taken to climb a tree 60metres high?

Kumaraguru College, Coimbatore


a. 35 b.37 c.32 d.34 (think the answer is 37)

8.) (This was the second last question) A bucket contains z drops.
and it leaks x drops in t secs.then the time required to empty the
bucket(in minutes)?

9.) 6 pipes fill or empty the cistern. find the number of emptying
pipes iff it takes 18hrs to fill and 18 hrs to empty.... (don't
remember the question exactly)

10.) the largest no: which is a factor of 1080 and 729

11.) No: of spheres of radius 1 that can be got from sphere of radius
(or diameter don't recall) 8

12.) (think the last but three question)Travelling at 3/4th the speed
a man is 20 minutes later then speed is??

13.) there are 6 keys and 6 locks. then number of combinations to be


tried out to get the actual solution
a. 5^6 b.6^5 (don't remember the rest)

14.) choosing 2 people out of 10 in how many combinations can a


particular person(some name) be always included....

15.) from 6 white balls and 7 black balls probability that 2 balls
drawn at random are of the same color?

16.) if a sales man gets successive gain of 15% and 20% then his
actual gain?
ans. 38

17.) a string of pearls such that 1/3 is lost and of that 1/4th is
missing, remaining is 20 then actual number of pearls?
ans. 40

18.) a man gets a gain of x%. but if he had sold at twice the cost
price, what will be his gain?(question not sure)
a. 2x b.200-2x c.100+x (not sure of the options)

19.) a clock was 7mts behind the actual time on 3 p.m. on wednesday
and 8 mts ahead of actual time on (not sure) 4 p.m. friday. when will
it show the correct time?

20.) boat moves upstream in 6 hrs and covers the same distance
downstream in 5 hrs. then speed of a raft floating?(accuracy of
question not sure)

21.) (this was the last question) no idea what it stands for........
some kind of notation like S(P(M((D(a,b),2))):P(M(S(D(a,b),........
options were
1. ab 2.(a-b)^2 3.(a+b)^2 4.none

22.) if x men working x hrs per day can do x units of work in x days,
then y men working y hrs/day would be able to complete how many units
of work i y days?
ans. y^3/x^2 (question in R.S. Agarwal)

Kumaraguru College, Coimbatore


23.) ( this was a question in the first page of the section II
booklet) a cone with radius----- and height -----. a hemisphere
covers the cone such that base of hemisphere meets that of the cone.
then the enclose volume.....(R.S. Agarwal consists of similar
questions)

24.) there was one more question on volume and surface area.....

25.) 1 Rs, 50 ps , 25 ps coins are in the ratio ---------, then the


number of 50 ps coins if they sum to ------Rs. (similar question in
R.S.Agarwal)

26.) there was one more question on coins i.e. abt getting a change
of 10ps and 25 coins for ------Rs.(how many possible combinations or
so possible)

27.) x/y+y/x=40/21(don't remember the exact value, believe this is


the one) find x and y

there were 2 questions on train and one was like:


28.) a goods train starts and after 2 hrs a passenger train at
4km/hr starts and overtakes the goods train after 4 hrs, then the
speed of goods train?

29.) 15hrs of boys work=6 hrs of women's work. 3/5 of the work is
done by -----boys and -----women. How much time would be taken by the
women to complete the work?

30.) using false weight a man gains -----%, for a k.g. then he uses a
weight of.....(this kind of question has some formula that can be
found in R.S.Agarwal)

31.) there was one question on L.C.M. and co primes(don't remember


the question)

32.) there was one question on triangle

33.) a figure was given a square with four corners shaded and asked
to find the area of the shaded portion.... ie area of square-area of
the regular octagon.....

7. Find the reminder when 333666777888999 divided by 3 or 9 or 11 ?


8. Which is the biggest perfect square amongst the following
15129, 12348, 23716, 20736
9. The greatest area of the following
(a) The radius of circle is 4
(b) The square of diagonal is 4
(c) The square of side is 4
10. The area of the maximum size of the circle described from the 10 square inch square?
11. In the series 0, 3, 8, 15,__ What is the next number?
12. X < 0, Y <> 0 then what is the possibility that the result is always positive?
Ans. xy
13. 3 red and 4 blue balls are in a basket. A member of PPTeam is drawing balls from the basket. What is the probablity
of getting the 3 red balls simultaneously?
14. Let ax2 + bx + c = 0
If the sum of the equal roots is equal to the product of the same roots.Then which of the following hold true

Kumaraguru College, Coimbatore


(a) a + b = 0
(b) a = 0
(c) c = 0
(d) a + c = 0

15. A fold density is 19 times greater than the water and for copper it is 9 times.At what ratio you can mix gold and
copper to get 15 times denser than water.
Ans. 3 : 2
16. Find the value of (1.99)2 Ans. 3.9601
17. There is a room with 6' x 8'. A 1' tile is fixed along the 4 walls in one row. How many 1" tiles require to finish the
work. Ans. 24
18. 2 persons can finish a job in 8 days. First person alone can finish the work in 24 days. How many daysdoes the
second person take to finish the job? Ans. 12 days
19. A 4" cube is painted in all its faces and then it is cut down into 1" blocks. How many 1" blocks are there even
without a single face being painted? Ans. 8
20. A cylinder is inserted in a sphere d/h = 2/3. Find the surface area of the cylinder ?
21. In a car wheel, two spokes cover 15 degree. Then for the entire car,how many spokes are there?
Ans. 24.
22. What is the angle of degree suspended when two hands of clock showing the time 2.30.
Ans. 105 degrees
23. The age difference between two brothers is 3 years. After 6 years the ratio between the age is 9:8 What are their
ages? Ans. 21 and 18
24. A person's salary is getting reduced by 20%. What percentage should be added to get back his original salary?
Ans. 25%
25. Two persons start at the same point, walk in opposite directions with 5km/hr and 5.5km/hr respectively.
What is the distance separated after 2 and half hrs? Ans. 26.25 (approx)
26. A person starts walking at a speed of 5km/hr through half the distance, rest of the distance he covers with a speed
4km/hr. Total time of travel is 9 hours. What is the maximum distance he can cover?
Ans. 40km.
27. Initially two cups of same volume are present with milk filled upto 3/5th and 4/5th of their volumes.Water is then
filled. Then two mixtures are mixed. Find the ratio of water to milk in the mixture.
Ans. 3 : 7
28. 16 grams of radioactive material decays into 8 grams in 10 years. How long will it take to decay to 1 gram ?
Ans. 70 yrs.
29. In a rectangle the length is increased by of the original length . By what proportion should the width be reduced so
that the area will be the same? Ans. 33
30. Find the nth number in the series is 1, -3, 5, -7.___ Ans. (-1)*(2n-1)
31. If a square is formed by the diagonal of the square as an edge, what is the ratio between the area?
Ans. 2
32. The perimeter of a rhombus is 52 units. One of its diagonal is 24 units.What is its second diagonals length?
Ans. 10
33. A cubical rectangular bar has the dimensions with the ratio 5 : 4 : 3. Its volume is 7500. What is the surface area of
the bar? Ans. 2350
34. In a class total 34 students, 16 are have a brother, 15 are have sisters, 9 students don't have either brothers or
sisters.Find the number of students having both brother and sisters.
Ans. 6
35. A batsman scored 18 runs in his 18th innings and that makes his average 18. Find his average upto the 17th innings?
Ans. 19
36. 6 women can do 75 units of work in 8 days by working 5hrs/day. In how many days can 4 women do 30 units of
work by working 8hrs/day ?
37. A persons salary iis decreased by steps of 20%, 15% and 10%. What will be the percentage decrease, if thesalary is
decreased in a single shot?
38. The ratio of the length : breadth : height of a cuboid is 5 : 4: 3, and the volume is 7500. What will be its surface area
?
39. If the circumference of a circle is 100 units, Then what will the length of the arc described by an angle of 20 degree
?

Kumaraguru College, Coimbatore


40. 3 persons started placementpapers with a capital of Rs.3000 . B invest Rs.600 less than A, C invest Rs.300 less than
B. Then what is the share amount of B in a profit of Rs.886 ?

1. In 1978, a kg of paper was sold at Rs25/-.


If the paper rate increases at 1.5% more than the inflation rate which is 6.5% a year,
then what wil be the cost of a kg of paper after 2 years?
(a) 29.12
(b) 29.72
(c) 30.12
(d) 32.65
(e) none of these
2. In A,B,C are having some marbles with each of them.
A has given B and C the same number of marbles each of them already have.
Then, B gave C and A the same number of marbles they already have.
Then C gave A and B the same number of marbles they already have.
At the end A,B,and C have equal number of marbles.
(i) If x,y,z are the marbles initially with A,B,C respectively.
Then the number of marbles B have at the end
(a) 2(x-y-z)
(b) 4(x-y-z)
(c) 2(3y-x-z)
(d) x + y-z Ans. (c)
(ii) If the total number of marbles are 72, then the number of marbles with A at the starting
(a) 20
(b) 30
(c) 32
(d) 39 Ans. (d)
3. If a car starts from A towards B with some velocity. Due to some problem in the engine after travelling 30km, the car
goes with 4/5 th of its actual velocity The car reaches B 45 min later to the actual time. If the car engine fails ofter
travelling 45km, the car reaches the destination B 36min late to the actual time What is the initial velocity of car and
what is the distance between A and B in km Ans. 20 & 130.
4. A person has Rs 100/- in his pocket, he can as 25 pencils or 15 books. He kept 15% of the money for travelling
expenses and purchased 5 pencils. So how many books he can purchase with the remaining money.

6. The values of shares (in Rs).of A, B and C from January to June are as follows.
Month A B C
January 30 60 80
February 35 65 85
March 45 75 65
April 40 75 82
May 55 75 85
June 50 75 80
i) During this period which share has undergone maximium fluctuation?
ii) In which month it is possible to buy B and C selling A?
iii) In which month the share values are very low?
iv) By purchasing one share of A and 4 each of B and C in the beginning of the period,
when shoudl these be sold to get maximum profit?

7. In a computer institute 9 languages can be taught.The module is of 6 months duration and of the six languages only
one can be taught each month. In addition to that BASIC is always taught and should be in first month itself
WORD PERFECT is to be taught in the preceeding week of WORD STAR.
FORTRAN can not be taught until COBAL is taught prior to that
BINO, FIFO can never be taught in single module
languages are BASIC, WORD STAR, WORD PERFECT, FORTRAN, COBAL, BINO, FIFO, LOTUS, C

Kumaraguru College, Coimbatore


i) If word star is in 3rd month , what could be in 6th month.
ii) If COBAL is in the 2nd month and BINO in 6th month. FORTRAN will be taught in which month.
8. In a class, except 18 all are above 50 years.
15 are below 50 years of age. How many people are there
(a) 30
(b) 33
(c) 36
(d) none of these. Ans. (d)
9. A square plate of some size is cut at four corners. Equal squares of the same size are cut and is formed as open box. If
this open box carries 128 ml of oil. What is the size of the side of the plate?
(a) 17
(b) 14
(c) 13
(d) None of these
10. In a square, all the mid points are joined. The inner square is shaded.
If the area of the square is A, what is the area of the shaded area?
11. Two questions on basic angles i.e given a circle, a few chords or diameter is drawn etc.
12. If the follwoing statements are given
@(a,b)= (a+b)/2
/(a,b)= a/b
*(a,b)= ab
If a=1, b=2 then find
i) /(a,(@(a,b),*(a,b)))
ii) */(a,@(*(a,b)))
13. If the follwoing statements are given
(x#y) = x + y- xy
(x*y) = (x + y)/2
i) Find the values of x, y will satisfy this equation (x#y)#(x*y) < (x#y)
ii) Find the values of x, y will satisfy this equation (a*b)#(b*c)< (a#b)*(b*c)

1. If two pencils cost 8 cents, then how much do 5 pencils cost? Ans. 20 cents
2. Some work is done by two people in 24 minutes. One of them can do this work alone in 40 minutes. How much time
does the second person take to do the same work ? Ans. 60 minutes
3. A car is filled with four and half gallons of fuel for a round trip.If the amount of fuel taken while going is 1/4 more
than the amount taken for coming, what is the amount of fuel consumed while coming back? Ans.2
gallons
4. The lowest temperature in the night in a city A is 1/3 more than 1/2 the highest during the day. Sum of the lowest
temperature and the highest temperature is 100 degrees. Then what is the low temp? Ans.40
degrees
5. Javagal, who decided to go to weekened trip should not exceed 8 hours driving in a day. The average speed of
forward journey is 40 miles/hr.Due to traffic on sundays, the return journey's average speed is 30 m/h. How far he can
select a picnic spot?
a) 120 miles
b) between 120 and 140 miles
c) 160 miles Ans. 120 miles
6. A salesperson by mistake multiplied a number and got the answer as 3, instead of dividing the number by 3.What is
the answer he should have actually got? Ans. 3
7. A building with height D shadow upto G. What is the height of a neighbouring building with a shadow of C feet.
Ans. (C*D)/G
8. A person was fined for exceeding the speed limit by 10 mph. Another person was also fined for exceeding the same
speed limit by twice the same. If the second person was travelling at a speed of 35 mph, find the speed limit.
Ans. 15 mph
9. A bus started from bustand at 8.00am, and after staying for 30 minutes at a destination, it returned back to the
busstand. The destination is 27 miles from the busstand. The speed of the bus is 18mph. During the return journey bus

Kumaraguru College, Coimbatore


travels with 50% faster speed.At what time does it return to the busstand? Ans.
11.00am
10. In a mixture, R is 2 parts and S is 1 part. In order to make S to 25% of the mixture, how much of R is to be added?
Ans.One part of R
11. Wind flows 160 miles in 330 min, for travelling 80 miles how much time does it require?
Ans. 2 hrs 45 mins
12. With a 4/5 full tank a vehicle can travel 12 miles, how far can it travel with a 1/3 full tank
Ans. 5 miles
13. There are two trees in a lawn. One grows at a rate 3/5 of the other in 4 years. If the total growth of trees is 8 ft. What
is the height of the smaller tree after 2 years
Ans. 1 1/2 feet
14. Refer to the figure below.A ship started from P and moves at a speed of I miles per hour and another ship starts from
L and moving with H miles per hour simultaneously.Where do the two ships meet? ||---g---||---h---||---i---||---j---||---k---
||---l---||
PG H I J K L are the various stops in between denoted by || . The values g, h, i, j, k, l denote the distance between the
ports. Ans. Between I and J, closer to J
15. If A is travelling at 72 km per hour on a highway. B is travelling at a speed of 25 meters per second on a highway.
What is the difference in their speeds in m/sec. Ans. 1 m/sec

1. There are 150 weights .Some are 1 kg weights and some are 2 kg weights. The sum of the weights is 260.What is the
number of 1kg weights? Ans. 40
2. A is driving on a highway when the police fines him for overspeeding and exceeding the limit by 10 km/hr.At the
same time B is fined for overspeeding by twice the amount by which A exceeded the limit.If he was driving at 35 km/hr
what is the speed limit for the road? Ans. 15 kmph
3. A moves 3 kms east from his starting point . He then travels 5 kms north. From that point he moves 8 kms to the
east.How far is A from his starting point? Ans. 13 kms
4. A car travels 12 kms with a 4/5th filled tank.How far will the car travel with 1/3 filled tank?
Ans. 5 kms
5. The sum of the digits of a two digit number is 8. When 18 is added to the number, the digits are reversed. Find the
number? Ans. 35
6. The cost of one pencil, two pens and four erasers is Rs.22 while the cost of five pencils, four pens and two erasers is
Rs.32.How much will three pencils, three pens and three erasers cost? Ans. 27
7. Fathers age is 5 times his son's age. 4 years back the father was 9 times older than son.Find the fathers' present age.
Ans. 40 years
8. What number should be added to or subtracted from each term of the ratio 17 : 24 so that it becomes equal to 1 : 2.
Ans. 10 should be subtracted
9. What is the 12th term of the series 2, 5, 8, .... Ans. 35
10. If 20 men take 15 days to to complete a job, in how many days can 25 men finish that work?
Ans. 12 days
11. In a fraction, if 1 is added to both the numerator at the denominator, the fraction becomes 1/2. If numerator is
subtracted from the denominator, the fraction becomes 3/4. Find the fraction.
Ans. 3/7
12. If Rs.1260 is divided between between A, B and C in the ratio 2:3:4, what is C's share?
Ans. Rs. 560
13. A shopkeeper bought a watch for Rs.400 and sold it for Rs.500.What is his profit percentage?
Ans. 25%
14. What percent of 60 is 12? Ans. 20%
15. Hansie made the following amounts in seven games of cricket in India: Rs.10, Rs.15, Rs.21, Rs.12, Rs.18, Rs.19
and Rs.17(all figures in crores of course).Find his average earnings. Ans. Rs.16 crore

Q34. A person wants to measures the length of a rod. First he measures with standing ideally then he measures
by moving parallel to the rod

Kumaraguru College, Coimbatore


(a) the length will decrease in second case
(b) length will be same
(c) length will increase in the second case.

Q37. If the word FADENCOMT equals 345687921 then

1. What is FEAT
2. Find representation of 2998

Q38. Given 10 alphabets out of which 5 are to be chosen. How many words can be made with atleast one repetition.

Q40. Find sum of 3 + 5/(1+22) + 7/(1 + 22 + 32) + ...... Ans. 3n/(1 + n)

1. How many degrees hours hand rotate in 10 minutes?


a) 6° b) 5° c) 4° d) None of the above Ans. (b)

2. 1/(1018) - 1/(1020) = ?
a) 99/1016 b) 99/1014 c) 99/1015 d) None of the above Ans. (a)

3. 0 <x < 1: Which is greater ?


a) 1/x2 b) 1/x c) x d) x 2 Ans. (a)

4. c=a/b ; a-1=c What is the relation between a&b.b=?


a) a/(a-1) b) a/(a+1) c) (a+1)/a d) None of the above Ans. (a)

5. The sum of 7 consecutive odd integers with 27 as the fourth number.


a) 183 b) 181 c) 185 d) 179 Ans. (b)

6. For (66666666666666)2 + 888888888888,what is the number at the unit's place


a) 2 b) 4 c) 8 d) None of the above Ans. (d)

7.32736 Express it in product of 3 numbers.


a) 41,42,43 b) 31,32,33 c) 32,34,33 d) 38,38,33 Ans. (b)

8. Radius of sphere is increased by 50%. By how much percentage is surface area is increased.
a) 150% b) 125% c) 128% d) 225% Ans. (b)

9. In which of the following , 2 as a common factor , can be eliminated.


a) log(x2)/log(y2) b) (logx*logx*logx)/(logy*logy*logy)
c) (logx*logx)/logy d) None of the above
Ans. (a)
30. On a single day, 14 children are admitted in a school by their mothers.2 are sisters, 3 are brothers,2 are brother and
sister and 2 are twins. The rest are singles. How many mothers came?
(a) 5 (b) 7 (c) 9 (d) 14

31.It will take 8hrs to fill a cistern. But due to leak at the bottom, it takes 10 hrs to fill it. In how many hrs, the full tank
will be emptied because of the leak.?
(a) 18 (b) 8 (c) 40 (d) 10 Ans. (c)

32. A does alone a piece of work in 4 days. B does the same piece of work alone in 8 days. C does alone a work in 10
days. In how many days will A, B, C together complete the work
(a) 40/19 (b) 19/40 (c) 1/22 (d) 2 Ans. (a)

33. In 3.5 Kg rod , there is 74% silver. If it is alloyed with a 0.5 Kg rod, the % of silver goes up to 84%. The percent of
silver in 0.5 Kg rod?

Kumaraguru College, Coimbatore


34. Two chords of lengths L1 and L2 are drawn in a circle. Their lengths are inversely proportional to the straight
distance joining the centre.Find the radius of circle.

35. A Kg of tea costs Rs 49.50 . But the supplier gives 10 gms less for every Kg he sold. What is the actual cost.
(a) 49.001 (b) 49.005 (c) 49.01 (d) Same Ans. (b)

A
B
C
D
E
A
X
12
8
20
6
B
12
X
12
5
9
C
20
8
X
4
7
D
3
15
6
X
10
E
12
5
8
3
X

A,B,C,D,E denote the stages.


X denotes the start of the stage.
The bus goes from A to E and E to A with back stops at B,C, &D.
For each the charge is Rs.0.70.
The numbers in the table are how many passengers are there in the bus upto that stage.( the numbers given the table are
not
correct)

36. Total no. of passengers in onward journey

37. Total amount in the conductors bag just before the bus reaches the stage C

38. How many Rs. 1.40 tickets are issued to passengers in backward journey.

Kumaraguru College, Coimbatore


39. If the bus breaks down between the stages C & D , the amount refunded to passengers.

40. If the ticket costs Rs.1.50, how much is the profit in Backward journey.

6. The monthly personal maintenance allowance for a family of an employee is determined by the average age of family
and size
of the family. Jacob was drawing rs.570 as PMA after he got married 6 years back.At present he draws Rs 720 as PMA
with
his family of 3 after the birth of son?

a. 2 years
b. 3 years
c. 4 years
d. 5 years

Ans. (d)

7. White collar sells a shirt for an amount. Due to off season sale white collar started offering a discount of 20% on tag
price.
come diwali, white collar offered a further discount of 10% on the reduced price. If i get the shirt for Rs 108 what was
the
original price?

a. Rs.180
b. 160
c. 145
d. none

Ans. (d)

8. A vertical stick 10-cm long casts a shadow 6 cm long on the ground under similar conditions a tower casts a shadow
10m
long determine the height of the tower to the 2nd place of the decimal.

a. 16.67m
b. 17.70m
c. 16.8m
d. none

Ans. (a)

9. In the fig given below ...... which curve rep.the monthly savage(the fig.cannot be recollected by the candidate)

a. LR
b. ST
c. PQ
d. none

Ans. (b)

10. Two poles of height 7m and 12m stand on a play ground. If the distance between their feet is 12m,find the distance
between their tops.

a. 12m
b. 13m
c. 11m
d. none

Kumaraguru College, Coimbatore


Ans. (b)

Direction : (18-20) : In column one are listed certain groups of individual while in column 2 are listedsome of the
characteristics against each group of individuals which are common to each member of that group

Coloumn1 Column 2

BCDFH TU
DEGH QRS
DEF PN
ACDE NS
GDE PVQ
DEG VQRS
CDE VNO
BCD VTS
ABD PT

18. Which characteristcs are found either in 'E' or 'F' or in both but not in 'H'?

a. QRS
b. NOP
c. PVT
d. UVW

Ans. (b)

19. Which character is common to 'B' &amp 'C' but is not present in 'H'?

a. U
b. V
c. W
d. T

Ans. (a)

20. Which characteristics are common to 'C' &amp 'E' but are not present in 'F'?

a. T,V
b. V,W
c. T,U
d. O,P

Ans. (d)

(1) 9 cards are there. u have to arrange them in a 3*3 matrix.


cards are of 4 colors.they are red,yellow,blue,green.
conditions for arrangement: one red card must be in first row
or second row.2 green cards should be in 3rd column.Yellow
cards must be in the 3 corners only. Two blue cards must be in
the 2nd row. Atleast one green card in each row.
Solution:
Yello Red Gren
Blu Blu Gren
Yello Gren Yello
2. 4 cards are placed on a table, each card has two colors. U
don't know the color of the back side of eachcard.4 persons A
B C and D are sitting on the table before the cards. They can

Kumaraguru College, Coimbatore


see Red, Green Red and blue .Out of the 4 poeple 2 always lie.
They see the color on the reverse side and give the following
comment
A: Yello/green
B: Neither Blue/nor Green
c: Blue/Yello
D: Blue/ Yello
find out the color on the other side of the 4 cards.

3.Red and brown tribes [FROM BARRONS GRE] Conditions to


get married with each other.

4. Venn diagram regarding Rich, muscular, soft-skinned,


employed, etc.,( Refer BARRONS GRE GUIDE)

------------------------------------------------------------------------
PART 2.
-------
1. SAKUNTALA DEVI'S PUZZLE BOOK : PUZZLES TO PUZZLE YOU.
problem no: 3. ( Brothers and Sisters)
A family I know has several children. Each boy in this
family has as many sisters as brothers but each girl has
twice as many brothers as sisters. How many brothers
and sisters are there?
ans: 4 boys and 3 girls.
2. No. of animals is 11 more than the no. of birds. If the
no. of birds were the no. of animals and no. of animals
were the no. of birds( ie., interchanging no.s of animals
and birds.), the total no. of legs get reduced by one fifth
(1/5). How many no. of birds and animals were there?
ans: birds:11,animals:22
3. In a soap company a soap is manufactured with 11 parts.
For making one soap you will get 1 part as scrap. At the
end of the day u have 251 such scraps. From that how many
soaps can be manufactured? ans: 22 + 2+ 1 = 25.
4. 2** |
3** | No. 7 does not occur in this
---------------- |
5** | multiplication.
*4* |
**3 | Find the product.
---------------- |
***** |
---------------- |
--------------------------------------------------
ans 2 8 1
322
-----
562
5620
84300
---------
9 0482
---------
5. There is a 5digit no. 3 pairs of sum is eleven each.
Last digit is 3 times the first one.
3 rd digit is 3 less than the second.
4 th digit is 4 more than the second one.

Kumaraguru College, Coimbatore


Find the digit.
ans : 25296.
6. There are five thieves, each loot a bakery one after the
other such that the first one takes 1/2 of the total no.
of the breads plus 1/2 of a bread. Similarly 2nd, 3rd,4th
and 5fth also did the same. After the fifth one no. of
breads remained are 3. Initially how many breads were there?
ans : 31.
7.ESCALATOR PROBLEM OF SAKUNTALA DEVI 'PUZZLES TO PUZZLE'book.
Problem No: Problem 27( Down the escalator)
ans : the no of steps in the stair way : 46.
8.Harbour line and Main line Problem of Sakuntala Devi Puzzle
book. Ans : 4/5.

1. A SOLID ICE OF 11 X 8 X2 INCHES IS MADE INTO ROD OF DIA 4 INCH. WHAT


IS THE LENGE OF ROD?

ANS: 3.5 INCH

2. THERE WERE 750 PEOPLE WHEN THE FIRST SONG WAS SUNG. AFTER EACH
SONG 50 PEOPLE ARE LEAVING THE HALL. HOWMANY SONGS ARE SUNG TO MAKE
THEM ZERO?

ANS:16

3. A PERSON IS CLIMBING OF 60 MTS . FOR EVERY MINUTE HE IS CLIMBING 6 MTS


AND SLIPPING 4 MTS . AFTER HOWMANY MINUTES HE MAY REACH THE TOP?

ANS: (60-6)/2 +1 :28

4. HOWMANY ZEROS ARE THERE IN THE PRODUCT OF THE INTEGER FROM 1TO 100?

ANS: 24( NOT GIVEN)

1 TO 10 -2 ZEROS
21 TO 30 -3 ZEROS : BECAUSE 25 = 5*5
22 *5
24 *5

5. A CAN DO WORK IN 2 HOURS B CAN DO A WORK IN 3 HOURS WHAT IS THE


SHORTEST TIME TYEY CAN FINISH THE WORK?

ANS: 1HOUR 12 MIN.

6..SALARY IS INCREASED BY 1200 ,TAX IS DECREASED FROM 12% TO 10% BUT PAYING
SAME AMOUNT AS TAX . WHAT IS THE PREVISIOUS SALARY?

ANS:6000

7. THE LEAST NO. WHICH IS WHEN DEVIDED BY 4,6,7 LEAVES A REMAINDER OF 2 ?

ANS: 86

Kumaraguru College, Coimbatore


8. A MAN DRIVING THE CAR AT TWICE THE SPEED OF AUTO ONEDAY HE WAS DRIVEN
CAR FOR 10 MIN. AND CAR IS FAILED. HE LEFT THE CAR AND TOOK AUTO TO GOTO
THE OFFICE . HE SPENT 30 MIN. IN THE AUTO. WHAT WILL BE THE TIME TAKE BY
CAR TO GO OFFICE?

ANS:25 MIN

9. A REPORT HS 20 WHEETS, EACH OF 55 LINES AND EACH SU;CH A LINE CONSISTS


OF 65 CHARACTERS. IF THE REPORT HAS TO BE RETYPED WITH EACH SHEET HAVING 65
LINES AND EACH LINE HAVE 75 CHARACTERS, THE PERCENTAGE OF REDUCTION OF
NO OF SHEETS IS CLOSEST IS TO?

ANS: 25%

10. OUT OF 100 FAMILIES IN NEIGHBOUR HOOD , 55 OWN RADIO, 75 OWN T.V
AND 25 OWN VCR. ONLY 10 FAMILIES HAVE ALLOF THESE, AND EACH VCR OWNER
HAS TV . IF 25 FAMILIES HAVE THE RADIO ONLY, THE NO. OF FAMILIES HAVE
ONLY TV ARE?

ANS: 30

) ONE RECTANGULAR PLATE WITH LENGTH 8INCHES,BREADTH 11 INCHES AND


2 INCHES THICKNESS IS THERE.WHAT IS THE LENGTH OF THE CIRCULAR ROD
WITH DIAMETER 8 INCHES AND EQUAL TO VOLUME OF RECTANGULAR PLATE?
ANS: 3.5INCHES
2) WHAT IS THE NUMBER OF ZEROS AT THE END OF THE PRODUCT OF THE NUMBERS
FROM 1 TO 100
3) in some game 139 members have participated every time one fellow will
get bye what is the number of matches to choose the champion to be
held?
ans: 138
4) one fast typist type some matter in 2hr and
another slow typist type the
same matter in 3hr. if both do combinely in how much time they
will finish.
ans: 1hr 12min
5) in 8*8 chess board what is the total number of squares
refer odel
ans:204
6) falling height is proportional to square of the time.
one object falls 64cm in 2sec than in 6sec from how much
height the object will fall.
7) gavaskar average in first 50 innings was 50 . after the 51st
innings his average was 51 how many runs he made in the 51st
innings
8)2 oranges,3 bananas and 4 apples cost Rs.15 . 3 ornages 2 bananas
1 apple costs Rs 10. what is the cost of 3 oranges, 3 bananas and
3 apples ANs Rs 15.
9)in 80 coins one coin is counterfiet what is minimum number of
weighings to find out counterfiet coin

Kumaraguru College, Coimbatore


10)in a company 30% are supervisors and 40% employees are male
if 60% of supervisors are male. what is the probability
that a randomly choosen employee is a male or female?

9. If 2men and 7boys can do a work in 16days.3men and 8boys cand do in


12 days 8menand 8boys ?
ans;6
10. Aperson wants 30% discount. But shopper gave 20% and 12%succesively
discount then the person gets
ans:29.6
11. The distance between A and B stations is 110 km. apart on a st.line.
One train starts at A at 7am. and travels towards B at 20kmph. Another
starts at B at 8am.and travels towards A at a speed of 25kmph. AT
what time they will meet?
ans:10am.
12. 2,2,6,52.5,157.5,630 find the wrong number
ans:replace 52.5 by 45
13. 02,12,28,722,152,312,632 find thef wrong number
ans. replace 28 by 32
14. Air resistance is proportional to square of the velocity at a speed
of 3kmph. the resistance is 0.000045 gm/cm. An aeroplance is
travelling at a speed of 250 kmph. What is the air resistance offered
in kg/mit
ans:0.03125
15. A person gives 10% discount and he revenues 10% in bargaining. The
printed price is 330. What is actual price?
ans:270
16. For what period of time for 1800 at 5% interest will give interest
of 390%
ans:13/3
17. 10 years back thef father age is 7 times of his son. After 2 years
twice the age of the father is equal to 5 times of the son. What are
their ages?
ans;f->38,S->14
18. Train travels 100km. in first 2hrs. and 200 km. in 3hrs. and 150
km. in 2.5hrs. What is the avg. speex?
ans;60kmph
19. A number when divided by 342 gives remainder 47 When the same
number is divided by 19,what would be the remainder ?
ans:9
20. A and B can do work in 6 days. B alone can do in 16 days. After
3 days of combined work B left. In how many days further A completes
remaining work?
ans:24/5 days
21. In a parking place there are 36 vehicles. After first one scooter
is there. AFter 2nd car two scooters, after 34d car and so on....
How many scooters are there in second half of the row?
ans; (YOU PLEASE DO THIS PROBLEM)
22. How many times do the hands of clock concide in a day?
ans;22
23. Fraction and simplification one problem is there. The exact answer
is 7/13.(this damn correct)
24. The least/greatest 4 digit numbers which when divided by 15,27,35,
42 leaves in each case a remainder 7 is

Kumaraguru College, Coimbatore


ans: 1897,9457

1. In a triangle ABC AB=AC then all the following are correct


exept one. Find that.
a) AB<AC+BC b)AC<AB+BC c)BC<AB+AC d) AB+BC>AC+BC

2. Ten years ago the father's age is seven times the son's
age. Two years later father's age is twice the son's. Then the
find the ages.

3. The father's age is 3 times the son's. After 15 years its age
becomes twice the son's. Find the son's age now.

4. Two times which are started from stations A and B which are
seperated by 110 km. One train leaves station A at 7 am. and
travels 20 kmph. The other train leaves station B at 8 am. will
speed of 25 kmph. Then at what time will they meet if they are
travelling in opposite direction.
a) 7 am. b) 10 am. c) 12 noon. d) 11 am.

5) x+2y=2 , 2x-y=4 then


i)x=2 ii)x=0 y=0 iii) x-y=2
a)i only b) ii only c) none d)i and ( ) only

6) In a family E is the father of two sons and a daugher who is


unmarried. Daugher in-law is an officer whose brother_in_law A
is Engineer. C is doctor. The sister of B is Violenist who learnt
Violin from B's wife. Then

6) What is the relationship between A&B. (ans. brothers)

7) Who is the wife of B (ans. officer)

8) What is the relation between E&B. (ans. father and son)

9) In a word COINCIDENCE the 1st letter is interchanged with


3ed letter and 2nd letter with 4th and so on. So what is the
seventh letter from the right after interchanging.
ans is D.

10) Minmise 123658/234586= (ans is 0.52)

11) A and B can do a work in 6 days. B alone can do in 16 days.


After 3 days B withdraws. Then how many days can be needed by A
to do the work alone.

12) Two men and Three boys can do a work in 6 days. 3 men and 5 boys
can do that work in 4 days. Then 8 men and 8 boys can do in how many

Kumaraguru College, Coimbatore


days.

13)
123 means LITTLE BRIGHT BOY and 435 means GIRL IS BRIGHT and 267
means LITTLE THING LLOGAL . Then LITTLE is represented by_______.
ans is 2.

14 to 17 ( R.S Agarwal suggested )


On same relation like HELMET-HEAD then
HIDE _ ? ( BOUND ans.)
18to 22
Assertion and reason following (R.S Agarwal suggested)

23 to 25
Having the opposite meaning of relation given (ENGLISH)

25 TO 30
Statements

30 to 35 ---> Figuring NON VERBAL

36 TO 38 ----> A simillarity between two figures is given.


You have to find the same simillar figure from the choice.

39 to 41 ----> Correction of sentences.

42. How many times a minute & hours hands will meet together in a
day. ans. 22 times.

43. In a total of 36 vehicles after one car there is one scooter.


After 2nd car there will be two scooters and after 3ed car there
will be 3 scooters so on . then find the number of scooters in
the right half of arrangement. ans. 13 check.

44. All Hoopes are Dupes


All fears are liars so
i) All hoopes are fears ii) All dupes are liars

45. & 46. like above.

47). Find the statement which gives correct explanartion to


the contradictory statements.
i). doctor told that "Drug adicts can not have their habit
ii). some surveys indicate that all the drug addicts left their
drug habit.
a) . Doctor do not know about drug addicts.
b). servays does not include every drug addict
c). Drug addiction is a bad habit.
d). will power make people leave their habbit.
48 to 54.

- - a - b b a - b - c like this.
[ simple Qns from R.s.agarwal]

55. A shop keeper selling a product for 10% discount to the market
price . but by bourgaining he got 10% profit of the costprice.
Market price is 300Rs. Find the cost price.

Kumaraguru College, Coimbatore


56. Customer needs 30% discount . But shopkeeper give 20% and 10%
succesively. Then
a). Whether customer got less profit than what he needed.
b).will he got more.
c). no loss
d). none

57. Cost price of two articles is 800. If he sells one article at


3/4 th of cost price and another article at 4/3 rd of cost price
Then he will loss 12 Rs. Then SP of article is.
a). 600,400 b). 500, 400 c). 500,500 d). none

58 What is the longest four digit numbr which when divided by 12,17,35
leavining same remainder 5
a) 8825 b)8725 c) 8025 d) none

59) If travels 300 meters in first two hours and next 200 mts in 3 hours
and 150 meters in 2.5 hours. Find the avg speed.

a) 56 b) 58 c)50 d) 60
ans) 60

1.|x-a| = a-x solve.


ans.x<=a

2.There is a six letterword UGANDA.How many ways u can arrange the letters
in the word in such a way that both A's are together.
ans.120

3.If two cards are taken one after another without replacing from a pack of
cards.what will be the probability for 2 cards to be drawn?
ans.1/13 x1/17

4.51x53x.....x59

ans.99!x25!/2 power 24x 49!x5!

5.The ratio of boys to girls is 6:4.60% of the boys and 40% of girls take
lunch in the canteen.What % of class takes lunch?
ans.52%

6.& 7. 2 simple problems from data sufficency(refer 1 or 2 test from ims)

data sufficiency:
a) only statement a is sufficient
b) only statement b '
c) both are necessary
d) both are not sufficient
7) Anna is the tallest girl
a) anna is taller than all boys
b) one ther similarly appers you have to study it cartefully and the choose the right answer

Kumaraguru College, Coimbatore


8) Zulus always speak truth and hulus always speak lies.There are three persons A,B,and C. A met B and says "Iam zulu
or iam hutus. We don't know what he exactly said then B meets C and says to C that A is a Zulu .Then C replies " No ,A
is a hutu.
q) How many zulus are there? (ans 2)
who must be a Zulu ? and B (check)
A father F has 5 sons p,q,r,s,t Not necessarily in this order. Two are of the same age The eldest and youngest cannot be
twins. T is elder to r and younger to q and s has three older brothers
q 1) who are twins
2) who is the oldest and youngest
3)who is the one having equal number of youngers and elders
8) There are 7 people who take a test among which M is worst. R is disqualified , P and S abtain same marks , T scores
less than S and Q scores less than P , N scores higher than every one.
N P S T Q R M ( may be correct it)
and some questions will come on this

1) A - G are 7 consecutive +ve integers not necessarily in the same order

1) B is the middle number


2) D is 3 less than c
3) the difference between F & A is equal in magnitude and sign
to the difference between E & C
4) Neither F nor C lie between E & G

a) What is the value of B-F

1 2 -1 -2 cannot be determined

b) which is greatest

F C A E cannot be determined

c) Given both A & B are primes what is the lowest value of E

8 6 9 12 cannot

2) Given that a,b,c,d,e each represent one of the digits between


1-9 and that the following multiplication holds

abcde
4
----------
edcba

What digit does e represent

a) 4
b) 6
c) 7
d) 8
e) none

1. How many butes does an array A(1:8,-2:2,1:5) require for storage if


each element of the array is 24 bits long.

Kumaraguru College, Coimbatore


200 480 600 800 none

2. begin

i:=0;
j:=0; | block d

loop:

if(i != 0)
i := i-1;
else
i := i+1;

i := i+1; | block a
j := j+1; | block b

if (j <= 25)
goto loop;

end | block c

a) What is the value of i at [c]


2?
b) How many times is the goto executed
25 ?
c) How many times is the loop executed if i is initialized to 1
in [d] 26
d) How many times is the loop entered if the block [b] is changed
to j=j+1 ?

e) What is the value of i at [c] interchanging blocks [a] and [b] ?


2?

Follow the instructions given below [ From 1 to 8 ]

1. A cause B or C but not both

2. F occurs only if B occurs

3. D occurs if B or C occurs

4. E occurs if only c occurs

5. J occurs only if E or F occurs

6. H occurs if E occurs

7. D causes G, H or Both.

8. G occurs if F occurs.

Questions
---------

Kumaraguru College, Coimbatore


1. If A occurs which of the following may occur

1. F & G (ii) E & H (iii) D

Ans
---
(a) 1 only (b) 2 only (c) 3 only (d) 1,2,3 or 2 & 3 but not 1

(e) 1,2 & 3

2. If B occurs which must occur

Ans
--- (a) F & G (b) D & G (c) D (d) G & H (e) J

3. If J occurs which must occur

Ans
---
(a) E (b) Both E & F (c) Either B or C (d) B (e) Both B & c

4. Which may occur as a result by a cause not mentioned.

(I) D (II) A (III) F

Ans
---
(a) I only (b) II (c) I & II (d) II & III (e) I,II,III

5. If E occurs which cannot occur.

(a) F (b) A (c) D (d) C (e) J

1. Mr.Mathurs jewels have been stolen from his bank locker .


The bank has lockers of 12 people which are arranged in an
array of 3 rows and 4 columns
like:

1234
5678
9 10 11 12

The locker belonging to JONES was to the right of BLACK'S locker


and directly above MILLAR'S.
BOOTH'S locker was directly above MILLAR'S.

Kumaraguru College, Coimbatore


SMITH'S locker was also above GRAY's (though not directly).
GREEN'S locker was directly below SMITH'S.
WILSON'S locker was between that of DAVIS and BOOTH.
MILLAR'S locker was on the bottom row directly to the right of
HERD'S.
WHITE'S locker was on the bottom right hand corner in the same
column as BOOTH'S.
Which box belonged to Mr.Mathurs?

Ans: Box number 9 belongs to Mr.Mathurs.

2. Fifty minutes ago if it was four times as many minutes past


three o'clock,
how many minutes is it to six o'clock?

Ans: Twenty six minutes.

3. If a clock takes 7seconds to strike 7, how long will the same


clock take to strike 10?

Ans: The clock strikes for the first time at the start and takes 7
seconds for 6 intervals-thus
for one interval time
taken=7/6.
Therefore, for 10 seconds there are 9 intervals and time
taken is 9*7/6=10 and
1/2 seconds.

4. Three criminals were arrested for shop lifting.


However, when interrogated only one told the truth in both
his statements, while the other
two each told one true
statement and one lie.
The statements were:

ALBERT :(a)Chander passed the merchandise. (b)Bruce created the


diversion.
BRUCE :(a)Albert passed the merchandise. (b)I created the
diversion.
CLIVE :(a)I took the goods out of the shop. (b)Bruce passed
them over.
Ans: Albert passed the goods.Bruce created the diversion..Clive
took the goods out of the shop.

5. Everyday in his business a merchant had to weigh amounts from 1


kg to 121 kgs,
to the nearest kg.
What are the minimum number of weight required and how heavy
should they be?

Ans: .The minimum number is 5 and they should weigh 1,3,9,27 and
81 kgs.

Kumaraguru College, Coimbatore


6. A hotel has 10 storeys.Which floor is above the floor below the
floor,
below the floor above the floor, below the
floor above the fifth.

Ans: The sixth floor.

7. Seven members sat around a table for three days for a


conference.
The member's names were Abhishek, Amol, Ankur, Anurag,Bhuwan
,Vasu and Vikram.
The meetings were chaired by Vikram.
On the first evening members sat around the table
alphabetically.
On the following two nights, Vikram arranged the seatings so
that he could have
Abhishek as near to him as
possible and abesent minded Vasu as far away as he could.
On no evening did any person have sitting next to him a
person who had previously been
his neighbour.
How did Vikram manage to seat everybody to the best advantage
on the second and third
evenings?

Ans:
Second evening:Vikram,Ankur,Abhishek,Amol,Vasu,Anurag and
Bhuwan.
Third evening :Vikram,Anurag,Abhishek,Vasu,Bhuwan,Ankur,Amol.

8. Two trains start from stations A and B spaced 50 kms apart at


the same time and speed.
As the trains start, a bird flies from one train towards the
other and on reaching the
second train, it flies back to the
first train.This is repeated till the trains collide.
If the speed of the trains is 25 km/h and that of the bird is
100km/h.
How much did the bird travel till the collision.

Ans: 100 kms.

9. Four prisoners escape from a prison.


The prisoners, Mr East, Mr West, Mr South, Mr North head
towards different directions
after escaping.
The following information of their escape was supplied:

The escape routes were The North Road, South Road, East Road and
West Road.
None of the prisoners took the road which was their namesake.
Mr.East did not take the South Road
Mr.West did not the South Road.
The West Road was not taken by Mr.East
What road did each of the prisoners take to make their escape?

Kumaraguru College, Coimbatore


Ans: Mr.East took the North Road
Mr.West took the East Road
Mr.North took the South Road
Mr.South took the West Road.

10. Complete the series:


5, 20, 24, 6, 2, 8, ?

Ans: 12 (as 5*4=20, 20+4=24, 24/4=6, 6-4=2, 2*4=8, 8+4=12).

Interview Section

--------------------------------------------------------------------------------
Questionnaire Index Page Top

Home | Our Services | Eligibility | About Us | Sign Up |


President's Note

--------------------------------------------------------------------------------

Copyright © 2001 Cassius Technologies Pvt Ltd. All rights


reserved.

1) A man collects cigarette stubs and makes one full cigarette


with every 8 stubs.
If he gets 64 stubs how many full cigarettes can he smoke.

Ans: 8+1=9

2) A soldier looses his way in a thick jungle. At random he walks


from his camp but
mathematically in an interesting fashion. First he walks one mile
East then half mile to North.
Then 1/4 mile to West, then 1/8 mile to South and so on making a
loop.
Finally how far he is from his camp and in which direction.

Ans: Distance travelled in north and south directions


1/2 - 1/8 + 1/32 - 1/128 + 1/512 - and so on
= 1/2/((1-(-1/4))
Similarly in east and west directions
1- 1/4 + 1/16 - 1/64 + 1/256 - and so on
= 1/(( 1- ( - 1/4))
Add both the answers

3) How can 1000000000 be written as a product of two factors


neither of them containing zeros

Ans: 2 power 9 x 5 power 9

Kumaraguru College, Coimbatore


4) Conversation between two mathematcians:
First : I have three childern. The product of their ages is 36.
If you sum their ages, it is exactly same as my
neighbour's door number on my left.
The second mathematician verfies the door number and says that it
is not sufficient.
Then the first says " Ok one more clue is that my youngest is
really the youngest". Immmediately the second mathematician
answers .
Can you answer the question asked by the first mathematician?
What are the childeren ages?

Ans 1,6 and 6

5) Light glows for every 13 seconds . How many times did it glow
between 1:57:58 and 3:20:47 am.

Ans : 383 + 1 = 384

6) 500 men are arranged in an array of 10 rows and 50 columns


according to their heights.
Tallest among each row of all are asked to fall out.
And the shortest among them is A.
Similarly after resuming that to their original podsitions
that the shortest
among each column are asked to fall out.
And the tallest among them is B .
Now who is taller among A and B ?

Ans A

7) A person with some money spends1/3 for cloths, 1/5 of the


remaining for food
and 1/4 of the remaining for travel.
He is left with Rs 100/- .
How much did he have with him in the begining ?

Ans: Rs 250/-

8) There are six boxes containing 5 , 7 , 14 , 16 , 18 , 29 balls


of either red or blue
in colour.
Some boxes contain only red balls and others contain only
blue.
One sales man sold one box out of them and then he says
" I have the same number of red balls left out as that of blue
".
Which box is the one he solds out ?

Ans: Total no of balls = 89 and (89-29 /2) = 60/2 = 30


and also 14 + 16 = 5 + 7 + 18 = 30

9) A chain is broken into three pieces of equal lenths containing

Kumaraguru College, Coimbatore


3 links each.
It is taken to a backsmith to join into a single continuous
one .
How many links are to to be opened to make it ?

Ans : 2.

10) Grass in lawn grows equally thick and in a uniform rate.


It takes 24 days for 70 cows and 60 days for 30 cows to
eat the whole of the grass.
How many cows are needed to eat the grass in 96 days.?

Ans : 20
g - grass at the beginning
r - rate at which grass grows, per day
y - rate at which one cow eats grass, per day
n - no of cows to eat the grass in 96 days
g + 24*r = 70 * 24 * y
g + 60*r = 30 * 60 * y
g + 96*r = n * 96 * y
Solving, n = 20.

Section B

1. Replace each letter by a digit.


Each letter must be represented by the same digit and no
beginning letter of a word can be 0.

ONE
ONE
ONE
ONE
-------
TEN

Ans: 0 =1, N = 8 ,E = 2, T = 7

2. Ann, Boobie, Cathy and Dave are at their monthly business


meeting.
Their occupations are author, biologist, chemist and doctor,
but not necessarily in that
order.
Dave just told the biologist that Cathy was on her way with
doughnuts.
Ann is sitting across from the doctor and next to the
chemist.
The doctor was thinking that Boobie was a goofy name for
parent's to choose,
but didn't say anything.
What is each person's occupation?

Ans: Since Dave spoke to the biologist and Ann sat next to the

Kumaraguru College, Coimbatore


chemist and across the doctor, Cathy must be the author
and Ann the biologist.
The doctor didn't speak, but David did, so Bobbie is the
doctor and Dave the chemist.

3. Sometime after 10:00 PM a murder took place.


A witness claimed that the clock must have stopped at the
time of the shooting.
It was later found that the postion of both the hands were
the same but their positions had interchanged.
Tell the time of the shooting (both actual and claimed).

Ans: Time of shooting = 11:54 PM


Claimed Time = 10:59 PM

4. Next number in the series is


1 , 2 , 4 , 13 , 31 , 112 , ?

Ans: 224.
No number has digits more than 4. All of them are 1 , 2,
4, 8 , 16 , 32 , 64 converted to numbers in base 5

5. Shahrukh speaks truth only in the morning and lies in the


afternoon,
whereas Salman speaks truth only in the afternoon.
A says that B is Shahrukh. Is it morning or afternoon and who is
A - Shahrukh or Salman.

Ans: Afternoon ; A is Salman.

6. Two trains starting at same time, one from Bangalore to Mysore


and other in opposite
direction arrive at their destination 1 hr and 4 hours
respectively after passing each other.
How nuch faster is one train from other?

Ans: Twice

7. There are 6 volumes of books on a rack kept in order ( ie


vol.1, vol. 2 and so on ).
Give the position after the following changes were
noticed.

All books have been changed


Vol.5 was directly to the right of Vol.2
Vol.4 has Vol.6 to its left and both weren't at Vol.3's place
Vol.1 has Vol.3 on right and Vol.5 on left
An even numbered volume is at Vol.5's place

Kumaraguru College, Coimbatore


Find the order in which the books are kept now.

Ans: 2 , 5 , 1 , 3 , 6 , 4

8. I bought a car with a peculiar 5 digit numbered licence plate


which on reversing could still
be read. On reversing value is increased by 78633.Whats the
original number if all digits were
different?

Ans: Only 0 1 6 8 and 9 can be readupside down.So on rearranging


these digits we get the
answer as 10968

9. The shape in the sketch below is that of a square attached to


half of a similar square.Divide it into four equal pieces

Ans: Hint : the figure can be divided into 12 equal triangles

10. Supposing a clock takes 7 seconds to strike 7. How mlong will


it take to strike 10?

Ans: 10 1/2 seconds.

Interview Section

--------------------------------------------------------------------------------
Questionnaire Index Page Top

Home | Our Services | Eligibility | About Us | Sign Up |


President's Note

--------------------------------------------------------------------------------

Copyright © 2001 Cassius Technologies Pvt Ltd. All rights


reserved.

1. At 6'o a clock ticks 6 times.


The time between first and last ticks is 30 seconds.
How long does it tick at 12'o clock.

Ans: 66 sec. (2 marks)

2. Three friends divided some bullets equally.


After all of them shot 4 bullets the total number of bullets
remaining is equal

Kumaraguru College, Coimbatore


to the bullets each had after division.
Find the original number divided.

Ans: 18 (2 marks)

Initially . x x x
Now x-4 x-4 x-4
Equation is 3x-12 = x

3. A ship went on a voyage.


After it had travelled 180 miles a plane statrted with 10
times the speed of the ship.
Find the distance when they meet from starting point.

Ans: 200miles. (2 marks)


Distance travelled by plane = 1/10 distance travelled by
ship + 180

4. Complete the Table given below:

Three football teams are there. Given below is the group


table. Fill in the x's

Played Won Lost Draw Goals For Goals Against


A22xxx1
B2xx124
C2xxx37

Ans: The filled table is given below


(4 marks)

Played Won Lost Draw Goals For Goals Against


A220071
B201124
C201137

5. There are 3 societies A, B, C.


A lent cars to B and C as many as they had already.
After some time B gave as many cars to A and C
as many as they have. After sometime c did the same thing.
At the end of this transaction each one of them had 24.
Find the cars each orginally had.

Ans: A had 39 cars, B had 21 cars & C had 12 cars


(4 marks)

6. There N stations on a railroad.


After adding X stations on the rail route 46 additional
tickets have to be printed.
Find N and X.

Ans. x=2 and N=11

Kumaraguru College, Coimbatore


Let initially, N(N-1) = t
After adding, (N+X)(N+X-1) = t+46
By trail and error method (4
marks)

7. Given that April 1 is tuesday.


A, B, C are 3 persons told that their farewell party was on

A - May 8, thursday
B - May 10,tuesday
C - June 5, friday
Out of A, B, C only one made a completetly true statement
concerning date,day and month
The other told two one told the day right and the other the date
right..
What is correct date, month, day.

Ans: B - (May 10) SUNDAY


C - June 6 (Friday).
(5 marks)

8. The Bulls, Pacers, Lakers and Jazz ran for a contest.


Anup, Sujit, John made the following statements regarding
results.

Anup said either Bulls or Jazz will definitely win


Sujit said he is confident that Bulls will not win
John said he is confident that neither Jazz nor Lakers will win
When the result cameit was found that only one of the above three
had made a correct statement.
Who has made the correct statement and who has won the contest.

Ans: Sujith; Lakers (5marks )

9. Five people A ,B ,C ,D ,E are related to each other.


Four of them make one true statement each as follows.

(i) B is my father's brother.


(ii) E is my mother-in-law.
(iii)C is my son-in-law's brother
(iv)A is my brother's wife.

Ans: (i) D
(ii) B
(iii) E
(iv) C (10 marks)

10. Some statements are given below:

L says all of my other four friends have money


M says that P said that exactly one among them has money
N says that L said that precisely two among them have money
O says that M said that three of the others have money
P, L and N said that they have money

Kumaraguru College, Coimbatore


All the above statement are false..
Who has money & who doesn't have any money?

(5 marks)

Interview Section

--------------------------------------------------------------------------------
Questionnaire Index Page Top

Home | Our Services | Eligibility | About Us | Sign Up |


President's Note

--------------------------------------------------------------------------------

Copyright © 2001 Cassius Technologies Pvt Ltd. All rights


reserved.
1. Father's age is three years more than three times the son's
age.
After three years, father's age will be ten years more than
twice the son's age.
What is the father's present age.

Ans: 33 years. (2 marks)

2. Find the values of each of the alphabets.

NOON
SOON
+MOON
----------
JUNE

Ans: 9326 (2 marks)

3. There are 20 poles with a constant distance between each


pole
A car takes 24 second to reach the 12th pole.
How much will it take to reach the last pole.

Ans: 41.45 seconds (2 marks)


Let the distance between two poles = x
Hence 11x:24::19x:?

4. A car is travelling at a uniform speed.


The driver sees a milestone showing a 2-digit number.
After travelling for an hour the driver sees another
milestone with the same digits in reverse order.
After another hour the driver sees another milestone
containing the same two digits.
What is the average speed of the driver.

Kumaraguru College, Coimbatore


Ans: 45 kmph (4 marks)

5. The minute and the hour hand of a watch meet every 65


minutes.
How much does the watch lose or gain time and by how much?

Ans: Gains; 5/11 minutes (4 marks)

6. Ram, Shyam and Gumnaam are friends.


Ram is a widower and lives alone and his sister takes care of
him.
Shyam is a bachelor and his neice cooks his food and looks
after his house.
Gumnaam is married to Gita and lives in large house in the
same town.
Gita gives the idea that all of them could stay together in
the house and share monthly expenses equally.
During their first month of living together, each person
contributed Rs.25.
At the end of the month, it was found that Rs 92 was the
expense so the remaining amount was distributed equally
among everyone.
The distribution was such that everyone recieved a whole
number of Rupees.
How much did each person recieve?

Ans. Rs 2 (4 marks)
(Hint: Ram's sister, Shyam's neice and Gumnaam's wife are
the same person)

7. Four persons A, B, C and D are playing cards.


Each person has one card, laid down on the table below him,
which has two different colours on either side.
The colours visible on the table are Red, Green, Red and
Blue.
They see the color on the reverse side and give the following
comment.

A: Yellow or Green
B: Neither Blue nor Green
C: Blue or Yellow
D: Blue or Yellow

Given that out of the 4 people 2 always lie find out the colours
on the cards each person.

Section B

1. From a vessel, 1/3rd of the liquid evaporates on the first


day.
On the second day 3/4th of the remaining liquid evaporates.
What fraction of the volume is present at the end of the
second day.

Ans: 50%

Kumaraguru College, Coimbatore


2. An orange glass has orange juice and white glass has apple
juice both of equal volumes.
50ml of the orange juice is taken and poured into the apple
juice.
50ml from the white glass is poured into the orange glass.
Of the two quantities, the amount of apple juice in the
orange glass and the amount of orange juice in the white glass,
which one is greater and by how much?

Ans: The two quantities are equal

3. There is a 4 inch cube painted on all sides.


This is cut down into of 1 inch cubes.
What is the no of cubes which have no pointed sides.

Ans: 8

4. Sam and Mala have a conversation.

Sam says I am certainly not over 40


Mala says I am 38 and you are atleast 5 years older than me
Now Sam says you are atleast 39
All the statements by the two are false.
How old are they really?

Ans: Mala = 38 yrs


Sam = 41 yrs.

5. Ram Singh goes to his office in the city, every day from his
suburban house.
His driver Gangaram drops him at the railway station in the
morning and picks him up in the evening.
Every evening Ram Singh reaches the station at 5 O' Clock.
Gangaram also reaches at the same time.
One day Ram Singh started early from his office and came to
the station at 4 O' Clock.
Not wanting to wait for the car he starts walking home.
Mangaram starts at normal time, picks him up on the way
and takes him back house, half an hour early.
How much time did Ram Singh walk?

6. In a railway station, there are two trains going.


One in the harbour line and one in the main line, each having
a frequency of 10 minutes.
The main line service starts at 5 o'clock and the harbour line
starts at 5.02A.M.
A man goes to the station every day to catch the first train
that comes.
What is the probability of the man catching the first train?

Ans: 0.8

Kumaraguru College, Coimbatore


7. A family X went for a vacation.
Unfortunately it rained for 13 days when they were there.
But whenever it rained in the mornings, they had clear
afternoons and vice versa.
In all they enjoyed 11 mornings and 12 afternoons.
How many days did they stay there totally?

Ans: 18

8. A survey was taken among 100 people to find their preference of


watching T.V. programmes.
There are 3 channels. Given the no of people who watch

at least channel 1
at least channel 2
at least channel 3
no channels at all
atleast channels 1and 3
atleast channels 1 and 2
atleast channels 2 and 3
Find the no of people who watched all three.

9. Albert and Fernandes have two leg swimming race.


Both start from opposite ends of the pool.
On the first leg, the boys pass each other at 18 m from the
deep end of the pool.
During the second leg they pass at 10 m from the shallow
end of the pool.
Both go at constant speed but one of them is faster.
Each boy rests for 4 seconds at the end of the first leg.
What is the length of the pool?

10. Each alphabet stands for one digit in the following


multiplication.

THIS
x IS
---------
XFXX
XXUX
------------
XXNXX
------------

What is the maximum value T can take?


Interview Section
1.An escalator is descending at constant speed.
A walks down and takes 50 steps to reach the bottom.
B runs down and takes 90 steps in the same time as A takes 10
steps.
How many steps are visible when the escalator is not
operating?

Ans: 150 steps

Kumaraguru College, Coimbatore


2. Every day a cyclist meets a train at a particular crossing.
The road is straight before the crossing and both are
travelling in the same direction.
The cyclist travels with a speed of 10 Kmph.
One day the cyclist comes late by 25 min. and meets the train
5km before the crossing.
What is the speed of the train?

Ans: 60 kmph

3. There are five persons with surnames Mukherjee, Misra, Iyer,


Patil and Sharma.
There are 4 persons having first or middle name of Kumar, 3
persons with Mohan, 2 persons with Dev and 1 Anil.
Either Mukherjee and Patil have a first or middle name of Dev
or Misra and Iyer have their first or middle name of Dev.
Of Mukherkjee and Misra, either both of them have a first or
middle name of Mohan or neither have a first or
middle name of Mohan.
Either Iyer or Sharma has a first or middle name of Kumar but
not both.
Who has the first or middle name of Anil?

Ans: Kumar Misra Dev


Mohan Iyer Dev
Kumar Patil Mohan
Mohan Sharma Kumar

4. Boys are allowed to watch football at C.V.Raman auditorium


subjected to conditions.

The boy over age 16 can wear overcoat


No boy over age 15 can wear cap
To watch the football either he has to wear overcoat or cap or
both
A boy with an umberella or above 16 or both cannot wear sweater.
Boys must either not watch football or wear sweater.
What is the appearence of the boy who is watching football.

5. A bird keeper has got P pigeons, M mynas and S sparrows.


The keeper goes for lunch leaving his assistant to watch the
birds.

Suppose p=10, m=5, s=8 when the bird keeper comes back, the
assistant informs the x birds have escaped. The bird keeper
exclaims: "Oh no! All my sparrows are gone."
How many birds flew away?
When the bird keeper comes back, the assistant told him that x
birds have escaped. The keeper realised that atleast 2 sparrows
have escaped.
What is minimum no of birds that can escape?

6. Answer the following questions based on the conditions from the

Kumaraguru College, Coimbatore


choices A, B, C, D, E as described below:
(A) if a definete conclusion can be drawn from condition 1
(B) if a definete conclusion can be drawn from condition 2
(C) if a definete conclusion can be drawn from condition 1 and 2
(D) if a definete conclusion can be drawn from condition 1 or 2
(E) no conclusion can be drawn using both conditions

person 1 says N<5


person says N>5
person 3 says 3N>20
person 4 says 3N>10
person 5 says N<8
What is the value of N?

a) 1. No of persons who speak false being less than no of persons


who tells the truth.
2. Person 2 is telling the truth.

b) 1. no of persong telling the truth is greater than no of


persons telling lies
2. person 5 is telling the truth.

7. There are N coins on a table and there are two players A & B.
You can take 1 or 2 coins at a time.
The person who takes the last coin is the loser.
A always starts first.

If N=7
(a) A can always win by taking two coins in his first chanse
(b) B can win only if A takes two coins in his first chance.
(c) B can always win by proper play
(d) none of the above

2. A can win by proper play if N is equal to


(a) 13 (b) 37 (c) 22 (d) 34 (e) 48
Ans: (e.)

3. B can win by proper play if N is equal to


(a) 25 (b)26 (c) 32 (d) 41 (e) none

4. if N<4, can A win by proper play always?


(a) Yes (b) No

8. Two twins have vertain peculiar charcteristics.


One of them always lies on Monday, Wednesday, Friday.
The other always lies on Tuesdays, Thursdays and Saturdays.
On the other days they tell the truth. You are given a
conversation.

Person A-- today is Sunday my name is Anil


Person B -- today is Tuesday, my name is Bill
What day is today?

Ans: Today is Tuesday.

Kumaraguru College, Coimbatore


9. There is a safe with a 5 digit number as the key.
The 4th digit is 4 greater than second digit, while 3rd digit
is 3 less than 2nd digit.
The 1st digit is thrice the last digit.
There are 3 pairs whose sum is 11.
Find the number.

Ans: 65292

10. A hotel has two wings,the east wing and the west wing.
Some east wing rooms but not all have an ocean view.
All west wing rooms have a harbour view.
The charge for all rooms is identical, except as follows
:

Extra charge for all harbour view rooms on or above the 3rd
floor
Extra charge for all ocean view rooms except those without
balcony
Extra charge for some harbour rooms on the first two floor & some
east wing rooms without ocean view but
having kitchen facilities.
Which of the following cannot be determined on the basis of the
nformation given:

I. Whether there are any rooms without a balcony for which an


extra charge is imposed.
II. Whether any room without a kitchen or a view involves an
extra charge.
III. Whether two extra charges are impsed for any room.

(A) I only
(B) II only
(C) III only
(D) II and III only
(E) I, II and III

(This question is from 1999 Barrons GRE Guide model Test 3 -


section 6, question 22)

Ans: (A)

Interview Section

--------------------------------------------------------------------------------
Questionnaire Index Page Top

Home | Our Services | Eligibility | About Us | Sign Up |


President's Note

--------------------------------------------------------------------------------

Copyright © 2001 Cassius Technologies Pvt Ltd. All rights


reserved.

Kumaraguru College, Coimbatore


1. Three friends divided some bullets equally. After all of them
shot 4 bullets the total number of bullets remaining is equal to
the bullets each had after division. Find the original number
divided.
Ans: 18 ( Initially . x x
x
Now x-4 x-4
x-4 ) Equation is 3x-12 = x

2. A ship went on a voyage. After it had traveled 180 miles a


plane started with 10 times the speed of the ship.
Find the distance when they meet from starting point.
Ans: 200miles. ( Distance traveled by plane =
1/10 distance traveled by ship + 180 )

3. Replace each letter by a digit. Each letter must be represented


by the same digit and no beginning letter of a word can be 0.
ONE
ONE
ONE
ONE
TEN
Ans: O =1, N = 8 ,E = 2, T = 7

4. In a railway station, there are two trains going. One in the


harbor line and one in the main line, each having a frequency of
10 minutes. The main line service starts at 5 o'clock and the
harbor line starts at 5.02A.M. A man goes to the station every day
to catch the first train that comes. What is the probability of
the man catching the first train?
Ans: 0.8

5. Next number in the series is: 1 , 2 , 4 , 13 , 31 , 112 ,


?
Ans: 224. (No number has digits more than 4. All
of them are 1 , 2, 4, 8 , 16 , 32 , 64 converted to base 5 )

6. Father's age is three years more than three times the son's
age. After three years, father's age will be ten years more than
twice the son's age. What is the father's present age?
Ans: 33 years.

7. Light glows for every 13 seconds . How many times did it glow
between 1:57:58 and 3:20:47 am.
Ans : 383 + 1 = 384

8. From a vessel, 1/3rd of the liquid evaporates on the first day.


On the second day 3/4th of the remaining liquid evaporates. What
fraction of the volume is present at the end of the second day.
Ans: 50%

9. Supposing a clock takes 7 seconds to strike 7. How long will it


take to strike 10?
Ans: 10 1/2 seconds.

10. There are 20 poles with a constant distance between each


pole. A car takes 24 second to reach the 12th pole.
How much will it take to reach the last pole.

Kumaraguru College, Coimbatore


Ans: 41.45 seconds (Let the distance
between two poles = x, Hence 11x:24::19x:? )

11. How can 1000000000 be written as a product of two factors


neither of them containing zeros
Ans: 2 power 9 x 5 power 9

12. Two trains starting at same time, one from Bangalore to Mysore
and other in opposite direction arrive at their
destination 1 hr and 4 hours respectively after passing each
other. How much faster is one train from other?
Ans: Twice

13. Every day a cyclist meets a train at a particular crossing.


The road is straight before the crossing and both are traveling in
the same direction. The cyclist travels with a speed of 10kmph.
One day the cyclist comes late by 25 min. and meets the train 5km
before the crossing. What is the speed of the train?
Ans: 60kmph

14. A man collects cigarette stubs and makes one full cigarette
with every 8 stubs. If he gets 64 stubs how many full cigarettes
can he smoke.
Ans: 8+1=9

15. The minute and the hour hand of a watch meet every 65 minutes.
How much does the watch lose or gain time and by how much?
Ans: Gains; 5/11 minutes

16. A survey was taken among 100 people to find their preference
of watching T. V. programs. There are 3 channels. Given the no of
people who watch

at least channel 1
at least channel 2
at least channel 3
no channels at all
at least channels 1and 3
at least channels 1 and 2
at least channels 2 and 3
Find the no of people who watched all three.
Ans.

17. Two trains start from stations A and B spaced 50kms apart at
the same time and speed. As the trains start, a bird flies from
one train towards the other and on reaching the second train, it
flies back to the first train. This is repeated till the trains
collide. If the speed of the trains is 25 km/h and that of the
bird is 100km/h. How much did the bird travel till the
collision.
Ans: 100kms.

18. Four persons A, B, C and D are playing cards. Each person has
one card, laid down on the table below him, which has two
different colors on either side. The colors visible on the table
are Red, Green, Red and Blue. They see the color on the reverse
side and give the following comment.
A: Yellow or Green

Kumaraguru College, Coimbatore


B: Neither Blue nor Green
C: Blue or Yellow
D: Blue or Yellow
Given that out of the 4 people 2 always lie find out the
colors on the cards each person.
Ans.

19. Sometime after 10:00 PM a murder took place. A witness claimed


that the clock must have stopped at the time of the shooting. It
was later found that the position of both the hands were the same
but their positions had interchanged.
Tell the time of the shooting (both actual and claimed).
Ans: Time of shooting = 11:54 PM
Claimed Time = 10:59 PM

20. Some statements are given below:

L says all of my other four friends have money


M says that P said that exactly one among them has money
N says that L said that precisely two among them have money
O says that M said that three of the others have money
P, L and N said that they have money
All the above statement are false. Who has money & who doesn't
have any money?
Ans.

21. The Bulls, Pacers, Lakers and Jazz ran for a contest. Anup,
Sujit, John made the following statements regarding results.

Anup said either Bulls or Jazz will definitely win


Sujit said he is confident that Bulls will not win
John said he is confident that neither Jazz nor Lakers will win
When the result came, it was found that only one of the above
three had made a correct statement. Who has made the correct
statement and who has won the contest.
Ans: Sujith; Lakers

22. There are five persons with surnames Mukherjee, Misra, Iyer,
Patil and Sharma. There are 4 persons having first or middle name
of Kumar, 3 persons with Mohan, 2 persons with Dev and 1 Anil.
Either Mukherjee and Patil have a first or middle name of Dev or
Misra and Iyer have their first or middle name of Dev. Of
Mukherkjee and Misra, either both of them have a first or middle
name of Mohan or neither have a first or middle name of Mohan.
Either Iyer or Sharma has a first or middle name of Kumar but not
both. Who has the first or middle name of Anil?
Ans: Kumar Misra Dev, Mohan Iyer Dev, Kumar Patil
Mohan, Mohan Sharma Kumar

23. Ann, Boobie, Cathy and Dave are at their monthly business
meeting. Their occupations are author, biologist, chemist and
doctor, but not necessarily in that order. Dave just told the
biologist that Cathy was on her way with doughnuts. Ann is sitting
across from the doctor and next to the chemist. The doctor was
thinking that Boobie was a goofy name for parent's to choose, but
didn't say anything. What is each person's occupation?
Ans: Since Dave spoke to the biologist and Ann sat
next to the chemist and across the doctor, Cathy must be the

Kumaraguru College, Coimbatore


author and Ann the biologist. The doctor didn't speak, but David
did, so Bobbie is the doctor and Dave the chemist.

24. There are 6 volumes of books on a rack kept in order ( i.e.


vol.1, vol. 2 and so on ). Give the position after the following
changes were noticed.

All books have been changed


Vol.5 was directly to the right of Vol.2
Vol.4 has Vol.6 to its left and both weren't at Vol.3's place
Vol.1 has Vol.3 on right and Vol.5 on left
An even numbered volume is at Vol.5's place
Find the order in which the books are kept now.
Ans: 2 , 5 , 1 , 3 , 6 , 4

25. A soldier looses his way in a thick jungle. At random he walks


from his camp but mathematically in an interesting fashion. First
he walks one mile East then half mile to North. Then 1/4 mile to
West, then 1/8 mile to South and so on making a loop. Finally how
far he is from his camp and in which direction.
Ans: Distance traveled in north and south
directions:
1/2 - 1/8 + 1/32 - 1/128 + 1/512 -
............. = 1/2/((1-(-1/4))
Similarly in east and west directions:
1- 1/4 + 1/16 - 1/64 + 1/256 -
................... = 1/(( 1- ( - 1/4)) Add both the answers

26. Conversation between two mathematicians:


First : I have three children. The product of their
ages is 36. If you sum their ages, it is exactly same as my
neighbor's door number on my left.
The second mathematician verifies the door number and says
that it is not sufficient. Then the first says " Ok one more clue
is that my youngest is really the youngest". Immediately the
second mathematician answers. Can you answer the question asked by
the first mathematician? What are the children ages?
Ans 1,6 and 6

27. 500 men are arranged in an array of 10 rows and 50 columns


according to their heights. Tallest among each row of all are
asked to fall out. And the shortest among them is A. Similarly
after resuming that to their original positions that the shortest
among each column are asked to fall out. And the tallest among
them is B. Now who is taller among A and B ?
Ans. A

28. There are six boxes containing 5 , 7 , 14 , 16 , 18 , 29


balls of either red or blue in color. Some boxes contain only red
balls and others contain only blue. One sales man sold one box out
of them and then he says, " I have the same number of red balls
left out as that of blue ". Which box is the one he sold out ?
Ans: Total no of balls = 89 and (89-29 /2) =
60/2 = 30 and also 14 + 16 = 5 + 7 + 18 = 30

29. Ram Singh goes to his office in the city, every day from his
suburban house. His driver Gangaram drops him at the railway
station in the morning and picks him up in the evening. Every

Kumaraguru College, Coimbatore


evening Ram Singh reaches the station at 5 O' Clock. Gangaram also
reaches at the same time. One day Ram Singh started early from his
office and came to the station at 4 O' Clock. Not wanting to wait
for the car he starts walking home. Mangaram starts at normal
time, picks him up on the way and takes him back house, half an
hour early. How much time did Ram Singh walk?
Ans.

30. A family X went for a vacation. Unfortunately it rained for 13


days when they were there. But whenever it rained in the mornings,
they had clear afternoons and vice versa. In all they enjoyed 11
mornings and 12 afternoons. How many days did they stay there
totally?
Ans: 18

31. There are N coins on a table and there are two players A & B.
You can take 1 or 2 coins at a time. The person who takes the last
coin is the loser. A always starts first.

If N=7
(a) A can always win by taking two coins in his first chance
(b) B can win only if A takes two coins in his first chance.
(c) B can always win by proper play
(d) none of the above
Ans.

2. A can win by proper play if N is equal to


(a) 13 (b) 37 (c) 22 (d) 34 (e) 48
Ans: (e.)

3. B can win by proper play if N is equal to


(a) 25 (b)26 (c) 32 (d) 41 (e) none
Ans.

4. if N<4, can A win by proper play always?


(a) Yes (b) No
Ans.
32. Mr. Mathurs jewels have been stolen from his bank locker. The
bank has lockers of 12 people which are arranged in an array of 3
rows and 4 columns like:

1234
5678
9 10 11 12

The locker belonging to JONES was to the right of BLACK'S locker


and directly above MILLAR'S.
BOOTH'S locker was directly above MILLAR'S.
SMITH'S locker was also above GRAY's (though not directly).
GREEN'S locker was directly below SMITH'S.
WILSON'S locker was between that of DAVIS and BOOTH.
MILLAR'S locker was on the bottom row directly to the right of
HERD'S.
WHITE'S locker was on the bottom right hand corner in the same
column as BOOTH'S.
Which box belonged to Mr. Mathurs?
Ans: Box number 9 belongs to Mr. Mathurs.

Kumaraguru College, Coimbatore


33. Five people A ,B ,C ,D ,E are related to each other. Four of
them make one true statement each as follows.
(i) B is my father's brother.
(ii) E is my mother-in-law.
(iii) C is my son-in-law's brother
(iv) A is my brother's wife.
Ans: (i) D (ii) B (iii) E
(iv) C

Interview Section

--------------------------------------------------------------------------------
Questionnaire Index Page Top

Home | Our Services | Eligibility | About Us | Sign Up |


President's Note

--------------------------------------------------------------------------------

Copyright © 2001 Cassius Technologies Pvt Ltd. All rights


reserved.

There is a 4 inch cube painted on all sides. This is cut down


into of 1 inch cubes. What is the no of cubes which have no
pointed sides.
Ans: 8

2. At 6'o a clock ticks 6 times. The time between first and last
ticks is 30 seconds. How long does it tick at 12'o clock.
Ans: 66 sec.

3. Complete the series: 5, 20, 24, 6, 2, 8, ?


Ans: 12 (as 5*4=20, 20+4=24, 24/4=6, 6-4=2,
2*4=8, 8+4=12).

4. Find the values of each of the alphabets.


NOON
SOON
+ MOON
JUNE
Ans: 9326

5. If a clock takes 7seconds to strike 7, how long will the same


clock take to strike 10?
Ans: The clock strikes for the first time at
the start and takes 7 seconds for 6 intervals-thus for one
interval time taken=7/6. Therefore, for 10 seconds there are 9
intervals and time taken is 9*7/6=10 and 1/2 seconds.

6. An escalator is descending at constant speed. A walks down and


takes 50 steps to reach the bottom. B runs down and takes 90 steps
in the same time as A takes 10 steps. How many steps are visible
when the escalator is not operating?
Ans: 150 steps

7. A chain is broken into three pieces of equal lengths containing

Kumaraguru College, Coimbatore


3 links each. It is taken to a blacksmith to join into a single
continuous one. How many links are to to be opened to make it ?
Ans : 2.

8. There is a safe with a 5 digit number as the key. The 4th digit
is 4 greater than second digit, while 3rd digit is 3 less than 2nd
digit. The 1st digit is thrice the last digit. There are 3 pairs
whose sum is 11. Find the number.
Ans: 65292

9. An orange glass has orange juice and white glass has apple
juice both of equal volumes. 50ml of the orange juice is taken and
poured into the apple juice. 50ml from the white glass is poured
into the orange glass. Of the two quantities, the amount of apple
juice in the orange glass and the amount of orange juice in the
white glass, which one is greater and by how much?
Ans: The two quantities are equal

10. The shape in the sketch below is that of a square attached to


half of a similar square. Divide it into four equal pieces

Ans: Hint : the figure can be divided into 12


equal triangles

11. Fifty minutes ago if it was four times as many minutes past
three o'clock, how many minutes is it to six o'clock?
Ans: Twenty six minutes.

12. Everyday in his business a merchant had to weigh amounts from


1 kg to 121kgs, to the nearest kg.
What are the minimum number of weight required and how
heavy should they be?
Ans: .The minimum number is 5 and they should
weigh 1,3,9,27 and 81kgs.

13. A car is traveling at a uniform speed. The driver sees a


milestone showing a 2-digit number. After traveling for an hour
the driver sees another milestone with the same digits in reverse
order. After another hour the driver sees another milestone
containing the same two digits. What is the average speed of the
driver.
Ans: 45kmph

14. A hotel has 10 storeys. Which floor is above the floor below
the floor, below the floor above the floor, below the
floor above the fifth.
Ans: The sixth floor.

15. Albert and Fernandes have two leg swimming race. Both start
from opposite ends of the pool. On the first leg, the boys pass
each other at 18 m from the deep end of the pool. During the
second leg they pass at 10 m from the shallow end of the pool.
Both go at constant speed but one of them is faster. Each boy
rests for 4 seconds at the end of the first leg. What is the

Kumaraguru College, Coimbatore


length of the pool?
Ans.

16. Shahrukh speaks truth only in the morning and lies in the
afternoon, whereas Salman speaks truth only in the afternoon. A
says that B is Shahrukh. Is it morning or afternoon and who is A -
Shahrukh or Salman.
Ans: Afternoon ; A is Salman.

17. A person with some money spends1/3 for cloths, 1/5 of the
remaining for food and 1/4 of the remaining for travel.
He is left with Rs 100/- . How much did he have with him in
the beginning ?
Ans: Rs 250/-

18. Ram, Shyam and Gumnaam are friends.


Ram is a widower and lives alone and his sister takes care of
him.
Shyam is a bachelor and his niece cooks his food and looks
after his house.
Gumnaam is married to Gita and lives in large house in the
same town.
Gita gives the idea that all of them could stay together in
the house and share monthly expenses equally.
During their first month of living together, each
person contributed Rs.25. At the end of the month, it was found
that Rs 92 was the expense so the remaining amount was distributed
equally among everyone. The distribution was such that everyone
received a whole number of Rupees. How much did each person
receive?
Ans. Rs 2 (Hint: Ram's sister, Shyam's niece
and Gumnaam's wife are the same person)

19. There are 3 societies A, B, C. A lent cars to B and C as many


as they had already. After some time B gave as many tractors to A
and C as many as they have. After sometime c did the same thing.
At the end of this transaction each one of them had 24. Find the
cars each originally had.
Ans: A had 39 cars, B had 21 cars & C had 12
cars

20. Sam and Mala have a conversation.

Sam says I am certainly not over 40


Mala says I am 38 and you are at least 5 years older than me
Now Sam says you are at least 39
All the statements by the two are false. How old are they
really?
Ans: Mala = 38 yrs, Sam = 41 yrs.

21. Each alphabet stands for one digit in the following


multiplication.
THIS
x IS
XFXX
XXUX
XXNXX What is the maximum value T can
take?

Kumaraguru College, Coimbatore


Ans: T max value = 4

22. Grass in lawn grows equally thick and in a uniform rate. It


takes 24 days for 70 cows and 60 days for 30 cows to eat the whole
of the grass. How many cows are needed to eat the grass in 96
days.?
Ans : 20
[Hint: g - grass at the beginning
r - rate at which grass grows, per
day
y - rate at which one cow
eats grass, per day n - no of cows to eat the grass in 96
days
g + 24*r = 70 * 24 * y
g + 60*r = 30 * 60 * y
g + 96*r = n * 96 * y,
Solving, n = 20. ]

23. Three criminals were arrested for shop lifting. However, when
interrogated only one told the truth in both his statements, while
the other two each told one true statement and one lie. The
statements were:

ALBERT :(a) Chander passed the merchandise. (b) Bruce created the
diversion.
BRUCE :(a) Albert passed the merchandise. (b) I created the
diversion.
CLIVE :(a) I took the goods out of the shop. (b) Bruce
passed them over.
Ans: Albert passed the goods. Bruce created the
diversion. Clive took the goods out of the shop.

24. I bought a car with a peculiar 5 digit numbered license plate


which on reversing could still be read. On reversing value is
increased by 78633.Whats the original number if all digits were
different?
Ans: Only 0 1 6 8 and 9 can be read upside down.
So on rearranging these digits we get the answer as 10968

25. There N stations on a railroad. After adding X stations on the


rail route 46 additional tickets have to be printed.
Find N and X.
Ans. x=2 and N=11 ( Let initially, N(N-1) = t;
After adding, (N+X)(N+X-1) = t+46;Trail and error method )

26. Complete the Table given below:


Three football teams are there. Given below is the group
table. Fill in the x's

Played Won Lost Draw Goals For Goals Against


A22xxx1
B2xx124
C2xxx37

Ans: The filled table is given below

Kumaraguru College, Coimbatore


Played Won Lost Draw Goals For Goals Against
A220071
B201124
C201137

27. A bird keeper has got P pigeons, M mynas and S sparrows. The
keeper goes for lunch leaving his assistant to watch the birds.

Suppose p=10, m=5, s=8 when the bird keeper comes back, the
assistant informs the x birds have escaped. The bird keeper
exclaims: "Oh no! All my sparrows are gone."
How many birds flew away?
When the bird keeper comes back, the assistant told him that x
birds have escaped. The keeper realized that at least 2 sparrows
have escaped.
What is minimum no of birds that can escape?
Ans.

28. Seven members sat around a table for three days for a
conference.
The member's names were Abhishek, Amol, Ankur, Anurag, Bhuwan
,Vasu and Vikram.
The meetings were chaired by Vikram.
On the first evening members sat around the table
alphabetically.
On the following two nights, Vikram arranged the seating so
that he could have Abhishek as near to him as
possible and absent minded Vasu as far away as he could.
On no evening did any person have sitting next to him a
person who had previously been his neighbor.
How did Vikram manage to seat everybody to the best advantage
on the second and third evenings?
Ans: Second evening: Vikram, Ankur, Abhishek,
Amol, Vasu, Anurag and Bhuwan.
Third evening :Vikram, Anurag,
Abhishek, Vasu, Bhuwan, Ankur, Amol.

29. Two twins have certain peculiar characteristics. One of them


always lies on Monday, Wednesday, Friday. The other always lies on
Tuesdays, Thursdays and Saturdays. On the other days they tell the
truth. You are given a conversation.

Person A-- today is Sunday my name is Anil


Person B -- today is Tuesday, my name is Bill
What day is today?
Ans: Today is Tuesday.

30. Four prisoners escape from a prison. The prisoners, Mr. East,
Mr. West, Mr. South, Mr. North head towards different directions
after escaping. The following information of their escape was
supplied:

The escape routes were The North Road, South Road, East Road and
West Road.
None of the prisoners took the road which was their namesake.
Mr. East did not take the South Road
Mr. West did not the South Road.
The West Road was not taken by Mr. East

Kumaraguru College, Coimbatore


What road did each of the prisoners take to make their
escape?
Ans: Mr. East took the North Road
Mr. West took the East Road
Mr. North took the South Road
Mr. South took the West Road.

31. A hotel has two wings, the east wing and the west wing. Some
east wing rooms but not all have an ocean view.
All west wing rooms have a harbor view. The charge for all
rooms is identical, except as follows :

Extra charge for all harbor view rooms on or above the 3rd floor
Extra charge for all ocean view rooms except those without
balcony
Extra charge for some harbor rooms on the first two floor & some
east wing rooms without ocean view but
having kitchen facilities.
Which of the following cannot be determined on the basis of the
information given:
I. Whether there are any rooms without a balcony for
which an extra charge is imposed.
II. Whether any room without a kitchen or a view
involves an extra charge.
III. Whether two extra charges are imposed for any
room.
(A) I only (B) II only (C) III
only (D) II and III only (E) I, II
and III
Ans: (A)

32. Given that April 1 is Tuesday. A, B, C are 3 persons told that


their farewell party was on

A - May 8, Thursday
B - May 10, Tuesday
C - June 5, Friday
Out of A, B, C only one made a completely true statement
concerning date, day and month. The other told two one told the
day right and the other the date right. What is correct date,
month, day.
Ans: B - (May 10) SUNDAY , C - June 6
(Friday).

33. Answer the following questions based on the conditions from


the choices A, B, C, D, E as described below:
(A) if a definite conclusion can be drawn from condition
1
(B) if a definite conclusion can be drawn from condition
2
(C) if a definite conclusion can be drawn from condition
1 and 2
(D) if a definite conclusion can be drawn from condition
1 or 2
(E) no conclusion can be drawn using both conditions

person 1 says N<5


person says N>5

Kumaraguru College, Coimbatore


person 3 says 3N>20
person 4 says 3N>10
person 5 says N<8
What is the value of N?
a) 1. No of persons who speak false being less than no of persons
who tells the truth.
2. Person 2 is telling the truth.
Ans.
b) 1. no of persons telling the truth is greater than no of
persons telling lies
2. person 5 is telling the truth.
Ans.

1. In 1978, a kg of paper was sold at Rs25/-. I f the


paper rate
increases at 1.5% more than inflation rate which is of
6.5% a year
, then what wil be the cost of a kg of paper after 2
years?
a)29.12 (b) 29.72 (c) 30.12 (d) 32.65 (e) none of these

2. In A,B,C are having some marbles with each of them. A


has giben
B and C the same number of marbles they already have to
each of them.
then, B gave C and A the same no. of marbles they have,
then C gave
A and B the same no. of marbles they have. At the
end A,B,and C have equal no. of marbles.
(i) If x,y,z are the marbles initially with A,B,C
respectively.
then the no of marbles B have at the end
(a) 2(x-y-z) (b) 4(x-y-z) etc.

(ii)If the total no. of marbles are 72, then the


no. of marbles with A at the starting
a. 20 b. 30 c. 32

3. If a car starts from A towards B with some velocity due


to some

Kumaraguru College, Coimbatore


problem in the engine after travelling 30km.If the car
goes with
4/5 th of its actuval velocity the car reaches B 45min
later to the
actual time. If the car engine fails ofter travelling
45km, the car reaches the destination B 36min late to the
actual
time , what is the initial velocity of car and what is the
distance between A and B in km
ans) 20 & 130.

4. A person has Rs 100/- in his pocket, he can as 25


pencils or 15books.
He kept 15% of the money for travelling expenses and
purchased 5
pencils.So how many books he can purchase with the
remaining money.

5. ten questions on analogies.


ex: hammer : carpenter ::
B
knife : butcher.

6. The values of shares A,B and C from january to june are


as follows.
OB
month A B C

JAN 30 60 80
FEB 35 65 85
MAR 45 75 65
APR 40 75 82
MAY 55 75 85
JUNE 50 75 80

i) During this period which share has undergone max


fluctuation?

ii) In which month it is possible to buy B and C selling


A?

iii) In which month the share values are very low?

iv) By purchasing one share of A and 4 each of B and C in


the beginning
of the period , to get max profit when this shares should
be sold?
v) ?

7. In a computer institute 9 languages can taught. The


module is
of 6 months duration and only six languages each of one
month can
be taught. In addition to that BASIC is always there and
should be in first month itself
# word perfect is to be taught in the preceeding week of
word star.
# FORTRAN can not be taught until COBAL is coarsed prior

Kumaraguru College, Coimbatore


to that
# BINO, FIFO never be taught in single module
languages are BASIC, WORD STAR, WORD PERFECT, FORTRAN,
COBAL, BINO, FIFO, LOTUS, C

i. Of the following which module is possible based on


above conditions.

ii) If word star is in 3rd month , what could be in 6th


month.

iii) If COBAL is in the 2nd month and BINO in 6th month


are there
in addition to the above condition, FORTRAN will be in
which month.

8. In a class , except 18 all are above 50 years. 15 are


below 50
years of age. how many people are there
a) 30 b) 33 c) 36 d) none of these.

9. A square plot of some size , at four corners equal


squares of
some size are cut and is formed as open box. If this open
box carries
128ml of oil. What is the size of the plate i.e. side
a.17 b.14 c.13

10. In a square , all the mid points are joined. the inner
square
is shaded. If the area of the square is A, what is the
shaded area?

it consists of number series.In some institutes alphabetical series is


given instead of number series.Iam having number series so iam sending
that.Please go through tha alphabetical tests also.
1. 19,24,20,25,21,26,? ans:22
2. 11,14,12,15,13,16,? ans: 14
3. 10,2,8,2,6,2,? a:4
4. 8,9,11,14,,18,23,? a:29
5. 25,25,22,22,19,19,? a:16
6. 14,2,12,4,10,6,? a:8
7. 7,16,9,15,11,14,? a:13
8. 40,42,39,44,38,46,? a:37
9. 3,18,4,24,5,30,? a:6
10. 18,20,22,20,28,20,? a:22

Kumaraguru College, Coimbatore


11. 18,20,10,12,4,6? a:0
12. 7,6,8,5,3,7,? a:4
13 9,18,21,25,20,? a:30
14 3,3,4,8,10,36,? a:33
15.30,28,25,20,34,28,? a:21
16. 4,8,16,32,64,128,? a:256
17. 8,16,24,32,40,48,? a:56
18. 13,11,14,12,15,13,? a:16
19. 6,18,36,108,216,648,? a:1296
20. 4,4,8,8,16,16,? a:32
21. 2,6,18,54,162,486,? a:1458
22. 4,20,35,49,62,74,? a:85
23. 10,18,15,23,20,28,? a:25
24. 4,10,8,14,12,18,? a:16
25 10,15,12,17,14,10,? a:16

part 2 consists of non-verbel reasoning(figures).So it is impossible


for
me to send those.(25 questions)

part 3 (quantitative)

1.A clerk multiplied a number by ten when it should have been divided
by
ten.The ans he got was 100.what should the ans have been?
a:1
2.If Rs20/- is available to pay for typing a research report & typist A
produces 42 pages and typist B produces 28 pages.How much should typist
A
receive?
a:Rs12/-
3.The average salary of 3 workers is 95 Rs. per week. If one earns
Rs.115 and second earns Rs.65 how much is the salary of the 3rd worker.
Ans.105.
4.A 16 stored building has 12000 sq.feet on each floor. Company A rents
7
floors and company B rents 4 floors. What is the number of sq.feet of
unrented floor space.
Ans.60000
5. During a given week A programer spends 1/4 of his time preparing
flow
chart, 3/8 of his time coding and the rest of the time in debugging the
programs. If he works 48 hours during the week , how many hours did he
spend debugging the program.
Ans. 18.
6. A company installed 36 machines at the beginning of the year. In
March
they installed 9 additional machines and then disconnected 18 in
August.
How many were still installed at the end of the year.
Ans. 27
7. A man owns 2/3 of the market research beauro business and sells 3/4
of
his shares for Rs. 75000. What is the value of Business.
Ans.150000
8. If 12 file cabinets require 18 feet of wall space, how many feet of

Kumaraguru College, Coimbatore


wall space will 30 cabinets require?
Ans.45
9.A computer printer produced 176,400 lines in a given day. If the
printer was in operation for seven hours during the day, how many lines
did it print per minute?
Ans.420

10. From its total income, A sales company spent Rs.20,000 for
advertising, half of the remainder on commissions and had Rs.6000 left.
What was its total income?
Ans.32000
11. On Monday a banker processed a batch of cheques, on Tuesday she
processed three times as many, and on Wednesday she processed 4000
cheques. In the three days, she processed 16000 cheques. How many did
she process on Tuesday?
Ans.9000
12. The cost of four dozen proof machine ribbons and five dozen
accouting
machine ribbons was Rs.160/-. If one dozen accounting machine ribbons
cost Rs.20/-, what is the cost of a dozen proof machine ribbons?
Ans.Rs.15
13. If a clerk can process 80 cheques in half an hour, how many cheques
can she process in a seven and one half hour day?
Ans.1200
14. In a library, there are two racks with 40 books per rack. On a
given
day, 30 books were issued. What fraction remained in the racks?
Ans.5/8
15. The average length of three tapes is 6800 feet. None of the tapes
is
less than 6400 feet. What is the greatest possible length of one of
the
other tapes?
Ans.7600
16. A company rented a machine for Rs.700/- a month. Five years later
the treasurer calculated that if the company had purchased the machine
and paid Rs.100/- monthly maintenance charge, the company would have
saved Rs.2000/-. What was the purchase price of the machine?
Ans.Rs.34000
17. Two computers each produced 48000 public utility bills in a day.
One
computer printed bills at the rate of 9600 an hour and the other at the
rate of 7800 an hour. When the first computer finished its run, how
many
bills did the other computer still have to print?
Ans.9000

18. If a salesman's average is a new order every other week, he will


break the office record of the year. However, after 28 weeks, he is
six
orders behind schedule. In what proportion of the remaining weeks does
he have to obtain a new order to break the record?
Ans.3/4
19. On a given day, a bank had 16000 cheques returned by customers.
Inspection of the first 800 cheques indicated that 100 of those 800 had
errors and were therefore the available immediately for data
processing.
On this basis, hwo many cheques would be available immediately for data

Kumaraguru College, Coimbatore


processing on that day?
Ans.14000
20. A company figured it needed 37.8 sq.feet of carpot for its
reception
room. To allow for waste, it decided to order 20% more material than
needed. Fractional parts of sq.feet cannot be ordered. At Rs.9/- a
sq.feet, how much would the carpet cost?
Ans.
a. Rs.324 b) Rs.405 c) Rs.410 d) Rs.414 e) Rs.685
21. A tape manufacturer reduces the price of his heavy duty tape from
Rs.30/- to Rs.28/- a reel and the price of a regular tape from Rs.24/-
to
Rs.23/- a reel. A computing centre normally spends Rs.1440/- a month
for
tapes and 3/4 of this is for heavy duty tapes. How much will they save
a
month under the new prices?
Ans.Rs.87
22. In a team of 12 persons, 1/3 are women and 2/3 are men. To obtain
a
team with 20% women how many men should be hired?
Ans.8
23. The dimensions of a certain machine are 48" X 30" X 52". If the
size
of the machine is increased proportionately until the sum of its
dimensions equals 156", what will be the increase in the shortest side?
Ans. 6"
24. In a certain company, 20% of the men and 40% of the women attended
the annual company picnic. If 35% of all the employees are man, what
percent of all the employees went to the picnic?
Ans.33%
25. It cost a college Rs.0.70 a copy to produce a Programme for the
homecoming football game. If Rs.15,000/- was received for
advertisements
in the programme, how many copies at Rs.0.50 a copy must be sold to
make
a profit of Rs.8000/- ?
Ans. 35000

CRITICAL REASONING : THERE WILL BE 13 PASSAGES WITH 50 QUESTIONS TIME 30 MIN.


HERE I AM SENDING ONLY SOME OF THE PASSAGES (these will give only rough idea)
(ANSWERS WILL BE AS YES/NO/CAN'T SAY we are giving our answers, please check.)

1. My father has no brothers. he has three sisters who has two Childs each.

1> my grandfather has two sons (f)


2> three of my aunts have two sons (can't say)
3> my father is only child to his father (f)
4> I have six cousins from my mother side (f)
5> I have one uncle (f)

2. Ether injected into gallbladder to dissolve gallstones. This type one-day treatment is
enough for gallstones not for calcium stones. This method is alternative to surgery for millions of people who are
suffering from this disease.

Kumaraguru College, Coimbatore


1> calcium stones can be cured in one day (f)
2> hundreds of people contains calcium stones(can't say)
3> surgery is the only treatment to calcium stones(t)
4> either will be injected into the gallbladder to cure the cholesterol
based gall stones(t).

3. Hacking is illegal entry into other computer. This is done mostly because of lack of knowledge of computer
networking with networks one machine can access to another machine. Hacking go about without knowing that each
network is accredited to use network facility.
1> Hacking people never break the code of the company which they
work for (can't say).
2> Hacking is the only vulnerability of the computers for the usage
of the data.(f)
3> Hacking is done mostly due to the lack of computer knowledge (f).
(there will be some more questions in this one )

4. Alpine tunnels are closed tunnels. In the past 30 yrs not even a single accident has been recorded for there is one
accident in the railroad system. Even in case of a fire accident it is possible to shift the passengers into adjacent wagons
and even the living fire can be detected and extinguished with in the duration of 30 min.

1> no accident can occur in the closed tunnels (True)


2> fire is allowed to live for 30 min. (False)
3> All the care that travel in the tunnels will be carried by rail
shutters.(t)
4>

5. In the past helicopters are forced to ground or crash because of the formation of the ice on the rotors and engines. A
new electronic device has been developed which can detect the water content in the atmosphere and warns the pilot if
the temperature is below freezing temp about the formation of the ice on the rotors and wings.

1> the electronic device can avoid formation of the ice on the wings (False).
2> There will be the malfunction of rotor & engine because of formation of ice (t)
3> The helicopters are to be crashed or down (t)
4> There is only one device that warn about the formation of ice (t).

6.In the survey conducted in mumbai out of 63 newly married house wives not a single house wife felt that the husbands
should take equal part in the household work as they felt they loose their power over their husbands. In spite of their
careers they opt to do the kitchen work themselves after coming back to home. The wives get half as much leisure time
as the husbands get at the weekends.

1> housewives want the husbands to take part equally in the household (f)
2> wives have half as much leisure time as the husbands have (f)
3> 39% of the men will work equally in the house in cleaning and washing

7. Copernicus is the intelligent. In the days of Copernicus the transport and technology development was less & it took
place weeks to communicate a message at that time. Where in we can send it through satellite with in no time ----------.
Even with these fast developments it has become difficult to understand each other.

1> people were not intelligent during Copernicus days (f).


2> Transport facilities are very much improved in now a days (can't say)
3> Even with the fast developments of the technology we can't live happily.(can't say)
4> We can understand the people very much with the development of communication (f).

Kumaraguru College, Coimbatore


Q8) senior managers warned the workers that because of the introductory of Japanese industry in the car market.
There is the threat to the workers. They also said that there will be the reduction in the purchase of the sales of car in
public. The interest rates of the car will be increased with the loss in demand.

1> Japanese workers are taking over the jobs of Indian industry (false)
2> managers said car interests will go down after seeing the raise in interest rates (true)
3> Japanese investments are ceasing to end in the car industry (false)
4> people are very much interested to buy the cars (false)

Q9) In the totalitarian days, the words have very much devalued. In the present day, they are becoming domestic that
is the words will be much more devalued. In those days, the words will be very much affected in political area. But at
present, the words came very cheap .we can say they come free at cost.

1> totalitarian society words are devalued.(false)


2> totalitarian will have to come much about words (t)
3> The art totalitarian society the words are used for the political speeches.
4>

Q10) There should be copyright for all arts. The reele has came that all the arts has come under one copy right
society,they were use the money that come from the arts for the developments . There may be a lot of money will come
from the Tagore works. We have to ask the benifiters from Tagore work to help for the development of his works.

1> Tagore works are came under this copy right rule.(f)
2> People are free to go to the because of the copy right rule.(can't say)
3> People gives to theater and collect the money for development.(can't say)
4> We have ask the Tagore resedents to help for the developments of art.(can't say)

1. 420% OF 7.79 = 32.718


2. 3427 / 95 = 115.7365
3. 10995 / 95 = 115.7365
4. 43 + 557 -247 = 357
5. 31070 * 3.082 = 9591.39
6. 48.7 + 24.9 - 8.7 = 64.90
7. 525.8 / 47.8 = 11
8. ( 135 - 30 -14) /7 - 6 + 2 = 3
9. 3 / 8 * 5.04 = 1.89
10. 697 / 219 = .0316
11. 8 / 64 + 64 / 16 = 4.14
12. 298 * 312 / 205 = 453.54
13. .33 * 1496 /13
14. .26 +1/8 = .385
15. 66.07 + 1/3 = 67.03
16. 284 + 1/4 = 3.09
17. 33 % OF 450 = 1.485
18. 907.54 / .3073 = 3002

Kumaraguru College, Coimbatore


QUANTITATIVE APTITUDE SECTION

1. Two pencils costs 8 cents, then 5 pencils cost how much

Sol: 2 pencils  8 cents => 1 pencil  4 cents

Therefore 5 pencils cost = 5 * 4 = 20 cents

2. A work is done by two people in 24 min. one of them can do this work a lonely
in 40 min. how much time required to do the same work for the second person.

Sol: (A+B) can do the work in = 1/24 min.

A alone can do the same work in = 1/40 min.

B alone can do the same work in = (A+B)’s – A’s = 1/24 – 1/40 = 1/60

Therefore, b can do the same work in = 60 min

3.A car is filled with four and half gallons of oil for full round trip. Fuel is taken
1/4 gallon more in going than coming. What is the fuel consumed in coming up?

Sol Before the trip, car is filled with = 4 ½ gallon of oil

Let ‘X’ be the quantity of fuel consumed for the trip in one direction

The fuel consumed while going = X + ¼

The fuel consumed while coming = X

Therefore, the fuel consumed for the trip = (X + ¼) + X = 4 ½

 2X + ¼ = 4 ½ => 2X = 4 ½ - ¼ => 2X = 4 ¼ => X= 2. approx

Therefore the fuel consumed while coming = 2 gallon

4. Low temperature at the night in a city is 1/3 more than 1/2 high as higher
temperature in a day. Sum of the low temperature and highest temp. is 100
degrees. Then what is the low temp?

Sol: 40 deg.

5. A person, who decided to go to weekend trip should not exceed 8 hours


driving in a day. Average speed of forward journey is 40 m/h. Due to traffic
in Sundays, the return journey average speed is 30 m/h. How far he can select
a picnic spot?

a) 120 miles
b) Between 120 and 140 miles
c) 160 miles

Kumaraguru College, Coimbatore


Answer: 120 miles

6. A salesperson multiplied a number and got the answer 3, instead of that number divided by 3.What is the
answer he actually has to get?

Sol: (1/3) *1 * 3 = 3, so number = 1

Divided by 3

Answer: 1/3.

7. A ship started from port and moving with I miles per hour and another ship started from L and moving
with H miles per hour. At which place these two ships meet?

|----|----|----|----|----|----|
port G H I J K L

Sol: Answer is between I and J and close to J or (L*I*H)/(H+I)

8. A building with height D shadow up to G. A neighbor building with what


height shadows C feet.

|----|----|----|----|----|----|----|
A B C D E F G H

Sol: B Ft. or CD/G

9. A person was fined for exceeding the speed limit by 10mph. Another person
was also fined for exceeding the same speed limit by twice the same. If the
second person was traveling at a speed of 35 mph, find the speed limit.

Sol: Let ‘x’ be the speed limit

Person ‘A’ was fined for exceeding the speed limit by = 10mph
Person ‘B’ was fined for exceeding the speed limit by = twice of ‘A’
= 2*10mph=20mph
given that the second person was traveling at the speed of 35mph

=> 35mph – 20mph = 15mph

Therefore the speed limit is =15 mph

10.A bus started from bus stand at 8.00am, and after 30 minutes staying at
destination, it returned back to the bus stand. The destination is 27 miles from
the bus stand. The speed of the bus is 18mph. In return journey bus travels
with 50% fast speed. At what time it returns to the bus stand?

Sol: 11.00am

11.In a mixture, R is 2 parts S is 1 part. In order to make S to 25% of the mixture,


how much r is to be added?

Sol: One Part

Kumaraguru College, Coimbatore


12. Wind flows 160 miles in 330 min, for 80 miles how much time required.

Sol:

13. With 4/5 full tank vehicle travels 12 miles, with 1/3 full tank how much
distance travels

Sol: ( 5 miles )

14. Two trees are there. One grows at 3/5 of the other in 4 years, total growth of
trees is 8 ft. what growth will smaller tree will have in 2 years

Sol: ( < 2 ft. )

15. A storm will move with a velocity of towards the center in hours, at the same
rate how much far will it move in hrs.

Sol: ( but the answer is 8/3 or 2 2/3 )

1. Two pencils costs 8 cents, then 5 pencils cost how much


(Ans:20 cents).
2. A work is done by the people in 24 min. one of them can do
this work a lonely in 40 min. how much time required to do the same
work for the second person.
(ans:60 min.)
3. A car is filled with four and half gallons of oil for full round
trip. fuel is taken 1/4 gallons mor3 in going than coming. what is
the fiel consumed in coming up? (2 gallons)
4. low temperature at the night in a city is 1/3 more than 1/2
hinge as higher temperature in a day. sum of the low temp and
higherst temp is 100C. then what is the low temperature (40 C)
5. A person who decided to go weekend trip should not exceed 8 hours
driving in a day Average speed of forward journy is 40 mph. due to
traffic in sundays, the return journey average speed is 30 mph.
how far he can select a picnic spot (120 miles).

6. A sales person multiplied a number and get the answer is 3,


instead of that number divided by 3. what is th answer he actually
has to get ? (1/3).
7. A ship started from port and moving with I mph and another ship
started from L and moving with H mph. At which place these two ships
meet ? ( Ans is between I and J and close to J)

!_____!_____!_____!_____!_____!_____!
port G H I J K L

8. A building with hight D ft shadow upto G A neighbour building with


what height shadow C ft is (B ft.)

!_____!_____!_____!_____!_____!_____!_____!
A B C D E F G H

Kumaraguru College, Coimbatore


9. A person was fined for exceeding the speed limit by 10 mph.Another
person was also fined for exceeding the same speed limit by twice
the same. If the second person was travelling at a speed of 35 mph.
find the speed limit (15 mph)

10. A bus started from bustand at 8.00a m and after 30 min staying at
destination, it returned back to the bustand. the destination is 27
miles from the bustand. the speed of the bus 50 percent fast speed.
at what time it retur4ns to the bustand (11.00)

11.in a mixture, R is 2 parts, S is 1 part. in order to make S to 25%


of the mixture, howmuch R is to be added ( one part).

12. wind flows 160 miles in 330 min, for 80 miles how much time
required.
13. with 4/5 full tank vehicle travels 12 miles, with 1/3 full tank
how much distance travels ( 5 miles).

14. two trees are there. one grows at 3/5 of the other. in 4 years,
total growth of trees is 8 ft. what growth will smaller tree will
have in 2 years. (<2ft)

15. A storm will move with a velocity of towords the center in


hours. At the same rate how much far will it move in hrs.
(but Ans is 8/3 or 2 2/3).

1. My father has no brothers. he has three sisters who has two childs each.

1> my grandfather has two sons (f)


2> three of my aunts have two sons (can't say)
3> my father is only child to his father (f)
4> I have six cousins from my mother side (f)
5> I have one uncle (f)

2. Ether injected into gallbladder to dissolve gallstones. This type one-day treatment is
enough for gallstones not for calcium stones. This method is alternative to surgery for millions of people who are
suffering from this disease.

1> calcium stones can be cured in one day (f)


2> hundreds of people contains calcium stones(can't say)
3> surgery is the only treatment to calcium stones(t)
4> either will be injected into the gallbladder to cure the cholesterol
based gall stones(t).

3. Hacking is illegal entry into other computer. This is done mostly because of lack of knowledge of computer
networking with networks one machine can access to another machine. Hacking go about without knowing that each
network is accredited to use network facility.
1> Hacking people never break the code of the company which they
work for (can't say).
2> Hacking is the only vulnerability of the computers for the usage
of the data.(f)
3> Hacking is done mostly due to the lack of computer knowledge (f).
(there will be some more questions in this one )

Kumaraguru College, Coimbatore


4. Alpine tunnels are closed tunnels. In the past 30 yrs not even a single accident has been recorded for there is one
accident in the railroad system. Even in case of a fire accident it is possible to shift the passengers into adjacent wagons
and even the living fire can be detected and extinguished with in the duration of 30 min.

1> no accident can occur in the closed tunnels (True)


2> fire is allowed to live for 30 min. (False)
3> All the care that travel in the tunnels will be carried by rail
shutters.(t)
4>

5. In the past helicopters are forced to ground or crash because of the formation of the ice on the rotors and engines. A
new electronic device has been developed which can detect the water content in the atmosphere and warns the pilot if
the temperature is below freezing temp about the formation of the ice on the rotors and wings.

1> the electronic device can avoid formation of the ice on the wings (False).
2> There will be the malfunction of rotor & engine because of formation of ice (t)
3> The helicopters are to be crashed or down (t)
4> There is only one device that warn about the formation of ice (t).

6.In the survey conducted in mumbai out of 63 newly married house wives not a single house wife felt that the husbands
should take equal part in the household work as they felt they loose their power over their husbands. In spite of their
careers they opt to do the kitchen work themselves after coming back to home. The wives get half as much leisure time
as the husbands get at the weekends.

1> housewives want the husbands to take part equally in the household (f)
2> wives have half as much leisure time as the husbands have (f)
3> 39% of the men will work equally in the house in cleaning and washing

7. Copernicus is the intelligent. In the days of Copernicus the transport and technology development was less & it took
place weeks to communicate a message at that time. Where in we can send it through satellite with in no time ----------.
Even with these fast developments it has become difficult to understand each other.

1> people were not intelligent during Copernicus days (f).


2> Transport facilities are very much improved in now a days (can't say)
3> Even with the fast developments of the technology we can't live happily.(can't say)
4> We can understand the people very much with the development of communication (f).

Q8) senior managers warned the workers that because of the introductory of Japanese industry in the car market.
There is the threat to the workers. They also said that there will be the reduction in the purchase of the sales of car in
public. The interest rates of the car will be increased with the loss in demand.

1> Japanese workers are taking over the jobs of Indian industry (false)
2> managers said car interests will go down after seeing the raise in interest rates (true)
3> Japanese investments are ceasing to end in the car industry (false)
4> people are very much interested to buy the cars (false)

Q9) In the totalitarian days, the words have very much devalued. In the present day, they are becoming domestic that
is the words will be much more devalued. In those days, the words will be very much affected in political area. But at
present, the words came very cheap .we can say they come free at cost.

1> totalitarian society words are devalued.(false)


2> totalitarian will have to come much about words (t)

Kumaraguru College, Coimbatore


3> The art totalitarian society the words are used for the political speeches.
4>

Q10) There should be copyright for all arts. The reele has came that all the arts has come under one copy right
society,they were use the money that come from the arts for the developments . There may be a lot of money will come
from the Tagore works. We have to ask the benifiters from Tagore work to help for the development of his works.

1> Tagore works are came under this copy right rule.(f)
2> People are free to go to the because of the copy right rule.(can't say)
3> People gives to theater and collect the money for development.(can't say)
4> We have ask the Tagore resedents to help for the developments of art.(can't say)

1. My Father is only child to his Father. My father has three


sisters. All are married and have two children each. (State True or
False Or Can't be determined )
1.1 My Grand father has two sons
1.2 I am having six cousins
1.3 I have three uncle

Part B (15 Questions )

1. Two Pencils 8 cents 5 Pencils cost ( Ans 20 cents)

2.A work is done by two people in 24 minutes. One of them alone can
do it in 40 minutes. How much time will the other person wiil take to
complete it ( 60 minutes)

3. A car is filled with 4.5 gallons of fuel for a round trip . Car
is taken 1/4 more than in going than coming up. What is the fuel
consumed in coming up ?

4. Low temperature at the night in a city is more than 1/2 high as


higher temperature are 100. Then What is low temperature (Ans 40)

5. Person who dicided to go to weekend trip should not exceed 8 hrs


driving in a day. Average speed of forward journey is 40 miles/hour .
Due to traffic in sundays ,in return journey average speed is 30mph .
How far he can select a picnic spot ? (ans 120 miles)

6.A sales person multiplied by a number and get the answer 3. Instead
of that number divided by 3. What is the answer she actually has to
get (1/3) * 1 *3 =3 so the no= 1 divided by three. The answer =1/3

7. A ship started from port and moving with 'I' miles/hour and other
ship started From L and moving with 'H' miles/hour . At the place
where these two ships will meet.

8. A building with height D shadows up to G. A Neighbour building


with what height shadows C feet.

9. A person was fined for exceeding the speed limit by 10mph. Another
person was also fined for exceeding the speed limit by twice the
same. If the second person was travelling at speed of 35mph . Find
the speed limit (ans. 15mph)

Kumaraguru College, Coimbatore


10. A bus started from the bus stand at 8Am and after staying 30
minutes at a destination return back to the bus stand. The
Destination is 27 miles from the bus stand . The Speed of the bus is
!8mph . In the return journey the bus travells with 50% fast speed.
At what time it is return to the bus stand (11am)

11. In a mixture R is two parts, S is one part . In order to mixture


how much R is to be added. Ans S is 25% of R

1)2 pencils costs 8 cents,then 5 pencils cost how much


> > a)20c
> > 2)a work is done by the people in 24 min.one of them can do this
work
> > a lonely in 40 min.how much time required to do the same work for
the
> > second person
> > a)60min
> > 3)a car is filled with four and half gallons of oil for full round
> > trip.fuel is taken 1/4 gallons more in going than coming.what is
the fuel
> > consumed in coming up.
> > a)2 gallons
> > 4)low temperature at the night in a city is 1/3 more than 1/2 hinge
as
> > higher temperature in a day.sum of the low temp and high temp is
100 c.
> > then what is the low temp.
> > a)40 centigrade
> > 5)a person who decided to go weekend trip should not exceed 8 hours
> > driving in a day average spped of forward journey is 40 mph.due to
> > traffic insundays the return journey average speed is 30 mph.how
far he
> > can select a picnic spot.
> > a)120 miles
> > 6)a sales person multiplied a number and get the answer is 3
> > instead of that number divided by 3.what is the answer he actually
> > has to get.
> > a)1/3
> > 7)a ship started from a port and moving with I mph and another ship
> > started from L and moving with H mph.at which place these two ships
> > meet.
> > a)between I and J and close to J
> > 8)a building with height D ft shadow upto G.A neighbour building
> > with what height shadow C ft is
> > a)B ft
> > 9)a person was fined for exceeding the speed limit by 10mph.another
> > person was also fined for exceeding the same speed limit by twice
the same
> > if the second person was travellling at a speed of 35 mph.find the
> > speed limit
> > a)15mph
> > 10)a bus started from bustand at 8.00am and after 30min staying at

Kumaraguru College, Coimbatore


> > destination it returned back to the bustand.the destination is 27
> > miles from the bustand the speed of the bus 50percent fast speed at
what
> > time it returns to the bustand.
> > a)11.00am
> > 11)in a mixture,R is 2 parts,s is 1 part in order to make s to 25%
of the
> > mixture,howmuch r is to be added
> > a)one part
> > 12)wind flows 160 miles in 330min.for 80 miles how much time
required
> > a)---
> > 13)with 4/5 full tand vehicle travels 12 miles, with 1/3 full tand
how
> > much distance travels
> > a)5 miles
> > 14)2 trees are there.one grows at 3/5 of the other.In 4 years
> > total growth of the trees is 8 ft.what growth will smaller tree
will have
> > in 2 years.
> > a)2 ft
> > 15)a storm will move with a velocity of --- towards the center in
---
> > at the same rate how much far will it move in hrs
> > a)8/3 or 2 2/3
> > 15 not clear
>>
>>
> > Reasoning
> > ----------
> > A.
> > my father had no brothers but his 3 sisters are all married and
each has 2
> > children.my grandfather has 2 sons.
> > 1.mu father was an only child F
> > 2.3 of my aunts have sons C
> > 3.i have six cousins on my mother's side C
> > 4. i have only one uncle F
>>
> > B.
> > Senior managers in a leading company said that new japanese
investment in india was transforming the car industry and warned that jobs
were under threat from japanese competition. they stated that increasing
> > competition would be coupled with an in evitable downturn i the car
market
> > and the recent rise in interst rates whaich had already hit demand.
> > 5.some senior managers said that more people will want to buy new
cars in
> > the future. F
> > 6.managers told workers that japanese workers are taking jobs away
from
> > indian workers in the car industry. F
> > 7.the managers issued their warning after a rise in interest rates.
T
> > 8.the increased rate of the interest will mean that japanese firms
will
> > cease to operate in this country. C
>>

Kumaraguru College, Coimbatore


> > C.Researchers in mumbai have found that certain types of gallstones
can be dissolved by injecting them with a gasoline additive in the form of
ether the ether is injected through a tube directly into a tube directly into
the gallbladder.the one day treatment works only on cholesterol-based stones.not those composed largely o
calcium.however as the cholesterol
> > stones are by far the most common typefor millions of gallstones
> > sufferers the treatment should offer a welcome alternative
tosurgery
 > the commonest option in nost hospitals.

> > 9.injecting ether into the gallbladder dissolves most gallstones
T
> > 10.surgery is the only treatment for calcium stones T
> > 11.hundreds of peoplecontains calcium stones C
> > 12.calcium stones will be cured in one day F
>>
> > D. organizing the home can be perceived as conferring power so large
> > numbers of women are unwilling to let go of chores, even when they
have careers. a survey found that, out of 65 new marriages not one single
wife expected her husband to share work equally. according to the family
policy studies center 81% working wives return home to do all the cooking. The average male has nearly half as much
more free time at weekends than his wife and the typical new father spends just 37 seconds a day talking to his baby.

> > 13.most working wives do not expect their husbands to share chores
> > equally. T
> > 14.the average wife has half as much free time at weekends as her
husband.
>>F
> > 15.some women collude in the unequal distribution of house hold
work because they want to retain control C
> > 16. 39% of all men with working wives do the cooking and all the
cleaning
>>F
>>
> > E. Confucius said that to know the future we have to understand the
past his time transport ,communications and scientific knowledge were less
> > developed than they are today. News took weeks to travel where as today
> > satellite links connect the continents virtually instantaneously. But
> > our technological advances in the field of communications seem not to have improved our capacity to understand
one another
> > 17.in Confucius day people were more intelligent C
> > 18.we understand each other better now than in Confucius time because
> > we can travel more quickly. F
> > 19.we have made great improvements in transport since Confucius day T
> > 20.none of our scientific discoveries has really improved our lives C
 >
 F.words in totalitarian systems have an unhealthy importance and in
such states now attempting to return to a more democratic normality there has been a natural inevitable and healthy
devaluation of words whereas previously a single word used in a sppech or even a play or poem could be a serious
political event now the words come cheaper almost free. politics is politics again and poetry only poetry
> > 21.totalitarian state devalue words T
> > 22.only non-totalitarian regimes produce poetry of political importance T
> > 23.writers under totalitarian regimes have to choose their words carefully T
> > 24. the democratic political system is healthier than others C
>
>
> G.statistics show that millions of vehicals have beencarried by
shuttle

Kumaraguru College, Coimbatore


> over the past 30 years through alpine tunnels withouty one ever
catching
> tire.in the alpine tunnels.drivers and passengers sit in
theirvhehicals on
> the shuttle trains.only one vehical has evercaught fire on the bussy
> french motorail equivalent system.this sort of accidents is not
possible
> in a closed shuttle. assertinos that a vehical fire will lead to
> catastophe have no basis. since the resoures exit do detect,control
and
> extingush a fire and to remove any persons present safely to an
adjoning
> wagon, leaving any surviving fire facing rapid extinction within a
wagan
> built to contain fire for 30 minutes. catastrophe seems very
unlikely.
>
> 25. if a car cauget fire in a rail shuttle, probably none would be
killed.
>
> (T)
> 26.at least one vehical has cauht fire in an alpine tunnel.(f)
> 27.if a fire started in a wagon, it would be allowed to burn itself
out in
> 30 minutes.(f)
> 28.if would theoreticaly be possible for a car to cath fire in
aclosed
> shuttle system.(F)

>
> H)every form of art is protected by copy write,upon the expiration
>
> of whitch the property passes into the public domain and becomes
freely
> available to any one wishing to exploit it commercialy. the time has
come
> when all treasures shoud pass to the controled of a trust,and by this
made
> readily available to anyone on pament of a fee or royality.the income
> fromthe works of tagore would alone be enarmous.these who now main
> financialbenifit from his genius should make some contribution to the
> welfare of the arts in general.
> 29.tagore's plays are not protected by copyright. F
> 30.tagore's decendants should be asked to make some contribution to
the
> arts C
> 31.instead of buying a ticket , theatregoers should pay a fee to
trust for
> the benefit of the arts. C
> 32. More people could go to the theatre if copy rightwere abolished
C

> I. Hacking is a crime made possible by a relatively new technology,


> which one of the reasons it is often poorly understood and reported.
> Many computers, but no means all,are now linked togetherin networks
which
> allow users on one computer to communicate with others on the same

Kumaraguru College, Coimbatore


> network.If a computer is not networked,no manipulation of its
datafrom
> another machine is possible.So long as users are authorised,
networking
> is just a way of making work easier and more productive.Hacking on
the
> other hand,is the unauthorised use of networks or unauthorised entty
> into the computers themselves.Most people do not need to break into
the
> networks they use,since they are already accredited users.
> 33. Most hackers are authorised to break into networks F
> 34. Computers are only vulerable to the unauthorised manipulation of
> their data via another computer if they are networked T
> 35.The main reason why it is relatively easy to break into a computer
is
> that few people understand the technology. C
> 36.Hackers do not work fot the firms whose networks they break into.
T

> J.Although invaders represent a threat to the conservation of flora


and
> fauna,there are two special cases in which invasion have been
deliberately
> brought about.One is the desire to control presents by natural
predators,
> whichmay have to be brought in from other countries.The second is
> releasing organisms into the wild(or on to farms,from which they
might
> escape)that are completely novel,beacause they have been genetically
> engineered.There is nothing intrinsically sinister about engineered
> organisms,but any novelty must be regarded as a potential invader.
> 37.Pest control does not threat the conservation of flora and fauna.
T
> 38.Genetically engineered organisms must always be regarded as poten-
> tially dangerous. F
> 39.Natural predators are work harmful than pests.T
> 40.Genetically engineered organisms escaped from the farm,they will
be
> pose a threat to wildlife. T
>

>>
> > K.electronics technology is coming to the rescue of helicopters
which can
> > be grounded or crash in icy conditions the machines are especially
> > vulnerable to the build up of ice on both their rotors and engine
air
> > intake when operating in cold damp conditions.the problem is 2 fold
> > ice increases the weight and the build upp makes the aerofoils
> > less efficient .now at last a detector has been devised which the
company
> > hopes will mean safer flightsand less frequent grounding.unlike
most
> > devices in use at present it can detect the liquid water content of
cloud
> > above freezing level.so the warning is ginven before the potential
hazard

Kumaraguru College, Coimbatore


> > is encountered.
>>
> > 39.an electronic device has been invented which will prevent the
build
> > up of ice on helicopter rotors F
> > 40.helicopters are sometimes grounded because in cold damp weather
their
> > engine air intakes and rotors malfuntion owing to the formatrion of
ice.
>>T
> > 41.only one device can at present detect the liquid water content
of
> > cloud above freezing level C
> > 42.in future fewer helicopters will crash or have to grounded T
>>

> > L.anyone whos has systematically examined faces will have perceived
a
> > prepondreance although not a proliferation of asymmtry.whether or
not
> > the expression is volitinal and self controlled or spontaneous
appears
> > to predict facial asymmetry as does the type of emotion
protrayed.positive
> > emotions are usually displayed symmetrically although a left sided
> > representation of a negative emotion is more common.posed
expressions and
> > negative emotions are less likely to be symmetrically represented.
> > 43.an angry person is more likely to have a lopsided expression
than
> > someone who is smiling T
> > 44.a deliberately assumed facial expression will always be
asymmetrical F
> > 45.an actor is likely to smile symmetrically when acting C
> 46.more self conscious people are likely to have less
asymmetrically
> facial expressions than those who are less aware of themselves T
>

> M. human existence is not susceptible of arbitary division between


> consciousness and unconsciousness.the conscious world invades and
shapes
> the activities of the unconscious while many of the great
achievements of
> humanity's waking hours were wholly or partly inspires bu dreams
.even if
> it could be argued that dreams precede experience such a dichotomy
could
> not be drawn as the influence of dreaming on the waking state would
remain
> unclear but as yet no common vocabulary exists to record the
substance
> of prenatal dreaming
> 47.sleep can be a creative state. T
> 48.it is difficult to tell whether a sleeper is dr3eaming or not C
> 49. if we knew what babies dream about before they are born we could

Kumaraguru College, Coimbatore


> show that conscious and unconscious mond influence one another F
> 50. it is untrue to claim that the conscious and unconscious worlds
never
> impinge on one another F
>

> New twenty questions


> --------------------
> N. disease x succeeds lung disease.it may also occur helthy persons
the
> person looks healthier from outside.the number of red blood cells in
> the blood increased and this leads to thickness fo the blood as a
result
> of which blood lacks inside the vessels .the person suffers heart
attact.
> one possible solution is to remove a litre of blood or to control the
> growth of red blood cells.
> 53.lung disease precedes the disease x T
> 54.a person who gets heart attack always has disease x F
> 55.people suffering from disease x look healthier C
> 56.one litre of blood must be removed to cure this T
>

> O.a starfire engine has been designed to work only for unleaded
> petrol.this crossfire engine is designed for both 1500 and `1800 cc
> they internally and externally look alike except for the difference
> in wheels ----,2000 cc is differentr from the above.two in terms of 3
> features --- ----- ---- on the bumber.
> 57.the engine run only on the leaded petrol F
> 58.1500 and 1800 cc look alike F
> 59.apart from cross fire engines they manfacture ---- F
> 60.internal structure of the 2000cc is same as that of both 1500 and
> 1800 cc C
>

> P.a weed killer to kill the weeds has been developed.the weed killer
> solution has to be applied to the growing tips. it need not to be
applied
> to all the tips.the solution has to be prepared in a can in one hand
> and the plants are taken in the other hand with a glove and are
immersed
> in the solution if we cannot immerse them in the solution the
solution
> can be applied to the roots with a brush.it used without care it can
> pose danger to other plants.
> 61.the solutions has to applied to growing leaves
> 62.to use the weedkiller first take the plants in the cn as then pour
the
> solution into it.
> 63.it is sufficient to apply it only to some roots
> 64.it effects to the other plants.
>

> Q.
> 65.flora 3 sofa set gives value to ur money

Kumaraguru College, Coimbatore


> 66.padding is used to increase durablity
> 67.flora will made all other products other than sofa sets
> 68.can we use wood as ----- T
>

> R.
>
> 69.the length of the cracker will be less than 12 cm F
> 70.the center part of the crackers is 4cm T
>
>-------------------------------------------------------------------------------
>
>
> these are the questions given from tcs paper
> dont answer all questions.
>
> In reasoning test at the end of each question we indicated
> F,T,C
> where F represents False
>" T " True
>" C " cant say.

>
>>
> >[1]In an year there are 4 fridays and 4 mondays in the month of
october.
> >on which day does the 20 november fall in the same year.
> >ans wednesday
>>
> >[2]Six people A,B,C,D,E and F went to cinema and want to sit as
> > (a) A is followed by B ,followed by c, followed by D ,followed
by E,
>> followed by F.
> > (b) B wants to sit on any side of A.
> > (c) C wants to sit by the side of A or B.
> > (d) D wants to sit by the side of A or B or C.
> > (e) E wants to sit by the side of A or B or C or D.
> > (f) F wants to sit by the side of A or B or C or D or E.
>>
>> now the question is how many combinations are possible to sit
as
>> mentioned above. ans:- 32
>>
>>
> >[3]In a mathematica there are 1,2,3,....9 cities and each number
> > corresponds to a city. Now an airflight route is to be
established
> > between two cities such that the numeral formed is divisible by
3.
> > (eg. if 1 and 2 are connected then it satisfies the condition).
> > Find out the number of cities which are connected to city 8

Kumaraguru College, Coimbatore


,either
> > directly or indirectly.
> >ans 5 cities
>>
> >[4]There are five sisters A,B,C,D,E. During deevali festival each
> > receive 4 gifts and gives 4 gifts to other sisters. Each member
has
> > his own strategy of gift distribution.
> > B gives all his gifts to A.
> > C gives 3 gifts to E.
>> determine how many gifts did each give to D.
> > ans a 1 ,b 0, c 1, e 2(check this)
>>
> >[5]There are 100 bulbs and 100 switches.Each switch is numbered
> > from 1 to 100 and each switch corresponds to each bulb.
> > (a)all switches are switched on.
> > (b)the switch nnumberes which are divisible by 2 are marked
>> and those switches which are on are put off and which are
>> off are put on.
> > (c)the switch numbers which are divisible by 3 are marked and
>> those switches which are on are put off and which are off
>> are put on.
> > (d)this process continued till the number 100 .
>> By the end how many switches are glowing.?
>>
> > ans : 10(1,4,9,16,25,36,49,64,81,100)
>>
> >[6]There are five letters A,B,C,D,E .Each represents a different
> > number.
> > (a) product of AB and CD is EEE.
> > (b) if AB is subtracted from the product E and CD then CC is
>> result.
> > Now what is the code which represents the product of AB and D.
>>
>>
> >[7]How many 10 digit numbers are there whose sum of the digits is
> > the number 4
> >hint: - 1010110000,2020000000 ,4000000000 etc...
> > ans 148 or 210 ( please check correct)
>>
>

1)At 6'o clock clock ticks 6 times. The time between first and
last ticks was 30sec. How much time it takes at 12'o clock.

Ans. 66 sec. 2 marks.

2)Three friends divided some bullets equally. After all of them


shot 4 bullets the total no.of remaining bullets is equal to that of
one has after division. Find the original number divided.

Ans. x x x
x-4 x-4 x-4
3x-12 = x
x= 6
ans is 18 2 marks

Kumaraguru College, Coimbatore


3)A ship went on a voyage after 180 miles a plane statrted with 10
times
speed that of the ship. Find the distance when they meet from
starting point.

Ans. 180 + (x/10) = x


x = 20
ans is 180+20=200miles. 2 marks

4) Fill the empty slots.


Three FOOTBALL teams are there. Given below the list of maches.

played won lost draw Goals for Goals against


A 2 2 *0 *0 *7 1

B 2 *0 *1 1 2 4
C 2 *0 *1 *1 3 7

the slots with stars are answers. 4 marks


BC drew with 2-2
A won on B by 2-0
a won on C by 5-1
( YOU HAVE TO FILL THE BLANKS AT APPROPRIATE STAR SYMBOLS.)`
5) There are 3 societies a,b,c. a lent tractors to b and c as many
as they had. After some time b gave as many tractors to a and c
as many as they have. After sometime c did the same thing.
At the end of this transaction each one of them had 24.
Find the tractors each orginally had.

Ans a had 39, b had 21, c had 12, 4 marks

6) There N stations on a railroad. After adding x stations 46


additional
tickets have to be printed. Find N and X.

Ans. let N(N-1) = t;


(N+x)(N+x-1) = t+46;
trail and error method x=2 and N=11 4 marks

7)Given that April 1 is tuesday. a,b,c are 3 persons told that their
farewell party was on
a - may 8, thursday
b - may 10,tuesday
c - june 8, friday
Out of a,b,c one is only correct one of the regarding month,day,date.
Other told two correct and the third person told all wrong.What is
correct date,month,day. 5 marks
(ans may be MAY 10 SUNDAY. check once again)
8)There are 4 parties. df,gs,dl(depositloss),ew ran for a contest.
Anup,Sujit,John made the following statements regarding results.
Anup said either df or ew will definitely win
sujit said he is confident that df will not win
John said he is confident that neither ew nor dl will win
the result has come. only one of the above three has made a correct
statement. Who has made the correct statement and who has won
the contest. 5 marks.
(ans DL )

Kumaraguru College, Coimbatore


9)Five people a,b,c,d,e are related to each other. Four of them make
one true statement each as follows.
i) b is my father's brother. (ans. d said this)
ii)e is my mother-in-law. ( b )
iii)c is my son-in-law's brother. ( e )
iv)a is my brother's wife. ( c )
who said each statement.
10 marks.

10) All members of d are also members of a


All '' e '' d
all '' c '' both a and b
not all '' a are members of d
not all '' d '' e
Some questions on these conditions.(5questions 5 marks)
11)
boys are allowed to watch football at c.v.Raman auditorium subjected
to
conditions.
i)the boy over age 16 can wear overcoat
ii)no boy over age 15 can wear cap
iii)to watch the football either he has to wear overcoat or cap
or both
iv) a boy with an umberella or above 16 or both cannot wear
sweater.
v) boys must either not watch football or wear sweater.

What is the appearence of the boy who is watching football.


Try to solve this question................

1. SAKUNTALA DEVI'S PUZZLE BOOK : PUZZLES TO PUZZLE YOU.


problem no: 3. ( Brothers and Sisters)
A family I know has several children. Each boy in this
family has as many sisters as brothers but each girl has
twice as many brothers as sisters. How many brothers
and sisters are there?
ans: 4 boys and 3 girls.
2. No. of animals is 11 more than the no. of birds. If the
no. of birds were the no. of animals and no. of animals
were the no. of birds( ie., interchanging no.s of animals
and birds.), the total no. of legs get reduced by one fifth
(1/5). How many no. of birds and animals were there?
ans: birds:11,animals:22
3. In a soap company a soap is manufactured with 11 parts.
For making one soap you will get 1 part as scrap. At the
end of the day u have 251 such scraps. From that how many
soaps can be manufactured? ans: 22 + 2+ 1 = 25.
4. 2** |
3** | No. 7 does not occur in this
---------------- |
5** | multiplication.
*4* |
**3 | Find the product.
---------------- |

Kumaraguru College, Coimbatore


***** |
---------------- |
--------------------------------------------------
ans 2 8 1
322
-----
562
5620
84300
---------
9 0482
---------
5. There is a 5digit no. 3 pairs of sum is eleven each.
Last digit is 3 times the first one.
3 rd digit is 3 less than the second.
4 th digit is 4 more than the second one.
Find the digit.
ans : 25296.
6. There are five thieves, each loot a bakery one after the

other such that the first one takes 1/2 of the total no.
of the breads plus 1/2 of a bread. Similarly 2nd, 3rd,4th
and 5fth also did the same. After the fifth one no. of
breads remained are 3. Initially how many breads were there?
ans : 31.
7.ESCALATOR PROBLEM OF SAKUNTALA DEVI 'PUZZLES TO
PUZZLE'book.
Problem No: Problem 27( Down the escalator)
ans : the no of steps in the stair way : 46.
8.Harbour line and Main line Problem of Sakuntala Devi
Puzzle
book. Ans : 4/5.
( More Puzzles book)
9.There are some chicken in a poultry. They are fed with
corn
One sack of corn will come for 9 days.The farmer decides to
sell some chicken and wanted to hold 12 chicken with him.
He cuts the feed by 10% and sack of corn comes for 30 days.
So initially how many chicken are there?
10.Two people X & Y walk on the wall of a godown in
opposite
direction. They meet at a point on one side and then go
ahead. X after walking for some time, walks in opposite
direction for 15 mtrs.Then again he turns back and walks
in the original direction. What distance did Y walk before
they met again, if X walks 11 mtrs by the time Y walks
8 mtrs.

1.My neighbour has seven children.Every brother has equal no. of


brothers and siters ,but each sister has twice as many brothers
as sisters?
Ans: 4,3
2.There are 11 more animals than birds in a pet shop.If there are
as many birds as animals and if there are as many animals as birds

Kumaraguru College, Coimbatore


the no. of legs is 4/5 of the original.Howmany birds and animals
are there?
Ans: 22,11
3.one soap can be made out of the scraps obtained while preparing
11 soaps.How many soaps can be prepared out of the scrap obtained
out of 251 soaps.(Take all possibilities)
Ans: 25
4.Find out the five digit number whose first digit is three times
that of it's fifth digit,the fourth digit is four more than the
second digit,the third digit is three less than the second digit,
and there are 3 pairs of digits such that each sum is 11?
Ans: 65292
5. 2XX
3XX
---
5XX
X4X
XX3
-------
XXXXX
-------
Find out the X's and seven is not there in the calculation.

Ans: 281x332
6.There are 5 burglars and once went to a bakery to rob it obviously
The first guy ate 1/2 of the total bread and 1/2 of the bread.
The second guy ate 1/2 of the remaining and 1/2 of the bread.
The third guy ,fourth guy and fifth guy did the same.After fifth
guy there is no bread left out.How many bread are there?
Ans:31
7.The main line train starts at 5.00AM and the harbour line train
starts at 5.02AM.Each train has the frequency of 10 minutes.If a
guy goes in the morning at a random time what is the probability
of he getting main line train?
Ans: 0.8
8.There is 66x33m rectangular area .Ram is 11/8 times faster than
Krishna.Both of them started walking at opposite ends and they
met at some point then,Ram said "See you in the other end"Then
they continued walking.After some time Ram thought he will have tea
so he turned back walked back 15 meters then he changed his mind
again and continued walking .How much Krishna has travelled by
the time they meet?
9.A farmer has C chickens.A sack of feed comes for 9 days.As the
feed cost is increasing the farmer sells some chickens and retains
12 chicken.If he reduces the feed quantity by 10% .Then he observes
that the feed comes for 30 days.What is C?
Ans:36
10.Escalator Problem.
A person walking takes 26 steps to come down on a escalator and it
takes 30 seconds for him for walking.The same person while running
takes 18 second and 34 steps.How many steps are there in the
escalator?
Ans:46

Anal Paper:
1. There are nine cards arranged in three rows and three columns.
Each row contains atleast one green card.There are three yellow
corners.Red is either in the first row or second row.There are

Kumaraguru College, Coimbatore


2 greens in the 3rd column and 2 blues in the second row.What is
the arrange ment of the cards
Ans: Y R G
BBG
YGY
2. There are eight cards.Each card has colour on both sides.There
are 2 blue 2 red 2 green 2 yellow.

(5). A + B + C +D = D + E + F + G = G + H + I =17.
IF A = 4 WHAT ARE THE VALUES OF D AND G. EACH LETTER
TAKEN
ONLY ONE OF THE DIGIT FROM 1 TO 9.
8MARKS

ANS : A = 4 ,B = 2, C =6, D = 5, E = 3, F = 8, G
= 1,
H = 7, I = 9.

1. 5,6,7,8,10,11,14, ?
2.
A 1 0 0 1 0 0 0 1
B 0 1 1 0 1 0 1 0
C 1 0 0 1 0 1 1 0
(AuB)nC = ? [ (A union B) intersection C = ?]
3. Find the Odd word .
Java, Lisp, Smalltalk, Eiffel
(One more wuestion of same type - Oddman out)

4.
Year 95 96 97 98 99
Members 100 1 70 10 50

i ) Which year has maximum members growth?


Two more questions on this. Please go
through Barrons GRE data interpretation problems

5. In Madras, temperature at noon varies according


to -t^2/2 + 8t + 3 (READ as : -t square /2 + ...),
where t is elapsed time. Find how much temp. More or
less in 4pm to 9pm. (May be we can solve it by
Definite Integration. Check any way)

6. A man, a woman, and a child can do a piece of work

Kumaraguru College, Coimbatore


in 6 days. Man only can do it in 24 days. Woman can do
it in 16 days and in howmany days child can do the
same work? ( Numbers are not correct. Problem model is
important)

7. What is the highest prime number that can be stored


in a 8-bit microprocessor?

8. In which of the system 384 is equal to 1234?


(Numbers are not correct. Important is only problem
model)

9. If D_MUQZM is coded as CENTRAL then RBDJK can be


coded as ---------

10. The size of a program is N. And the memory


occupied by the program is given by M = square root of
100N. If the size of the program is increased by 1%
then how much memory now occupied ?

11. French, English, German

1. How many more or less


speak English than French?
2. What % people speak all the three languages?
3. What % people speak German but not English?

12. The size of the bucket is N kb. The bucket fills


at the rate of 0.1 kb per millisecond. A programmer
sends a program to receiver. There it waits for 10
milliseconds. And responce will be back to programmer
in 20 milliseconds.
Based on above information one question is there.
13. A power unit is there by the bank of the river 750
mts. A cable is made from power unit to power a plant
opposite to that of the river at 1500mts. The cost of
the cable below water is Rs. 15/- per meter ansd cost
of cable on the bank is Rs. 12/- per meter.
Based on above information one question is there.
14. Match the following.

i. brother - sister a. Part of


ii. alsecian - dog b. Sibling
iii. sentence - paragraph c. Type of
iv. d.

15. If the vertex (5,7) is placed in the memory. First


vertex (1,1) 's address is 1245 and then address of
(5,7) is ----------

16. A Planar solid cube contains how many vertices,


howmany corner points and how many faces?

17. Which of the equation satisfies the graph?


4 equations given.

18. Square of the <<term>> is directly proportional to

Kumaraguru College, Coimbatore


the cube of the <<another term>> . If the <<term>> is
'a' and <<another term>> is 'b' then which one is
correct?
4 options given like a square / b cube =const...

Part III Critical reasoning


---------------------------

4 passages and each 3 questions.

Passage 1 : Barrons GRE Analytical Ability ->Practice


Excercise questions 7-11. ( passage is : The office
staff of the XYZ corporation.....) In 1996 edition
page number 377.
Any way Iam giving the passage for those who do not
have Barrons book.

The office staff of the XYZ corporation presently


consists of three book keepers (A, B and C) and five
secretaries (D,E,F,G, and H). Management is planning
to open a new office in another city using 3
secretaries and 2 book keepers of the present staff.
To do so they plan to seperate certain individuals
who do not function well together. The following
guidelines were established to set up the new office.

1. Book keepers A and C are constantly finding fault


with one another and should not be sent as a team to
the new office.

2. C and E function well alone but not as a team. They


should be seperated.

3. D and G have not been on speaking terms for many


months. They should not go together.

4. Since D and F have been competing for promotion,


they should not be a team.

Questions :
1. If A is to be moved as one of the book keeper which
of the following CANNOT be a possible working unit?
a. ABDEH b. ABDGH c. ABEFH d. ABEGH e.
ABFGH
ANS) b
2. If C and F are moved to the new office , how many
combinations are possible?
a. 1 b.2 c.3 d. 4 e. 5
ANS) a
3. If C is sent to new office, which member of the
staff CANNOT go with C?
a. B b. D c. F d. G e. H
ANS) b
4. Under the guidelines developed, which of the
following MUST go to the new office?
a. B b. D c. E d. G e. H
ANS) a

Kumaraguru College, Coimbatore


5. If D is going to the new office which of the
following is/are true?
I. C cannot go
II. A cannot go
III. H must also go
a. I only
b. II only
c. I and II only
d. I and III only
e. I, II, III
ANS)d
NOTE : Qustion paper contains project leads and
programmers not book keepers and secretaries.

Passage 2 : Barrons GRE Analytical Ability ->Practice


Excercise questions 14-17. ( passage is : After
months of talent searching for an .....) In 1996
edition page number 377.
Any way Iam giving the passage for those who do not
have Barrons book.
After months of talent searching for an
administrative assistant to the president of the
college the field of applicants has been narrowed down
to five (A,B,C,D and E). It was announced that
finalist would be chosen after a series of all-day
group personal interviews were held. The examining
committee agreed upon the following procedure.
1. The interviews will be held once a week.
2. Three candidates will appear at any all-day
interview session.
3. Each candidate will appear atleast once.
4. If it is necessary to call applicants for
additional interviews, no more than one such applicant
should be asked to appear the next week.
5. Because of the details in the written applications,
it was agreed that whenever candidate B appears, A
should also be present.
6. Because of the travel difficulties, it was agreed
that C will appear for only one interview.
Questions:
1. At the first interview, the following candidates
appear: A, B and D. Which of the following
combinations can be called for the interview to be
held the next week?
a. BCD b. CDE c. ABE d. ABC e. ADE
ANS) b
2. Which of the following is a possible combinations
for interviews in two consecutive interviews?
a. ABC ; BDE
b. ABD; ABE
c. ADE; ABC
d. BDE; ACD
e. CDE; ABC
ANS) c
3. If A, B and D appear at the interviw and D is
called for additional interview the following week,
which the two candidates may be asked to appear with
D?

Kumaraguru College, Coimbatore


I. A
II. B
III. C
IV. E
a. I and II
b. I and III only
c. II and III only
d. II and IV only
e. III and IV only
ANS)e
4. Which of the following correctly state(s) the
procedure followed by the search committee?
I. After the second interview, all appicants have
appeared atleast once.
II. The committee sees each applicant a second time.
III. If a third session is held it is possible fir akk
applicants to appear atleast twice.
a. I only
b. II only
c. I and II only
d. III only
e. I and III only
ANS)a

Passage 3 is some thing like this. There are swimmers


J,K,L,M,N,O,P and different conditions given.
Passage 4 is like . A former planted trees in 4X4
rows. No of plant verieties are 6. Like red, green
etc. Questions based on these arragements.

1. WHICH IS THE NEXT NO:


5,6,7,8,10,11,14,? ANS: 18
2. BFGE CODED AS CEHD THEN CODE PVHDJ ANS:
QUICK
3. FIND THE NO. OF Y FOLLOWED BY W BUT THAT IS NOT
FOLLOWED BY Z.
Y W R U D D Y W Z ...............
4. WHAT IS THE LARGEST PRIME NO THAT IS STORED IN 8
BIT PATTERN
ANS: 253 (NOT SURE)
5.WHICH WILL GIVE GOOD STANDARD DEVIATION
1. (7,0,-7,0,7) 2. (7,-7,7,-7,7) 3. (1,0,-1,0,1)
6. WHICH IS NOT A SIDE OF A RECTANGULAR
1. (2,3,4) 2.(3,4,7) 3. (3,5,9)
7.WHICH SHAPE WILL BE OBTAINED BY USING THESE VALUES
OF X ,Y
X Y
0 0.00001
10 1.02
100 1.72
1000 3.00
9999 4.72
8. WHICH EQUATION THAT BEST SUITS THIS CURVE

Kumaraguru College, Coimbatore


A LINE CUTS X AT -1 WHEN Y=0 AND X=0 WHEN Y=3 AND
GOES UPWARD
Y

X
9. A MAN , WOMAN AND A BOY JOINDLY DID A JOB IN 6
DAYS. A MAN ALONE FINISHES IN 10 DAYS, A WOMEN ALONE
FINISH IN 24 DAYS. THEN HOW MANY DAYS THE BOY CAN
TAKE TO FINSH?

10. FOR TEMPERATURE A FUNCTION IS GIVEN ACCORDING TO


TIME : ((t**2)/6) + 4t +12
WHAT IS THE TEMPERATURE RISE OR FALL BETWEEN
4.AM TO 9 AM

11. AN AEROPLANE STARTS FROM A (SOME LATITUDE IS GIVEN


ACCORDING TO PLACE)AT 2 AM LOCAL TIME TO B(SOME
LATITUDE). TRAVELLLING TIME IS 10 HOURS. WHAT IS THE
LOCAL TIME OF B WHEN IT REACHES B

12. A FILE IS TRANSFERRED FROM A PLACE TO


ADESTINATION CAPABLE OF 10 KB . THEY GIVEN SOME RATE
OF TRANSFER. U HAVE FIND A EQUATION THAT BEST SUIT
THIS.

13. IN A PLANAR CUBE , THE NO. OF VERTICES, NO OF


EDGES AND NO OF FACES ARE
1. 6,6,6 2. 4,8,12 3.... 4.........

14. VENN DIAGROM below


1. HOW MANY PERSON KNOW ENGLISH MORE THAN FRENCH
2. HOW MUCH % OF PEOPLE KNOWS ALL THE 3 LANGUAGES
3. HOW MUCH % OF PEOPLE THOSE WHO KNOWS FRENCH AND
GERMAN AND NOT ENGLISH

FRENCH

15. CORRECT CHART WITH CORRECT VALUES

X- YEAR
Y- NO OF PERSONS
1. AVERAGE NO. OF PERSONS FROM 1995 - 1999
2. WHICH YEAR HAS LARGE DIFFERENCES IN NO OF PERSONS
3. IF 10% OF PEOPLE LEAVES THE OFFICE IN 1998 THEN,
HOW MANY FRESH CANDIDATES CAN BE ADDED IN THE NEXT
YEAR

16. WHAT IS THE VALUE OF


M(373, 7) + R(6.8) -T(3.4) + R( 3.4)
M- MODULAS R- ROUNDOFF T- TRUNCATE
17. WHAT IS THE VALUE OF
% # % (5) + # % # (2)
WHERE % - DOUBLING #- RECIPROCAL

18. MATCH THE FOLLOWING


A B

Kumaraguru College, Coimbatore


1. SENTENCE, PARAGRAPH 1. TYPE OF
2. BASMATI, WHEAT 2. A PART OF
3. BROTHER, SISTER 3. NOT A TYPE OF
4. BREIGAL, DOG 4. SIBLING
ANS: 1-> 2 2->1 3->3 4->4

19. G (0) =1 G (1)= -1 IF G (N)=2* (G (N-1))- 3(G


(N-2)) THEN WHAT IS THE VALUE
OF G (4)?
20. A 0 0 0 0 1 1 1 1
B 0 0 1 1 0 0 1 1
C 0 1 0 1 0 1 0 1
(AUB)PC

21. TIME DEGREE


2 7 ° 6' 43.15"
4 5° 31' 4.3"

THEN WHAT WILL BE THE DEGREE WHEN TIME IS 3 O CLOCK

22. THREE COMPANIES WORKING INDEPENDENTLY AND GET


SAVINGS 10%,
20%, 25%. IF THE COMPANIES WORK JUST OPPOSITE THEN
WHAT WILL BE THE NET SAVING?

23. WHICH ONE WILL BE THE EXACT POWER OF 3


(i) 2768 (ii)2678 (III) 2187

24. SOME RELATION THAT IS DEDUCE TO


A (POW 2) DIRECT PROPORTIONAL TO X (POW 3)
B (POW 2) DIRECT PROPORTIONAL TO Y (POW 3)
SOME FOUR ANSWERS WERE GIVEN
ANS: ALL OF THE ABOVE

25. 900 M WIDE 3000 M WIDTH


SOMETHING I CAN'T REMEMBER SOME VALUES ARE GIVEN
BY AIR PER M Rs. 4 BY GROUND PER M Rs. 5
THEN WHERE U WILL CUT ADD MAN OUT

26. 1.JAVA 2.SMALLTALK 3.LISP 4.EIFFEL

27. 1.SAP 2.ARP 3.WAP 4.TCP IP

28. WHICH IS THE PERFECT ONE AMONG THE 4


1. 2x +3y=4 2. x + y= -1 3. Y=2x+ 3

29-32. I FORGOT

PART III
CRITICAL REASONING
-----------------
I FEEL VERY EASY

Q1-Q 3
A,B,C - PROGRAMMERS D,E,F,G,H- DATABASE HANDLERS
NEW OFFICE IS GOING TO BE STARTED WITH 2 PROGRAMMERS
AND 3 DATABASE HANDLERS

Kumaraguru College, Coimbatore


A AND C CANNOT GO TOGETHER
C AND E CANNOT GO TOGETHER
D AND F CANNOT GO TOGETHER

1.) two pencils cost 8 cents. then 5 pencils cost?


(20 cents)

2. A work is done by the people in 24 minutes. One of them can do


this work alone in 40 minutes. How much time to do the same work
for the second person?
(60 minutes)

3. A car is filled with four and half gallons of fuel for a round trip.
Fuel is taken 1/4 more in going then coming. What is the fuel
consumed in coming up? (2 gallons)

4.Low temperature at the night in a city is 1/3 more than 1/2 high as
higher temperature in a day. Sum of the low tem. and highest temp.
is 100 degrees. Then what is the low temp? (40 deg.)

5. A person, who decided to go to weekened trip should not


exceed 8 hours driving in a day. Average speed of forward journey
is 40 m/h. Due to traffic in sundays, the return journey average speed
is 30 m/h. How far he can select a picnic spot?
a) 120 miles
b) between 120 and 140 miles
c) 160 miles
ans: 120 miles
6. A salesperson multiplied a number and get the answer 3,
instead of that number devided by 3.
what is the answer he actually has to get?
1x3=3
so number = 1
devided by 3, the ans. is 1/3.
7. A ship started from port and moving with I miles per hour and another
ship started from L and moving with H miles per hour.
At which place these two ships meet?

|----|----|----|----|----|----|
port G H I J K L

8. A building with height D shadow upto G. A neighbour building with


what height shadows C feet.

|----|----|----|----|----|----|----|
A B C D E F G H

9. A person was fined for exceeding the speed limit by 10 mph.


Another person was also fined for exceeding the same speed limit
by twice the same. If the second person was travelling at a
speed of 35 mph, find the speed limit.. (15 mph)

10.A bus started from bustand at 8.00am, and after 30 minutes staying

Kumaraguru College, Coimbatore


at destination, it returned back to the busstand. The destination
is 27 miles from the busstand. The speed of the bus is 18mph. In
return journey bus travels with 50% fast speed.
At what time it returns to the busstand? (11.00am).

11.In a mixture, R is 2 parts, S is 1 part. In aoder to make S to


25% of the mixture, howmuch r is to be added?

SECTION-1 PART-3 marks-50 questions-50 30 min

CRITICAL REASONING

It is very difficult to reproduce them and so we are giving here the


first question of the section so that you may have a general idea of the
section.Be careful time management is important.
YOU HAVE TO MARK YES, NO OR WE CAN'T SAY..
My father is only child to his father.My father has three sisters.
All are married and have two children each.

1) My grandfather has two sons.


2) I am having six cousins.
3) I have three uncles.

Q1.Last year pandit was thrice his sister's age.Next year he is only twice her age.After 5 years what is pandit's age.
a)2 b)12 c)11 d)14
ans:b

Q2.A box of 150 packets consists of 1kg packets and 2kg packets. Total weight of box is 264kg. How many 2kg
packets are there
a) 96 b)67 c)100 d)114
ans:d

Q3.Which is more economical of the following


a)2kg -- 30/-
b)8kg -- 160/-
c)5kg -- 80/-
ans:

Q4.Satish earns 240 weekly.12% of big amount + earning weekly = 540


what is the big amount
a)3200 b)3600 c)2500 d)1000
ans: c

Q5. Jalia is twice older than qurban . If jalia was 4 years younger, qurban was 3 years older ther diff. between their ages
is 12 years
what is the sum of thier ages
a)67 b)57 c)36 d)none
ans: b

Kumaraguru College, Coimbatore


CTS Apti Question paper (Colour: Yellow)

There are other three colours also. We received only yellow Question papers.

1. If all the 6 are replaced by 9, then the algebraic sum of all the numbers from 1 to 100(both inclusive) varies
by Ans: 330

2. The total no. of numbers that are divisible by 2 or 3 between 100 and 200(both inclusive) are
Ans:67

3. From a pack of cards Jack, Queen, King & ace are removed. Then the algebraic sum of rest of the cards is
Ans:216

4. The average temperature of days from Monday to Wednesday is 37 degree Celsius and that of from
Tuesday to Thursday is 34 degrees. The temperature of Thursday is 4/5th of Monday. Then the temperature
of Thursday is
Ans: 36 degrees
5. Swetha, Tina, Uma and Vidya are playing a gambling. In this different people lose in different games-
in the reverse alphabetical order. The rule is that if one loses she should double the amount of others. At
the end of 4th game each of them have same amount of money (Rs.32). Which one of them started with the
least amount? (6) Which one of them started with the largest amount of money? (7.) At the end of the 2nd
game what is the amount of money with uma?
Ans: Vidya, Swetha, Rs.8
8. A cube of 12 mm is painted on all its side. If it is made up of small cubes of size 3mm. If the
big cube is splitted into those small cubes, the number of cubes that remain unpainted is
Ans: 8

9. B is 50% faster than A. If A starts at 9 A.M. and B starts at 10 A.M. A travels at a speed of 50 km/hr. If A
and B are 300 kms apart, The time when they meet when they travel in opposite direction is
Ans:12 noon

10. A graph will be there. Inside the graph sheet there will be a Quadrilateral. We have to count the number
of squares in the Quadrilateral.

11. You are having 31kg of rice. You are provided with a 1kg stone for weighing. In how many weights the
31kg of rice can be weighed. Ans: 5

12. A starts at 11:00AM and travels at a speed of 4km/hr. B starts at 1:00PM and travels at 1km/hr for the
first 1hr and 2km/hr for the next hr and so on. At what time they will meet each other.
Ans:

13. There are 80 coins, among them one coin weighs less compared to other. You are given a physical
balance to weigh. In how many wieghings the odd coin can be found.
Ans:
14. Dia of the circle 4cm. The shaded part is 1/3 of the square area. What is the side of the square.
Ans: root of 3pi

15. A,B,C, can do a work in 8,14,16 days respectively. A does the work for 2 days. B continues from it and
finishes till 25% of the remaining work. C finishes the remaining work. How many days would have
taken to complete the work Ans:

Kumaraguru College, Coimbatore


16. Raja went to a beauty contest .his wife was eager to know the result he told that the lady wear a yellow
sari was winner. Miss. Andhra Pradesh Miss. Utter Pradesh, Miss. Maharashtra, Miss. West Bengal were
the participants all the participants sat in a row. The conditions are (A) The woman wore yellow sari won
the competition. (B) Miss. West Bengal was neither the runner-up or winner.(C) Miss. West Bengal was
not at either ends.(D) Miss. Maharastra wore the white sari.(E) The women wore white sari and yellow
sari sat at extreme ends.(F) The runner-up and winner did not sit together. [This was the passage given
and the questions were easy]

17. The ratio of white balls and black balls is 1:2. If 9 gray balls is added it becomes 2:4:3. Then what is
number of black balls. Ans:12

18. There are 10 coins. 6 coins showing head. And 4 showing tail. Each coin was randomly flipped (not
tossed) seven times successively.after flipping the coins are 5 heads 4 tails one is hided the hided coin
will have what.

19. Two cars are 500 cm apart. each is moving forward for 100 cm at a velocity of 50 cm/s and receding
back for 50 cm at 25 cm/s at what time they will collide with each other.

20. People near the sea shore are leading a healthy life as they eat fish.but people at other part of the city are
also healthy. Inference.

21. It is found from research that if u r a drunken then u have a less chance for chronic heart diseases.
Inference.

22. A-B+c>A+B-C i) B is +ve, ii) B is –ve when it will hold true.

23. i) C.P is Rs 120 and profit is 30%


ii) C.P is Rs 210 and profit is 20%
we can find the S.P by using
i)only ii)only both i &ii neither i&ii

24. How will u find distance between Nagpur and Mumbai?


I took one hour more when I travel at 80 km/hr than at 90 km/hr.

25. 100 coins were collected by four persons each collected more than 10 each collected a different
number each was an even number find what is the max possible no of coins, two more questions based on the same
passage.

26. A car travels from B at a speed of 20 km/hr. The bus travel starts from A at a time of 6 A.M. There is a
bus for every half an hour interval. The car starts at 12 noon. Each bus travels at a speed of 25 km/hr. Distance
between A and B is 100 km. During its journey , The number of buses that the car encounter is

27. The ratio of the ages of the father and the son is 5:3, After 10 years it will be in the ratio 3:2. What will be
their ages.
28. There was a Island. In that Island there was Rubys and Emeralds. Those were available in plenty. 0.3 kg
of ruby is 4 lakhs and 0.4 kg of emeralds is 5 lakhs. Jayanth is buying 12 kg of Ruby and emerald.
Choices will be given . Jayanth has to carry both ruby and emerald to the maximum profit.
29. Varun buys 8 books,10 pens and 2 pencils and Babu buys 6 books, 5pens and 5 pencils. Babu pays 50%
more than Varun. What is the amount Varun spends in buying pencils.
30. Prakash and Revathi rent a fancy shop. Prakash imposes the following restrictions on Revathi for buying
clips,stickers and lip sticks. The number of clips is twice the no. of stickers. The no. of lipsticks should be
more than the sum of clips and stickers. Cost of clip is 1 rupee. Cost of lipstick is twice the clips. Cost of
1 lipstick is the cost of four stickers. Then What is the amount that Prakash spents for Revathi.
Answer choices will be given.

CTS GCT 28 MAY 2003

Kumaraguru College, Coimbatore


SANTOSH
MAYA
BALAJI
SHANTI

CSE AND ECE TOTAL SHORTLISTED------24+26

This is the CTS paper that was given to GCT. This is the only college they went in Coimbatore. Then there were three
papers- pink, violet, and yellow. The paper they gave here was pink. The criterion was above 6.5 with no current
arrears. They also saw the 10th and 12th marks along with the entrance marks. They opened only for cse and ece. I cud
collect some of the questions. It doesn’t hav the full paper. Actually the test is for an hour. There was quans, annals,
reading comprehension, logical reasoning. The problems were more of R.S Agarwal. The others were just like in GRE.

1. Using the digits 1,5,2,8 four digit numbers are formed and the sum of all possible such numbers.
ans:106656

2. Four persons can cross a bridge in 3,7,13,17 minutes. Only two can cross at a time. find the minimum time taken
by the four to cross the bridge.
ans:20

3. Find the product of the prime numbers between 1-20


ans..9699690

4. 2,3,6,7--- using these numbers form the possible four digit numbers that are divisible by 4. ans.----8

5. Two trains are traveling at 18kmph and are 60 km apart. There is fly in the train. it flies at 80kmph. It flies and
hits the second train and then it starts to oscillate between the two trains. At one instance when the two trains
collide it dies. Distance traveled by the fly when both trains collide is Ans.---12km

6. there are 1000 doors that are of the open-close type. When a person opens the door he closes it and then opens
the other. When the first person goes he opens-closes the doors ion the multiples of 1 i.e., he opens and closes all
the doors. when the second goes he opens and closes the doors 2, 4 6 8 resly. Similarly when the third one goes he
does this for 3 6 9 12 15th doors resly. Find number of doors that are open at last.
Ans:square numbers

7.There are 9 balls of this one is defective. Find the minimum no. of chances of finding the defective one.Ans
3times

8. There are coins of Rs.5, 2,1,50p,25p,10p,5p. Each one has got a weight. Rs 5 coin weighs 20gms.find the
minimum number of coins to get a total of 196.5gms.

9.A can do a work in 8 days, B can do a work in 7 days, C can do a work in 6 days.
A works on the first day, B works on the second day and C on the third day resly.that is they work on alternate
days. When will they finish the work.(which day will they finish the work)
Ans: 7 7/168 days

10.A batsman scores 23 runs and increases his average from 15 to 16. find the runs to be made if he wants top inc
the avg to 18 in the same match.
ans: 39runs.

11.A man sells apples. First he gives half of the total apples what he has and a half apple.
Then he gives half of the remaining and a half apple. He gives it in the same manner. After 7 times all are over.
How many apples did he initially have.
ans: 127 apples.

Kumaraguru College, Coimbatore


12.In a club there are male and female members. If 15 female quit then the number of females will become
double the number of males. If 45 males quit no. of female becomes five times the number of males. Find the
number of females.
ans: females:175,males:80

13.When I was married 10 years back my wife was the sixth member of my family. Now I have a baby. Today
my father was dead and I had a new baby.now the average age of my family is the same as that when I was
married. Find the age of my father when I was married.
ans:50

14.I and two of my friends were palying a game. For each win I get Rs 3. totally I had three wins. Player 2 got
Rs9 and player 3 got Rs 12. how many games had been played.

15.A person gives a secret to two other persons in 5 minutes. How long will he take to tell the secret to 768
people.

16.There are 40 seats in a bus. People agree to share the money for the number of seats. The total money
comes to 70.37. how many seats were free.
9 seats.Rs.2.27

17.I had Rs100 and I play. If I win I will hav Rs110 and if I lose I will hav Rs90. at the end I hav 2 wins and 2
loses. How much do I hav.

18.There were sums related to diagrams. They asked to calculate the areas of the circle, rectangle that were
enclosed in other objects. They were simple.

20. In a village, there is flood. In one village causalities were less than the other. Why?

Ans : There were better health care centres(HCC).

21. A question on Pythagoras Theorem. Ans. 20

23. The distance between Mumbai & Calcutta is 25000 Km. Train goes from Mumbai to Culcutta for which

Speed & Time are given. From C->M Speed alone is give. Of the above conditions which is not required. (Not

Complete)

ans: The distance 25,000Km is not required. Because, Speed * Time = Distance. So only two conditions

are required.

24. m < n & x>y Which is false?

Ans: x-m < y-n

25. A person has Rs.100. If he wins he gains 10%. If he loses the game, he loses 10%. He wins twice and

loses twice. How much he has at the end?

Ans: Always less than 100.

26. Area of Shaded portion is ?

Kumaraguru College, Coimbatore


Ans : 115.5

Verbal

27. In A tribal group two groups live in different climatic conditions. Ear Sensitivity is tested and found that

one has more when compared to other. What is the reason.

Ans. Depends on the physical place and condition he is living.

There were many questions on logical reasoning.


Eg:
There are two identical islands. Same tribe live in the islands. But their receptiveness varies.
This is the question.
There were four choices and we have to select the most appropriate one.
For the above one the answer is ----- because of climatic changes
There was a question in which they gave a polygon with all the external angles. we have to calculate the asked
interior angle.

CTS 2003 – nit jsr


Aptitude test: 60Qs Time 1 hr

Booklet color : Blue

1)

Diameter of circle is d, Find length of string.(outer string that covers the circle)

Ans: d (pi + 3)

2) Diamond\’s value is proportional to its weight2 .When the diamond broke wts of pieces in ratio 1:2:3;4:5.

Kumaraguru College, Coimbatore


Total loss in value is 85,000.What is the value of the diamond twice the wt of the original diamond.

Ans : 45,000

3)Person X join a job at 20 yrs.First 3 years sal =10,000 p.a. Afterwards every year inc of 2,000 per year for 10 year.
Then sal become const till retirement. at retirement avg sal is 25,000. ( thro’out career) what age he retires.?

4)In an island there r tribals who speak lang of atmost 4 words.Lang consists of 4 alphabets.How many words can be
formed in that language? Ans 340

5) It was found that the cause for the malaria was the swamp marsh and so r swamps were drained .Mosquito the real
cause for malaria due to lack of breeding grounds (Swamps) also was wiped out. What does this illustrate?

(Ans : (Possible) when many conditions form a result eradication of one cause also eradicates the result)

6)An officer kept files on his table at various times in the order 1,2,3,4,5,6 .Typist can take file from top whenever she
has time and type it.What order she cann’t type.?

(Ans : 4,5,6,2,3,1)

7)A and B r fighting .B fires 3 times as many missiles as A. Total hits: total misses = 1/7 .B’s misses 357.B’s hits – A’s
hits = 66.A’s hits?

8)40 shots taken.50p for a hit.10p for a miss.(he have to give).Finally he has Rs.5.How many hits.? Ans 15

9)Find avg of a,b,c,d,e .Given data : avg of any 4 num =avg of any 3 num 2)(a+b)² = 36

Which of the abv are sufficient?

10)What is the difference in times btwn clk 1 & clk2.

1) both show same time 6 hrs back 2 ) 1 clk gains 1 min an hr,clk2 gains 2 min an hour.
Like abv….

11)A takes 9 strides to B’s 7 strides. A stride = 1meter.B stride =1.2m B gets the start of 24m.What dist should A travel
to overtake B?

12) Tortoise gets 100 m head start. Hare is 10 times faster as tort. What is the dist traveled by hare to catch up tort.?

13)4 weights r weighed in pairs. Weights of pairs are determined as 103,105,106,106,107,109 What is the min wt?
Ans 51

14) Constant cost = 300


and 1.75 / copy. How many copies should he sell at 7.75 /copy to make a profit.

15)

Kumaraguru College, Coimbatore


Find the perimeter ? ANs 28

16) 20 members avg =10.5. 3 memb of 11.5,12.5,13.5 left and 3 memb of 10.5,12.5,14.5 joined along with a teacher of
21 yr.Now avg = ….Ans 11.

17)

Find the area of shaded. Radius of circle = 1cm…..Arcs r drawn with center at circumference.

18)

Kumaraguru College, Coimbatore


Find the area of the shaded portion?

19) Solid cube of 6 * 6 * 6. This cube is cut into to 216 small cubes.(1 * 1 * 1).the big cube is painted in all its
faces. Then how many of cubes are painted at least 2 sides. (Ans 56)

20) A Bacteria is doubling at every 4 min. After 40 min 1024 bact. Then 256 when>…..? Ans : 32 min

21) A bag contains 3 balls of 11 different colors each. Find the min no of chances to find at least 3 balls of same
color?

Ans : 23
22) If x² < 4 then 100/x is….? Ans : 100/x > 50 & 100/x < -50.

23) If [x] is the int less than x and |x| is the abs val of x.Then max of [x]/|x| is Ans d)none

24) A work in 12 days b in 15 days. Find the no of days if they work on alternate days. Ans 13 ¼

25) A,B,C r positive int.Out of them 2 r odd. Then 5²a + ( b-5)3 (c-3)² = ? Ans : always odd.

26) A squarer side is 5cm.If a square of side 10cm is hinged @ the center of the prev square. when they r rotated
common area to both squares (Ans : Does not change)

27) 3p² + pq + 5q² is even. If

a) If p is odd, q is odd
b) If p is even, q is odd
c) If p is odd ,p is even
d) Atleast one of p and q is odd.

Choices are given. Ans : 4) None of these

28) A lady has to feed a dog for the one week from Monday to Sunday .She has food types M,N,O,P,Q,R,S .

MNOP  protein enriched RS - vitamin enriched. Vitamin enriched cannot be fed on consecutive days.
Conditions given : M should be fed before S.
M should be fed before Q.
R Should be fed before S.
Before N and Q there should be four types.
Based on this 3 ?s are asked.All r easy to answer…

Kumaraguru College, Coimbatore


29) A man bought at the cost of 5 plums a rupee and 2 oranges a rupee.He sells 10 plums and 6 oranges at the
selling price of 4 plums a rupee and 3 oranges a rupee.What is his gain or loss? Ans loss of 50p.
30) Out of 32 books the cost of 10 books is Rs. 50 each and he got a profit of 4%. He sells 15 books at a profit of
3.8461% on the selling price of Rs 70. The remaining cost is 576. The remaining books are sold at Rs 74.
What is his total profit.
31) Two solutions have milk & water in the ratio 7:5 and 6:11.Find the proportion in which these two solutions
should
Be mixed so that the resulting solution has 1 part milk and 2 parts water?
a)35:3 b)21:36 c)not possible Ans :c

Kumaraguru College, Coimbatore

You might also like